TCM Case Studies: Pediatrics 9787117156684

TCM Pediatrics is concerned with children’s growth and development along with physiological and pathological conditions

1,230 124 12MB

English Pages [824]

Report DMCA / Copyright

DOWNLOAD FILE

Polecaj historie

TCM Case Studies: Pediatrics
 9787117156684

Table of contents :
Cover
Sealed Cover
Editors
CIP data in Chinese
CIP data in English
Authors
About the Authors
Editorial Board for the TCM Case Studies in the International Standard Library of Chinese Medicine
Preface
Table of Contents
General Introduction
PHYSIOLOGICAL, ETIOLOGICAL AND PATHOLOGICAL CHARACTERISTICS
DIAGNOSIS
PEDIATRIC THERAPEUTICS
HOW TO USE THIS BOOK
LEARNING TIPS
PART I Lung Diseases
Chapter 1 Common Cold
Chapter 2 Tonsillitis
Chapter 3 Cough
Chapter 4 Pneumonia with Panting and Cough
Chapter 5 Asthma
Chapter 6 Recurrent Respiratory Tract Infection
PART II Spleen Diseases
Chapter 7 Thrush
Chapter 8 Aphtha
Chapter 9 Vomiting
Chapter 10 Abdominal Pain
Chapter 11 Diarrhea
Chapter 12 Aversion to Food (Anorexia)
Chapter 13 Food Accumulation
Chapter 14 Malnutrition
Chapter 15 Nutritional Iron Deficiency Anemia
Chapter 16 Vitamin D-Deficiency Rickets
PART III Heart-Liver Diseases
Chapter 17 Sweating Syndrome
Chapter 18 Viral Myocarditis (VMC)
Chapter 19 Anaphylactoid Purpura
Chapter 20 Attention Deficit Hyperactivity Disorder
Chapter 21 Tourette Syndrome (Multiple Physical Tics)
Chapter 22 Infantile Convulsion
Chapter 23 Epilepsy
PART IV Kidney Diseases
Chapter 24 Acute Glomerulonephritis
Chapter 25 Nephrotic Syndrome
Chapter 26 Urinary Frequency
Chapter 27 Enuresis
Chapter 28 Five Types of Developmental Delay and Five Kinds of Flaccidity
PART V Infectious Diseases
Chapter 29 Measles
Chapter 30 Exanthema Subitum
Chapter 31 Rubella
Chapter 32 Chickenpox
Chapter 33 Hand-Foot-and-Mouth Disease
Chapter 34 Mumps
Chapter 35 Epidemic Encephalitis B
Chapter 36 Infectious Mononucleosis
Chapter 37 Pertussis
Chapter 38 Scarlatina
PART VI Neonatal Diseases
Chapter 39 Scleroderma Neonatorum
Chapter 40 Fetal Jaundice
PART VII Other Diseases
Chapter 41 Summer Fever
Chapter 42 Mucocutaneous Lymph Node Syndrome
Appendix I: Pinyin-English Formula Cross Reference
Appendix II: List of Cited Sources

Citation preview

Editors Project Managers: Zeng Chun, Harry F. Lardner & Liu Shui Copy Editor: Yu Yang Book Designer: Li Xi Cover Designer: Li Xi Typesetter: Shan Si

CIP data in Chinese 图书在版编⽬(CIP)数据 ⼉科学= Pediatrics /王孟清,吴潜智,雷⽟娥主编. —北京:⼈民 卫⽣出版社,2014 (英⽂版中医病案教育系列) ISBN 978-7-117-15668-4 Ⅰ. ①⼉… Ⅱ. ①王…②吴…③雷… Ⅲ. ①中医⼉科学-英⽂ Ⅳ. ①R272 中国版本图书馆CIP数据核字(2014)第099188号 ⼈卫社官⽹ www.pmph.com 出版物查询,在线购书 ⼈卫医学⽹ www.ipmph.com 医学考试辅导,医学数据库服务,医 学教育资源,⼤众健康资讯 版权所有,侵权必究! 中医病案教育系列:⼉科学(英⽂版) 主 编:王孟清 吴潜智 雷⽟娥 出版发⾏:⼈民卫⽣出版社(中继线010-59780011) 地 址:中国北京市朝阳区潘家园南⾥ 19 号 世界医药图书⼤厦B座 邮 编:100021

⽹ 址:http:∥www.pmph.com E - mail:pmph @ pmph.com 购书热线:010-59787592 010-59787584 010-65264830 开 本:787×1092 1/16 版 次:2014年 ⽉第1版 2014年 ⽉第1版第1次印刷 标准书号:ISBN 978-7-117-15668-4/R·15669 打击盗版举报电话:010-59787491 E-mail:WQ @ pmph.com (凡属印装质量问题请与本社市场营销中⼼联系退换)

CIP data in English Website: http://www.pmph.com/en Book Title: TCM Case Studies: Pediatrics 中医病案教育系列:⼉科学 Copyright © 2014 by People’s Medical Publishing House. All rights reserved. No part of this publication may be reproduced, stored in a database or retrieval system, or transmitted in any form or by any electronic, mechanical, photocopy, or other recording means, without the prior written permission of the publisher. Contact address: No. 19, Pan Jia Yuan Nan Li, Chaoyang District, Beijing 100021, P.R. China, phone/fax: 8610 5978 7584, E-mail: [email protected] For text and trade sales, as well as review copy enquiries, please contact PMPH at [email protected] Disclaimer This book is for educational and reference purposes only. In view of the possibility of human error or changes in medical science, the author, editor, publisher and any other party involved in the publication of this work do not guarantee that the information contained herein is in any respect accurate or complete. The medicinal therapies and treatment techniques presented in this book are provided for the purpose of reference only. If readers wish to attempt any of the techniques or utilize any of the

medicinal therapies contained in this book, the publisher assumes no responsibility for any such actions. It is the responsibility of the readers to understand and adhere to local laws and regulations concerning the practice of these techniques and methods. The authors, editors and publishers disclaim all responsibility for any liability, loss, injury, or damage incurred as a consequence, directly or indirectly, of the use and application of any of the contents of this book. First published: 2014 ISBN: 978-7-117-15668-4/R·15669 Cataloguing in Publication Data: A catalogue record for this book is available from the CIP-Database China. Printed in The People’s Republic of China

Authors Contributors Yang Qian Attending Physician of TCM Pediatrics, the First Hospital Affiliated to Hunan University of CM, Changsha, China Li Ying, M.S. TCM Resident Physician of TCM Pediatrics, the First Hospital Affiliated to Hunan University of CM, Changsha, China Li Yan, M.S. TCM Resident Physician, the First Hospital Affiliated to Hunan Traditional Chinese Medical College, Zhuzhou, China Li Hua, M.S. TCM Resident Physician of TCM Pediatrics, the First Hospital Affiliated to Hunan University of CM, Changsha, China Yu Yan-lan, M.S. TCM Nurse-in-Charge, Head Nurse, the First Hospital Affiliated to Hunan University of CM, Changsha, China Zou Hua, M.S. TCM

Lecturer, Hunan Traditional Chinese Medical College, Zhuzhou, China Ou-yang Yao Associate Chief Physician of TCM Pediatrics, Director of Pediatrics, Liuyang TCM Hospital of Hunan Province, Liuyang, China Zhou Shan, M.S. TCM Resident Physician, the First Hospital Affiliated to Hunan University of CM, Changsha, China Hu Yan, M.S. TCM Resident Physician, the First Hospital Affiliated to Hunan University of CM, Changsha, China Huang Sheng Attending Physician, the First Hospital Affiliated to Hunan University of CM, Changsha, China Huang Ting, M.S. TCM Resident Physician, the First Hospital Affiliated to Hunan University of CM, Changsha, China Jiao Luo-jia, M.S. TCM Associate Professor, Hunan University of CM, Changsha, China Zhan Wen, M. S. TCM Resident Physician, Yiyang City Central Hospital, Hunan Province, Changsha, China Translation Committee Translator-in-Chief

Lei Yu-e Associate Professor, Hunan University of CM Translators Deng Lian Hunan University of CM Xie Yue-xiang Hunan University of CM Yan Yan Hunan University of CM Zhang Dan Hunan University of CM English Editors Harry F. Lardner, Dipl.Ac. Editor and Project Manager of International Publication Department, People’s Medical Publishing House William Bryan Ellett, Austin, Texas, USA

About the Authors

王孟清教授 Dr. Wang Meng-qing has been teaching TCM Pediatrics for 30 years, currently acting as Director of TCM Pediatrics Department at the First Hospital Affiliated to Hunan University of Chinese Medicine, where over 150 thousand children are treated each year. His research projects mainly focus on pulmonary system diseases, as well as spleen and stomach diseases, with an emphasis on asthma, pneumonia and diarrhea. Dr. Wang has a particular interest in alternative methods of medicinal administration including transdermal administration, atomizing inhalation and external applications. Dr. Wang has taken charge of 9 major research projects including those funded by the National Natural Science Foundation of China, and has won 4 major awards including a second prize of Science and Technology Award sponsored by the China Association of Chinese Medicine. He has published over 40 academic papers and 16 monographs on pediatrics.

吴潜智教授 Wu Qian-zhi first learned acupuncture and Chinese medicine while apprenticing with a so-called “barefoot doctor” in the Chinese countryside near his hometown. His formal studies began in 1978 at Luzhou Medical College, after which he entered the Chengdu University of Traditional Chinese Medicine. In Chengdu, he studied most notably with Li Ke-guan, whose father was the founder of the University in early 1930s. Wu’s first book, Qu Wei Zhong Yi, was published prior to his graduation, and in 1993 he became one of the youngest professors ever employed by the university. By 1993, Prof. Wu had published 5 other books as well as dozens of academic papers. After visiting Germany and Austria in 1994 and 1995, Prof. Wu took a teaching position at the Academy of Oriental Medicine at Austin, Texas, where he now serves as Vice President. He is a former Commissioner for the National Certification Commission for Acupuncture and Oriental Medicine (NCCAOM), chairman of the Chinese Herbal Test Development Committee of NCCAOM, and currently acts as a site visitor for the Accreditation Commission for Acupuncture and Oriental Medicine. Prof. Wu continues to lecture on Pediatric tui na in yearly continuing education courses.

雷⽟娥副教授 Lei Yu-e, an associate professor of Hunan University of Chinese Medicine, has been engaged in English language and medical English teaching for about 30 years. Her research interest is on translation of Chinese medicine and intercultural communication. She has translated Cyber-Based Bilingual Teaching Platform for Chinese Medical Formulas, and TCM Case Studies: Pediatrics.

Editorial Board for the TCM Case Studies in the International Standard Library of Chinese Medicine Executive Directors Li Zhen-ji (李振吉) Vice Chairman and Secretary-General, World Federation of Chinese Medical Societies, Beijing, China Hu Guo-chen (胡国臣) President and Editor-in-Chief, People’s Medical Publishing House, Beijing, China Directors Liang Fan-rong (梁繁荣) Professor and Doctoral Supervisor of Acupuncture and Moxibustion, and President of Chengdu University of TCM, Chengdu, China He Qing-hu (何清湖) Professor and Doctoral Supervisor of Chinese External Medicine, and Vice President of Hunan University of CM, Changsha, China

General Coordinators Liu Shui (刘⽔) Vice-Director of International Publication Center, People’s Medical Publishing House, Beijing, China Wang Han-bing (王寒冰) Former Vice-director of International Cooperation and Exchange Department, Chengdu University of TCM, Chengdu, China Harry F. Lardner, Dipl.Ac. Editor and Projector Manager of International Publication Center, People’s Medical Publishing House, Beijing, China Members Al Stone, L.Ac., DAOM Cai Guang-xian (蔡光先), M.S. TCM Chief Physician, Professor, Doctoral Supervisor and Academic Leader of TCM Internal Medicine, Hunan University of CM; Director of Key Laboratory of TCM Internal Medicine Sponsored by Provincial Government and Ministry of Education, Changsha, China Cara O. Frank, L.OM. Dipl. OM. Educational Director of the Department of Chinese Herbology, Won Institute of Graduate Studies, Glenside, PA, USA Chen Yun-hui (陈云慧), Ph.D. TCM

Lecturer of Chinese Materia Medica, School of International Education, Chengdu University of TCM, Chengdu, China He Qing-hu (何清湖), Ph.D. TCM Professor and Doctoral Supervisor of Chinese External Medicine, and Vice President of Hunan University of Chinese Medicine, Changsha, China Hu You-ping (胡幼平) Professor and Doctoral Supervisor, Acupuncture and Tui Na College/The Third Affiliated Hospital, Chengdu University of TCM, Chengdu, China Huang Ying (黄莺) Chief Physician, Professor and Vice-Director of the Department of Dermatology, Teaching Hospital of Chengdu University of TCM, Chengdu, China Jake Paul Fratkin, OMD, L.Ac. Continuing Education Faculty, American College of Traditional Chinese Medicine, San Francisco; Continuing Education Faculty, Oregon College of Oriental Medicine, Portland; Continuing Education Faculty, Austin College of Oriental Medicine, Austin, Texas, USA James Flowers PhD Candidate, College of Oriental Medicine, Wonkwang University, South Korea; Secretary General, International Association for the Study of Traditional Asian Medicine Lei Lei (雷磊), Ph.D. TCM

Chief Physician, Professor & Doctoral Supervisor of TCM Gynecology; Academic Leader of TCM Gynecology, National Key Discipline of the State Administration of Traditional Chinese Medicine, Hunan University of CM, Changsha, China Lei Yu-e (雷⽟娥) Associate Professor, Hunan University of CM, Changsha, China Liu Bai-yan (刘柏炎), Ph.D. Integrative Medicine Professor and Doctoral Supervisor of TCM Internal Medicine, Hunan University of CM; Director of Hunan Provincial Key University Laboratory of TCM Internal Medicine, Changsha, China Ma Xi-tao (马喜桃) Resident Physician, Teaching Hospital of Chengdu University of TCM, Chengdu, China Misha R. Cohen, OMD, L.Ac. Clinic Director, Chicken Soup Chinese Medicine; Research Specialist in Integrative Medicine, UCSF Institute for Health and Aging; Associate Member, UCSF Comprehensive Cancer Center; Director, MRCE Foundation Pär Rufus Scott, MOAM, Lic. Ac. Peng Qing-hua (彭清华), Ph.D. TCM Chief Physician, Professor and Doctoral Supervisor of TCM Ophthalmology, the First Hospital Affiliated to Hunan University of

Chinese Medicine; President of the School of International Education, Hunan University of CM, Changsha, China Portia Barnblatt, DAOM, L.Ac. Dipl.Ac. Portia Chan Acupuncture Clinic, San Francisco, U.S.A.; Clinical supervisor for American College of Traditional Chinese Medicine, San Francisco, U.S.A Suzanne Robidoux, Ph.D, DOM, L.Ac. Post Doctor Laureates of Beijing University of Chinese Medicine, Specialized in Classical Medicine and Neurology, Beijing, China Wan Peng (万鹏), M.S. TCM Lecturer and Attending Physician of Chinese External Medicine, School of Clinical Medicine/Teaching Hospital of Chengdu University of TCM, Chengdu, China Wang Jing (王静), Ph.D. TCM Lecturer of TCM Gynecology, Chengdu University of TCM, Chengdu, China Wang Meng-qing (王孟清), Ph.D. TCM Professor, Chief Physician and Ph.D. Supervisor of TCM Pediatrics; Director of TCM Pediatrics Department and TCM Pediatrics Teaching Office, the First Hospital Affiliated to Hunan University of CM, Changsha, China William R. Morris, PhD, DAOM, L.Ac. President, AOMA Graduate School of Integrative Medicine, Austin, Texas, USA

Wu Jun-mei (吴俊梅), Ph.D. of Acupunctureand Moxibustion Professor and Master Supervisor, Acupuncture and Tui Na College/The Third Affiliated Hospital, Chengdu University of TCM, Chengdu, China Wu Qian-zhi (吴潜智), M.S. TCM Vice President of Faculty, Professor and Licensed Acupuncturist, AOMA Graduate School of Integrative Medicine, Austin, Texas, USA Ye Xiao (叶晓), M.S. TCM Lecturer, Foreign Languages College, Zhejiang Chinese Medical University, Hangzhou, China Zeng Chun (曾纯), M.S. TCM Editor and Projector Manager of International Publication Center, People’s Medical Publishing House, Beijing, China Zeng Sheng-ping (曾升平) Chief Physician and Professor of TCM Rheumatology and Immunology, Chengdu University of TCM, Chengdu, China Zhang Yan-hong (张艳红) Associate Professor, English Teaching and Research Office, Hunan University of CM, Changsha, China Zheng Qi (郑琦) Freelance Translator/Interpreter, Beijing, China Sponsored by World Federation of Chinese Medical Societies

Preface Teaching with case studies is a hands-on, problem-solving and interactive teaching method. By studying representative cases, students can enhance their clinical capacity through gaining insight into the application of theoretical knowledge; cultivating this ability is a critical aspect of effective TCM education. This book is aimed at students and junior practitioners of Traditional Chinese Medicine in order to reinforce basic knowledge and enhance clinical thinking as illustrated through case records. The key diagnostic characteristics and treatment methods of TCM Pediatrics are the focus of this text. TCM Pediatrics is concerned with children’s growth and development along with physiological and pathological conditions and their treatment with traditional Chinese medicinals, acupuncture, tui na and other therapeutic methods. As an important TCM clinical discipline with distinct features, the study of Chinese Medical Pediatrics links closely with other basic and clinical courses and also plays a vital role in enhancing students’ clinical thinking and practice in general. This book is mainly intended as a tool to be employed in the teaching of TCM. Following each case, there are exercises with questions broadly based on the disease category in order to compensate for the lack of breadth offered by a single case. In this way, the student is given the opportunity to review and consolidate their basic knowledge of the disease.

The cases here are compiled mainly by pediatricians from Hunan University of Chinese Medicine who have long been engaged in clinical practice and teaching, and all cases involve commonly seen conditions in the TCM pediatric department. However, this is our first attempt in editing clinical case teaching materials for students of Chinese Medicine abroad, so some inadequacy is inevitable. It is our sincere hope that the field of Chinese medicine can flourish abroad as it does within China, and to this end, we encourage our readers to provide us with feedback so that our future efforts can be improved. Wang Meng-qing March 6th, 2014

Table of Contents

General Introduction PHYSIOLOGICAL, ETIOLOGICAL AND PATHOLOGICAL CHARACTERISTICS DIAGNOSIS PEDIATRIC THERAPEUTICS HOW TO USE THIS BOOK LEARNING TIPS PART I Lung Diseases Chapter 1 Common Cold Chapter 2 Tonsillitis Chapter 3 Cough Chapter 4 Pneumonia with Panting and Cough Chapter 5 Asthma Chapter 6 Recurrent Respiratory Tract Infection PART II Spleen Diseases Chapter 7 Thrush Chapter 8 Aphtha Chapter 9 Vomiting Chapter 10 Abdominal Pain Chapter 11 Diarrhea Chapter 12 Aversion to Food (Anorexia) Chapter 13 Food Accumulation Chapter 14 Malnutrition Chapter 15 Nutritional Iron Deficiency Anemia Chapter 16 Vitamin D-Deficiency Rickets PART III Heart-Liver Diseases Chapter 17 Sweating Syndrome Chapter 18 Viral Myocarditis (VMC) Chapter 19 Anaphylactoid Purpura Chapter 20 Attention Deficit Hyperactivity Disorder

Chapter 21 Tourette Syndrome (Multiple Physical Tics) Chapter 22 Infantile Convulsion Chapter 23 Epilepsy PART IV Kidney Diseases Chapter 24 Acute Glomerulonephritis Chapter 25 Nephrotic Syndrome Chapter 26 Urinary Frequency Chapter 27 Enuresis Chapter 28 Five Types of Developmental Delay and Five Kinds of Flaccidity PART V Infectious Diseases Chapter 29 Measles Chapter 30 Exanthema Subitum Chapter 31 Rubella Chapter 32 Chickenpox Chapter 33 Hand-Foot-and-Mouth Disease Chapter 34 Mumps Chapter 35 Epidemic Encephalitis B Chapter 36 Infectious Mononucleosis Chapter 37 Pertussis Chapter 38 Scarlatina PART VI Neonatal Diseases Chapter 39 Scleroderma Neonatorum Chapter 40 Fetal Jaundice PART VII Other Diseases Chapter 41 Summer Fever Chapter 42 Mucocutaneous Lymph Node Syndrome Appendix I: Pinyin-English Formula Cross Reference Appendix II: List of Cited Sources

General Introduction The term “pediatrician” was first mentioned in China during the Warring States Period, with Bian Que being the first pediatrician on record. The Records of the Grand Historian states, “Bian Que was famous around the country…When he came to Xian Yang and was told that the Qin people were very fond of children, he then became a pediatrician”.(扁鹊名闻天 下……来入咸阳,闻秦⼈爱⼩⼉,即为⼩⼉医) Pediatrics is an important part of Chinese medicine, deserving special attention in all dynasties. Sun Si-miao, in his Tang Dynasty book, Important Formulas Worth a Thousand Gold Pieces for Emergency (Bèi Jí Qiān Jīn Yào Fāng) said, “To keep the people prosperous, raising children is first and foremost. Without healthy babies, there are no healthy adults”.(⽣民之道, 莫不以养⼩为⼤,若无于⼩,卒不成⼤) Education in pediatrics existed early in history; pediatric degrees were granted by the Imperial Academy of Medicine as early 624 CE. Students qualified as pediatricians by passing an examination after 5 years of study, similar to today’s pediatric training. In the Song Dynasty, the Imperial Physician’s Bureau established a pediatrics department as one of its nine departments, with 20 out of every 300 medical students belonging to this department. Many pediatricians from different dynasties are famous either for their pediatric theories or for the clinical treatments that they devised. Of them,

Qian Yi (1032-1113), was known as the “Saint of Pediatrics”; his work, Key to Diagnosis and Treatment of Children’s Diseases (Xiăo Ér Yào Zhèng Zhí Jué) continues to be an important textbook in current pediatric practice.

PHYSIOLOGICAL, ETIOLOGICAL AND PATHOLOGICAL CHARACTERISTICS Children grow and develop very quickly. The younger they are, the faster the rate of growth and development. Because children are different from adults in many respects, the appropriate treatment will often differ as well.

Physiological Characteristics Even though children grow and develop quickly, their internal organs are not yet well-developed; the organs are described as fragile and immature with insufficient qi and lower resistance to attack by pathogens. Children have a lower tolerance for polluted environments as well as many medications. Based on the philosophical view of yin-yang theory in Systematic Differentiation of Warm Diseases (Wēn Bìng Tiáo Biàn), Wu Ju-tong (1758-1836) said, “Young children have immature yang that is not sufficient, and immature yin that is not well-developed”(⼩⼉稚阳未充, 稚阴未长者也). This pediatric concept is also referred to as “tender yin and delicate yang”. Second, children experience vigorous life activity along with rapid growth and development. Young children have a rapid growth period that involves physical changes in body size and shape, physiological functions, and biological maturation. The thinking, calculating, language and motor

skills are also developing and maturing rapidly. This physiological change in children is described as a “pure-yang phenomenon” indicating vigor, exuberant vitality, and continuously rapid growth and development. The theories of “tender yin and delicate yang” and the “pure-yang phenomenon”represent two aspects of a young child’s physiological features. The former reflects that yin and yang qi are relatively insufficient, while the latter stresses that they are vigorous in growth and development. In fact, these two aspects are not at all contradictory.

Etiological Characteristics The same pathogens cause diseases in children as in adults, but children may react differently and their level of susceptibility is very different. Even different childhood age groups have different levels of susceptibility to various etiological factors. Firstly, young children can be easily attacked by external pathogens, as their defensive system is insufficient. Cross-infections occur particularly easily at preschools and elementary schools. Children are also highly susceptible to pestilential pathogens with highly contagious abilities; these conditions are characterized by sudden onsets and a tendency to develop into epidemics. Secondly, inappropriate feeding can cause digestive problems, as the child spleen and stomach are still delicate. Children are less able to control their food intake and can easily under-or over-consume food, or have unhealthy food preferences and unbalanced diets. Because the spleen and stomach are the foundations of postnatal essence, damage to the spleen and stomach will have a negative impact on growth and development.

Therefore, the establishment of good eating habits is crucial for all young children. Thirdly, genetic causes are major contributing factors in disease etiology. Premature birth or difficult labor (such as abnormal fetal positioning and resulting birth injuries including head hematoma, birth trauma, fractures, even suffocation, etc.) are also included in the congenital factors. All pregnant women should have regular prenatal care and obstetric visits to reduce the congenital impact to the minimum. Fourth, children are highly susceptible to some emotional disorders. The hearts of infants and children are delicate and their spirits are weak, with fear and sudden fright being the emotions that most often cause damage. Such damage may manifest as night-crying, palpitations, panic attacks, bed-wetting or convulsions. On the other hand, if a child’s desires are not satisfied or are neglected, or if there is a lack of communication with others, the child will likely experience sadness. Excessive thinking and contemplation due to loneliness can damage the heart and spleen, causing poor appetite, fullness, focal distention and cramping. However, when children are over-indulged, they may become excessively dependent. Later in life, heavy study loads and high expectations from the parents can trigger stress, anxiety and even psychiatric disturbances. Finally, other factors must be considered. In modern society, worldwide environmental pollution, food contamination, pesticides, excessive hormone levels and radioactive materials can also damage the fetus and child. Improper treatment or care also threatens the lives of many children.

Pathological Characteristics

First, due to the immature and delicate zang-fu organs and the insufficiency of qi in the bodies of infants and children, they easily contract disease; furthermore, due to their vigorous life activities and pure yang, pathological conditions change quickly. The younger the child, the greater the immaturity of the zang-fu organs, and the disease onset and speed of pathological change is also more rapid. The major manifestations of these changes occur between patterns of cold, heat, deficiency and excess. These patterns can easily inter-transform or may all appear simultaneously. Secondly, the pure and clear visceral qi of children also enables quick recovery. Children have a “pure yang” body, also called “young yang”; their life activity is vigorous, they have pure and clear qi, and are full of vitality, so they usually respond well to both internal and external treatments. Because they have fewer chronic conditions and the pathogenic factors are relatively simple, they do not require long recovery periods. With clear and accurate pattern differentiation and diagnosis, children will generally recover faster than adults.

DIAGNOSIS Ancient pediatricians called pediatrics the “silent department”, in that infants and young children cannot communicate well and as such may not be able to describe their illnesses and symptoms with precision. They also are often frightened, anxious or uncomfortable when they see a practitioner; some may have had negative experiences with vaccinations or injections. Loud crying or uncooperative attitudes may also affect the accuracy of diagnosis. Diagnostic methods for pediatric diseases are still based on the four examinations of inspection, listening and smelling, inquiry and palpation. More attention is paid to inspection of facial complexion and the color of lips, eyelids, and forehead, and inspection of the venules on the index fingers of children less than 3 years of age is a special diagnostic method. Detailed information can be gathered through the parents and through simple inquiries to the child; to ensure an accurate diagnosis, both sources of information are important.

PEDIATRIC THERAPEUTICS First, treatment must be timely, accurate and cautious. Because the symptoms of illness develop and change more quickly in children, timely treatment is always required. Delayed treatment may allow the pathogens to run deeper, and symptoms to become more complicated. The diagnosis should be accurate and the treatment precise, as any mistake could lead to a crisis. To prevent impairment of the appetite or damage to the digestive organs, also avoid excessively cold or sedating medicinals. Secondly, because children are more sensitive and respond to treatment more quickly, it is important to maintain a relatively simple approach; simplified formulas may be more appropriate. This is also why pediatric tui na works very well with young children. The general principle for pediatric formula prescription is to use fewer medicinals and lighter dosages; this can actually yield a superior curative effect. Heavy, greasy or cloying medicinals should be avoided as they interfere with digestion. Thirdly, attention should be paid to protecting the spleen and stomach. As the root of the post-natal organs following birth, growth and development rely mainly upon the spleen and stomach. Congenitally insufficient children especially need post-natal nourishment and replenishment. In fact, the absorption of all orally administered medicinals depends on the function of spleen and stomach. Fourth, pediatric diseases change quickly and move quickly; therefore, using remedies to treat and prevent forthcoming illness is also very

important. This approach helps to subdue the serious effects of an illness while stopping further disease progression. Fifth, apply supplementing medicinals with care. Children are “pure yang” in nature, so they generally do not require additional boosting and supplementing medicinals. In fact, long-term supplementation can lead to sexual precocity, as many such medicinals contain natural hormones. If children contract external pathogens or have phlegm-dampness retention or food stagnation, it is especially critical to avoid replenishing and boosting formulas; these interfere with the removal of pathogens, and negative effects may result. Supplementing and boosting formulas should be applied only when there is no excess condition present. Finally, determine the dosages with great precision. Dosing will vary according to the child’s age, individual situation, disease conditions, and the practitioner’s experience, as well as the patient’s sensitivity to Chinese medicinals. Attacking medicinals for diaphoretic, purgative and diuretic purposes should be used cautiously; these include má huáng (Herba Ephedrae), fù zĭ (Radix Aconiti Lateralis Praeparata), dà huáng (Radix et Rhizoma Rhei), bā dòu (Fructus Crotonis) and máng xiāo (Natrii Sulfas). The total medicinal dosage in a child’s decoction can be calculated according to the following ratios: For newborns, use 1/6 of an adult dosage. For infants, use 1/3 to 1/2 of the adult dosage. For young children use 2/3 or the same dosage as adults, and for school-age children, use the same relative dosage as for an adult. The above adult dosages refer to the general normal dosage rather than the maximum dosage. A dose of medicine is often decocted twice. The total amount of the decoction is controlled within 15 to 30 ml for newborns, 50 to 100 ml for infants, 150 to 250 ml for young children and 250 to 400 ml for school-age children. The total daily amount

of the decoction is usually administered in 2 to 4 portions daily, 3 to 4 for an acute illness, or 2 to 3 for a chronic disease.

HOW TO USE THIS BOOK Each disease in the book of case records consists of the following sections: Introduction: Provides a brief introduction to the disease, including the definition, general characteristics and epidemiology. Clinical Essentials: To provide an overall picture of the associated TCM patterns and their treatment; this section also summarizes the clinical experience of senior practitioners and modern experts regarding both internal and external treatments, biomedical knowledge, and modern research. Case Study: This section is the main focus of the text; the case studies include the chief complaints, pattern differentiations, diagnoses, treatment principles, formulas, and formula analyses. Other treatment modalities are outlined as well, including Chinese prepared medicines, medicinal injections, external treatments, acupuncture, etc. Commentary and Discussion: To ensure that all of the above items are explained and evaluated completely, this section includes a commentary by the co-author with a review of important points regarding diagnosis, treatment principles and formula modifications. Study Questions/Case Scenarios: A greater knowledge of each disease is acquired by the student through the study question practice exercises. Additional case scenarios are presented with multiple-choice questions to provide a case-based learning approach.

LEARNING TIPS A thorough understanding of both the disease and the individual case is made possible by inclusion of the following aspects into the case records:

Comprehensive Collection of Clinical Data Pediatric conditions change and transmit more quickly, and for this reason a comprehensive collection of clinical data is always required. The data should be collected not only through traditional evaluation of the tongue, pulse and veins on the index finger, but also through physical assessment and lab research including blood tests, X-rays, MRI and CT scans. Vital signs are always recorded at each visit. For example, for children with acute abdominal pain, nausea, and vomiting, special attention should be paid to palpating the abdomen for abdominal sounds, tenderness, rebound tenderness, and muscle tightness in order to assess acute abdomen. Otherwise, treatment without the exclusion of acute abdomen may make the disease more complicated or even lead to a fatal condition. In fact, many infantile diseases discussed in this book include their biomedical names, such as mucocutaneous lymph node syndrome, acute glomerulonephritis, hand-foot-and-mouth disease, and Vitamin Ddeficiency rickets, all of which require laboratory research to support the disease diagnosis. In order to have a better understanding of these diseases, one should be familiar with the biomedical etiologies, pathological

mechanisms and diagnoses of these diseases. In order to deal with the often rapidly changing clinical condition, the pediatrician must have strong biomedical knowledge.

Diagnosis and Evaluation A complete TCM diagnosis may include the following six aspects: TCM or biomedical disease diagnosis, pattern/differential diagnosis, the location of disease, the property of the disease, principles of treatment, and formulas and modifications based on the evaluation. These several aspects are separate, but also related; the therapeutic methods and formulas are based upon accurate principles of treatment, while the principles of treatment derive from a correct diagnosis and evaluation. Without knowing the location of the problem, we cannot understand the stage of disease and which specific organs require treatment. Without knowing the properties of the disease, we cannot select the correct treatment principles. All of the above six aspects are closely linked, and they all rely on the collection of initial data and its subsequent analysis.

Multiple Treatment Skills and Techniques To achieve superior therapeutic effects with pediatric conditions, the use of multiple treatment styles is highly recommended. Each case includes one or several formulas, many of which are from traditional pediatric classics, making this book very unique. For a deeper understanding of how the formula was created, the commentary for each case provides the source of the formulas and the formulating theory. Each case also includes different treatment skills and techniques, such as internal and external

treatment therapies. Chinese medicinals decoctions and prepared medicines are also applied in each case. In addition, young children with high fevers or convulsions are often given Chinese medicinal injections or strong prepared medicines designed to deal with such emergent situations. External therapies refer to pastes, fumigation, washes, insufflations and smelling; these methods are often applied in dermatological cases. Acupuncture, tui na and cupping are also commonly used; however, special attention should be paid to the needling depth, frequency, duration, and level of stimulation. The focus of this book is to demonstrate the clinical reality of how TCM theory actually manifests in the pediatric clinic. It is our sincere hope that by reading and applying the experience shared in this book, that any practitioner can render improved diagnoses and more effective treatments for a variety of pediatric conditions. Wu Qian-zhi March 6th, 2014

PART I Lung Diseases Chapter 1 Common Cold The common cold is a common, externally contracted disease, with the main manifestations being fever, nasal congestion, sneezing, and cough. The primary etiologies of a childhood cold involve wind-evil invasion accompanied by any of the other five pathogens which include cold, heat, summerheat, dampness, and dryness, or there may be an invasion of seasonal and epidemic toxins. Children’s upright qi insufficiency and declined bodily resistance along with changeable weather patterns like abnormal cold or hot weather, or cold attack while bathing, and improper nursing all may contribute to the invasion of exogenous pathogens. This disease is mainly located in the lungs, although the liver and spleen may also be involved. The key disease mechanism is defensive yang obstruction with failure of lung qi to disperse. Common clinical patterns include wind-cold, wind-heat, summerheat, and seasonal epidemic toxin often accompanied by phlegm, stagnation, and even convulsion in severe cases.

CLINICAL ESSENTIALS In diagnosing and treating the common cold, eight-principle pattern differentiation is the first priority, identification of pestilential cold and influenza the second, and identification of any complications or accompanying symptoms the third. Common colds in infants are mainly caused by an attack of pathogenic wind, which is a yang pathogen. A child is of “pure yang” constitution, with yang frequently in excess and yin is often in deficiency. When exogenous pathogens are contracted, they easily transform into heat, due to excessive yang. When a child is fed with rich foods that easily transform into heat, the child also becomes more susceptible to the common cold. Clinically, wind-heat and cold-heat types of common cold are most prevalent, so treatment with a combination of acrid-cool and acrid-warm medicinals is often the best choice. Heat stays at the wei level for very short time before moving into the qi level. Then, high fever occurs and persists for some time. Dr. Wang Jingan holds that it is most critical to treat at the qi level, although fever in an infant will occur in all four levels with rapid transmission and changing symptoms. Therefore, clinically, it is important to fully expel pathogens from the exterior so that none remain inside the body. Dr. Wang impresses upon us that body fluids are the source of vigor, so bitter and cold purgative medicinals should be avoided. Based on Bái Hŭ Tāng (White Tiger Decoction) from Treatise on Cold Damage (Shāng Hán Lùn) and Qīng Hāo Biē Jiă Tāng (Sweet Wormwood and Turtle Shell

Decoction) from Systematic Differentiation of Warm Diseases (Wēn Bìng Tiáo Biàn), Dr. Wang created Qīng Xuān Dăo Zhì Tāng (Light-Diffusing Stagnation-Moving Decoction), a basic formula for common colds in children.

● In this formula, a large dosage of the chief medicinal shēng shí gāo (Gypsum Fibrosum) is used; it is non-toxic, acrid-sweet, and very cold. Because it can release the muscles by venting the exterior, it is known as a panacea for clearing excess heat from the yangming channel; thus the saying, “shí gāo is the only choice for excess heat in febrile disease”.

● For frenetic movement of the blood caused by high fever, shí gāo is combined with qīng hāo (Herba Artemisiae Annuae), bái wēi (Radix et Rhizoma Cynanchi Atrati) and sāng yè (Folium Mori).

● Dà qīng yè (Folium Isatidis) clears heat and resolves toxins, cools the blood and discharges heat.

● Chái hú (Radix Bupleuri) and jīng jiè (Herba Schizonepetae) dispel the embedded heat by venting heat from the ying level through the qi level.

● Tiān huā fĕn (Radix Trichosanthis) can boost yin and clear heat by preserving body fluids. When used together with shān zhā (Fructus Crataegi), shén qū (Massa Medicata Fermentata), and bīng láng (Semen Arecae), it can not only protect the spleen (middle jiao) by promoting digestion and guiding out (food) stagnation, but also stop other medicinals from damaging upright qi. Combined together, the formula medicinals act to clear heat, resolve toxicity, dispel pathogens, and guide out food stagnation through the sweat and stools.

Yang Jie-bin, a well-known acupuncturist in China, needles LI 4 (hé gŭ), BL 12 (fēng mén), DU 14 (dà zhuī), and tài yáng (EX-HN5) as deep as one cun, while also selecting other points based on the presentation. He also cups with strong drainage until the skin is dark red, with BL 12 (fēng mén) needles retained for 20 min after the arrival of qi. Lifting, thrusting, and rotation is applied once every 3 min.

● With nasal congestion, add LI 20 (yíng xiāng). ● With fever, add LI 11 (qū chí). ● With sore throat, prick LU 11 (shào shāng). ● With aversion to cold, cauterize DU 14 (dà zhuī) for 10 min with a burning moxa roll.

CASE STUDY Female, age 2. Initial Visit: June 4th, 2009 Chief Complaint: Fever and cough for one day. History: This child began to have a fever yesterday afternoon, with her highest temperature reaching 39.5℃. There was also a stuffy nose, sneezing, and cough. Her temperature was brought down with Western medicine including compound aminophenazone and cephalosporin-type antibiotics. But shortly afterwards, her temperature went up again. Signs and Symptoms: Fever, slight sweating, thirst, polydipsia, cough with little sputum, a runny nose with yellow discharge, poor appetite and digestion, yellow urine, constipation for 2 days, a red tongue with a thin yellow coating that was greasy coating at the tongue root, and purple finger venules reaching the wind pass. Past History: A weak constitution and susceptibility to the common cold. Physical Examination: BT 38.9 ℃. Level Ⅰ swelling of tonsils on both sides of the throat. Clear breath sounds in both lungs, rales not heard. Heart rate 130 beats/min, normal heart beat with strong heart sounds, murmurs not heard. Laboratory Examination: WBC 8.0×109/L, N 0. 45, L 0.54, PLT 135×109/L.

Pattern Differentiation

With delicate organs and bowels, children are more likely to contract exogenous pathogens which invade from the mouth, nose, skin, and body hair. When wei yang is obstructed, fever and aversion to cold result. The nose is the orifice of the lung, so when exogenous pathogens invade the upper body through the lung channel, symptoms like nasal congestion and runny nose will occur. When exogenous pathogens invade the lung, the lung fails to diffuse and govern descent, causing cough. With the child spleen often being deficient and exogenous pathogens entering the interior, the spleen and stomach are impaired and cannot function well, leading to poor appetite. When wind-heat disturbs internally, heat consumes fluids to bring about thirst and dark scanty urine. The lung and large intestine are interior-exteriorly related. Therefore, when the lung fails to diffuse and govern descent, qi in the bowels is obstructed and constipation occurs. The red tongue with a yellow greasy coating is a result of wind-heat invading the exterior. This disease is located in the lung, and also involves the spleen, and this is a wind-heat exterior pattern. The key disease mechanism is defensive yang being obstructed with lung qi failing to disperse.

Diagnosis Common cold due to wind-heat

Clinical Treatment

Principles: Release the exterior with acrid-cool medicinals, dissolve phlegm, resolve accumulation Formula: Modified Yín Qiào Săn (Lonicera and Forsythia Powder) [银翘散加减]

2 doses [Formula Analysis] Lián qiào, jīn yín huā, dà qīng yè and băn lán gēn release the exterior and clear heat. Bò he, jié gĕng, niú bàng zĭ and băn lán gēn scatter wind and dissipate heat, diffuse the lung, and soothe the throat. Jīng jiè and dàn dòu chĭ vent the exterior while assisting the acrid-cool medicinals to vent pathogens through the exterior. Lú gēn and zhú yè clear heat and promote fluid production. Gān căo clears heat, resolves toxins, and harmonizes all formula medicinals. Prepared Medicines: Yín Qiào Jiĕ Dú Piàn (Lonicera and Forsythia Toxin-Resolving Tablets), 2-4 tablets per time, 2-3 times a day.

Acupuncture Points: LI 4 (hé gŭ), BL 12 (fēng mén), DU 14 (dà zhuī), tài yáng (EXHN5), GB 20 (fēng chí), LI 11 (qū chí), LU 5 (chĭ zé), LU 10 (yú jì), LI 20 (yíng xiāng). Manipulations: DU 14 (dà zhuī) can be pricked to bleed and cupped. Needle all other points with drainage. Treat once or twice a day. Second Visit After 2 doses of the above formula, her temperature returned to normal, but she still had retained pathogens with occasional coughing. Other signs and symptoms included poor appetite, constipation, yellow scanty urine, light purple finger venules, and sore throat. Principles: Release the exterior with acrid-cool medicinals, promote digestion, guide out food stagnation Formula: Modified Yín Qiào Săn (Lonicera and Forsythia Powder) and Băo Hé Wán (Harmony-Preserving Pill) [银翘散合保和丸加减]

3 doses [Formula Analysis] Based on the previous formula, add xìng rén to relieve cough. Xuán shēn soothes the throat. Jiāo shān zhā and jī nèi jīn promote digestion.

Prevention and Nursing Enhancing bodily resistance is the key to preventing upper respiratory infections; increased outdoor activities and adequate amounts of exercise are encouraged. Be cautious during sudden weather changes and dress appropriately. Drink boiled water and eat easily digestible foods.

COMMENTARY AND DISCUSSION The common cold is one of the most common pediatric diseases seen in clinic as children often become cross-infected at school. At the beginning, symptoms are similar to allergies, and patients often wonder about the differences. The main difference is that allergies have more localized symptoms due to blockage of wei qi in the five sense organs, such as itching of the eyes, nose, ears, and throat with sneezing, earaches, sinus congestion, etc. The common cold has more generalized symptoms that include fever, chills, headache, and cough, in addition to some local symptoms. Common cold is a result of wei qi and ying qi disorder and involves the lungs and other internal organs. Children with weak bodily constitutions are prone to catching external pathogens. In this case, the typical symptoms indicated wind-heat invading the upper jiao, particularly the lungs; the lung is considered as the “Imperial Carriage Roof”, due to its location. There is a ratio of one to 10 between fever and pulse, meaning if the temperature raises one degree centigrade, then the pulse increases by 10 beats/min. This is why her pulse was 130 beats/min, a clear clue for wind-heat invasion. Even though the patient had spleen qi deficiency as a chronic and constitutional problem, the focus of the treatment here is to dispel windheat from the upper jiao and lungs for the acute onset. The medicinals applied during the initial treatment included lián qiào, jīn yín huā, dà qīng yè, bò he, jié gĕng, niú bàng zĭ, jīng jiè, dàn dòu chĭ, lú gēn, and zhú yè.

Most of these medicinals are the flowers, leaves, sprouts and seeds of plants. They are lighter, clearer, and some of them are also aromatic, acting to dispel and eliminate pathogens from the upper jiao area. In the Qing Dynasty, Wu Ju-tong (1758-1836) said, “to treat upper jiao problems, the medicinals applied should be as light as feathers” (治上焦如⽻). This ensures that the medicinals will work on the upper jiao, and that pathogens are removed through the skin, head, and sense organs. This also prevents pathogens from running deeper, from the upper jiao to the middle or lower jiao, as may occur when heavier medicinals are applied. Also, childhood diseases are relatively simple and easy to treat, and lighter and clearer herbs do not interfere with internal organ systems by leaving dampness or phlegm retention nor do they cause damage to the internal organs. A quick recovery from a disease with diminished or vanished symptoms is an essential factor in judging the success of a treatment, but protecting the body from damage due to the treatment is probably a more important factor. This is particularly true in children, as they are in a period of quick growth and development. Any damage to their internal organs may delay and interfere with such development, also resulting in potentially severe diseases. Would you prefer to remove a child’s symptoms in 2 days but leave a lot of digestive dysfunctions and damage to his body constitution due to the harsh properties of the medicinals, or to remove a child’s fever and other symptoms in 3 days without damage to the child’s internal organs and vital substances? Clearly the second choice is superior. Based on the above theory, during the second visit, jiāo shān zhā, jī nèi jīn are added to protect digestion and prevent food stagnation. When all excess symptoms are gone, shān yào (Rhizoma Dioscoreae) can be ground

into a powder and fed to infants daily as a food supplement to improve digestion and supplement the lung, spleen, and kidney qi. Modern research has also found that shān yào powder can improve the immune system and prevent the common cold.

STUDY QUESTIONS 1. What are the key points for diagnosis in this case? 2. Since fever is the principal symptom for both summerheat fever and common cold caused by summerheat pathogens, how can you tell the difference between them?

Answers 1. Key points for diagnosing a child’s common cold involve noting sudden changes of weather patterns like extreme heat or cold, exposure to patients with the common cold, or possible attack by exogenous pathogens. Main signs and symptoms include fever, aversion to wind and cold, nasal congestion, runny nose, sneezing, and cough, often with accompanying symptoms of worsened cough, gurgling with sputum, stuffiness and fullness of the chest and epigastric regions, acid regurgitation, vomiting, foul breath, foul-smelling stools, irregular stools, or restless sleeping, fright, or even convulsion. Hemogram testing shows normal TWBC (total white blood cells) or slightly lower TWBC in those with a viral infection. It shows higher TWBC and PMNs (neutrophilic granulocyte) in those with bacterial infection. Etiological testing: viral isolation and corresponding bacterial detection is suggested.

2. Both summerheat fever and common cold due to summerheat pathogens often occur in summer and fever is the main symptom, but they involve different pathogens and natures. Summerheat fever is caused by exposure to summerheat, and characterized by slow onset, a longer course of disease, rising or falling of the body temperature synchronized with variations in the weather, no runny nose, no nasal congestion, thirst with a desire for drinks, lack of sweating, increased urination, and generally good spirits. The common cold caused by summerheat pathogens is characterized by a short course of disease, fever with no or less sweating, body temperature unaffected by the weather, headache, heavy sensations in the body, cough, sore throat, nausea and vomiting.

CASE SCENARIOS The following cases present variations of this condition. After familiarizing yourself with the possible common pattern presentations and appropriate formulas for treatment; use the following exercises to test your overall understanding of the condition. 1. A patient, age 4, had fever for one day. Other signs and symptoms included aversion to wind, little sweating, slight headache, nasal congestion, runny nose with yellow discharge, slight cough, sore throat, red tongue with a thin yellow coating, and a rapid pulse. What should be the first formula prescribed? A. Sāng Jú Yĭn (Mulberry Leaf and Chrysanthemum Beverage) B. Jīng Fáng Bài Dú Săn (Schizonepeta and Saposhnikovia ToxinResolving Powder) C. Yín Qiào Săn (Lonicera and Forsythia Powder) D. Xīn Jiā Xiāng Rú Yĭn (Newly-Supplemented Mosla Beverage) E. Yù Píng Fēng Săn (Jade Wind-Barrier Powder) 2. A patient, age 2, had a fever for 2 hours. The signs and symptoms included fever, aversion to cold, lack of sweating, nasal congestion, runny nose, slight cough, normal throat, poor appetite, a light red tongue with a thin white coating, and red finger venules. What is the presenting pattern? A. Wind-cold type common cold B. Wind-heat type common cold

C. Common cold due to summerheat D. Common cold due to seasonal pathogens E. Common cold with phlegm 3. The patient, age 7, had a fever. His temperature did not come down with perspiration. Other symptoms included dizziness, headache, chest distress, fatigue, nausea, fidgeting, thirst, poor appetite, loose stools, scanty yellow urine, a red tongue with greasy yellow coating, and a rapid pulse. What is the presenting pattern? A. Wind-heat type common cold B. Wind-cold type common cold C. Common cold with food retention D. Common cold due to summerheat E. Common cold due to seasonal pathogens 4. A patient, age 5, had a fever. Signs and symptoms included aversion to cold, lack of sweating, nasal discharge, runny nose, slight cough, stuffiness and fullness of the chest and epigastric regions, acid regurgitation, vomiting, foul breath, foul-smelling stools, scanty yellow urine, a red tongue with thick greasy coating, and a slippery pulse. What is the presenting pattern? A. Wind-heat type common cold B. Wind-cold type common cold C. Diarrhea due to damp-heat D. Common cold due to summerheat

E. Common cold with food retention 5. A patient, age 18 months, has had a fever for a day. Signs and symptoms included persistent fever, aversion to cold, lack of sweating, nasal congestion, fright, convulsion, crying, restless sleep, dry stools, scanty yellow urine, a red tongue, and purple finger venules reaching the qi pass. In addition to releasing the exterior, what other principles should be applied? A. Clear heat and resolve toxins B. Clear heart heat and calm the mind C. Clear heat and suppress fright D. Calm the liver and extinguish wind E. Harmonize the ying and wei levels 6. A patient, age 9, had a fever for half a day. Signs and symptoms included aversion to cold, decreased sweating, headache, nasal congestion, runny nose, sore throat, slight cough, a red tongue with thin yellow coating, and a floating rapid pulse. What formula should be prescribed? A. Sāng Jú Yĭn (Mulberry Leaf and Chrysanthemum Beverage) B. Má Xìng Shí Gān Tāng (Ephedra, Apricot Kernel, Gypsum and Licorice Decoction) C. Yín Qiào Săn (Lonicera and Forsythia Powder) D. Xīn Jiā Xiāng Rú Yĭn (Newly-Supplemented Mosla Beverage) E. Pŭ Jì Xiāo Dú Yĭn (Universal Relief Toxin-Removing Beverage)

7. The patient, age 9, had a fever for 2 hours. Signs and symptoms included high fever, aversion to cold, lack of sweating, headache, conjunctival congestion, pain all over the body, nausea, stomachache, a red tongue with a yellow coating, and a rapid pulse. What formula should be prescribed? A. Jīng Fáng Bài Dú Săn (Schizonepeta and Saposhnikovia ToxinResolving Powder) B. Yín Qiào Săn (Lonicera and Forsythia Powder) C. Xīn Jiā Xiāng Rú Yĭn (Newly-Supplemented Mosla Beverage) D. Má Xìng Shí Gān Tāng (Ephedra, Apricot Kernel, Gypsum and Licorice Decoction) E. Yín Qiào Săn and Pŭ Jì Xiāo Dú Yĭn (Lonicera and Forsythia Powder and Universal Relief Toxin-Removing Beverage) 8. A patient, age 10 months, had signs and symptoms including fever, lack of sweating, fidgeting, crying, sore throat, poor appetite, vomiting fresh milk or food, loose stools, yellow and scanty urine, red tongue with greasy yellow coating, and rapid pulse. What is the proper treatment principle? A. Release the exterior with acrid-warm medicinals B. Release the exterior with acrid-cool medicinals C. Clear heat and resolve toxins D. Clear summerheat and release the exterior E. Release the exterior and remove stagnation

9. A patient, age 5, had a fever for 2 days. His signs and symptoms included fever, aversion to cold, lack of sweating, headache, nasal congestion, runny nose, cough, gurgling with sputum, a light red tongue with a white coating, and a floating slippery pulse. In addition to releasing the exterior, what other principles should be applied? A. Warm the lung and dissolve phlegm B. Purify the lung and dissolve phlegm C. Diffuse the lung and dissolve phlegm D. Clear the lung and dissolve phlegm E. Dissolve phlegm with dryness-damp medicinals

Answers 1. C 2. A 3. D 4. E 5. C 6. C 7. E 8. E 9. C

Chapter 2 Tonsillitis Tonsillitis is a common disease of the pulmonary system in children, caused by the sloughing of the palatine tonsil, resulting from the invasion of pathogens into the throat, with such signs and symptoms as sore throat, swelling, and even purulent tonsils. Classified as unilateral tonsillitis, bilateral tonsillitis, or eroded tonsillitis, it attacks in all seasons and is most commonly found in children over age 4. Tonsillitis has several causes. First, it is caused by the invasion of wind-heat pathogens into the throat through the nose and mouth. Second, it occurs when exuberant lung-stomach heat contracted by external pathogens invade the throat upwardly. Third, it occurs when a yin-deficient constitution is attacked by pathogenic heat, damaging yin and causing deficiency fire to flame upward. It is also caused by lung and spleen qi deficiency and the insecurity of the wei-exterior, which can also result in recurrences of the disease. The location of tonsillitis is mainly in the lung and stomach, but the kidney may be involved. The main pathological factor is heat, and the pathogenesis involves heat accumulation in the throat and qi and blood stagnation which burns the surface of the throat. Children have more serious symptoms as compared to adults, where the condition is usually accompanied by high fever. Most patients will be cured efficiently with treatment, although infants experience a longer course of disease; the disease may be persistent or may recur. Edema, bì syndrome,

and palpitations may be found in some older children as a result of inefficient or insufficient treatment. Common clinical patterns include wind-heat invading the pharynx, exuberant lung and stomach heat, lung and kidney yin deficiency, and lung and spleen qi deficiency.

CLINICAL ESSENTIALS The primary concern in the pattern differentiation for tonsillitis is to distinguish exterior from interior, and deficiency from excess. The secondary concern is to identify whether such heart-kidney transmuted patterns as rheumatic fever and acute nephritis are present. Tonsillitis can be treated in different ways, depending on pattern differentiation:

● The main treatment principles are to clear heat, resolve toxins, soothe the throat and dissipate masses.

● For wind-heat invading the pharynx, treatment must scatter wind, clear heat, reduce swellings and soothe the throat.

● For exuberant lung and stomach heat, treatment must clear heat, resolve toxins, drain fire and soothe the throat.

● For lung-kidney yin deficiency, treatment must enrich yin and subdue fire to soothe the throat.

● For lung-spleen qi deficiency, treatment must supplement the lungs to consolidate the exterior and fortify the spleen to boost qi.

● Simultaneously, oral medicines should be given, and medical powders may be sprayed on the affected area. Dr. Li Jia-feng holds that acute tonsillitis in children most often results from excessive interior heat caused by the invasion of wind-heat pathogens, or by heat accumulated in the stomach and spleen because of excessive spicy food intake with heat pathogens invading upwardly and accumulating.

Dr. Li most often prescribes Huà Biăn Tāng (Tonsillitis-Resolving Decoction), consisting of sāng yè (Folium Mori) 10g, xuán shēn (Radix Scrophulariae) 10g, mài dōng (Radix Ophiopogonis) 10g, qián hú(Radix Peucedani ) 10g, jié gĕng (Radix Platycodonis) 10g, băn lán gēn (Radix Isatidis) 10g, shè gān (Rhizoma Belamcandae) 10g, zhī mŭ (Rhizoma Anemarrhenae) 6g, chăo huáng qín (Radix Scutellariae) 6g, jīn yín huā (Flos Lonicerae Japonicae) 10g, bò he (Medicinala Menthae) 6g, tiān huā fĕn (Radix Trichosanthis) 10g, qīng dài (Indigo Naturalis) 6g, gān căo (Radix et Rhizoma Glycyrrhizae) 3g. The formula acts to clear heat, resolve toxins, reduce swellings and soothe the throat. It is quite effective for this disease. Acupuncture treatment for pediatric tonsillitis has rarely been reported. Some contemporary doctors suggest using acupuncture together with bloodletting therapy for recurrent pediatric tonsillitis, needling ĕr jiān (EXHN6) as the main point and pricking the other three points of LI 1 (shāng yáng), LU 11 (shào shāng) and LV 3 (tài chōng) with a three-edged needle. This treatment has proved to have some effect on the disease. Other doctors treat this disease by combining foot massage with pricking ĕr jiān (EX-HN6). In this way, endocrine function will be adjusted, immunity will be enhanced, orifices will be opened and the mind calmed. As the balance between yin and yang is improved, the disease will be cured and the body strengthened.

CASE STUDY Male, age 7. Initial Visit: June 7th, 2005 Chief Complaint: Fever and sore throat for 6 days. History: This child began to have fever and sore throat 2 days after eating some fried food a week ago. He was then taken to hospital and diagnosed with acute tonsillitis. A 7-day course of amoxicillin and antiviral medicines taken orally showed little effect. Signs and Symptoms: High fever, aversion to cold, runny nose, sore throat, especially when eating, dry cough without phlegm, poor appetite and digestion, thirst with a desire to drink, loose stools 2 or 3 times a day, yellow urine, a red tongue with a thin and yellow coating, and a floating slippery rapid pulse. Past History: A weak constitution with susceptibility to cold. Physical Examination: BT 39.5 ℃. Thin, red lips, sore throat and swelling of the tonsils on both sides of the throat with beads of pus on the left side. Rough breath sounds heard in both lungs, dry or wet rales not heard. Heart rate 128 beats/min, normal heartbeat with strong heart sounds, no murmurs detected. Flat and soft abdomen without tenderness or rebound tenderness, normal bowel sounds. Laboratory Examination: WBC 16.8×109/L, N 0.81, L 0.10, PLT 204×109/L, HGB 108 g/L, ESR 45 mm/h, CRP 15.1 mg/L.

Pattern Differentiation

This child was fond of spicy food, which led to the accumulation of heat in his stomach. He contracted wind-heat pathogens at the same time, which invaded his lungs. The pathogenic heat penetrated inward, invading the throat upward along the channel, accumulating in the tonsils, scorching the surface of the throat, and resulting in a sore throat with tonsillitis. When exterior pathogens have not been cleared and wind-heat remains in the wei-exterior, symptoms like runny nose, cough, aversion to cold, and floating pulse occur. When there is exuberant heat in the lungs and stomach scorching body fluids, symptoms like thirst with a desire for drinks occur. If the heat is accompanied by dampness, loose stools may appear. Both red tongue with a thin and yellow coating and a floating and slippery pulse are manifestations of exuberant lung and stomach heat. The cause of this case is heat pathogen. Its pathogenesis is heat pathogen accumulating in the throat with qi and blood stagnation, which burned the surface of the throat, accompanied by exterior wind-heat and dampness.

Diagnosis Tonsillitis due to exuberant lung and stomach heat

Clinical Treatment Principles: Clear heat, resolve toxins, drain fire and soothe the throat Formula: Supplemented Niú Bàng Gān Jié Tāng (Arctium, Licorice and Platycodon Decoction) [⽜蒡⽢桔汤加味]

2 doses Patients should be fed appropriate food to release the exterior and remove dampness. When oral medicines are given, a medical powder may be sprayed on the diseased region to improve the treatment. [Formula Analysis] Niú bàng zĭ, shè gān, shān dòu gēn, jié gĕng, and xuán shēn resolve toxins and soothe the throat. Lián qiào, huáng qín, hŭ zhàng, and gān căo resolve toxins and eliminate abscesses. Shí gāo clears heat and relieves fidgeting. Qīng hāo and zĭ sū yè disperse and vent pathogens. Huò xiāng and huá shí clear heat and drain dampness. Prepared Medicines: Liù Shén Wán (Six Spirits Pill). One pill, 3 times daily. Place the pill at the middle of the tongue and let it melt, or take with warm water.

Acupuncture Points: LU 11 (shào shāng), LI 4 (hé gŭ), LU 5 (chĭ zé), ST 44 (nèi tíng), SJ 1 (guān chōng), LU 10 (yú jì). Manipulations: Select 2 or 3 points for each treatment, used interchangeably. Prick LU 11 (shào shāng) to bleed. Second Visit After 2 days of treatment, the child began to have a low afternoon fever with a slightly itching sore throat with an abnormal sensation, a dry cough with a small amount of phlegm, feverish palms and soles, red cheeks, dry stools, a red tongue with a thin coating, and a thready pulse. His temperature was 37.8 ℃ and there were red swollen tonsils covered by a small amount of pus. The fever was relieved after the initial treatment, and the aversion to cold and runny nose had disappeared. The remaining signs and symptoms suggested a main pattern of lung and kidney yin deficiency caused by pathogenic heat damaging yin. Principles: Enrich yin, subdue fire and soothe the throat Formula: Modified Yăng Yīn Qīng Fèi Tāng (Yin-Nourishing LungClearing Decoction) [养阴清肺汤加减]

3 doses [Formula Analysis] Xuán shēn, shēng dì huáng, mài mén dōng, tiān huā fĕn, and lú gēn nourish yin and soothe the throat. Mŭ dān pí and chì sháo cool the blood and dissipate blood stasis. Băn lán gēn and zhè bèi mŭ resolve toxins and dissipate masses. Xià kū căo and hăi zăo soothe the throat and dissipate masses. Dì gŭ pí and hú huáng lián nourish yin and clear heat.

COMMENTARY AND DISCUSSION This is a case of possibly three or four patterns in combination: windheat invading the pharynx, exuberant lung-stomach heat, lung and kidney yin deficiency, and lung and spleen qi deficiency. This makes the initial differential diagnosis much more difficult. The child did not have a history of common cold or upper respiratory infection. Instead, he ate some fried food and had a sore throat the next day. This easily guides us to a pattern diagnosis of excess heat in the stomach due to irregular diet. The high fever, and thirst, sore throat, yellow urine, red tongue and slippery and fast pulses support this evaluation. However, it does not explain the aversion to cold, runny nose, cough and a floating pulse. These signs must be due to invasion of external pathogens, even though the child had no obvious medical history to support the diagnosis. Because of the external pathogens, the child had both exterior- and interior-type patterns, and attention must be paid to treating both. This explains why shí gāo was applied to clear heat internally with lián qiào to dispel heat externally along with other medicinals that drain heat, remove toxins and benefit the throat. Logically, interior and exterior heat together should cause constipation, but this case had loose stools. This is a sign of damp-heat, which made the case even more complicated. Huò xiāng and huá shí were applied to clear heat and drain dampness. This case leads us to a discussion of the differences between heat, fire and toxic-heat with fire. They all are yang pathogens, but they have different levels of severity. Heat is milder than fire, while toxic-heat and fire

is stronger than fire. Symptomatically, heat causes more generalized symptoms such as high fever, a red face, dark yellow urine, constipation, profuse sweating, a desire for cold drinks and so on. Fire takes responsibility for some localized symptoms such as sore throat, carbuncles, toothache and so on. When severe swelling and pus form, the internal heat and fire have become toxic. In this case, Liù Shén Wán was applied as a prepared medicine to remove the toxic-heat and fire; it acts to clear toxic-heat, sedate internal heat, resolve phlegm-heat and calm the shen (spirit). It is commonly used for a variety of severe heat problems including acute tonsillitis, acute mumps, and high fever with delirium. Liù Shén Wán is not appropriate for sore throat in the early stage of a wind attack, but it is applicable for sore throat due to internal heat. Without the prepared medicine Liù Shén Wán, the function of draining toxins and removing swelling in this case would have been much diminished. But great care must be taken in the application of Liù Shén Wán, as it contains toxic medicinals with strong side effects. It is not approved by the FDA.

STUDY QUESTIONS 1. What are the prognoses for this disease? 2. What were the manifestations of lung and kidney yin deficiency at the second visit?

Answers 1. Compared to adult patients, pediatric patients have more serious symptoms, usually accompanied by an ardent fever. Most patients will heal with proper treatment. Infants, however, suffer from a longer course of disease, sometimes remaining affected for a longer period, or they may suffer from recurrences. In some older children, delayed or insufficient treatment can result in edema, bi syndrome, or palpitations. 2. This patient had long been suffering from tonsillitis, so heat pathogens had damaged yin. The insufficient lung yin caused a dry cough with little phlegm and dry stools. The insufficient kidney yin caused a low afternoon fever and red cheeks. The red tongue with little coating and a thready, rapid pulse are both manifestations of yin deficiency.

CASE SCENARIOS The following cases present variations of this condition. After familiarizing yourself with the possible common pattern presentations and appropriate formulas for treatment, use the following exercises to test your overall understanding of the condition. 1. A patient, age 7, caught a cold a week ago with such signs and symptoms of fever, stuffy, runny nose, and headache, none of which improved with treatment. He now has an itching sore throat. There is a red tongue with a thin white coating, and a floating rapid pulse. Medical examination shows swollen tonsils (Level Ⅱ). What is the proper diagnosis? A. Common cold, wind-heat type B. Tonsillitis due to wind-heat accumulation C. Cough, wind-heat type D. Thrush, heat accumulated in the heart and spleen E. Asthma and cough caused by pneumonia with wind-heat blocking the lung 2. A patient, age 2, has significantly red and swollen tonsils with a sore throat. The child is usually on a liquid diet. Signs and symptoms include a high fever, thirst, dry stools, constipation, scanty yellow urine, a red tongue with a yellow coating, and purple finger venules. What is the best treatment formula?

A. Yín Qiào Săn (Lonicera and Forsythia Powder) B. Sāng Jú Yĭn (Mulberry Leaf and Chrysanthemum Beverage) C. Zhú Yè Shí Gāo Tāng (Lophatherum and Gypsum Decoction) D. Má Xìng Shí Gān Tāng (Ephedra, Apricot Kernel, Gypsum and Licorice Decoction) E. Niú Bàng Gān Jié Tāng (Arctium, Licorice and Platycodon Decoction) 3. A patient, 10 months of age, began to have a fever 3 days ago with the highest temperature of 38.8℃. He now has signs and symptoms of hoarseness, frequent choking and coughing, slight gurgling phlegm heard in the throat, dry stools, scanty yellow urine, a red tongue with a thin and yellow coating, and purple finger venules. Medical examination shows swollen tonsils (Level Ⅲ). What is the presenting pattern? A. Lung-stomach yin deficiency B. Exuberant heat pathogens C. Wind-heat blocking the lungs D. Wind-heat invading the throat E. Dryness-heat damaging the lungs 4. A patient, 1 year of age, began to have a fever, runny nose, and headache 5 days ago and now has an itching sore throat. There is a red tongue with a thin white coating, and a floating pulse. Medical examination shows swollen tonsils (Level Ⅱ). What are the correct principles of treatment for this case? A. Clear heat, resolve toxins, soothe the throat, dissipate masses

B. Scatter wind, release the exterior, clear and soothe the throat C. Scatter wind, clear heat, resolve toxins, soothe the throat D. Nourish yin to moisten the lung, soothe the throat, dissipate masses E. Scatter wind, clear heat, soothe the throat, dissipate masses 5. A patient, age 2, has significantly red and swollen tonsils and a sore throat caused by delayed treatment of a common cold. This child is usually on a liquid diet. Signs and symptoms include a high fever, thirst, dry stools, constipation, scanty yellow urine, a red tongue with a yellow coating, and purple finger venules. What is the presenting pattern? A. Wind-heat accumulation B. Lung-stomach yin deficiency C. Exuberant heat pathogens D. Dry-heat damaging the lungs E. Wind-heat blocking the lungs 6. A patient, age 5, has significantly red and swollen tonsils with a sore throat from a cold. This child is usually on a liquid diet. Signs and symptoms include a high fever, thirst, dry stools, constipation, scanty yellow urine, a red tongue with a yellow coating, and purple finger venules. What is the best Chinese prepared medicine for this case? A. Dān Shēn Zhù Shè Yè (Salvia Injection) B. Shēn Mài Zhù Shè Yè (Red Ginseng and Ophiopogon Injection) C. Huáng Qí Zhù Shè Yè (Astragalus Injection)

D. Qīng Kāi Líng Zhù Shè Yè (Clear and Disperse Heat Injection) E. Yán Hŭ Níng Zhù Shè Yè (Potassium Sodium Dehydroandroan Drographolide Succinate for Injection) 7. A patient, age 9 has significantly red, swollen tonsils and a sore throat from delayed treatment of a common cold. This child is usually on a liquid diet. Signs and symptoms include a high fever, thirst, scanty yellow urine, a red tongue with a yellow coating, and purple finger venules. He is unwilling to take oral Chinese medicine. What is the best treatment formula for this child? A. Liù Yī Săn (Six-to-One Powder) B. Bīng Péng Săn (Borneol and Borax Powder) C. Qīng Dài Săn (Indigo Powder) D. Yù Shū Săn (Jade Axis Powder) E. Qiān Jīn Săn (Thousand Gold Pieces Powder)

Answers 1. B 2. E 3. D 4. E 5. C 6. D

7. B

Chapter 3 Cough Cough is a disease of the pulmonary system commonly found in children. In Western medicine, the symptom of coughing is sometimes referred to as tracheitis or bronchitis. It may attack in any season, but is more prevalent in spring and winter. Children of all ages may develop this disease, with infants being especially susceptible. Coughing in children can result from external contraction or from internal damage, more commonly the former, usually due to the contraction of pathogenic wind. The main internal causes of this disease involve weaknesses of lung and spleen. The condition is mainly located in the lung, and the spleen is often involved. The pathomechanism involves lung failure to diffuse and purify, which causes lung qi to move adversely as external pathogens invade the lung through the nose or skin and body hair. The child spleen is generally insufficient, where spleen deficiency produces phlegm that accumulates upward into the lung. A long course of coughing caused by external pathogens damages the upright qi, so the condition may eventually transform into a cough associated with internal damage.

CLINICAL ESSENTIALS The guiding principle for the diagnosis of cough is eight-principle pattern differentiation. External contraction and internal damage must first be differentiated. If the cough is caused by external contraction, patterns of wind-cold or wind-heat must be further differentiated. If caused by internal damage, deficiency-excess pattern identification and cold-warm pattern identification must be applied. Common clinical pattern types include externally contracted cough with wind-cold or wind-heat patterns, and coughing patterns associated with internal factors such as phlegm-heat, phlegm-dampness, qi deficiency and yin deficiency. Cough should always be treated by facilitating lung qi to diffusion and descent. The fundamental principles for the treatment of externally contracted cough are to scatter external pathogens and diffuse lung qi. On the basis of this principle, proper treatment diffuses the lung and dissipates cold or diffuses the lung and clears heat, depending on the differentiation of cold or heat patterns. When treating externally contracted cough with intense pathogenic qi and relatively strong upright qi, cloying or astringent medicinals should not be given too early, as this may result in pathogens retained internally. Cough associated with internal damage should be treated by first identifying the location and nature of the disease. For patients with excess phlegm, treatment should either clear the lung to dissolve phlegm or dry dampness to dissolve phlegm, depending on the differentiation of phlegm-

heat or phlegm-dampness patterns. For patients with yin deficiency, however, treatment should either fortify the spleen to supplement the lung and boost qi to dissolve phlegm, or yin must be nourished to moisten the lung and clear heat, depending on the differentiation of qi deficiency and yin deficiency. Pediatrician Li Xiu-liang believes that cough in children is more often than not related to phlegm, with the spleen being the source of phlegm production. Cough in children is caused by stagnation of body fluids because of qi failure to transport fluids and by phlegm coagulation resulting from indigestion of food and drink. Failure to absorb the essence of food and drink and fluid metabolism disorder is the general pathomechanism for the production of phlegm-rheum. Treatment should address both lung and spleen. First, banking up earth to generate metal can prevent pathogenic invasion. Second, activating the spleen will move qi to disperse phlegm and relieve cough. As to daily care, Dr. Li suggests that patients take care in their daily life to improve the adaptability to temperature and environment. In addition, he stresses the importance of daily care of the spleen and stomach. Because the child spleen and stomach function of transportation and transformation is weaker than at other times, cold food and drink as well as greasy or spicy foods should be avoided. Cough in children is rarely treated by acupuncture, although some doctors will treat externally-contracted cough by applying seven-star needling on both sides of the neck along the C5 to C7 vertebrae (cervical jia ji points) and RN 22 (tiān tū), or along the route of the hand taiyin lung channel. Moderate moxibustion can also be applied to DU 14 (dà zhuī), BL 12 (fēng mén), BL 13 (fèi shù) and LU 7 (liè quē).

Others treat pediatric cough with thumb-pressing on BL 14 (jué yīn shù) to boost yang qi and promote the movement of qi and blood. After yin qi and yang qi have been harmonized, TCM decoctions are administered according to pattern differentiation.

CASE STUDY Female, age 6. Initial Visit: August 11th, 2009 Chief Complaint: Fever and cough for 2 days. History: This child had cough, fever, aversion to wind, and a runny nose. Ibuprofen and amoxicillin relieved the fever and runny nose slightly, but the cough and expectoration remained. Signs and Symptoms: Coughing with yellow phlegm, sore throat and thirst, fever, aversion to wind, insomnia, poor appetite, normal urine and stools, a red tongue with a thin yellow coating, and a rapid floating pulse. There was no nasal congestion, breathlessness or wheezing. Physical Examination: BT 37.8℃, throat congestion, no swelling of the tonsils, rough breathing sounds in both lungs, dry or wet rales not heard. Laboratory Examination: Routine blood test: WBC 9.8×109/L, N 0. 70, L 0.28, PLT 199×109/L, HGB 112 g/L. Chest X-ray showed increased lung markings indicating bronchitis.

Pattern Differentiation When wind-heat invades the lung, lung qi fails to purify and diffuse, leading to coughing and a runny nose. When wind-heat burns the body fluids, yellow sticky phlegm is produced. The pharynx is the door to the lung; therefore, when lung heat damages fluids, thirst and sore throat will result. When pathogens invade the skin and body hair, the disorder of qi

flow can cause fever and aversion to wind. The red tongue with a thin yellow coating and the floating rapid pulses are both manifestations of exterior wind-heat.

Diagnosis Cough due to wind-heat

Clinical Treatment Principles: Scatter wind, clear heat, diffuse the lung to relieve cough Formula: Modified Sāng Jú Yĭn (Mulberry Leaf and Chrysanthemum Beverage) [桑菊饮加减]

3 doses

[Formula Analysis] Sāng yè and jú huā scatter wind-heat. Bò he, lián qiào and dà qīng yè vent pathogens with acrid-coolness, clear heat, and release the exterior. Xìng rén, qián hú and jié gĕng diffuse the lung and relieve cough. Lú gēn clears heat and promotes body fluid production. Huáng qín clears and diffuses lung heat. Tŭ niú xī gēn and xuán shēn soothe the throat and reduce swelling. Sāng bái pí clears the lung and relieves cough. Gān căo harmonizes all formula medicinals.

Prepared Medicines

● Jí Zhī Táng Jiāng (Acute Bronchitis Syrup), 5-10 ml, 3 times daily. ● Shé Dăn Chuān Bèi Yè (Snake Gall and Sichuan Fritillaria Bulb Oral Liquid), 10 ml, 2 or 3 times daily.

Acupuncture Points: LU 7 (liè quē), LI 4 (hé gŭ), BL 13 (fèi shù), DU 14 (dà zhuī), LU 11 (shào shāng) Manipulations: Select 2 or 3 points for each treatment and needle alternately. Prick LU 11 (shào shāng) to bleed.

Tui Na

Points: fèi jīng (肺经), rǔ páng (乳旁), ST 18 (rŭ gēn), RN 17 (dàn zhōng), BL 13 (fèi shù), BL 20 (pí shù), ST 36 (zú sān lĭ) Manipulations: Apply pushing, kneading, pressing and kneading, and grasping techniques. Second Visit After 3 days of treatment, the cough was relieved, though she still coughed occasionally with heavy and dull sounds. The pain in her pharynx was relieved, and there was no nasal congestion, fever, aversion to wind or insomnia. But she still had no desire for drinks or food. Her stool and urine were normal. Her tongue was red with a white greasy coating, and her pulse was slippery. This fever and aversion to wind had disappeared and her cough was relieved, which indicated that her illness was moderated, though some internal dampness remained. Principles: Diffuse the lung, dry dampness, resolve phlegm Formula: Modified Sāng Jú Yĭn (Mulberry Leaf and Chrysanthemum Beverage) [桑菊饮加减]

3 doses [Formula Analysis] Sāng yè and jú huā scatter wind-heat. Bò he, lián qiào and dà qīng yè vent pathogens with acrid-cool medicinals and clear heat to release the exterior. Xìng rén, qián hú and jié gĕng diffuse the lung and relieves coughing. Sāng bái pí clears lung heat to relieve cough. Yì yĭ rén, bàn xià and fú líng diffuse the lung and dry dampness. Shén qū and shān zhā fortify the spleen. Gān căo harmonizes all formula medicinals.

COMMENTARY AND DISCUSSION The lungs are clear and fragile and cannot tolerate the invasion or retention of internal and external pathogens such as the six evils, dampness and phlegm. Therefore, coughing can be considered as a protective reaction of the human body to remove external or internal pathogens retained in the lungs, even though this effort often injures the lung tissues with some aftereffects. In this case, the young girl had a clear history of external wind-heat invasion with symptoms of coughing, yellow phlegm, a sore throat with thirst, fever and aversion to wind. The red tongue with a thin yellow coating and the floating rapid pulses confirm the differentiation, and a modification of Sāng Jú Yĭn was administered promptly. Most of the medicinals in this prescription are very clear in nature and light in weight, such as sāng yè, jú huā, and bò he because the pathogens are located in the upper jiao, including the lungs. Therefore, the pathogens should be removed from the upper body through scattering and ascending therapies. According to Wu Ju-tong, the author of this prescription from Systematic Differentiation of Warm Diseases (Wēn Bìng Tiáo Biàn), the medicinals for diseases in the upper jiao “…should be as light as a feather, otherwise the heavy medicinals do not ascend”. (治上焦如⽻,非轻不举) Another prescription, Yín Qiào Săn (Lonicera and Forsythia Powder), is also for upper jiao diseases; it shares more than half of the ingredients in Sāng Jú Yĭn, including lián qiào, jié gĕng, bò he, lú gēn and gān căo. Why then was Yín Qiào Săn not applied in this case? The main reason is that Yín

Qiào Săn is for the beginning of a warm disease with symptoms in the wei stage, while Sāng Jú Yĭn is more focused on the lungs with cough. Sāng Jú Yĭn also has a milder heat-clearing effect than Yín Qiào Săn. In Systematic Differentiation of Warm Diseases, Wu Ju-tong explains the formulating theory of Sāng Jú Yĭn: “Cough is due to heat damaging the collaterals of the lungs. If a patient does not have a high fever, then the disease is not severe. Slight thirst shows that the heat is not severe. To prevent giving strong medicinals for a lighter case, Sāng Jú Yĭn was created to have a milder effect”. (咳,热伤肺络也。身不甚热,病不甚也。渴⽽ 微,热不甚也。恐病轻药重,故另⽴轻剂⽅) By recognizing Wu Ju-tong’s statement, we can understand why other heat-clearing medicinals like dà qīng yè, huáng qín, and sāng bái pí were added to the prescription.

STUDY QUESTIONS 1. At the second visit, what were the manifestations of dampness in this case? 2. Why were medicinals for fortifying the spleen used at the second visit? 3. Why is an externally contracted-type cough the most common type of cough in children?

Answers 1. The manifestations of dampness at the second visit were a heavy cough, no desire for drink, lack of appetite, a red tongue with a white greasy coating, and a slippery pulse. 2. The child spleen is relatively deficient, which often impairs the transportation and transformation of dampness. When damp-turbidity encumbers the spleen, a poor appetite results, and when dampness accumulates to produce phlegm, a heavy cough results. A red tongue with a greasy white coating and a slippery pulse are manifestations of dampness. Therefore, foods that can fortify the spleen should be taken. 3. The lung, among the five viscera, is the most delicate organ. With “thin skin and an open fence”, children’s viscera and bowels are especially susceptible to infection by external pathogens. Connected to the throat, opening into the orifices in the nose, and connected externally with the skin and body hair, the lung governs qi and controls breathing. External

pathogens usually invade through the mouth and nose or skin and body hair, attacking the lung first. Cough occurs when the lung fails to diffuse and govern descent.

CASE SCENARIOS The following cases present variations of this condition. After familiarizing yourself with the possible common pattern presentations and appropriate formulas for treatment, use the following exercises to test your overall understanding of the condition. 1. A patient, age 5, has been coughing over the past week with abundant sticky yellow phlegm difficult to expectorate. Occasional gurgling phlegm is also heard in his throat. He has a fever and a desire to drink, but scanty yellow urine. His tongue is red with a greasy yellow coating. His pulse is slippery and rapid. What is the proper treatment formula? A. Qīng Jīn Huà Tán Tāng (Melt-Clearing Phlegm-Transforming Decoction) B. Sāng Jú Yĭn (Mulberry Leaf and Chrysanthemum Beverage) C. Shā Shēn Mài Dōng Tāng (Adenophora and Ophiopogon Decoction) D. Má Xìng Shí Gān Tāng (Ephedra, Apricot Kernel, Gypsum and Licorice Decoction) E. Huáng Lián Jiĕ Dú Tāng (Coptis Toxin-Resolving Decoction) and Sān Ào Tāng (Rough and Ready Three Decoction) 2. A patient, age 3, has been suffering from cough and fever for 2 days with a temperature of 38.2℃, a stuffy nose with thick discharge, slight

sweating, aversion to wind, cough, sore throat, a red tongue with a thin yellow coating, and shallow purple finger venules. What are the principles of treatment? A. Scatter wind and dissipate cold, diffuse the lung, relieve cough B. Scatter wind and clear heat, diffuse the lung, relieve cough C. Clear heat, dissolve phlegm, diffuse the lung, relieve cough D. Clear heat, dissolve phlegm, nourish yin to moisten the lung E. Clear heat, eliminate phlegm, diffuse the lung, relieve cough 3. A patient, age 2, has been ill for 2 days with a congested and runny nose, frequent sneezing, ceaseless heavy coughing, slight throat congestion, a normal tongue body with a thin white coating, and shallow red finger venules. What is the presenting pattern? A. Wind-cold cough B. Wind-heat cough C. Phlegm-heat cough D. Qi-deficiency cough E. Yin-deficiency cough 4. A patient, age 2, has signs and symptoms including a breathless cough, fever, aversion to wind, a sore throat, and a red tongue with a thin yellow coating. What is the best formula for treatment?

A. Yín Qiào Săn (Lonicera and Forsythia Powder) and Má Xìng Shí Gān Tāng (Ephedra, Apricot Kernel, Gypsum and Licorice Decoction) B. Yín Qiào Săn (Lonicera and Forsythia Powder) and Sāng Jú Yĭn (Mulberry Leaf and Chrysanthemum Beverage) C. Huáng Lián Jiĕ Dú Tāng (Coptis Toxin-Resolving Decoction) and Má Xìng Shí Gān Tāng (Ephedra, Apricot Kernel, Gypsum and Licorice Decoction) D. Wǔ Hŭ Tāng (Five Tiger Decoction) and Tíng Lì Dà Zăo Xiè Fèi Tāng (Lepidium/Descurainiae and Jujube Lung-Draining Decoction) E. Qīng Níng Săn (Clearing and Calming Powder) and Tíng Lì Dà Zăo Xiè Fèi Tāng (Lepidium/ Descurainiae and Jujube Lung-Draining Decoction) 5. A patient, 6 months of age, has been suffering from a paroxysmal spasmodic cough for more than 2 months, especially serious in the evening. He coughs with dozens of whooping coughs each time and with cock’s crow-like echoes at the end. When he coughs, his face turns red, the waist bends, the neck stretches, the tongue sticks out, tears are shed and he has a runny nose at the same time. His tongue is red with thin yellow coating, and his finger venules are shallow purple. What is the proper diagnosis? A. Common cold with phlegm B. Wind-heat cough C. Pertussis D. Acute hoarseness E. Asthma

6. A patient, age 7, has been ill for 2 months with a dry cough and a small amount of sticky phlegm that is difficult to expectorate. Over the past month, he has developed thirst, a dry itching throat, hoarseness, afternoon tidal fever, feverish palms and soles, a red tongue with little coating, and a thready rapid pulse. What is the presenting pattern? A. Wind-heat cough B. Phlegm-heat cough C. Phlegm-damp cough D. Qi deficiency cough E. Yin deficiency cough 7. A patient, age 6, has been ill for a month. He had a fever at the beginning, with a temperature between 37.5 ℃ and 37.8 ℃ and a paroxysmal cough with a small amount of phlegm. Over the past 25 days, his fever has come down, but his cough is even worse with the accumulation of a large amount of rumbling thin white phlegm in his throat. He feels tired and lacks appetite, his tongue is light red with a white greasy coating, and his pulse is slippery. What is the best treatment formula? A. Má Xìng Shí Gān Tāng (Ephedra, Apricot Kernel, Gypsum and Licorice Decoction) B. Qīng Jīn Huà Tán Tāng (Melt-Clearing Phlegm-Transforming Decoction) C. Shā Shēn Mài Dōng Tāng (Adenophora and Ophiopogon Decoction)

D. Modified Sān Ào Tāng (Rough and Ready Three Decoction) and Èr Chén Tāng (Two Matured Substances Decoction) E. Rén Shēn Wŭ Wèi Zĭ Tāng (Ginseng and Schisandra Decoction)

Answers 1. A 2. B 3. A 4. A 5. C 6. E 7. D

Chapter 4 Pneumonia with Panting and Cough Pneumonia with panting and cough is a common pulmonary system disease with clinical manifestations of fever, cough, phlegm accumulation, breathlessness and nasal flaring. Severe cases may present with open-mouth raised-shoulder breathing with difficulty, a pale complexion and cyanotic lips. This disease has both internal and external causes. Externally, it is caused by invasion of pathogenic wind or following further development of colds with coughing. Internal causes involve children’s vulnerable organs and relative insecurity of the wei-exterior due to the immature bodies and qi in children. When wind pathogens attack the lung-wei through the nose, mouth, skin and body hair, lung qi becomes disordered and is transformed into heat that burns body fluids. The body fluids are then condensed into phlegm, blocking the air passages so that lung qi fails to purify, diffuse, and govern descent; signs and symptoms include cough, breathlessness, gurgling phlegm in the throat and nasal flaring. The condition can develop into pneumonia with panting and severe coughing due to the lung qi obstruction. This disease is mainly located in the lung, and the spleen is often involved, but the heart and liver may become affected as well. Phlegm-heat is the main pathological result of this disease, and obstruction of lung qi is

the key pathomechanism. Common clinical patterns include wind-cold blocking the lung, wind-heat blocking the lung, phlegm-heat blocking the lung, toxic-heat blocking the lung, yin deficiency and lung heat, and lungspleen qi deficiency. Transmuted patterns include heart yang deficiency and debilitation, and pathogens sinking into the jueyin in more severe cases.

CLINICAL ESSENTIALS Common pattern and transmuted pattern differentiation is the first necessity in clinic. In the early stage of the common pattern, cold patterns and heat patterns must be first differentiated. In the developing stage, heatpredominant patterns and phlegm-predominant patterns must be differentiated. Finally, in the late stage, patterns of qi deficiency and yin deficiency should be differentiated. For patients with excess phlegm obstruction, qi must be directed downward and phlegm must be eliminated. For patients with serious panting, panting must be relieved and qi supplemented. For patients with qi stagnation and blood stasis, blood must be invigorated and stasis dissolved. Because the lung and the large intestine are exteriorly-interiorly related, when internal heat is intensely high, cathartic drugs should be applied to unblock the bowels and discharge heat. For patients with transmuted patterns, treatment should be adjusted accordingly, either to warm and supplement heart yang or to calm the liver and extinguish wind. A long course of disease usually causes yin deficiency, lung dryness and lingering pathogens. Therefore, sweet-cold medicinals together with medicinals that moisten the lung and dissolve phlegm should be applied so that residual heat is cleared at the same time. For patients with lung-spleen qi deficiency, the spleen should be fortified, yang reinforced, and the lung supplemented to consolidate the exterior; the main consideration here is to reinforce upright qi.

Dr. Hu Cheng-qun believes that on account of the intense heart, liver and stomach fire and the combination of phlegm and heat, the treatment of pneumonia with panting and coughing in children should be clinically compounded with such methods as clearing heart fire, draining liver fire, dispelling stomach fire and resolving binding constraint.

● Cough caused by heart fire is commonly manifested by cough with little phlegm, flaming heat on face and forehead, a sweaty head, neck and back, and dribbling yellow urine. Dăo Chì Săn (Red-Guiding Powder) is the selected formula.

● Cough caused by liver fire is manifested commonly as dry cough without phlegm and heavy breathing, which usually becomes chronic. Normally, gurgling phlegm is not heard. Treatment focuses more on draining liver fire than on dissolving phlegm. Modified Chái Hú Shū Gān Săn (Bupleurum Liver-Soothing Powder) is the selected formula.

● With intense stomach fire manifested by bad breath, sweatiness, poor appetite and dry stools, select Qīng Wèi Săn (Stomach-Heat-Clearing Powder) with added zhī mŭ (Rhizoma Anemarrhenae).

● With phlegm-fire binding constraint manifested by phlegm-heat together with swollen glands, add qīng pí (Pericarpium Citri Reticulatae Viride) and xià kū căo (Spica Prunellae) to clear internal fire. Pneumonia may also be treated with auricular plaster therapy, which may be more agreeable for children than standard acupuncture. Selected ear points include pharynx larynx (yān hóu), trachea (qì guăn), lung (fèi), spleen (pí), kidney (shèn), shen men (shén mén), subcortex (pí zhì xià), endocrine (nèi fēn mì) and brain stem (năo gàn). Wáng bù liú xíng (Semen Vaccariae) seeds are placed at the center of a small square adhesive plaster

and pressed onto each point. Alternate between ears every 3 days. Patients should press the ear points 2 to 3 times each day, for 2 to 3 min each time.

CASE STUDY Male, age 4 months. Initial Visit: December 15th, 2008 Chief Complaint: Fever, cough and breathlessness for 4 days. History: The child developed a persistent fever 4 days ago, coughing hoarsely and finding it hard to expectorate the sticky phlegm in his throat. He cried uneasily and breathlessly with nasal flaring, his face and lips were flaming red, and there was scanty yellow urine. He was given oral erythromycin with limited effect. Signs and Symptoms: Fever, cough with panting, gurgling phlegm, frequent spitting, no sweating, uneasiness and groaning, a poor appetite, constipation, retention of urine, a red tongue with a yellow coating, and purple finger venules reaching the qi-pass. Physical Examination: BT 39.0℃, HR 158 beats/min, RR 48 times/min, BP 85/65 mmHg, average nourishment and development, clear consciousness, poor spirits, panting and cough, blue lips, strong, dry rales heard in both lungs, light wet rales heard in the bottoms of both lungs, especially the right lung, soft abdomen, liver extends 1 cm below the right rib, spleen not palpable. Laboratory Examination: Routine blood test: WBC 18.78×109/L, N 0. 75. Chest X-ray shows small patchy shadows in both lungs.

Pattern Differentiation

Heat invading lung-wei caused lung fluids to become scorched and subsequently condensed to phlegm; the phlegm combined with the internal heat to block the lung with signs and symptoms of fever, cough, breathlessness, nasal flaring and gurgling phlegm in the throat. Intense heattoxin led to a flaming red face and thirst; and qi stagnation and blood stasis caused cyanotic lips. The lung failing to purify and descend affected the spleen-stomach’s function of ascending and descending, resulting in stagnation of turbid qi with bowel excess symptoms such as poor appetite and constipation. The red tongue with a yellow coating is also associated with phlegm-heat obstructing the lung. This disease is mainly located in the lung, and the spleen is involved. The pattern involves intense phlegm and heat combined, and is a branch excess pattern.

Diagnosis Pneumonia with panting and cough due to phlegm-heat blocking the lung

Clinical Treatment This case involves a branch excess pattern manifested mainly by combined phlegm and heat, which blocks the lung and obstructs lung qi. Treatment should open the lung, and eliminate phlegm to relieve cough and panting. The major concern in opening the lung is to restore the lung qi functions of diffusion, dispersion, purification and descent so as to relieve cough and panting.

Principles: Clear heat, eliminate phlegm, open the lung, relieve panting Formula: Modified Wŭ Hŭ Tāng (Five Tigers Decoction) and Tíng Lì Dà Zăo Xiè Fèi Tāng (Lepidium/Descurainiae and Jujube Lung-Draining Decoction) [五虎汤合葶苈⼤枣泻肺汤加减]

3 doses [Formula Analysis] Zhì má huáng, xìng rén and qián hú diffuse the lung and relieve cough. Shēng shí gāo, huáng qín, yú xīng căo and gān căo clear the lung and discharge heat. Sāng bái pí, tíng lì zĭ and zĭ sū zĭ drain the lung and eliminate phlegm. Xì chá purifies the lung and eliminates phlegm. Dān shēn invigorates blood and dispels stasis. Prepared Medicines: Tán Rè Qīng Zhù Shè Yè (Phlegm-Heat Clearing Injection) 0.5 ml/kg diluted with 5% glucose injection 100-250 ml for intravenous infusion, once daily.

Acupuncture Main Points: LU 7 (liè quē), LI 4 (hé gŭ), BL 13 (fèi shù). Supplementary Points: LU 11 (shào shāng), ST 40 (fēng lóng), LI 11 (qū chí), RN 12 (zhōng wăn). Manipulation: Needle all points with drainage. Puncture the points rapidly, with no needle retention. Second Visit After 3 days of treatment, the child had a temperature of 38.5℃ and began to sweat, which brought down his temperature. He could expectorate phlegm with ease, and his cough and panting were greatly relieved. He was given another 3 decoctions, lowering his temperature to 37.5 ℃. He now had no nasal flaring, but the panting, breathlessness, chest pain and constipation still remained. This child’s fever, cough and panting were relieved by the first treatment, which indicated that the branch excess symptoms had been successfully addressed. However, pathogens still remained internally, the intense heat still caused constipation, and the turbid phlegm had not been fully eliminated. Principles: Diffuse the lung, relieve cough, unblock the bowels, discharge heat Formula: Modified Wŭ Hŭ Tāng (Five Tigers Decoction) and Tíng Lì Dà Zăo Xiè Fèi Tāng (Lepidium/Descurainiae and Jujube Lung-Draining Decoction) [五虎汤合葶苈⼤枣泻肺汤加减]

3 doses [Formula Analysis] The symptoms were relieved, so he was given a modified decoction of the original formula. Tiān zhú huáng and chén pí clear heat and dissolve phlegm. Shēng dà huáng unblocks the bowels and discharges heat.

Prevention and Nursing Infants should be well-nourished to improve their constitutions. Complications and secondary infections should be prevented. With severe coughing, the child may be held up and patted on the back to clear the airway. Secretions in the respiratory tract should be cleared quickly, and their body positions should be adjusted to help with the discharge of phlegm.

COMMENTARY AND DISCUSSION The panting and cough discussed here are two symptoms of pneumonia, which is different from the cough and asthma discussed previously. Pneumonia is the name of a biomedical disease, the diagnosis of which is based on physical assessment and laboratory research such as Xrays and blood tests. For example, in this case, the physical examination, blood test and X-rays indicated pneumonia, while the TCM differentiation was phlegm-heat blocking the lung with qi and blood stagnation. As the child was in a relative emergent situation, two strong formulas, Wŭ Hŭ Tāng and Tíng Lì Dà Zăo Xiè Fèi Tāng were applied together. Wŭ Hŭ Tāng derives from The Complete Works of [Zhang] Jing-yue (Jĭng Yuè Quán Shū) (1624) but is actually based on Zhang Zhong-jing’s classic formula, Má Xìng Shí Gān Tāng (Ephedra, Apricot Kernel, Gypsum and Licorice Decoction), which facilitates the flow of lung qi, clears heat and calms wheezing by downbearing rebellious lung qi. This is a traditional and commonly used formula for pneumonia. The fifth “tiger”in this formula is xì chá, which acts to clear internal fire and purify the lung. In the Grand Compendium of Materia Medica (Bĕn Căo Gāng Mù), Li Shi-zhen said, “tea is bitter and cold. It is yin within yin with the characteristics of sinking and descending; therefore it is good for draining fire. Fire tends to cause the hundred diseases. After fire is drained, the upper jiao then remains clear” (茶苦⽽寒,阴中之阴,沉也,降也,最能降⽕。⽕为百病,⽕降则上 清矣). As the pneumonia in this case was quite severe, Wŭ Hŭ Tāng was not strong enough, so another classic prescription Tíng Lì Dà Zăo Xiè Fèi Tāng

was selected. This formula derives from Zhang Zhong-jing’s Essentials from the Golden Cabinet (Jīn Guì Yào Lüè). The chief medicinal tíng lì zĭ can definitely be considered as another “tiger”medicinal that acts to drain the lung, reduce phlegm, calm wheezing, move water and reduce edema. Because of its harsh properties, dà zăo (Fructus Jujubae) is added to harmonize and neutralize the side effects. As the child also had constipation as a result of the lung-large intestine relationship, a small amount of dà huáng was added as another “tiger” herb to drain heat, purge accumulation, promote bowel movement, and invigorate blood and dispel blood stasis. The application of these “tiger-like” medicinals such as zhì má huáng, shēng shí gāo and tíng lì zĭ should be monitored and limited to only a short time period. As soon as the symptoms are controlled, their application should be ceased. In this case, yú xīng căo and huáng qín were added to expel toxic-heat and clear the lungs; both can be considered as natural antibiotics for pneumonia. Dān shēn was also applied to promote blood circulation for the cyanotic signs, as qi stagnation tends to cause blood stasis.

STUDY QUESTIONS 1. What are the causes and pathomechanism of pneumonia transmuted patterns with panting and cough in children? 2. How to differentiate coughing in the wind-heat blocking the lung pattern with panting from cough due to wind-heat?

Answers 1. The lungs govern qi and link with all the vessels. The development of this disease, as a result of intense pathogenic qi, weak upright qi, or failed or improper treatment, will harm other organs. When the lungs fail to purify and descend, it may cause spleen-stomach failure of ascending and descending; as the clear qi fails to move upward and turbid qi downward, bowel excess symptoms such as abdominal distention and constipation appear. When heat-toxin is intense and sinks into the jueyin, liver wind can lead to convulsions or even coma. The lungs govern qi, and the heart governs the blood and vessels. Qi flow promotes blood transportation, while qi stagnation causes blood stasis. Therefore, when lung qi is blocked, impaired qi movement will lead to poor blood flow; symptoms of qi stagnation and blood stasis manifest here as a pale face with cyanotic lips and nails. If the upright qi is unable to defeat the pathogenic qi, more severe qi stagnation and blood stasis can produce critical conditions associated with heart qi deficiency or heart yang deficiency and debilitation.

2. Signs and symptoms of fever, cough, a yellow tongue coating and rapid pulses may be observed clinically in patterns of both wind-heat blocking the lung with panting and cough and wind-heat invading the lung with cough. In addition, lung symptoms such as breathlessness, nasal flaring, slight panting and gurgling phlegm in the throat may accompany the wind-heat blocking the lung pattern. Signs and symptoms of lung qi failing to purify with wei-exterior disharmony include an uneasy cough, a runny nose with turbid discharge, headache, and aversion to wind will accompany the wind-heat invading the lung pattern, but breathlessness and nasal flaring will not.

CASE SCENARIOS The following cases present variations of this condition. After familiarizing yourself with the possible common pattern presentations and appropriate formulas for treatment, use the following exercises to test your overall understanding of the condition. 1. A 1-year-old child has been suffering from paroxysmal cough for 2 days; he is coughing uneasily and breathlessly with turbid phlegm. He has also suffered from a fever for the past 2 days, with a temperature between 37.5℃ and 38℃. Other signs and symptoms include a normal pharynx, a light red tongue with a thin white coating, and light red finger venules. What is the presenting pattern in this case? A. Wind-cold blocking the lung B. Wind-heat blocking the lung C. Phlegm-heat blocking the lung D. Lung-spleen qi deficiency E. Yin-deficiency and lung heat 2. A patient, 5 months of age, has been suffering from fever and cough for the past 3 days with panting for 1 day. Other signs and symptoms include a high temperature, aversion to wind, breathless coughing, slight sweatiness, thirst, a red pharynx, a red tongue with a thin yellow coating, and purple finger venules. What is the presenting pattern?

A. Wind-cold blocking the lung B. Wind-heat blocking the lung C. Phlegm-heat blocking the lung D. Toxin-heat blocking the lung E. Yin-deficiency and lung heat 3. A patient, age 2, has been suffering from a fever over the past 3 days, and coughing over the past 2 days. Presenting symptoms and signs include fever, severe coughing, breathlessness, nasal flaring, non-production of tears, a flaming red face and lips, uneasiness, dribbling yellow urine, constipation, a red tongue with greasy yellow coating, and static purple finger venules. What are the proper treatment principles? A. Scatter wind, clear heat, diffuse the lungs, relieve cough B. Clear the lungs, dissolve phlegm, relieve cough and panting C. Clear heat, eliminate phlegm, open the lungs, relieve panting D. Diffuse the lungs with acrid-coolness, clear heat, eliminate phlegm E. Clear heat, resolve toxins, drain the lungs, open obstruction 4. A patient, age 9, has been suffering from fever and cough over the past 2 days with fever, aversion to wind, cough, breathlessness, large production of yellow phlegm, thirst, a red pharynx, a red tongue with a thin white coating, and a floating rapid pulse. What are the proper treatment principles? A. Diffuse the lungs with acrid-warmth, eliminate phlegm, relieve cough

B. Diffuse the lungs with acrid-coolness, clear heat, eliminate phlegm C. Clear heat, eliminate phlegm, diffuse the lung, relieve panting D. Clear heat, resolve toxins, drain the lung, open obstruction E. Nourish yin, clear the lung, moisten the lung, relieve cough 5. A patient, age 4, has been suffering from a persistent high fever over the past 3 days and has been coughing severely with breathlessness, nasal flaring, uneasiness, convulsion of the limbs, occasional coma and delirium, upward-facing eyes, a dark red tongue, and finger venules reaching the lifepass. What are the proper treatment principles? A. Diffuse the lungs with acrid-coolness, clear heat, eliminate phlegm B. Clear heat, resolve toxins, drain the lung, open obstruction C. Clear heat, eliminate phlegm, open the lung, relieve panting D. Warm and supplement heart yang, rescue counterflow and desertion E. Calm the liver, extinguish wind, clear heart heat, open the orifices 6. A patient, age 5, has been suffering from persistent protracted pneumonia with panting and cough over the past month with a low fever, night sweats, a dry cough without phlegm, poor appetite, a flaming red face, a red tongue with a peeled coating, inadequate saliva, and a thready rapid pulse. What is the proper treatment formula? A. Mài Mén Dōng Tāng (Ophiopogon Decoction) B. Shā Shēn Mài Dōng Tāng (Adenophora and Ophiopogon Decoction)

C. Qīng Hāo Biē Jiă Tāng (Sweet Wormwood and Turtle Shell Decoction) D. Bŭ Fèi Ē Jiāo Tāng (Lung-Supplementing Donkey-Hide Gelatin Decoction) E. Zhī Băi Dì Huáng Tāng (Anemarrhena, Phellodendron and Rehmannia Decoction) 7. A patient, 6 months of age, has had fever and cough over the past 2 days with a high fever, occasional uneasiness, severe coughing, breathlessness, nasal flaring and dyspnea. Around 5 am he was found with a pale complexion, blue lips, cold limbs, palpable lumps underneath the right ribs, a purple tongue with a thin coating, and purple static finger venules. This case should be differentiated as which pattern of pneumonia with panting and cough? A. Wind-heat blocking the lung B. Phlegm-heat blocking the lung C. Heart yang deficiency and debilitation D. Pathogens sinking into the jueyin E. Heat pathogens blocking the lung 8. A patient, 5 months of age, has been suffering from a fever and breathless cough with panting for 3 days. He had a fever of 38.5℃ and was given Shuāng Huáng Lián Kŏu Fú Yè (Lonicera, Scutellaria and Forsythia Oral Liquid) to bring down his temperature, which failed. Over the past 3 days, signs and symptoms included a high fever, uneasiness, cough with gurgling phlegm, breathless panting, dyspnea, thirst, dry stools, a flaming

red face, dry lips, a red tongue with a yellow coating, and a slippery rapid pulse. What is the presenting pattern? A. Wind-heat blocking the lung B. Toxic-heat blocking the lung C. Yin deficiency and lung heat D. Wind-cold blocking the lung E. Phlegm-heat blocking the lung 9. A patient, age 2, has been suffering from a high fever over the past 3 days with uneasiness, cough with excessive phlegm, breathlessness and nasal flaring. This morning, signs and symptoms included coma and delirium, lockjaw, stiff neck, convulsion of the limbs and upward-facing eyes. After an intravenous injection of diazepam, the convulsion of his limbs ceased immediately. His tongue was dark red, and his finger venules were purple, reaching to the life-pass. What are the proper treatment methods? A. Calm the liver, extinguish wind, clear heart heat, open the orifices B. Release the exterior with acrid-coolness, clear heat, suppress fright C. Clear heat, resolve toxins, drain the lung, open obstruction D. Warm and supplement heart yang, rescue counterflow and desertion E. Clear heat, eliminate phlegm, open the lung, relieve panting 10. A patient, age 8, has been suffering from a cough over the past 10 days, fever over the past 7 days, and breathless panting for 1 day. He caught a cold a week ago and began to dry cough with little production of phlegm.

Over the past 3 days, he coughed even worse with gurgling phlegm in his throat. He was given Líng Yáng Qīng Fèī Săn (Antelope Horn LungClearing Powder), but it did not relieve his symptoms; he began to cough breathlessly with panting, feeling uneasy at times, and fine wet rales were heard in his lungs. Laboratory examination showed normal WBC counts and mycoplasma pneumoniae (+), and X-ray shows light patchy shadows in the lower middle part of the right lung. What is the correct diagnosis? A. Respiratory syncytial virus (RSV) pneumonia B. Adenovirus pneumonia C. Staphylococcal aureus pneumonia D. Mycoplasma pneumonia E. Gram-negative bacillus pneumonia

Answers 1. A 2. B 3. E 4. B 5. E 6. B 7. C 8. E

9. A 10. D

Chapter 5 Asthma Asthma is a recurrent asthmatic lung disease that commonly attacks in childhood, characterized by wheezing and panting with breathlessness, gurgling phlegm in the throat and lengthened expiration. The seriously ill may be unable to lie down and exhibit difficult open-mouth raised-shoulder breathing with shivering, blue lips, and their hands holding the abdomen. Asthma may have both internal and external causes. Internal causes involve deficiencies of the lung, spleen and kidney, which is the root of this disease. External causes include contraction of external pathogens (the most common pathogenesis), exposure to foreign matter or strong smells, and overconsumption of salty or sour foods. Generally speaking, the pathogenesis involves internal obstruction of qi, external contraction, and condensed phlegm accumulated in the chest. Asthma is clinically characterized by branch excess manifestations at the time of attack. Common patterns in the attack stage include cold wheezing, heat wheezing, exterior cold with interior heat, and lung excess with kidney deficiency. The disease is characterized by signs and symptoms of upright qi deficiency in the remission stage, with pattern types including lung and spleen qi deficiency, spleen and kidney yang deficiency, and lung and kidney yin deficiency.

CLINICAL ESSENTIALS This disease should be treated according to the presenting signs and symptoms in the attack or the remission stage. Primary treatment is given to the lung, with methods applied to expel pathogens, reduce phlegm, diffuse the lung and direct qi downward, applied accordingly during the stage of attack. During the stage of remission, upright qi must be reinforced to address the root, the functions of the lung, spleen and kidney must be adjusted, and the root of long-accumulated phlegm must be eliminated. A compound treatment is preferred, as asthma is a recurring disease. Prof. Liu Bi-chen views pediatric asthma as being caused by hidden phlegm in the lung, hidden wind in the liver, and invasion of external wind in the lung. His empirical formula acts to regulate the lung, calm the liver, warm the kidney, direct qi downward, dissolve phlegm and relieve panting: The formula contains: xīn yí (Flos Magnoliae) 10g, cāng ĕr zĭ (Fructus Xanthii) 10g, xuán shēn (Radix Scrophulariae) 10g, băn lán gēn (Radix Isatidis) 10g, shān dòu gēn (Radix et Rhizoma Sophorae Tonkinensis) 5g, gōu téng (Ramulus Uncariae Cum Uncis) 10g, dì lóng (Pheretima) 10g, zĭ shí yīng (Fluoritum) 15g and qín pí (Cortex Fraxini) 10g. Among the ingredients, xīn yí, cāng ĕr zĭ, xuán shēn, băn lán gēn and shān dòu gēn diffuse the lung and unblock the orifices to smooth qi movement and dispel pathogens to protect the lung and safeguard the interior. These medicinals disperse external pathogens and regulate and smooth qi movement during the stage of attack. In the stage of remission, they block the invasion of pathogens and prevent external wind invasion

that may activate hidden wind-phlegm. Gōu téng, qín pí and zĭ shí yīng are effective medicinals to treat wheezing. The main formula may be modified and applied both in the attack and remission stages, as all the medicinals combine well to treat both branch and root. Acupuncturist Liu Gong-wang views pediatric asthma as being caused by the combination of internally generated damp-heat with external pathogens that invade and block the lung. Therefore, hua tuo jia ji points should be selected to treat the corresponding zang-fu organs as well as the five sensory organs and five bodily constituents. Point selection includes hua tuo jia ji points (T1-T5), BL 13 (fèi shù) and BL 17 (gé shù) with cupping following collateral bloodletting, LI 11 (qū chí), SP 10 (xuè hăi), and SP 6 (sān yīn jiāo).

● Hua tuo jia ji points should be needled obliquely toward the spinal column; the patient should feel soreness at the needled points.

● Needles should be inserted perpendicularly into LI 11 (qū chí), SP 10 (xuè hăi) and SP 6 (sān yīn jiāo) with even supplementation and drainage; the patient should feel soreness at the needled points propagating upward or downward. Retain the needles for 30 min.

● Point bloodletting should be conducted on BL 13 (fèi shù) and BL 17 (gé shù) followed by flash-fire cupping for 5 min. Treat once every 3 days; two sessions constitute one course of treatment.

CASE STUDY Female, age 5. Initial Visit: November 9th, 2009 Chief Complaint: Cough with panting and chest stuffiness for 3 days. History: The child began to cough with panting, chest stuffiness and abdominal distention 3 days ago as a result of catching a cold. As the application of bricanyl and salbutamol had little effect, she was taken to the hospital for TCM treatment. Signs and Symptoms: Wheezing, gurgling phlegm in the throat, expectoration of sticky yellow phlegm, inability to lie down due to chest stuffiness that was particularly serious in the evening, low spirits, blue lips, thirst, dry stools, yellow dribbling urine, a red tongue with a greasy yellow coating, and a slippery rapid pulse. Past History: The patient had been suffering from asthma for 2 years, which usually attacked when the weather changed unexpectedly or when she caught a cold or played sports. Both Chinese and Western medicines were given, but neither was effective. Physical Examination: BT 39℃, HR 120 beats/min, RR 30 times/min, full consciousness, low spirits, pale complexion, nasal flaring, positive for three depression signs, pharynx hyperemia, hyperresonance heard upon lung percussion, great wheezing heard in auscultation, tender abdomen, liver and spleen not palpable. Routine Blood Test: WBC 4.2×109/L, N 0. 48, L 0. 43, E 0.34×109/L.

Lung Function Test: Tidal breath flow volume loop severely blocked, bronchodilatation test positive. Chest X-ray: Increased radiolucency of both lungs.

Pattern Differentiation A contraction of external pathogens activated the long-accumulated internal phlegm. The phlegm and qi became bound and blocked in the airway resulting in difficult breathing, chest stuffiness, gurgling and wheezing, and phlegm accumulated in the throat. Recurrences had damaged lung qi and yin, and the accompanying external contraction of wind-heat further promoted the development of sticky yellow phlegm and thirst. The red tongue with a greasy yellow coating, and the slippery rapid pulse reflect phlegm and heat combining internally. The disease was located in the lung, spleen and kidney. During the stage of attack, phlegm and heat congested the upper jiao. This is a case of a branch excess pattern type.

Diagnosis Asthma due to heat wheezing with heat-phlegm blocking lung qi

Clinical Treatment Principles: Clear the lung, clear phlegm, relieve coughing and panting

Formula: Modified Má Xìng Shí Gān Tāng (Ephedra, Apricot Kernel, Gypsum and Licorice Decoction) and Sū Tíng Wán (Perilla Fruit and Lepidium/Descurainiae Pill) [麻杏⽯⽢汤合苏葶丸加减]

6 doses As asthma is a recurring disease, a compound treatment is preferred. Auricular plaster therapy and Western medicines are also applicable. [Formula Analysis] Zhì má huáng, xìng rén and qián hú diffuse the lung and relieve cough. Shēng shí gāo and huáng qín clear the lung and clear heat. Tíng lì zĭ, zĭ sū zĭ and sāng bái pí drain the lung and relieve panting. Shè gān and zhĭ qiào direct qi downward and dissolve phlegm. Jiāng cán clears heat and relieves spasm. Zhú lì clears up phlegm and directs qi downward.

Guā lóu rén directs counterflow downward and dredges the bowels. Zhì gān căo protects stomach qi against being damaged by coldnatured shí gāo. It can also harmonize the actions of shí gāo and má huáng. Prepared Medicines: Xiào Chuăn Kē Lì (Asthma Granules), 10g twice daily, mixed with warm water. External Therapy: Bái jiè zĭ (Semen Sinapis) 21g, yán hú suŏ (Rhizoma Corydalis) 21g, gān suì (Radix Kansui) 12g and xì xīn (Radix et Rhizoma Asari) 12g are ground to a fine powder and divided into 3 equal parts; one part is used every 10 days. Mix ginger juice with the powder, stir until gelatinous, and form several dime-sized lumps. Press them separately onto BL 13 (fèi shù), BL 15 (xīn shù), BL 17 (gé shù) and RN 17 (dàn zhōng), and remove after 4 hours. If the skin turns red or small poxes appear, remove the medicinals earlier. Conduct this treatment 3 times in the hottest month of the summer, 10 days apart, for three successive years. Second Visit The chest stuffiness, panting and breathlessness disappeared after 6 doses. Remaining signs and symptoms included weak coughing, panting, sweating, a pale complexion, poor appetite, loose stools, a pale tongue with a thin coating, and a thready pulse. Harsh breath sounds were heard upon lung auscultation, but wheezing was not detected. Principles: Fortify the spleen , boost qi, supplement the lung, consolidate the exterior Formula: Modified Rén Shēn Wŭ Wèi Zĭ Tāng (Ginseng and Schisandra Decoction) and Yù Píng Fēng Săn (Jade Wind-Barrier Powder)

[⼈参五味⼦汤合⽟屏风散加减]

6 doses [Formula Analysis] Rén shēn and wŭ wèi zĭ supplement qi and astringe the lung. Fú líng and bái zhú fortify the spleen and supplement qi. Huáng qí and fáng fēng boost qi to consolidate the exterior. Bàn xià and jú hóng dissolve phlegm and relieve cough. Duàn mŭ lì consolidates essence and arrests sweating. Chăo shān zhā promotes digestion by helping the movement of the bowels. Chăo biăn dòu fortifies the spleen and removes dampness. Gān căo benefits qi, relieves cough, and harmonizes all formula medicinals.

Prevention and Nursing

Avoid any inducing factors such as external contraction, the smell of smoke or paint, dust mites, pollen, and stimulating foods such as seafood and cold drinks. Pay attention to changes in the weather. Avoid catching a cold when going out in wintertime. In seasons when asthma is liable to attack, avoid excessive physical exercise and extreme emotions. Encourage children to take an active part in daily activities and physical exercise in order to strengthen the constitution.

COMMENTARY AND DISCUSSION Of all zang-fu organs, the lung is located at the highest point; therefore, the lung has traditionally been called the “imperial carriage roof”. The lung is also connected to the external environment through the nose and skin pores. Because of these factors, the lung is most sensitive to changes in season and climate, and has the closest relationship with the external environment. In this case, asthma usually recurred when the weather changed or when she caught a cold. Traditionally, long-accumulated phlegm is considered the root of asthma, which is the key difference between asthma and other respiratory diseases like bronchitis or pneumonia. Asthma has wheezing as a main symptom. In Chinese medicine, all noisy symptoms are caused by the interaction between dampness retention, water or phlegm, and the abnormal movement of qi. For example, when there is middle jiao dampness and stomach qi counterflow, the result is nausea and vomiting. When there is phlegm in the chest and throat and lung qi counterflow, there is wheezing; this explains why asthma has wheezing, but other diseases of the lung may not. Based on this theory, eliminating phlegm becomes the primary treatment, so in this case, phlegm-removing medicinals such as tíng lì zĭ, zhĭ qiào, zhú lì and guā lóu rén were applied at the initial visit to treat the root. The Yellow Emperor’s Inner Classic (Huáng Dì Nèi Jīng) states, “the heart governs blood, while the lung controls qi” (⼼主⾎,肺主⽓).

When the lung system is out of balance, qi disorders often result. As the etiologies are different, it is important to know whether the lung qi disorder is due to failure of lung qi dispersion or due to lung qi failing to descend. With asthma, for example, external environment-induced asthma is associated with failure of lung qi dispersion, while internal environmentrelated asthma is due to lung qi failing to descend. The treatments are also different. For externally-induced asthma, dispersing and facilitating the movement of lung qi is critical, so acrid and pungent medicinals such as má huáng (Herba Ephedrae), qián hú (Radix Peucedani), zĭ wăn (Radix et Rhizoma Asteris) and băi bù (Radix Stemonae) are commonly applied to spread and disperse lung qi. In this case, Má Xìng Shí Gān Tāng, a formula for common cold-induced asthma was applied to disperse lung qi. For internally-induced asthma, descending lung qi is the key strategy; medicinals such as zĭ sū zĭ (Fructus Perillae), lái fú zĭ (Semen Raphani), bàn xià (Rhizoma Pinelliae), hòu pò (Cortex Magnoliae Officinalis) and zhĭ qiào (Fructus Aurantii) are often used. Internal asthma often involves the liver, large intestine and other organs, so treatment should also address those related issues. Because the dispersing and descending functions of the lung are mutually affected no matter which aspect was disordered first, a patient will often show disorders of both dispersing and descending. For this reason, the Sū Tíng Wán was added to descend lung qi and to remove phlegm. The combination of the two medicinal prescriptions ensures that the dispersing and descending functions of lung qi are both well-adjusted. Asthma is a chronic and stubborn disease involving the body constitution and genetic factors. Because the kidney stores congenital

essence, all inherited diseases involve the kidney. This means that asthma has a deep location, and is complicated. Therefore, “treating the upper (lung) during the acute episode while treating the lower (kidney) at the chronic stage” is the best overall treatment strategy. External treatments with bái jiè zĭ, yán hú suŏ, gān suì and xì xīn are not so practical in North America and Europe, as Western people cannot generally accept the blistering that results from this kind of moxibustion. The damage to the skin is often considered as a medical accident which can become a legal issue unless the client signs a consent agreement.

STUDY QUESTIONS 1. What are the key diagnostic points for asthma? 2. What is the most important problem to consider in the treatment of asthma in the stage of attack? In addition to treatment according to pattern differentiation, what other methods may be applied?

Answers 1. There are several key points for diagnosis:

● Recognize that asthma usually attacks suddenly with premonitory symptoms of sneezing and cough. When it attacks, patients usually develop panting, breathlessness, gurgling phlegm in the throat and paroxysmal coughing. The seriously ill will be unable to lie down, they will feel uneasy, and their lips will become blue.

● Most patients have a history of recurrent asthma with attacks usually related to specific factors such as a sudden weather change, catching a cold or being over-exposed to heat, or when taking or exposing oneself to certain anaphylactic substances.

● Patients usually have a history of asthma in infancy, or a family history of asthma.

● Upon auscultation of the lung during an attack, wheezing is heard especially at expiration, and expiration is usually lengthened. When infection follows, wet rales can be detected.

● Other signs are evident on a hemogram. Patients who suffer from bronchial asthma have a normal WBC count and a possibly increased EOS count. When bacterial infection accompanies asthma in the lung, WBC and NEU counts may be elevated. 2. During the attack stage, the main approach to treatment is to relieve panting. Apart from treatments for commonly differentiated patterns of asthma, other methods and medicines can also be applied accordingly:

● Treatment may be required to dispel wind and arrest convulsion. Gōu téng (Ramulus Uncariae Cum Uncis), dì lóng (Pheretima), jiāng cán (Bombyx Batryticatus), chán tuì (Periostracum Cicadae) as well as wú gōng (Scolopendra) and quán xiē (Scorpio) can dispel wind and arrest convulsion with reasonable effects on the bronchospasm and panting.

● Treatment may focus on dissolving phlegm and unblocking the bowels. The lung and the large intestine are internally-externally related, so if internal phlegm blocks the lung so that bowel-qi is blocked, the lung will fail to diffuse and govern descent. For patients with strong constitutions who suffer from severe cough and panting with blocked bowel-qi, medicinals such as qiān niú zĭ (Semen Pharbitidis), zào jiăo cì (Spina Gleditsiae), dà huáng (Radix et Rhizoma Rhei), máng xiāo (Natrii Sulfas), bīng láng (Semen Arecae) and zhĭ shí (Fructus Aurantii Immaturus) may be applied to unblock the bowels and direct lung qi downward.

● Treatment may focus on invigorating blood and dissolving stasis. Medicinals like táo rén (Semen Persicae), dāng guī (Radix Angelicae Sinensis), chuān xiōng (Rhizoma Chuanxiong), dān shēn (Radix et Rhizoma Salviae Miltiorrhizae) and chì sháo (Radix Paeoniae Rubra) may be applied to dredge the lung collaterals and dissolve phlegm-stasis. But in

order to dissolve the phlegm-stasis, compound application of substances that can move qi is recommended, because qi flow can promote blood transportation and also facilitate the movement of phlegm.

● Treatment may warm yang and govern floating yang. For patients with severe panting, great cold, dripping sweat, extreme anxiety and a pale complexion, medicinals such as fù zĭ (Radix Aconiti Lateralis Praeparata), ròu guì (Cortex Cinnamomi), shān zhū yú (Fructus Corni), cí shí (Magnetitum), lóng gŭ (Os Draconis; Fossilia Ossis Mastodi), mŭ lì (Concha Ostreae) and jiāo mù (Semen Zanthoxyli) should be applied to warm the kidney, restore yang, and govern floating yang.

● Increased dosages of anti-asthmatic medicinals with specific effects may also be applied; for example má huáng (Herba Ephedrae) is most effective when used in larger amounts, and not prepared. Jiāo mù (Semen Zanthoxyli) and tiān jiāng ké (Pericarpium Metaplexis) are also applicable medicinals with anti-asthmatic effects.

CASE SCENARIOS The following cases present variations of this condition. After familiarizing yourself with the possible common pattern presentations and appropriate formulas for treatment, use the following exercises to test your overall understanding of the condition. 1. A patient, age 7, has been suffering from cough and panting for 2 days with signs and symptoms including cough, panting, a high voice with strong breath, wheezing, gurgling phlegm in the throat, chest stuffiness, abdominal distention, expectoration of sticky yellow phlegm, fever, thirst, dry throat, constipation, a red tongue with a yellow coating, and a slippery pulse. What is the presenting pattern? A. Cold wheezing B. Heat wheezing C. Exterior cold- interior heat D. Lung excess and kidney deficiency E. Lung-kidney yin deficiency 2. A patient, age 9, has been suffering from cough and panting for the past 3 days with signs and symptoms of cough, gurgling phlegm in the throat, panting, breathlessness, a congested and runny nose, sneezing, expectoration of sticky yellow phlegm, constipation, a red tongue with a white coating, and a floating slippery pulse.

What is the presenting pattern? A. Cold wheezing B. Exterior cold-interior heat C. Heat wheezing D. Lung excess and kidney deficiency E. Lung-kidney yin deficiency 3. A patient, age 10, has been suffering from recurrent cough and panting for the past 4 years with signs and symptoms of coughing weakly, a low voice with weak breathing, spontaneous sweating, aversion to wind, a pale complexion, low spirits, reticence, emaciation, loose stools, a light red tongue with a thin white coating, and a soft thready pulse. What is the presenting pattern? A. Cold wheezing B. Lung excess and kidney deficiency C. Lung-kidney yin deficiency D. Lung-spleen qi deficiency E. Spleen-kidney yang deficiency 4. A patient, age 7, has been suffering from cough and panting for the past 2 months along with a recurrent cough, weak panting, difficult expectoration of phlegm, a flaming red face, night sweats, feverish feelings in the palms and soles, a red tongue with little coating, and a thready rapid pulse. What is the presenting pattern?

A. Cold wheezing B. Lung excess and kidney deficiency C. Lung-kidney yin deficiency D. Lung-spleen qi deficiency E. Spleen-kidney yang deficiency 5. A patient, age 3, has been suffering from recurrent cough and panting for the past half year. He has been suffering from a fever and paroxysmal cough for 2 days, with his temperature fluctuating between 38 ℃ and 39 ℃. Yesterday, he developed breathless panting, expectoration of sticky yellow phlegm, wheezing, gurgling phlegm in the throat, thirst, a red pharynx, constipation, a red tongue with a yellow coating, and a slippery and rapid pulse. What are the correct principles of treatment? A. Warm the stomach, dissipate cold, dissolve phlegm, relieve panting B. Clear lung heat, eliminate phlegm, relieve cough and panting C. Release the exterior, clear the interior, relieve cough and panting D. Drain the lung, supplement the kidney, treat both branch and root E. Nourish yin, clear heat, supplement and boost lung and kidney 6. A patient, age 5, has a two-year history of asthma and recurrent colds. Over the past 2 weeks he has developed breathlessness, sweating, a weak cough, a pale complexion, low spirits, reticence, loose stools, a pale tongue with a thin white coating, and a thready soft pulse. Which is the best treatment formula? A. Sān Zĭ Yăng Qīn Tāng (Three-Seed Filial Devotion Decoction)

B. Dà Qīng Lóng Tāng (Major Green Dragon Decoction) C. Rén Shēn Wŭ Wèi Zĭ Tāng (Ginseng and Schisandra Decoction) and Yù Píng Fēng Săn (Jade Wind-Barrier Powder) D. Jīn Guì Shèn Qì Wán (Golden Cabinet’s Kidney Qi Pill) E. Liù Jūn Zĭ Tāng (Six Gentlemen Decoction) 7. A patient, age 4, has been suffering from recurrent cough and panting for the past 3 years. Since last week he has developed an uneasy cough, weak panting worse in the evening, a flaming red face, night sweats, feverish feelings in palms and soles, excessive urination at night, a red tongue with a peeled coating, and a thready rapid pulse. What is the best treatment formula? A. Má Xìng Shí Gān Tāng (Ephedra, Apricot Kernel, Gypsum and Licorice Decoction) B. Mài Wèi Dì Huáng Wán (Ophiopogon and Rehmannia Pill) C. Shā Shēn Mài Dōng Tāng (Adenophorae and Ophiopogon Decoction) D. Sū Zĭ Jiàng Qì Tāng (Perilla Fruit Qi-Descending Decoction) E. Yù Píng Fēng Săn (Jade Wind-Barrier Powder) 8. A patient, age 5, has been suffering from cough and panting for one day along with aversion to cold, fever, cough, panting, gurgling phlegm in the throat, large production of thin phlegm, cold limbs, a runny nose, pale face, a pale tongue with a white slippery coating, and a floating slippery pulse.

What is the best treatment formula? A. Má Huáng Tāng (Ephedra Decoction) B. Huá Gài Săn (Florid Canopy Powder) C. Jīn Fèi Căo Săn (Inula Powder) D. Sū Tíng Wán (Perilla Fruit and Lepidium/Descurainiae Pill) E. Xiăo Qīng Lóng Tāng (Minor Green Dragon Decoction) and Sān Zĭ Yăng Qīn Tāng (Three-Seed Filial Devotion Decoction)

Answers 1. B 2. B 3. D 4. B 5. B 6. C 7. B 8. E

Chapter 6 Recurrent Respiratory Tract Infection Recurrent respiratory tract infection (RRTI) refers to the recurrent infection of the upper or lower respiratory tracts. Many children are susceptible to RRTI, and some suffer from frequent recurrence. In TCM, the disease falls under the category of common cold, which has a broader context within Chinese medicine. RRTI occurs more often in children with constitutional deficiency. Patients are most susceptible to recurrence of this disease during intense weather changes in the spring and winter seasons, with less prevalence in summertime. This disease is most commonly found in children between 6 months and 6 years of age, especially in infants between 1 and 3 years of age.

CLINICAL ESSENTIALS The key point in pattern differentiation of recurrent respiratory tract infection in children is to differentiate the relative growth and decline of upright qi and pathogenic qi. Pathogenic excess is the main pattern in the period of infection. In the period of lingering, however, the main pattern is upright qi deficiency and pathogenic qi lingering. In the period of recovery, the main pattern is upright qi deficiency.

● In the period of infection, patients have usually contracted pathogenic wind. Therefore, differentiation must be made between patterns of wind-cold and wind-heat, as well as between pathogens residing interiorly or exteriorly. The pathogenesis must be ascertained while also differentiating root deficiency patterns from branch excess manifestations.

● In the period of lingering, pathogenic toxins are controlled, and manifestations of deficiency appear. As signs of upright qi deficiency surpass those of pathogenic qi excess, deficiency and damage of the lung, spleen and kidney deficiency patterns must be differentiated in the following ways: Patients with lung deficiency will suffer qi weakness. Patients with spleen deficiency will suffer impaired transportation. Patients with kidney deficiency will suffer weakened bones and other signs of kidney deficiency. In the period of lingering, treatment should be conducted by reinforcing upright qi while concurrently dispelling pathogens. When upright qi is restored, pathogenic qi retreats automatically.

● In the period of recovery, treatment should be conducted on the basis of fortifying the root, either by supplementing qi and consolidating the

exterior, by regulating ying and wei, or by supplementing the kidney to strengthen bone. Treatment is most important in the period of recovery. During this time, upright qi must be supplemented to support the expelling of the remaining pathogenic qi; by supporting upright qi, the patient will also be less likely to suffer from recurrence.

● Recurrent respiratory tract infections in children can also be treated with plaster therapy: Dry and grind to powder 3 parts of bái jiè zĭ (Semen Sinapis), 2 parts of xì xīn (Radix et Rhizoma Asari) and 1 part of gān suì (Radix Kansui); mix the powder with a 50% fresh ginger juice solution, stirring until it becomes gelatinous, and form into several 2 cm×0.5 cm pie-shaped lumps. Apply treatment bilaterally to BL 13 (fèi shù), BL 43 (gāo huāng), dìng chuăn (EX-B1) and RN 22 (tiān tū) by pressing the plaster onto the points and fixing it with dressing. Retain for 2 to 6 hours. Treat three times a year, 10 days apart, during the summer season (according to the lunar calendar). The hot climate helps promote yang qi, especially in people with yang deficiency-type constitutions. The acridwarm medicinals act to dredge the channels, unblock the collaterals, warm yang, boost qi and dispel external pathogens. The above points act to regulate the ascending and descending of lung qi while warming and supplementing the spleen and kidney for prevention or treatment.

CASE STUDY Male, age 6. Initial Visit: November 21st, 2006 Chief Complaint: Recurrent common colds for 2 years with susceptibility to pneumonia. History: The patient contracted pneumonia at the beginning of the previous year, followed by recurrent upper respiratory tract infection. Signs and symptoms included a red posterior pharyngeal wall, fever, frequent cough, runny nose, and swelling of the tonsils. The condition recurred 6 to 10 times in the past year, roughly once every one or two months. Signs and Symptoms: A low fever, slight cough, runny nose, sore throat, aversion to cold and heat, cold sweating, flaccid muscles, a weak floating and rapid pulse, a light red tongue with a thin white coating, and purple finger venules. Physical Examination: BT 37.8℃, thin body, flaming red lips, swelling of the tonsils on both sides of the throat without apparent pus, rough breath sounds heard in both lungs, dry or wet rales not heard, heart rate 128 beats/min, normal heart beat with strong heart sounds, murmurs not heard, flat soft abdomen without tenderness or rebound tenderness, normal bowel sounds. Laboratory Examination: WBC 9.8×109/L, N 0.41, L 0.50, PLT 178×109/L, HGB 109 g/L.

Pattern Differentiation

The patient had a weak constitution which caused greater susceptibility to recurrent upper respiratory tract infections and the common cold. Because of deficient wei-yang, pathogenic qi invasion leads to a low fever and cough associated with the common cold. When ying and yin are not secured, cold sweating develops. When lung-wei is not secure, low fever, cough and runny nose develops. When pathogenic toxins linger, a red pharynx and swelling of the tonsils develop. The weak floating rapid pulse, the light red tongue with a thin white coating, and the purple finger venules are signs of ying-wei disharmony with pathogenic toxins lingering. The disease in this case was located in the lung, and caused by yingwei disharmony with lingering pathogenic toxins. The nature of the case involves root deficiency with branch excess. Upright qi insufficiency and insecurity of the wei-exterior resulted in recurrent external contraction with lingering of pathogenic toxins. For this reason, the disease recurred shortly after it had resolved.

Diagnosis Common cold due to ying-wei disharmony with lingering pathogenic toxins

Clinical Treatment Principles: Fortify upright qi, secure the exterior, harmonize ying and wei Formula: Modified Huáng Qí Guì Zhī Wŭ Wù Tāng (Astragalus and Cinnamon Twig Five Substances Decoction)

[黄芪桂枝五物汤加减]

5 doses were prescribed. [Formula Analysis] Huáng qí boosts qi to consolidate wei. Guì zhī and shēng jiāng warm yang and dissipate cold. Bái sháo regulates ying and astringes yin. Zhì gān căo and dà zăo harmonize the middle. Duàn lóng gŭ and duàn mŭ lì consolidate the exterior and arrest sweating. Fú xiăo mài boosts qi to consolidate the exterior.

Băi bù, xìng rén and zhì kuăn dōng huā diffuse the lung and relieve cough. Qīng hāo and lián qiào clear and diffuse lung heat. Băn lán gēn, xuán shēn and zhè bèi mŭ soothe the throat, dissolve phlegm and reduce swelling. Prepared Medicines: Yù Píng Fēng Kŏu Fú Yè (Jade Wind-Barrier Oral Liquid), 10 ml, twice daily. Administer for 3 to 6 successive months. Ear Acupuncture Points: Pharynx larynx (yān hóu), trachea (qì guăn), lung (fèi), large intestine (dà cháng), spleen (pí), kidney (shèn), endocrine (nèi fēn mì), subcortex (pí zhì xià), shen men (shén mén), brain stem (năo gàn), ear apex (ĕr jiān) (prick to bleed). Manipulations: Disinfect the skin with 75% alcohol. Place a wáng bù liú xíng (Semen Vaccariae) ear-seed onto a 0.4 cm×0.4 cm square of adhesive plaster and attach one to each point, then press each point for a few minutes. 6 days constitute one course of treatment. Second Visit After 5 doses, the fever, aversion to cold, cough and nasal congestion were gone. The sweating continued, especially when active or sleeping in the evenings. He had recently developed a poor appetite, a pale complexion and loose stools. This indicated that the exterior pathogens had been dispelled, with lung-spleen deficiency and insufficiency of qi and blood remaining. Principles: Fortify the spleen to boost qi, supplement the lung to consolidate the exterior

Formula: Supplemented Yù Píng Fēng Săn (Jade Wind-Barrier Powder) [⽟屏风散加味]

7 doses were prescribed. [Formula Analysis] Huáng qí supplements qi and consolidates the exterior. Bái zhú, dăng shēn and shān yào fortify the spleen and boost qi. Duàn mŭ lì and nuò dào gēn astringe the exterior and arrest sweating. Chén pí fortifies the spleen and dissolves phlegm. Wŭ wèi zĭ consolidates the exterior and arrests sweating. Dāng guī and jī xuè téng nourish blood and regulate ying. Jī nèi jīn, chăo gŭ yá and jiāo shān zhā promote appetite and improve digestion.

Chăo yì yĭ rén and fú líng fortify the spleen and remove dampness.

COMMENTARY AND DISCUSSION Recurrent respiratory tract infection often occurs in children with weak constitutions, particularly those with wei qi deficiency. Because wei qi is generated in the spleen, spread by the lung, and rooted in the kidney, strengthening of the lung, spleen and kidney is the key to effective treatment. Common prescriptions include Rén Shēn Bài Dú Săn (Ginseng Toxin-Resolving Powder), Shēn Sū Yĭn (Ginseng and Perilla Beverage), Zài Zào Săn (Renewal Powder), Yù Píng Fēng Săn and so on. Of these prescriptions, Rén Shēn Bài Dú Săn from Key to Diagnosis and Treatment of Children’s Diseases (Xiăo Ér Yào Zhèng Zhí Jué) is particularly designed for children with primary qi deficiency and recurrent exterior patterns; the formula was later applied broadly in adults and the elderly with poor constitutions or chronic disease. Investigations of Medical Formulas (Yī Fāng Kăo) explained that this formula acts to strengthen upright qi and remove pathogenic toxins, thus is called “the powder for overcoming pathogenic influences”. Rather than any of the prescriptions listed above, this case was treated with Huáng Qí Guì Zhī Wŭ Wù Tāng, a prescription from Essentials from the Golden Cabinet (Jīn Guì Yào Lüè) which is said to augment qi, warm and harmonize the channels, and unblock painful obstruction. It is normally indicated for painful obstruction of the blood due to a mild attack of external wind, with symptoms of superficial numbness, muscular lethargy, and a weak choppy and tight pulse.

So why was the child given the modified Huáng Qí Guì Zhī Wŭ Wù Tāng instead of the other more typical representative prescriptions mentioned above? The essential reason is that the pattern diagnosis for this case was disharmony between wei qi and ying qi. Huáng Qí Guì Zhī Wŭ Wù Tāng is derived from Guì Zhī Tāng (Cinnamon Twig Decoction), with the only difference being that in Huáng Qí Guì Zhī Wŭ Wù Tāng, gān căo (Radix et Rhizoma Glycyrrhiza) has been replaced by huáng qí. Wang Mian-zhi said, “The difference between huáng qí and gān căo is that huáng qí augments qi and acts on the area between skin and muscle, while gān căo replenishes qi but works on the heart organ”. Here, huáng qí acts to augment wei qi and stabilize the exterior. Huáng Qí Guì Zhī Wŭ Wù Tāng has a stronger function for strengthening wei qi than does Guì Zhī Tāng, even though the latter is a typical prescription for adjusting wei qi and ying qi in cases of superficial deficiency. Huáng Qí Guì Zhī Wŭ Wù Tāng also includes Mŭ Lì Săn (Oyster Shell Powder) to stabilize the exterior; with the addition of Yù Píng Fēng Kŏu Fú Yè (Jade Wind-Barrier Oral Liquid), the exterior-stabilizing effect is significantly increased to help prevent recurrence. Recurrent respiratory tract infections involve a root deficiency with excess branch manifestations. In the acute stage, one should focus on eliminating the excess pathogens, and in the chronic stage the constitutional deficiency should be strengthened. When a child presents with both excess and deficiency, supplementing and draining should be used simultaneously. In this case, the child had a red pharynx and swelling of the tonsils due to pathogenic toxins lingering, therefore qīng hāo, lián qiào, băn lán gēn, xuán shēn and zhè bèi mŭ were applied to clear lung heat, benefit the throat, dissolve phlegm and reduce swelling.

STUDY QUESTIONS 1. What are the key points in the pattern differentiation of recurrent respiratory tract infection in children? 2. What were the manifestations of lung and spleen deficiency at the second visit?

Answers 1. The key point in pattern differentiation for recurrent respiratory tract infections in children is to differentiate the relative growth and decline of upright qi and pathogenic qi. Pathogenic excess is predominant during the period of infection, where in the lingering period the main pattern is upright qi deficiency with pathogenic qi lingering internally; in the period of recovery, the main pattern is upright qi deficiency. In the period of infection, patients usually contract pathogenic wind; therefore, differentiation must be made between patterns of wind-cold and wind-heat, as well as between exterior and interior patterns. Whether mainly involving root deficiency or branch excess, the pathogenesis of the disease must also be ascertained. In the period of lingering, pathogenic toxins are under control as signs of deficiency appear. With upright qi deficiency predominating over pathogenic qi excess, patterns of lung, spleen and kidney deficiency and impairment must be differentiated.

2. The lung deficiency was manifested by his recurrent contraction of external pathogens, spontaneous sweating, and lingering cough with panting. Spleen deficiency was manifested by the yellowish complexion, poor appetite and loose stools.

CASE SCENARIOS The following cases present variations of this condition. After familiarizing yourself with the possible common pattern presentations and appropriate formulas for treatment, use the following exercises to test your overall understanding of the condition. 1. A patient, age 3, has suffered from the common cold 7 times and severe cough 3 times over the past year. He is thin and has copious phlegm accumulated in his throat. His tongue is light red with a thin white coating. The pulse is floating and rapid. What is the correct diagnosis? A. Common cold B. Cough C. Asthma D. Pneumonia with panting and cough E. Recurrent respiratory tract infection 2. A patient, age 1.5, is a premature infant and the younger brother of a set of twins whose mother has a history of recurrent common cold. Between the third month after his birth and the present, he has contracted a cold 6 times, cough 5 times and pneumonia with panting and cough 2 times. What is the main cause of his condition? A. Congenital insufficiency and constitutional weakness B. Improper breeding and inappropriate care

C. Lack of outdoor activities, unable to bear cold D. Improper medication damaging upright qi E. Upright qi deficiency with lingering pathogens triggering the disease following external contraction 3. A patient, age 2.5, has suffered from recurrent respiratory tract infection about 10 times over the past year. He usually develops aversion to wind, cold sweating, and swollen tonsils. He has a light red tongue with a thin white coating, and purple finger venules. What is the best treatment formula? A. Modified Yín Qiào Săn (Lonicera and Forsythia Powder) B. Modified Sāng Jú Yĭn (Mulberry Leaf and Chrysanthemum Beverage) C. Modified Jīng Fáng Bài Dú Săn (Schizonepeta and Saposhnikovia Toxin-Resolving Powder) D. Modified Huáng Qí Guì Zhī Wŭ Wù Tāng (Astragalus and Cinnamon Twig Five Substances Decoction) E. Modified Yù Píng Fēng Săn (Jade Wind-Barrier Powder) 4. A patient, age 3, is susceptible to cold, catching cold more than 10 times a year, and was diagnosed with recurrent respiratory tract infection. Signs and symptoms include aversion to wind and heat, inability to bear coldness, sweating, a pale complexion, slack muscles, persistently red tonsils, a light red tongue with thin white coating, and a floating weak pulse. What is the presenting pattern?

A. Ying-wei disharmony with lingering pathogenic toxins B. External contraction of wind cold and the failure of lung-wei to diffuse C. External contraction of wind cold and disharmony between ying and wei qi D. Lung-spleen deficiency and qi and blood insufficiency E. Lung qi deficiency and pathogenic toxins lingering 5. A patient, 15 months of age, is unable to walk by himself and has been suffering from recurrent colds. He is thin with a pale complexion. He usually begins to sweat when active, or at night when sleeping. He feels vexing heat in his chest, palms and soles. His hair is thin, and there is eversion at the edge of his ribs. His tongue is red with little coating, and the finger venules are light red. What is the presenting pattern? A. Ying-wei disharmony with lingering pathogenic toxins B. External contraction of wind cold with failure of lung-wei to diffuse C. Lung-spleen deficiency with qi and blood insufficiency D. Kidney deficiency, essence and blood malnourishment, weak bones E. Lung qi deficiency with pathogenic toxins lingering 6. A patient, age 7, has been suffering from recurrent colds with cough and panting. Other signs and symptoms include a yellowish complexion, light-colored lips, aversion to wind, sweating, a poor appetite, slack muscles, loose stools, a light red tongue with a thin white coating, and a moderate weak pulse.

What is the presenting pattern? A. Ying-wei disharmony with lingering pathogenic toxins B. External contraction of wind cold and the failure of lung-wei to diffuse C. Lung-spleen deficiency with qi and blood insufficiency D. Kidney deficiency, essence and blood malnourishment E. Lung qi deficiency with lingering pathogenic toxins 7. A patient, age 8, has been suffering from recurrent cough due to frequent external pathogenic contraction. Over the past year he has contracted colds with coughing about 10 times. Current signs and symptoms include a yellowish complexion, poor appetite, slack muscles, loose stools, cough, sweating, a light red tongue and a weak pulse. What are the correct principles of treatment? A. Reinforce upright qi, fortify the exterior, regulate ying and wei B. Nourish yin, clear heat, supplement lung and kidney C. Fortify the spleen, boost qi, supplement the lung, fortify the exterior D. Supplement the kidney, strengthen bone, nourish yin, warm yang E. Warm the lung, dissipate cold, dissolve phlegm, relieve panting 8. A patient, age 4, has been suffering from recurrent colds with coughing for 2 years, with more than 10 occurrences each year. Over the past week, he has developed coughing, sweating, and a poor appetite with loose stools. Examination shows normal tonsils, light-colored lips, a light red tongue, and a weak rapid pulse. What is the best treatment formula?

A. Supplemented Yù Píng Fēng Săn (Jade Wind-Barrier Powder) B. Supplemented Băo Hé Wán (Harmony-Preserving Pill) C. Supplemented Bŭ Shèn Dì Huáng Wán (Kidney-Supplementing Rehmannia Pill) D. Supplemented Rén Shēn Wŭ Wèi Zĭ Tāng (Ginseng and Schisandra Decoction) E. Supplemented Liù Jūn Zĭ Tāng (Six Gentlemen Decoction) 9. A patient, age 5, has been suffering from recurrent colds with coughing and panting. He usually has a poor appetite, and sweats easily when active and also when sleeping. He sleeps restlessly and feels vexing heat in his chest, palms and soles. He is thin with pigeon-breast and a pale complexion, a red tongue, and a weak pulse. What is the best treatment formula? A. Yù Píng Fēng Săn (Jade Wind-Barrier Powder) B. Sì Jūn Zĭ Tāng (Four Gentlemen Decoction) C. Guì Zhī Tāng (Cinnamon Twig Decoction) D. Guī Pí Tāng (Spleen-Restoring Decoction) E. Bŭ Shèn Dì Huáng Wán (Kidney-Supplementing Rehmannia Pill)

Answers 1. E 2. A 3. D

4. A 5. D 6. C 7. C 8. A 9. E

PART II Spleen Diseases Chapter 7 Thrush Thrush, also known as candidiasis or moniliasis, is an oral disease manifested by white crumbs in the oral cavity or on the surface of tongue, sometimes described as appearing similar to the mouth of a goose. The disease, actually caused by oral inflammation due to candida albicans, is common in newly born infants, children with long course of malnutrition or diarrhea, and children with long-term use of broad-spectrum antibiotics or hormones. Newly born infants primarily get the disease by infection from the birth canal, or from unclean nipples or milk-feeding equipment. Thrush can be differentiated into deficiency and excess patterns. Excess is caused by fetal heat accumulation, an unclean oral cavity infected by turbid filth, or by pathogens accumulated in the heart and spleen. Deficiency is primarily caused by insufficient fetal endowment, such as in premature babies with imperfect growth, tender skin and likely mucocutaneous damage, or by deficiency fire flaming upward through the channels; this occurs in children with disharmony following another disease, or in those with a long course of diarrhea or dysfunction leading to severe fluid damage, spleen and stomach deficiency, and consumption of both qi and yin.

The tongue is the associated sense organ of the heart. The foot shaoyin kidney channel reaches upwardly to the tongue root, and the foot taiyin spleen channel connects with the tongue body and scatters over the bottom. Thus, thrush occurs on the tongue and in the mouth, with its pathological location being the heart, spleen and kidney. The three organs are infected by turbid filth or by deficiency leading to heat with heat-toxin flaming upward through the channels to fumigate and scorch the mouth and tongue. The two common clinical patterns are thrush due to heat accumulation in the heart and spleen, and thrush due to deficiency fire flaming upward.

CLINICAL ESSENTIALS The typical clinical symptom of thrush is white curd-like pieces appearing on the tongue, lips, buccal mucosa, gums and palate which are spotty with small pieces at the beginning, later fusing into layers. The thrush is difficult to wipe off, or results in visible red flesh and rough superficial erosion if wiped off by force. The primary diagnosis can be made from the patient’s age, disease history and typical clinical symptoms; a definite diagnosis can be made from the results of microscopy, in which mold spores and hyphae can be found. It is essential in TCM treatment to first differentiate patterns of deficiency from excess. Cases of excess are manifested by a short course of disease, sudden onset, high temperature, red color of the face, mouth and tongue, more thick white crumbs in the throat and nasal cavity, a slippery and rapid pulse, and purple stagnation in the finger venules. Deficiency cases are manifested by a long course or chronic disease, symptoms of yin deficiency and interior heat, a pale complexion or with red cheeks, no fever or low fever, fewer and scattered white crumbs, a red tongue, a thready weak rapid pulse, and light purple finger venules. Excess patterns are treated primarily by draining heat, and deficiency patterns by nourishing yin. It is also important to distinguish the severity of the disease. Poor appetite, smooth breathing, and a limited range of thrush indicate a mild case. High temperature, a sluggish mind, layers of crumbs over a large range even to the point of congesting the trachea and causing difficulty in breathing, suckling or feeding indicate a severe case.

Cases of excess heat in the heart channel show signs and symptoms of unease, much crying, and scanty dark urine. Cases of excess heat in the spleen channel include dry and hard stools. Heat accumulation in the spleen may cause interior dampness and fluid metabolism disorders with dampness and heat leading to a red tongue with a thick yellow coating, and purple stagnation in the finger venules reaching the wind-pass.

CASE STUDY Female, age 6 months. Initial Visit: April 3rd, 1993 Chief Complaint: Refusing feeding, with excessive salivation for 1 day. History: The child, with a high temperature for 3 days, was diagnosed with an upper respiratory tract infection in the community clinic. His temperature became normal yesterday after antibiotic infusion therapy. Last night the child cried much, feeling uneasy and refusing to be fed. This morning, the child was found to be more uneasy and was salivating more than before, so she was sent to our hospital for treatment. Signs and Symptoms: Mental unease, increased sucking and crying, salivation from the mouth, dark yellow urine, no fever, dry hard stools, a red tongue with a thick yellow coating, and purple stagnation in finger venules reaching the wind-pass. Past History: She had a strong constitution. Physical Examination: BT 37.9℃, the mucocutaneous layer of the oral cavity and tongue was covered with white crumbs that were difficult to clear, the mucocutaneous zone around the mouth was redder and darker than before. Other signs and symptoms included a light red throat, normal breath sounds in both lungs, a normal heart beat with regular rate, no pathologic murmurs, a soft abdomen with no pain when pressing, no swelling of the lower extremities.

Laboratory Examination: An oral swab test indicated mold spores and hyphae.

Pattern Differentiation Physical examination found that the disease was confined to the oropharynx, not involving the cardiopulmonary system or the abdomen. An oral swab test indicated fungal infection in the oral cavity. The child had a high fever resulting from external pathogens and improper nursing. As a total yang constitution, a child is easily attacked by external pathogens which lead to heat, where heat-toxin flames upward through the channels to fumigate and burn the mouth and tongue, causing the white crumbs. The child’s disease belongs to an excess pattern manifested by a large amount of white crumbs in the oral cavity and on the tongue, a dark red mucocutaneous area around the mouth along with tongue and pulse signs of heat accumulation and dampness. As the tongue is the sensory organ of the heart, and the mouth is the orifice of the spleen, the child was diagnosed with thrush due to accumulated heat-toxins in the heart and spleen.

Diagnosis Thrush due to heat accumulation in the heart and spleen

Clinical Treatment Principles: Clear heart heat and drain spleen heat

Formula: Modified Qīng Rè Xiè Pí Săn (Heat-Clearing SpleenDraining Powder) [清热泻脾散加减]

2 doses [Formula Analysis] Huáng qín and zhī zĭ clear excess fire in the heart channel. Huáng lián and shí gāo clear accumulated heat in the spleen channel. Shēng dì huáng clears heat, nourishes yin and cools the blood. Dàn zhú yè and dēng xīn căo clear heat, subdue fire and conduct heat downward. Gān căo regulates and harmonizes all formula medicinals. Dà huáng unblocks the bowels and discharges heat. Huá shí clears heat and removes dampness. Prepared Medicines: Bīng Péng Săn (Borneol and Borax Powder), apply to the oral cavity 3 to 4 times a day. Western Medicine: Wash the mouth with a 3% sodium bicarbonate solution to inhibit mold growth.

Second Visit April 5th, 1993. Discharging the heat-toxins through normal stool, the child recovered to normal breast suckling with a much smaller amount of white crumbs in the oral cavity, and less salivation. Principles: Clear ying level heat, resolve toxins, discharge heat, nourish yin Formula: Modified Qīng Yíng Tāng (Ying Level Heat-Clearing Decoction) [清营汤加减]

3 doses [Formula Analysis] Xuán shēn, shēng dì huáng and mài dōng are sweet-cold medicinals that clear heat and nourish yin. Yín huā and dàn zhú yè clear the heart, resolve toxins and discharge heat externally. Xiān shí hú enriches and nourishes stomach yin and promotes fluid production to quench thirst.

Gān căo harmonizes all the other formula medicinals. The child recovered after taking 3 doses.

COMMENTARY AND DISCUSSION Thrush, as an oral disease, has manifestations occurring in the oral cavity or on the surface of the tongue. In Chinese medicine, the heart opens to the tongue, while the spleen opens to the mouth; therefore, there is no argument about the organs or the treatment principles of clearing and draining toxic-heat from the heart and spleen. In addition to toxic heat, damp-heat and turbid filth are also very important pathogenic factors in this disease. The symptoms include thick and white colored crumbs in the oral cavity or on the surface of the tongue which are defined as a combination of turbid-dampness with toxic heat and fire. It is said that candidiasis and other diseases caused by mold or fungus most likely involve dampness-evil. This knowledge leads to a better understanding of the essential formula and modifications for this case. The basic herbal formula for thrush is Qīng Rè Xiè Pí Săn. This formula is from the Golden Mirror of the Medical Tradition (Yī Zōng Jīn Jiàn) (1742). The author, Wu Qian, said that toxic-fire can accumulate during pregnancy, and that Qīng Rè Xiè Pí Săn drains damp-heat from the spleen and clears toxic-fire from the heart organ. Unlike Xiè Huáng Săn (Yellow-Draining Decoction) and Qīng Wèi Săn (Stomach-Heat-Clearing Powder), which contain fáng fēng (Radix Saposhnikoviae), huò xiāng yè (Folium Agastachis), shēng má (Rhizoma Cimicifugae), etc. for raising and dispersing pathogens, or other heat and fire clearing formulas, Qīng Rè Xiè Pí Săn drains damp-heat and heart fire through urination with zhī zĭ, dēng xīn căo and fú líng, in the original formula. Because dampness tends to move downward, and because heart fire can transmit to the small intestine

and cause urinary disorders, Qīng Rè Xiè Pí Săn follows the pathological tendency and provides ways for the damp-heat and heart fire to leave the body. The practitioner in this case must have a great understanding of this formula. Fú líng was replaced by dàn zhú yè, huá shí, and shēng gān căo in order to strengthen the function of draining damp-heat and heart fire through urination. Shēng dì huáng, dàn zhú yè and shēng gān căo remind us of Dăo Chì Săn (Red-Guiding Powder), a famous formula for clearing heart fire through urination. Creating the treatment strategy in accordance with the pathological tendency of a disease is a very intelligent method of treatment, which can get twice the result with half the effort. The practitioner of this case surely followed this approach.

STUDY QUESTIONS 1. What diseases are similar to thrush, and how can we differentiate between them? 2. How can we differentiate the patterns of deficiency and excess, and how can each be treated?

Answers 1. The key points in learning to treat thrush involve the essential signs and symptoms in clinical diagnosis, the pattern differentiation, and the formulas for treatment. Meanwhile, we must differentiate thrush from diphtheria, which is more common in children between 2 and 6 years of age; the signs and symptoms include denser, pale white crumbs tightly attached to the mucosa, mostly on the throat, though on the root of the tongue or palate in some cases, where peeling by force will cause bleeding. Diphtheria, commonly more severe than thrush, is accompanied with the systemic symptoms of fever and asthenia. Thrush, with whiter crumbs that are easy to peel, lighter systemic symptoms and without fever, is more common in newly born infants, deficient children with a long course of disease, and older children with long-term use of antibiotics. 2. Thrush is not hard to diagnose because it presents with typical clinical symptoms and is common in a certain group. Thus, the key point is to first differentiate deficiency from excess. Judging from local or systemic symptoms, thrush has two patterns. It is either due to heat accumulation in

the heart and spleen, or due to deficiency fire flaming upward. The former pattern, commonly with a short course of disease and in people with strong constitutions, usually has signs and symptoms including fever with red face, lips and tongue, and thicker crumbs even reaching the throat and nasal cavity. The latter pattern is usually manifested by recurrence and a long course of disease with a weak constitution and other signs and symptoms including a pale complexion with red cheeks, no fever or low fever, and fewer and thinner crumbs around the mouth. According to the treatment principle of harmonizing yin and yang by consuming the excess and supplying the deficiency, thrush due to heat accumulation in the heart and spleen should be treated by clearing heart and draining the spleen, resolving toxins and draining fire, with Qīng Rè Xiè Pí Săn, consisting of huáng lián, huáng qín, shān zhī, shēng shí gāo, shēng dì huáng, chì fú líng (Poria Rubra) and dēng xīn căo, etc. Thrush due to deficiency fire flaming upward should be treated by nourishing yin and discharging fire, and guiding fire back to original qi, with Zhī Băi Dì Huáng Wán (Anemarrhena, Phellodendron and Rehmannia Pill), consisting of shú dì huáng (Radix Rehmanniae Praeparata), shān yào (Rhizoma Dioscoreae), shān zhū yú (Fructus Corni), fú líng (Poria), zé xiè (Rhizoma Alismatis), dān pí (Cortex Moutan), zhī mŭ (Rhizoma Anemarrhenae) and huáng băi (Cortex Phellodendri Chinensis), etc. TCM treatment has remarkable effects on the clinical symptoms of thrush.

CASE SCENARIOS The following cases present variations of this condition. After familiarizing yourself with the possible common pattern presentations and appropriate formulas for treatment, use the following exercises to test your overall understanding of the condition. 1. What are the main locations of thrush? A. Heart, liver and spleen B. Heart, lungs and spleen C. Heart, liver and kidney D. Heart, spleen and kidney E. Heart, lungs and kidney 2. A child with thrush has signs and symptoms including crumbs filling the oral cavity and tongue, a dark red color around the mouth, a red complexion, an uneasy disposition, crying when suckling, dry hard stools, scanty dark urine, a red tongue, and purple finger venules. What is the presenting pattern? A. Deficiency fire flaming upward B. Excessive heat in the upper jiao C. Heat accumulation in the heart and spleen D. Excessive heat in the lung and stomach E. Heat and dampness in the spleen and stomach

3. A patient has thin scattered crumbs in the oral cavity and on the tongue, a slightly dark color around the mouth, a pale complexion and red cheeks, hot palms and feet, dry mouth without thirst, a tender red tongue with little coating, and a thready weak rapid pulse. What is the proper treatment formula? A. Wŭ Wèi Huà Dú Dān (Five Ingredients Toxin-Removing Elixir) B. Zhī Băi Dì Huáng Wán (Anemarrhena, Phellodendron and Rehmannia Pill) C. Niú Huáng Jiĕ Dú Wán (Bovine Bezoar Toxin-Resolving Pill) D. Xiăo Ér Huà Dú Dān (Children Toxin-Removing Elixir) E. Qīng Rè Xiè Pí Săn (Heat-Clearing Spleen-Draining Powder)

Answers 1. D 2. C 3. B

Chapter 8 Aphtha Aphtha is an oral disease manifested by yellow or white ulcerations on the surface of oral cavity, lips, tongue, gums and palate, along with pain and salivation, and fever in some cases. Swallow-mouth-like stomatitis refers to cases with ulcers on the sides of the lips and mouth, while aphthous stomatitis refers to the cases with red erosions all over the oral cavity along with pain. The disease is mostly caused by wind-heat attacking the spleen, fetal heat accumulation, improper feeding, and deficiency fire flaming upward. The location of the disease is in the heart, spleen, stomach and kidney, with their channels connecting to the oral cavity. Thus, internal pathogenic infection or interior damage transforms into heat and fire, which flames upward through the channels, fumigating and steaming the mouth and the tongue to cause aphtha. The disease is usually caused by heat and fire, the excess-fire mostly from exterior wind-heat or interior damage from milk feeding; the deficiency-fire is commonly caused by a yin-deficient constitution, yin damage from febrile disease, yang damage following a long course of disease, and deficiency yang floating. Common patterns of the disease include wind-heat attacking the spleen, spleen and stomach heat accumulation, heart fire flaming upward, and floating of deficiency-fire.

CLINICAL ESSENTIALS Since aphtha is primarily caused by fire and heat, eight-principle pattern differentiation is the first priority for determining patterns of excessor deficiency-fire, followed by identification of the disease location according to zang-fu signs and symptoms. The basic principle in treatment of aphtha is to clear heat and subdue fire. Excess patterns should be treated mainly by clearing heat, resolving toxins and draining fire in order to drain and discharge heat downward. Deficiency patterns should be treated by supplementing deficiency with, depending on the signs and symptoms, treatments for nourishing yin, clearing heat and subduing fire, warming and supplementing the spleen and kidney, or guiding deficiency fire to its origin. Pediatric expert Cao Song-zhao makes clearing heart and draining fire the priority in the treatment of aphtha, while also adding medicinals according to various clinical patterns. The selected formula is Dăo Chì Săn (Red-Guiding Powder) with modifications:

● For accumulation, add dà huáng (raw Radix et Rhizoma Rhei) to unblock the bowels and discharge heat.

● For summer aphtha with dampness, add bitter medicinals such as cāng zhú (Rhizoma Atractylodis) and huáng băi (Cortex Phellodendri Chinensis) to dry dampness

● For yin deficiency, add nán shā shēn (Radix Adenophorae), mài dōng (Radix Ophiopogonis), shí hú (Caulis Dendrobii) and zhú yè (Folium Phyllostachydis Henonis) to nourish yin and subdue fire.

● Select modified Jiāo Tài Wán (Peaceful Interaction Pill) to restore interaction between the heart and kidney and guide fire to its origin. Dr. Cao advocates coordinating internal and external treatments by prescribing different powders according to the presenting pattern:

● With a dark red sore surface with pain, select Xī Lèi Săn (Tin-Like Powder) or Zhū Huáng Săn (Pearl and Bezoar Powder).

● With hollow sore surfaces, select Yăng Yīn Shēng Jī Săn (YinNourishing Flesh-Engender Powder).

● With yin deficiency with vigorous fire, select Xū Shí Săn (Deficiency and Excess Powder). Other powdered prescriptions include:

● Grind tiān nán xīng (Rhizoma Arisaematis) 3-6g, dà huáng (Radix et Rhizoma Rhei) 6-9g, huáng lián (Rhizoma Coptidis) 9-12g and wú zhū yú (Fructus Evodiae) 10-15g into a powder. Mix with vinegar to make a sticky paste. Apply to the soles of the feet in the evening, for 12 to 24 hours; two times in total.

● Grind wú zhū yú 15g into a powder, mix with vinegar to make a sticky paste. Apply to bilateral KI 1 (yŏng quán) for 3 consecutive days.

CASE STUDY Female, age 14 months. Initial Visit: January 20th, 1978 Chief Complaint: Pain in the oral cavity and fever for 6 days. History: The child got a fever 6 days ago, with a temperature of about 39 ℃ that was not brought down by Western medicines. The child refuses to be fed and salivates with mouth ulcers and red and swollen gums. Signs and Symptoms: Fever, erosions in the mouth, red swollen gums, vexation and agitation, dry hard stools, dark yellow urine, a red tongue with thin greasy coating, and purple stagnation of the finger venules to the qi pass. Physical Examination: BT 39℃, erosions on the tongue, lips and mouth, red swollen gums, and swollen lymph nodes under the jaw.

Diagnosis Aphtha due to heart and stomach heat flaming upward

Clinical Treatment Principles: Clear the heart and drain fire, clear the stomach, unblock the bowels Formula: Modified Dăo Chì Săn (Red-Guiding Powder)

[导⾚散加减]

3 doses [Formula Analysis] Huáng lián clears and drains heart fire. Zhú yè clears heart and eliminates vexation. Mù tōng conducts heat downward. Gān căo clears heat and resolves toxins. Shēng shí gāo and shēng dà huáng clear and drain stomach heat. Lián qiào and Bì Yù Săn scatter and dissipate heat and clear damp-fire. Prepared Medicines: Apply Bīng Péng Săn (Borneol and Borax Powder) to the oral cavity, 3 times daily. Acupuncture Points: RN 23 (lián quán), ST 6 (jiá chē), LI 4 (hé gŭ), LI 11 (qū chí), HT 5 (tōng lĭ), HT 7 (shén mén), HT 9 (shào chōng), ST 36 (zú sān lĭ). Manipulations: Choose 2 or 3 points, alternately, from those listed. Use perpendicular or oblique insertion to 0.3 to 0.5 cun on facial points. Needle 0.1 cun deep or bleed HT 9 (shào chōng). 0.5 to 1.5 cun perpendicular

insertion on other points with moderate stimulation, retain all needles for 5 to 10 min. Tui Na

● Clear xīn jīng (⼼经) and calm gān jīng (肝经), clear tiān hé shuĭ (天

河⽔), clear xiăo cháng (⼩肠), and knead xiăo tiān xīn (⼩天⼼).

● Clear wèi jīng (胃经), clear băn mén (板门) and push liù fǔ (六腑) downward, clear dà cháng (⼤肠), and push qī jiē gǔ downward (tuī xià qī jiē gǔ, 推下七节骨). Second Visit After taking 3 doses of the prescribed formula, the child had normal stools and no symptoms of fever, oral ulcer or agitation. This indicates that heart fire had been drained and the stomach heat had been cleared, while the dark yellow urine indicates that some heart fire remains. Further treatment is applied to clear the residual heat. Principles: Clear and drain heart heat Formula: Modified Dăo Chì Săn (Red-Guiding Powder) [导⾚散加减]

3 doses

[Formula Analysis] Lú gēn and huā fĕn nourish yin and promote body fluid production to restore fluids damaged in the later period of disease.

Prevention and Nursing Keep the oral cavity clean, and rinse the mouth with warm water after meals and before sleep. Eat more fresh vegetables, fruits and light foods. Avoid excessively hot drinks and milk, and avoid damaging the mucocutaneous tissues, clean the oral cavity gently. Rinse the mouth frequently with a decoction of jīn yín huā (Flos Lonicerae Japonicae), yĕ jú huā (Flos Chrysanthemi Indici), băn lán gēn (Radix Isatidis), dà qīng yè (Folium Isatidis) and gān căo.

COMMENTARY AND DISCUSSION Keeping in mind the differences between heat, fire and toxins as discussed in the comments for tonsillitis, aphtha is most likely due to excessive fire and deficient fire, as this is a localized pediatric problem. It is correctly stressed previously that aphtha is mostly caused by fire and heat, so the first priority is to determine patterns of excess- or deficiency-fire. Internally, excess types of aphtha can be caused by stomach fire or heart fire as the stomach organ connects with the mouth through the channels; heart fire may either flare up and cause ulcerations in the mouth and tongue, or descend to the small intestine and cause urinary disorders such as frequent urgent urination with burning pain. Dăo Chì Săn was applied to clear the heart and to guide out fire through the urine. There are two common prescriptions for clearing and draining stomach fire, Xiè Huáng Săn (Yellow-Draining Powder) and Qīng Wèi Săn (Stomach-Heat-Clearing Powder). Xiè Huáng Săn is a prescription from the Key to Diagnosis and Treatment of Children’s Diseases (Xiăo Ér Yào Zhèng Zhí Jué). It clears smoldering fire from the spleen and stomach and is mainly for aphtha, mouth ulceration, bad breath, and so on. Qīng Wèi Săn has the functions of clearing the stomach and cooling blood and is for toothache, gum bleeding or swelling and painful cheeks, mumps, and so on. The differences between these two prescriptions are that Xiè Huáng Săn combines medicinals for the purposes of clearing, draining, ascending and dispersing, and the focus is on both spleen and stomach. Because of

this, the original name of Xiè Huáng Săn was Xiè Pí Săn (Spleen-Draining Powder). Qīng Wèi Săn focuses on the stomach organ with medicinals for cooling blood and removing toxins. In this case, the chief ingredients of these two prescriptions, shēng shí gāo and huáng lián were applied. To increase the effect of clearing fire from the yangming organ, shēng dà huáng was added for purging the heat accumulation. The topical prescription, Bīng Péng Săn, contains zhū shā (Cinnabaris) and may not be applied in North America.

STUDY QUESTIONS 1. Which zang-fu organs are primarily affected by aphtha? 2. Why is qi and yin emphasized in the development of aphtha?

Answers 1. TCM claims that the oral cavity is closely related with several zangfu organs through their associated channels. The spleen has the mouth as its orifice, and lips as its outward manifestation. The spleen channels are distributed under the tongue. The heart has the tongue as its orifice, and the heart channels are distributed on the tongue. Kidney channels relate with the throat and tongue, and cheeks and gums connect with the stomach and large intestine. Teeth are ascribed to the kidneys. Channels such as the ren mai and du mai connect with the oral cavity, lips and tongue. Therefore, aphtha indicates local disease in the oral cavity, while the location of the disease also involves the heart, spleen, stomach and kidneys. Regardless of external contraction of pathogens, internal food damage or upright qi deficiency, aphtha is closely related to the disorders of these four zang-fu organs. 2. Fire and heat is the inevitable mechanism for childhood aphtha, which is likely to consume yin-fluids. Thus, it is important to emphasize the growth and decline of qi and yin in the development of the disease. The pattern of excess fire may turn into a pattern of deficiency-fire if the child is not treated, or is treated incorrectly. If the deficiency fire is not removed

then damage to qi and yin is likely, and infection by internal pathogens leads to a deficiency-excess complex pattern. The long course of yin deficiency may cause damage to qi as well as yin; the deficiency of both qi and yin may then cause the aphtha to become chronic.

CASE SCENARIOS The following cases present variations of this condition. After familiarizing yourself with the possible common pattern presentations and appropriate formulas for treatment, use the following exercises to test your overall understanding of the condition. 1. A child has ulcers on the surface of the oral cavity, lips, tongue, gums and palate. Other signs and symptoms include pain around the ulcers, red and dark ulcers, annoyance, unease, refusal to eat, halitosis, much salivation, constipation, yellow urine, fever, red throat, a red tongue with thin yellow tongue coating, and a wiry rapid pulse. What is the presenting pattern? A. Wind-heat attacking the spleen B. Heat accumulation in the heart and spleen C. Heart fire flaming upward D. Floating of deficiency fire E. Excessive heat in the sanjiao 2. A one year-old child has 7 or 8 rice-sized ulcers on the surface of her oral cavity, lips, tongue, gums and palate, along with pain, unease and much crying. What is the correct diagnosis? A. Thrush B. Perleche

C. Aphtha D. Severe aphtha E. None of the above 3. A child, age 2, had many red and dark ulcers on his gums and oral cavity for one day; there is halitosis, much salivation, fever, and a red tongue with a yellow coating. What is the presenting pattern? A. Heart fire flaming upward B. Wind-heat attacking the spleen C. Heat accumulation in the heart and spleen D. Floating of deficiency fire E. Liver-gallbladder damp-heat 4. A child, age 4, played outside yesterday, then dark-colored ulcers were found on the tip and sides of the tongue. Other signs and symptoms included pain, difficulty in eating, annoyance, unease, thirst, scanty dark urine, a red tongue tip, and a thin yellow tongue coating. What is the best treatment approach? A. Scatter wind, disperse fire, clear heat, resolve toxins B. Nourish yin, subdue fire, guide fire to its origin C. Clear heart, cool blood, drain fire, resolve toxins D. Scatter wind, release the exterior, drain fire, resolve toxins E. Promote digestion, remove stagnation, clear heat, resolve toxins

5. A child, age 3, has a slim body and occasional tongue ulcers that are not red in color. There is tiredness, red cheeks, little pain, no thirst, and a red tongue with little coating. What is the proper formula? A. Shā Shēn Mài Dōng Tāng (Adenophora and Ophiopogon Decoction) B. Yăng Wèi Zēng Yè Tāng (Stomach-Nourishing Humor-Increasing Decoction) C. Yì Huáng Săn (Yellow-Transforming Powder) D. Liù Wèi Dì Huáng Wán (Six-Ingredient Rehmannia Pill), with added ròu guì (Cortex Cinnamomi) E. Zhī Băi Dì Huáng Wán (Anemarrhena, Phellodendron and Rehmannia Pill), with added fù zĭ (Radix Aconiti Lateralis Praeparata)

Answers 1. A 2. C 3. C 4. C 5. D

Chapter 9 Vomiting Vomiting in children is a disease manifested by the upward reversal of food or milk from the stomach to the mouth, which is caused by the loss of harmony of the stomach, stomach qi failing to descend, and ascending counterflow of qi. The condition may affect children of all ages, but clinical cases typically involve infants or younger children. Common clinical causes of vomiting include external pathogens damaging the stomach, accumulation and stagnation of milk and food, accumulation of heat in the stomach, cold and deficiency in the stomach and spleen, and liver qi invading the stomach. These problems impact stomach function, causing ascending counterflow of stomach qi, which in turn causes vomiting. The location of the disease is the stomach, and it is closely related to the liver and spleen. The foundational mechanisms involve loss of stomach harmony with qi failing to descend, and ascending counterflow of qi.

CLINICAL ESSENTIALS This disease is easily diagnosed by its main clinical manifestation of upward reversal of milk or food from the stomach into the mouth, along with general symptoms including belching, acid regurgitation, nausea, low spirits, and bloated gastric cavity and abdomen; usually symptoms of diarrhea, fever, or tiredness are absent. The basic treatment principles are to harmonize the stomach and descend counterflow qi. Clinical diagnosis and treatment must first differentiate the signs and symptoms into patterns of excess or deficiency. The acute nature and short course of the disease indicate an excess pattern, clinically manifested by exuberance of the pathogen with no deficiency of upright qi. A deficiency pattern is manifested by the moderate nature of the disease and its presence over a long course, along with a resultant upright qi deficiency. Excess patterns may be further differentiated into patterns of external pathogens damaging the stomach, accumulation and stagnation of food, heat in the stomach, and liver qi invading the stomach.

● External pathogens damaging the stomach manifests with vomiting with concurrent fever, sensitivity to cold, a light red tongue with a thin white coating, and a floating and tight pulse. The pattern is treated by releasing the exterior and harmonizing the stomach.

● Accumulation and stagnation of food, usually associated with an unrestrained diet, manifests with sour vomitus, a red tongue with a thick greasy coating, a rapid and slippery pulse, and purple finger venules. The pattern is treated by promoting the digestion of food and milk.

● Heat in the stomach manifests with frequent vomiting and immediate vomiting after feeding, along with the signs and symptoms of blazing interior heat such as thirst, a hot body, sour or dry stools, difficult urination and dark urine, a red tongue with a yellow coating, and a rapid pulse. The pattern is treated by clearing stomach heat and draining stomach fire.

● Liver qi invading the stomach is due to emotional factors and manifests with belching, acid regurgitation and vomiting along with discomfort in the chest, bloated abdomen, tendency to anger, frequent crying, a red tongue, and a wiry pulse. The pattern is treated by soothing the liver.

● Deficiency cold of the spleen and stomach manifests with vomitus containing undigested food soon after meals, along with signs and symptoms of yang deficiency and cold such as a pale complexion, cold limbs, a pale tongue with a white coating, and a moderate and weak pulse. The pattern is treated by warming the middle and dissipating cold. Chen Xiu-yuan suggests that vomiting should generally be treated with Èr Chén Tāng (Two Matured Substances Decoction) with added huò xiāng (Herba Agastachis) and shā rén (Fructus Amomi), also modifying the formula according to patterns of deficiency or excess, cold or heat, and exterior or interior, as follows:

● Wú zhū yú (Fructus Evodiae), dīng xiāng (Flos Caryophylli), and gān jiāng (Rhizoma Zingiberis) should be added to the formula for cold symptoms.

● Huáng lián (Rhizoma Coptidis), huáng qín (Radix Scutellariae), mài dōng (Radix Ophiopogonis), shā shēn (Radix Adenophorae seu Glehniae), and zhú rú (Caulis Bambusae in Taenia) should be added to generate fluid and clear heat.

● Hòu pò (Cortex Magnoliae Officinalis), shān zhā (Fructus Crataegi), shén qū (Massa Medicata Fermentata), and mài yá (Fructus Hordei Germinatus) should be added for excess signs and symptoms.

● Huáng qí (Radix Astragali) and rén shēn (Radix et Rhizoma Ginseng) should be added for deficiency signs and symptoms.

● Cases of vomiting caused by wind-cold, ingesting cold food, and nighttime milk feeding should be treated with Dìng Tù Zĭ Jīn Hé (VomitingArresting Purple Gold Pit).

● Cases of heat vomiting caused by latent summerheat should be treated initially with Xiāng Rú Săn (Mosla Powder), and later with modified Zhú Rú Tāng (Bamboo Shavings Decoction) along with Fú Líng Hòu Pò Tāng (Poria and Officinal Magnolia Bark Decoction) if symptoms of catching a cold and cough are present. Attention must be paid to the conversion between deficiency and excess and cold and heat to prevent a transmuted pattern of the disease. Failure in the treatment of excess may convert into deficiency, which may lead to a deficiency-excess complex due to external pathogens, food stagnation, and qi constraint. Conversely, a long course of cold vomiting may convert into heat vomiting, the long course of which may damage yang and cause cold and heat in complexity. Additionally, since children’s constitutions are “tender yin and tender yang”, violent vomiting may cause significant fluid damage and qi desertion following humor (thick fluid) desertion. This in turn leads to yin exhaustion and yang desertion, and further transmuted patterns of reversal counterflow and desertion. Also, a long course of vomiting may cause spleen and stomach deficiency and damage with consumption of qi and blood, which, if prolonged, can cause infantile malnutrition.

Vomiting manifests in many diseases in Western medicine. These include digestive system disorders, acute or chronic gastritis, and peptic ulcer. Vomiting also manifests in diseases of other systems such as acute upper respiratory tract infections, and various forms of encephalitis.

CASE STUDY I Male, age 11 months. Initial Visit: July 15th, 1995 Chief Complaint: Vomiting for 3 days. History: Over the last 3 days, the child vomited 3 to 5 times a day with no known cause, with no spraying of vomited stomach contents after feeding, and without symptoms of fever, diarrhea or convulsion. His parents gave him Huò Xiāng Zhèng Qì Yè (Agastache Qi-Correcting Oral Liquid) to no effect, and sent him to the hospital for further treatment. Signs and Symptoms: Vomiting, vomiting right after feeding, sour vomitus, thirst, desire for cold drinks, fever, annoyance, dry hard stools, and scanty dark urine. His tongue was red with a yellow coating. He had purple finger venules. Past History: The child had a strong constitution. Physical Examination: BT 38℃, weight 10 kg. Normal development in body and nutrition, a clear mind and normal spirit, plain and soft fontanel, crying with tears, normal elasticity in skin, no disorder in cardiopulmonary examination, soft abdomen, active bowel sounds, soft cervical vertebra, negative Kernigʹs sign and Brudzinskiʹs sign. Laboratory Examination: WBC 5.8×109/L, N 0.74, L 0.26, Hb 112 g/L, PLT 206×1012/L.

Pattern Differentiation

The child in this case should be diagnosed with vomiting since he is an infant with an acute condition manifested mainly by vomiting with no spray containing stomach contents, and he is without fever, diarrhea, or convulsion. Due to the short course of the disease and no deficiency of upright qi, this case should be diagnosed as an excess pattern. The immediate vomiting after feeding, sour vomitus, and other accompanying signs and symptoms show the heat nature of the disease and the likely location of the stomach. The child had an unrestrained diet where food transformed into heat accumulation in the stomach, leading to stomach fire flaming upward. This is why the child vomited with sour odor immediately after meals. The heat accumulated in the stomach and consumed fluids, so the child felt thirsty. The interior heat led to a hot body and irritation. His dry hard stools, scanty dark urine, red tongue with a yellow coating, and purple finger venules all indicate stomach heat. Vomiting caused by blazing interior heat is characterized by immediate vomiting after meals, frequent vomiting, vomitus with odor, and other signs of exuberant interior heat.

Diagnosis Vomiting due to stomach heat and upward reversal of qi

Clinical Treatment Principles: Clear heat and drain fire, harmonize the stomach, descend upward reversal of qi

Formula: Modified Huáng Lián Wēn Dăn Tāng (Coptis GallbladderWarming Decoction) [黄连温胆汤加减]

3 doses

CASE STUDY II Female, age 4. Initial Visit: February 21, 1996 Chief Complaint: Vomiting for 2 days. History: The child had an unrestrained diet during the Chinese Spring Festival and has been vomiting for the last 2 days. Associated signs and symptoms included vomiting 3 or 4 times a day, vomitus containing undigested foot with a sour odor, halitosis, no appetite, bloated gastric cavity and abdomen, and a thin stool once a day with a very sour odor. She had no fever, headache, nor abdominal pain. Her tongue was red with a thick greasy coating; pulses were rapid and slippery. The child’s vomiting got better after taking shén qū (Massa Medicata Fermentata) boiled by her parents; she was then sent to the hospital for further treatment. Past History: Good general health and a strong constitution. Physical Examination: BT 36.8 ℃, weight 15 kg. Normal development in body and nutrition, clear mind and normal spirit, normal skin elasticity, cardiopulmonary examination normal, a soft abdomen without pain when pressed, active bowel sounds, soft cervical vertebra, negative Kernigʹs sign and Brudzinskiʹs sign. Laboratory Examination: Routine stool test showed no abnormity.

Pattern Differentiation

Due to the short course of disease, and because there is no obvious deficiency of upright qi, this is an excess pattern of acute vomiting; supporting signs and symptoms include vomitus containing undigested food, no appetite for milk, halitosis, a bloated gastric cavity and abdomen, thin stools with sour odor.

Diagnosis Vomiting due to food accumulation and milk stagnation in the stomach

Clinical Treatment Principles: Promote digestion, harmonize the stomach, descend counterflow qi Formula: Modified Băo Hé Wán (Harmony-Preserving Pill) [保和丸加减]

3 doses [Formula Analysis] Jiāo shān zhā, jiāo shén qū, and jī nèi jīn promote digestion and remove accumulation and stagnation.

Lái fú zĭ, chén pí, and fă bàn xià rectify qi and descend counterflow to stop vomiting. Fú líng rectifies the spleen and percolates dampness. Lián qiào clears constrained heat and rectifies qi. Dà fù pí moves stagnation and eliminates bloating. Gān căo regulates and harmonizes all formula medicinals. Tui Na: Arc-push băn mén (板门), arc-push nèi bā guà (内八卦), knead RN 12 (zhōng wăn), separating-pushing abdomen (fēn tuī fù yīn yáng, 分推腹阴阳), knead ST 36 (zú sān lĭ), push from héng wén (横纹) to băn mén (板门), rub abdomen clockwise. Second Visit The child vomited less after the first treatment and had normal stools, while the symptoms of poor appetite and occasional bloated abdomen continued. She was prescribed further treatment with Băo Hé Wán taken orally 2 times a day, along with Jīn Shuāng Qí (Live Combined Bifidobacterium and Lactobacillus Tablets) to supply probiotics to the intestines and promote digestion. The child recovered 4 days later.

Prevention and Nursing It’s advisable to feed the child with no rich food, but with rice porridge for the purpose of protecting stomach yin.

COMMENTARY AND DISCUSSION Because infantile diseases are caught easily and transmitted quickly, proper clinical identification and differential diagnosis are critical. This is particularly true of vomiting, as vomiting may be an early sign of acute infectious diseases including epidemic encephalitis and encephalitis B. In these two cases, we saw that physical assessments had been done to check the fontanel, cervical vertebra, and Kernigʹs sign and Brudzinskiʹs sign. These tests can screen for meningeal irritation, a symptom of which is vomiting. Such screening is of utmost importance, because if epidemic encephalitis and encephalitis B are not treated promptly, they can be fatal. Vomiting may also be a sign of acute abdominal diseases including intestinal obstruction and intussusception. Therefore, checking bowel sound and examining the abdomen are also important. We saw that both cases had negative results for the abdominal examination. Utilizing a popular home remedy, the parent in the first case fed the child Huò Xiāng Zhèng Qì Yè. This formula has the functions of releasing the exterior, transforming dampness, regulating qi, and harmonizing the middle jiao for externally contracted wind-cold with concurrent internal injury due to stagnation. There was no improvement, which indicates that the vomiting was not cold-type vomiting. This was further proven by the later-shown symptoms of “thirst, desire for cold drinks, immediate vomiting after feeding, sour vomitus, fever, annoyance, dry hard stools, scanty dark urine, a red tongue with yellow coating, and purple finger venules”.

Modified Huáng Lián Wēn Dăn Tāng was then prescribed. Even though there were no follow-up visits, we can assume that the case resolved quickly, as infantile diseases can be easily and quickly cured due to children’s clear and healthy visceral qi. The child in the second case had a history of overeating, and she got the disease during the Chinese Spring Festival. Thus, we can presume food stagnation, and the symptoms of “vomiting with undigested food [and] thin stool with sour odor” prove this presumption. Modified Băo Hé Wán was given to promote digestion and remove accumulation while descending counterflow qi and harmonizing the stomach. When a medicinal prescription matches the presenting pattern, one can expect great results in a very short period of time; this is particularly true of infantile diseases. Vomiting over a longer time period may result in dehydration. This is considered as a body fluid and qi deficiency, as body fluids carry qi. In this situation, Jú Pí Zhú Rú Tāng (Tangerine Peel and Bamboo Shavings Decoction) from Formulas to Aid the Living (Jì Shēng Fāng) may be prescribed to supplement qi and nourish yin, as rén shēn and mài dōng are both included in the prescription. It is interesting to note that these three herbal prescriptions contain the formula Èr Chén Tāng. A famous herbalist at the Beijing University of Chinese Medicine, Prof. Wang Mian-zhi, holds that Èr Chén Tāng is derived from Wēn Dăn Tāng (Gallbladder-Warming Decoction), a formula in Sun Si-miao’s Important Formulas Worth a Thousand Gold Pieces for Emergency (Bèi Jí Qiān Jīn Yào Fāng). As a basic prescription to dry dampness, transform phlegm, regulate qi, and harmonize the middle jiao, Èr

Chén Tāng is commonly applied for lung and stomach problems with qi counterflow and damp-phlegm retention. Chen Xiu-yuan also believed that the application of Èr Chén Tāng was an intelligent treatment for the general treatment of vomiting, and that with modification it can be applied for cold-, heat-, excess-, and deficiency-types of vomiting. The modification of medicinal prescriptions is an art which can verify one’s overall knowledge of Chinese medicine.

STUDY QUESTIONS 1. What are the features of their clinical manifestations respectively because there are different factors to vomiting? 2. Illustrate the principles in treating vomiting.

Answers 1. The pathogenesis of vomiting is the loss of harmony and failure of qi descending in the stomach, so treatment should be based on harmonizing the stomach and descending the counterflow. Vomiting should be cured naturally by harmonizing the stomach and descending qi from the differentiation of cold or heat, deficiency or excess and exterior or interior cause. It’s necessary to learn about the similarities and differences in various patterns of vomiting. Take the two patterns for example, the pattern of deficiency and cold in spleen and stomach and the pattern of external wind-cold for example, they are different by deficiency and excess though sharing the similar cold signs and symptoms. Vomiting due to external wind-cold is caused by its invading the internal organs such as stomach, which is manifested by the symptoms of lung-wei pattern, such as sudden vomiting, clear and cold vomitus, fever and fear for cold, thus, it belongs to the excess pattern; while vomiting due to deficiency and cold in spleen and stomach is mainly caused by indigestion of milk and food or by the long accumulation of food, arising mainly from the insufficient granary, serious disease or long course of disease, deficiency and cold in spleen and stomach, insufficient center yang or failure of descending of stomach qi,

which is manifested by the symptoms of general deficiency and cold and poor center yang, such as vomiting a while after meals, thin and clear vomitus, light in color and without any taste, vomitus containing undigested food, thus, it belongs to the pattern of deficiency. 2. The pathogenesis of vomiting is the loss of harmony and failure of qi descending in the stomach, treated under the basic principle of harmonizing the stomach and descending the counterflow. The key point in diagnosis is to differentiate deficiency from excess, and cold from heat, and give treatment according to their pattern, with the basic principle of harmonizing the stomach and descending the counterflow.

CASE SCENARIOS The following cases present variations of this condition. After familiarizing yourself with the possible common pattern presentations and appropriate formulas for treatment, use the following exercises to test your overall understanding of the condition. 1. A child, age 2, vomited occasionally over the past 2 days, along with rotten vomitus with sour smell, dry hard stools, uneasiness and much crying, 37.6 ℃ in temperature, a red tongue with a thick greasy coating, a bloated abdomen aggravated by pressure, and purple and stagnated finger venules. What is the most likely pattern? A. The excess pattern of yangming bowel syndrome B. Externally-contracted wind-cold C. Stomach heat and qi counterflow D. Accumulation and stagnation of milk and food E. Liver qi invading the stomach 2. A child, age 8, had too many snacks at a picnic today and had sudden pain in his abdomen on the way home; other signs and symptoms include a bloated gastric cavity and abdomen, frequent vomiting with a strong odor, feeling better after vomiting, and a red tongue with yellow coating. What is the pattern? A. Upward reversal of gallbladder fire B. Accumulation and stagnation of milk and food

C. Stomach heat and qi counterflow D. Deficiency and cold of the spleen and stomach E. Liver qi invading the stomach 3. A child, age 6, who is fond of food with much fat and sugar in it, had repeated vomiting in the past 2 weeks, immediate vomiting after meals, frequent belching with sour smell. He got no better after taking medicines for promoting digestion, removing stagnation, and stopping vomiting. Now, he vomits or retches, dry mouth and throat, much noise in the stomach cavity, dysphoria, tidal fever, red lips and cheeks, a red tongue with light coating, and a rapid and thready pulse. What is the proper formula for his symptoms? A. Lián Pò Yĭn (Coptis and Officinal Magnolia Bark Beverage) B. Băo Hé Wán (Harmony-Preserving Pill) C. Xuán Fù Dài Zhĕ Tāng (Inula and Hematite Decoction) D. Jú Pí Zhú Rú Tāng (Tangerine Peel and Bamboo Shavings Decoction) E. Huáng Lián Wēn Dăn Tāng (Coptis Gallbladder-Warming Decoction) 4. A child, age 7, had repeated and irregular vomiting in the past 3 months, along with thin clear vomitus, emaciation, cold pain in the stomach cavity, discomfort in the abdomen, weak breathing, unwilling to talk, sloppy stools, a light red tongue with white coating, and a slow and moderate pulse. Which of the following is the best formula to modify? A. Lĭ Zhōng Wán (Center-Regulating Pill) B. Dīng Yú Lĭ Zhōng Tāng (Clove and Evodia Center-Regulating Pill)

C. Xiăo Jiàn Zhōng Tāng (Minor Center-Fortifying Decoction) D. Dāng Guī Jiàn Zhōng Tāng (Chinese Angelica Center-Fortifying Decoction) E. Wú Zhū Yú Tāng (Evodia Decoction) 5. A child, age 4, vomited one time after arguing with his peer and being blamed, vomited 2 times again that night. He was sent to the hospital for treatment, and his parent told the doctor his symptoms included depression, no desire for communication, occasional belching, poor appetite, and vomiting the contents of his stomach with little odor. He had a red tongue with a thin greasy coating, and wiry pulse. What is the most likely pattern of his vomiting? A. Vomiting due to indigestion B. Vomiting due to fright C. Vomiting due to stomach cold D. Vomiting due to stomach heat E. Liver qi invading the stomach

Answers 1. C 2. B 3. D 4. B 5. E

Chapter 10 Abdominal Pain Abdominal pain is a common disease in young children, manifested mainly by pain around the navel or pain below the stomach cavity and above the pubis. Pain below the stomach cavity and above the navel is classified as pain in the upper abdomen; in the center part of the abdomen below the navel is classified as pain in the lower abdomen; pain around the navel, at either or both sides of the lower abdomen is classified as pain in the lesser abdomen. Abdominal pain has five common causes: pathogenic cold invasion, accumulation and stagnation of milk and food, heat accumulation in the stomach and intestines, deficiency cold in the zang-fu, and qi stagnation and blood stasis. The location of disease primarily involves the liver, spleen, the six bowels and the associated channels. Pathogenic attack on the zang-fu organs or channels may cause zang-fu function disorders which obstruct the flow of channel qi and cause qi constraint, eventually resulting in abdominal pain.

CLINICAL ESSENTIALS Since there are many reasons for children’s abdominal pain, to make an immediate and accurate diagnosis, we must know certain details such as the age of the child, occurrence, development and changes of his or her pain, the location, nature, duration and degree of the pain, and also the pain’s cause and accompanying symptoms. This unit discusses children’s abdominal pain without acute surgical indications. TCM commonly differentiates such cases as patterns of deficiency, excess, heat, cold, qi, blood, food, and parasites. The mechanism of abdominal pain usually involves qi stagnation and stoppage, as it is said that when there is stoppage, there is pain; the long course of pain may also lead to generation of stasis. In terms of the nature of pain, heavy pain indicates a pattern of excess, while persistent pain indicates a pattern of deficiency. Severe pain aggravated by pressure indicates a pattern of excess, while dull pain moderated by pressure indicates a pattern of deficiency. Pain that becomes more severe after meals indicates a pattern of excess, while pain moderated after meals indicates a pattern of deficiency. Pain worsened by a hot compress indicates a pattern of excess, while pain moderated by a hot compress indicates a pattern of deficiency. Wandering and migratory pain indicates a pattern of excess, while fixed stabbing pain indicates a pattern of blood stasis. Wang Jing-an, the famous expert of Chinese medical pediatrics, points out that cases of cold and excess are more common than cases of deficiency

and heat. Abdominal pain should be treated with cold and warmth at the same time, using a basic formula containing sū gĕng (Caulis Perillae), chén xiāng (Lignum Aquilariae Resinatum), guăng mù xiāng (Radix Aucklandiae), tán xiāng (Lignum Santali Albi), chăo xiāng fù (dry-fried Rhizoma Cyperi), and huáng lián (Rhizoma Coptidis). This basic formula consists mostly of acrid-warm medicinals with a paradoxical assistant—the bitter-cold huáng lián; with acrid medicinals to open and bitter medicinals to promote descent, this formula use warm and cold medicinals simultaneously to nourish the stomach and clear heat.

● Abdominal pain due to cold accumulation should be treated with Sàn Hán Zhĭ Tòng Tāng (Cold-Dissipating Pain-Relieving Decoction) with liáng jiāng (Rhizoma Alpiniae Officinari), bái dòu kòu (Fructus Amomi Kravanh) and chén pí (Pericarpium Citri Reticulatae) added to the basic formula.

● Abdominal pain due to food accumulation should be treated with Xiāo Shí Zhĭ Tòng Tāng (Digestion-Promoting Pain-Relieving Decoction) with shān zhā (Fructus Crataegi), shén qū (Massa Medica Fermentata), gŭ yá (Fructus Setariae Germinatus) and bīng láng (Semen Arecae) added to the basic formula.

● Abdominal pain due to deficiency and cold should be treated with Jiàn Zhōng Zhĭ Tòng Tāng (Center-Fortifying Pain-Relieving Decoction) with yí táng (Saccharum Granorum), gŭ yá, mài yá (Fructus Hordei Germinatus), bái dòu kòu, and bái zhú (Rhizoma Atractylodis Macrocephalae) added to the basic formula.

● Abdominal pain due to parasite accumulation should be treated with Ān Huí Zhĭ Tòng Tāng (Roundworm-Calming Pain-Relieving Decoction)

with wū méi (Fructus Mume), xì xīn (Radix et Rhizoma Asari), liáng jiāng, mù tōng (Caulis Akebiae), chuān liàn zĭ (Fructus Toosendan), bīng láng, wú zhū yú (Fructus Evodiae), xuán fù huā (Flos Inulae), dāng guī (Radix Angelicae Sinensis), and chuān jiāo (Pericarpium Zanthoxyli) added to the basic formula. Some practitioners treat abdominal pain using points from Master Tong’s Extra Points; these points often show a complete and immediate effect. Insert needles until the arrival of qi, usually 1.7 to 5 cm. Needling manipulation is unnecessary. Retain all needles for 20 to 30 min.

CASE STUDY Male, age 3. Initial Visit: January 16th, 2003 Chief Complaint: Occasional abdominal pain for the last 2 months. History: The child began having occasional abdominal pain of unknown origin 2 months ago. Since then he has had a poor appetite, mental fatigue, cold hands and feet, and uneasy sleep. The abdominal pain was not sharp, and was sometimes moderated by massaging the abdomen. Signs and Symptoms: A pale complexion, poor appetite, and unformed stools. His tongue was light red with a white coating; pulses were moderate. Past History: The child was born premature, and therefore had a weak constitution. No information was available about a history of allergies to medicines or food, vaccination history was normal. Physical Examination: BT 36.8 ℃, HR 102 beats/min, RR 25 times/min, weight 13 kg. Soft abdomen, thin subcutaneous fat less than 0.6 cm, no shifting dullness, liver and spleen not palpable. Laboratory Examination: WBC 8.2×109/L, N 0.63, L 0.25, Hb 130g/L, PLT 206×1012/L.

Pattern Differentiation Because of the child’s weak constitution and the long course of disease, his abdominal pain is due to deficiency. In winter, instead of

ascending, yang qi is hibernated, which leads to a general deficiency of yang qi in the human body. Meanwhile, as spleen yang is not warmed, there is indigestion of water and grain, inhibited blood and qi movement, and spasms of blood vessels and channels. These are causative factors for the child’s abdominal pain and the associated signs and symptoms. The nature of the pain also suggests spleen yang deficiency. His tongue and pulse both indicate deficiency cold of the spleen and stomach.

Diagnosis Abdominal pain due to spleen and stomach deficiency cold

Clinical Treatment Principles: Warm the middle, regulate the spleen, moderate and relieve pain Formula: Modified Xiăo Jiàn Zhōng Tāng (Minor Center-Fortifying Decoction) and Lĭ Zhōng Wán (Center-Regulating Pill) [⼩建中汤合理中丸加减]

14 doses [Formula Analysis] Guì zhī, gān jiāng, and yí táng warm and promote middle yang. Dăng shēn, bái zhú, and dà zăo rectify the spleen and boost qi. Bái sháo and yí táng moderate and relieve pain. Gān căo regulates and harmonizes all the other medicinals. Prepared Medicines: Fù Zĭ Lĭ Zhōng Wán (Aconite CenterRegulating Pill), 2-3g, 2-3 times a day. Acupuncture Points: ST 36 (zú sān lĭ), LI 4 (hé gŭ), RN 12 (zhōng wăn), BL 20 (pí shù), BL 21 (wèi shù). Manipulations: Insert the needle quickly, twirl or lift and thrust with supplementation. Retain for 15 min or until the stomach pain resolves. Second Visit The abdominal pains had stopped, the child‘s spirit was improved and he slept better at night. After 2 weeks of medicinal treatment, the child’s symptoms were alleviated and the deficiency signs of the spleen and stomach were improved. Therefore, the main issue is damage to the spleen and stomach. Principles: Regulate the spleen and promote transportation Formula: Modified Liù Jūn Zĭ Tāng (Six Gentlemen Decoction) [六君⼦汤加减]

7 doses [Formula Analysis] Dăng shēn, shān yào, and bái zhú boost qi and rectify the spleen. Mài yá, shén qū, and shān zhā promote digestion and help transportation. Chén pí moves qi and makes the supplementing medicinals act to supplement without causing stagnation. Gān căo regulates and harmonizes all formula medicinals.

Prevention and Nursing The key in prevention is to eat clean foods, avoid uncooked foods, and to avoid unrestrained meals. Meanwhile, keep the abdomen warm, and avoid strenuous exercise immediately after meals. Patients with abdominal pain due to deficiency cold should have warm, sweet foods, and drink warm or hot liquids in small amounts over several times if vomiting is a problem.

COMMENTARY AND DISCUSSION Abdominal pain is present in many diseases, including acute abdomen and a group of emergent abdominal diseases, such as appendicitis, that can be fatal. To rule out acute abdomen, special attention must be paid when palpating the abdomen for abdominal sound, tenderness, rebound tenderness, and muscle tightness. Treatment to the exclusion of acute abdomen, when it has not been ruled out, may result in complications or increased severity of the disease. Because children are usually unable to precisely describe the location, character and severity of abdominal pain, a practitioner must collect clinical data carefully through physical assessment and the four traditional diagnostic methods to make an accurate diagnosis. Acute abdomen can be ruled out in this case, because the abdomen is “flat and soft”, no dynamic murmur is heard, and the liver and spleen are not readily palpable. On the first visit, two prescriptions from the Treatise on Cold Damage (Shāng Hán Lùn) were combined for this patient. Xiăo Jiàn Zhōng Tāng warms and supplements the middle jiao and moderates spasmodic abdominal pain. It also soothes the liver and relaxes spasms, as it includes sháo yào (Radix Paeoniae) and gān căo. The Treatise on Cold Damage states, “Cold attack with a hesitant yang pulse and a wiry yin pulse may include spasmodic abdominal pain. Xiăo Jiàn Zhōng Tāng should be prescribed. If the disease persists, Xiăo Chái Hú Tāng (Minor Bupleurum Decoction) should then be prescribed” (伤寒,阳 脉涩,阴脉弦,法当腹中急痛,先与⼩建中汤,不瘥者,⼩柴胡汤主

之). The mention of Xiăo Chái Hú Tāng here clearly implies that spasmodic abdominal pain involves the liver. Zhang Zhong-jing also said, “Cold attack for more than 2-3 days with palpitation and restlessness should be prescribed Xiăo Jiàn Zhōng Tāng” (伤寒⼆三⽇,⼼中悸⽽烦者,⼩建中汤主之). Thus, Xiăo Jiàn Zhōng Tāng is also indicated in situations in which yin and yang are both deficient. In his other classic text Essentials from the Golden Cabinet (Jīn Guì Yào Lüè), Zhang Zhong-jing also applies Xiăo Jiàn Zhōng Tāng for deficiency taxation, jaundice, and women’s abdominal pain, all of which are defined as deficiency patterns. It should be noted that the ingredients in Xiăo Jiàn Zhōng Tāng are mostly the same as those in Guì Zhī Tāng (Cinnamon Twig), but the dosage of sháo yào (Radix Paeoniae) is doubled with the addition of yí táng. Clearly, when the dosages of ingredients change within a prescription, the functions, indications, and name of the prescription may also change. Because the child in this case was diagnosed with deficiency cold in the spleen and stomach, Xiăo Jiàn Zhōng Tāng was too weak to warm the spleen yang. Therefore, Lĭ Zhōng Wán, another prescription from the Treatise on Cold Damage, was combined with Xiăo Jiàn Zhōng Tāng. To further increase the warming function, the child was also prescribed a prepared medicine, Fù Zĭ Lĭ Zhōng Wán. Here, fù zĭ (Radix Aconiti Lateralis Praeparata) pairs with gān jiāng (Rhizoma Zingiberis) to warm yang and eliminate interior cold. Warm needle technique or moxibustion could also be applied to strengthen the warming effect of the medicinals. 2 weeks later, the diagnosis was spleen and stomach qi deficiency, thus many warming medicinals were no longer needed. Therefore, modified Liù Jūn Zĭ Tāng was issued to strengthen the middle jiao. It is commonly

known that Liù Jūn Zĭ Tāng is based on Sì Jūn Zĭ Tāng. Even though Sì Jūn Zĭ Tāng is not from the Treatise on Cold Damage, two prescriptions in the text are very similar to Sì Jūn Zĭ Tāng. One is the previously mentioned Lĭ Zhōng Wán, and the other is Fù Zĭ Tāng (Aconite Decoction), containing fù zĭ, fú ling (Poria), rén shēn (Ginseng Radix), and bái zhú. Sì Jūn Zĭ Tāng can be accurately described as Lĭ Zhōng Wán plus Fù Zĭ Tāng, with the deletion of fù zĭ and gān jiāng, as they are too warming. Sì Jūn Zĭ Tāng is from Beneficial Formulas from the Taiping Imperial Pharmacy (Tài Píng Huì Mín Hé Jì Jú Fāng). The evolution from Lĭ Zhōng Wán and Fù Zĭ Tāng to Sì Jūn Zĭ Tāng took over 800 years to complete. Other similar examples exist in the history of Chinese medicinal formulas. For instance, Liù Wèi Dì Huáng Wán (Six-Ingredient Rehmannia Pill), from Key to Diagnosis and Treatment of Children’s Diseases (Xiăo Ér Yào Zhèng Zhí Jué) was derived from Shèn Qì Wán (Kidney Qi Pill), from the Essentials from the Golden Cabinet. Sì Wù Tāng is also very similar to both Dāng Guī Săn (Chinese Angelica Powder) (dāng guī, bái sháo, chuān xiōng [Rhizoma Chuanxiong], huáng qín [Radix Scutellariae] and bái zhú for restless fetus disorder) and Dāng Guī Sháo Yào Săn (Chinese Angelica and Peony Powder) (dāng guī, sháo yào, chuān xiōng, bái zhú, zé xiè [Rhizoma Alismatis] and fú líng [Poria] for abdominal pain and edema during pregnancy) in the Essentials from the Golden Cabinet. Tracing the development of Chinese medicinal formulas gives us a better understanding of the classic formulas and how they have been altered over time. At the follow-up visit, the child was given Liù Jūn Zĭ Tāng, which is a combination of Èr Chén Tāng (Two Matured Substances Decoction) and Sì Wù Tāng. Shān zhā, shén qū, and mài yá were also added to improve digestion and prevent food stagnation and damp-phlegm retention. Shān

yào was added to strengthen the spleen, stomach, and kidney, and to improve the child’s appetite. Both symptoms and root were carefully treated and a satisfactory result was achieved.

STUDY QUESTIONS 1. Why is abdominal pain primarily treated by regulating qi movement and freeing the channels and collaterals? 2. How can we differentiate the patterns of qi, blood, food, and parasites? 3. How can we prevent the disease?

Answers 1. The six bowels should be unblocked, and the channels should be uninhibited. Abdominal pain is caused by an attack of the bowels and organs by cold and pathogens, which leads to damage of the stomach and intestines by milk and food, poor spleen yang, and stagnation and stasis of the channels. These, in turn, cause inhibited qi movement with channel disorders and stagnation which eventually cause abdominal pain. Thus, the basic principle in the treatment of abdominal pain is to rectify qi movement and promote the free-flow of the channels. 2. Abdominal pain due to qi stagnation may occur from time to time without a fixed location. Abdominal pain due to blood stasis, often with a history of injury, fracture or surgery, may lead to stabbing pain with a fixed location that are worsened by pressure; there may be a feeling of fullness and hardness at certain parts of the abdomen, or with palpable masses. With abdominal pain due to accumulation of parasites, worm eggs may be found upon microscopic examination of the stool; there is occasional pain around

the navel, and focal distention that comes when the parasites gather and goes when the parasites disperse. Abdominal pain due to food accumulation, likely associated with an unrestrained diet, may present with sour or rotten belching, vomitus containing undigested food, and a bloated stomach cavity. 3. Avoid unclean foods, uncooked cold fruits, vegetables or drinks, and unrestrained meals. Pay attention to changes in the climate, protect the abdomen from cold, and avoid strenuous exercise immediately after meals.

CASE SCENARIOS The following cases present variations of this condition. After familiarizing yourself with the possible common pattern presentations and appropriate formulas for treatment, use the following exercises to test your overall understanding of the condition. 1. A child, 10 months of age, cried occasionally last night along with the following signs and symptoms: active bowel sounds, a pale complexion, cold forehead with sweat, reduced crying when pressure was applied to the abdomen, thin and unformed stools in small amounts 2 times this morning, a light red tongue with thin white coating, and red finger venules. What is the most desirable formula? A. Yăng Zàng Tāng (Viscera-Nourishing Decoction) B. Dà Chéng Qì Tāng (Major Purgative Decoction) C. Xiăo Jiàn Zhōng Tāng (Minor Center-Fortifying Decoction) D. Dīng Yú Lĭ Zhōng Wán (Clove and Evodia Center-Regulating Pill) E. Xiāng Shā Píng Wèi Săn (Amomum Stomach-Calming Powder) 2. A child, 3 years old, has had abdominal pain from time to time over the past 3 months. His pain is relieved by pressure and released by warmth. He also has a pale complexion, unformed stools, a light red tongue with a white coating, and a deep and moderate pulse. What is the presenting pattern in this case? A. Accumulation and stagnation of milk and food

B. Deficiency cold of the spleen and stomach C. Middle cold in the abdomen D. Blood stasis E. Heat accumulation in the stomach and intestines 3. A child, 10 months old, cried occasionally last night along with the following symptoms: active bowel sounds, a pale complexion, cold forehead with sweat, reduced crying with pressure on the abdomen, thin unformed stools in small amounts 2 times this morning, and a light red tongue with a thin white coating. Which is the most appropriate treatment formula? A. Yăng Zàng Tāng (Viscera-Nourishing Decoction) B. Dà Chéng Qì Tāng (Major Purgative Decoction) C. Xiăo Jiàn Zhōng Tāng (Minor Center-Fortifying Decoction) D. Dīng Yú Lĭ Zhōng Wán (Clove and Evodia Center-Regulating Pill) E. Xiāng Shā Píng Wèi Săn (Amomum Stomach-Calming Decoction)

Answers 1. A 2. B 3. A

Chapter 11 Diarrhea Diarrhea is a disease with many causes, the main clinical manifestation being increasingly frequent and thinner stools. In China, diarrhea is one of the most common diseases in children, predominantly attacking children aged 6-24 months, with half of them under 12 months of age. The disease occurs year-round, but is more prevalent in summer and autumn. Children’s diarrhea is most frequently caused by either external pathogens or internal damage from diet. The six pathogenic factors change with the seasons, with summerheat and dampness prevailing in summer and autumn, and wind and cold in winter and spring; however, any of these factors can cause diarrhea. As the spleen prefers dryness and is averse to dampness, external pathogenic dampness and internal turbid-dampness are said to most easily encumber and obstruct the spleen; it is also said that “no diarrhea comes without dampness”. In fact all kinds of diarrhea are associated with dampness, even though they may involve wind, cold, heat, deficiency, or excess. The stomach is the reservoir of water and grain (food and drinks), and the spleen governs transportation and transformation. Therefore, if the stomach and spleen are in harmony, the water and grain will decompose well to produce qi and blood and build ying-wei. Children grow quickly, which places a heavy burden on the spleen and stomach; thus it is said that “in children the spleen is often insufficient”. Improper feeding will cause impaired transportation and transformation,

failure in ascending and descending, and finally disease in the stomach and spleen. When water is pathologically transformed into dampness and grain causes food stagnation, food in stomach cannot be transported nor transformed, and thus descends as diarrhea. The pathological location is the spleen and stomach. The stomach governs the intake and decomposition of food and drinks, while the spleen governs the transportation and transformation of the essence of food and water. External pathogens or internal food damage may impact the physiological function of the splenic transportation and transformation, causing diarrhea, so diarrhea is said to be rooted in the spleen-stomach. Diarrhea mostly attacks children under 24 months because their stomach and spleen are still insufficient and easily attacked by external pathogens or affected by improper milk feeding; moreover, if the child’s spleen and kidney is yang-depleted, excessive dampness may accumulate in the body, further leading to diarrhea. Fulminant diarrhea and heat diarrhea will damage yin-fluids and consume yin, while cold and chronic diarrhea are likely to impair yang qi and lead to yang collapse. Because yang is engendered by yin, the exhaustion of yin-fluids will cause yang qi desertion in severe cases. In children who have excessive downpouring-type diarrhea, there may be damage to qi and yin at beginning, followed by damage to yin and yang; more critically deteriorated cases result in yin exhaustion and yang desertion. Common clinical pattern types include damp-heat, wind-cold, food damage, spleen deficiency, and spleen-kidney yang deficiency. Common transmuted patterns include impairment of both qi and yin, and yin exhaustion and yang desertion.

CLINICAL ESSENTIALS In the diagnosis of diarrhea, attention must be paid to two key points; one being observation of the changes in the frequency, nature, and form of the stool, and secondly we must consider the possibility of other physiological causes or infection. Since many children will experience regular attacks of wind-cold or seasonal pathogens, excessive milk feeding, or consumption of unclean foods, there are a wide variety of possible stool manifestations. These include stools being markedly more frequent, thin stools or watery light-yellow stools, unformed like paste-like stools, stool containing milk blocks, undigested food, or small amounts of mucus, and yellow-green or tan watery stools with a sour smell. Patients may also have vomiting, fever, poor appetite, and thirst. Microscopic examination of the stool may find fat globules or small amounts of leukocytes or erythrocytes. In clinical diagnosis, diarrhea should be differentiated from bacterial dysentery, which is mostly manifested by an acute condition with more frequent stools that are jelly-like, thin and containing pus and blood along with abdominal pain and urgency with rectal heaviness. A routine stool test for bacterial dysentery reveals phagocytes and a large amount of pyocytes and erythrocytes; dysentery bacilli may also be present. With diarrhea, eight-principle pattern differentiation through analysis of the signs and symptoms is the first priority. One must recognize patterns of cold or heat, deficiency or excess, the yin or yang of transmuted patterns, and the degree of severity. Because diarrhea can be acute or take a long course, common patterns can be divided into fulminant diarrhea and chronic

diarrhea. The former is mostly an excess pattern, while the later is mainly a pattern of deficiency or deficiency complicated by excess. Finding the root of diarrhea is also of great significance in clinical treatment; there are a number of causative factors including external invasion, improper diet, and spleen and stomach weakness.

● In cases of external contraction, the root of diarrhea must be found in wind, cold, summerheat, or dampness.

● With diarrhea due to improper diet, it must be clear what food the patient ate, whether he or she ate excessively, or consumed unclean or special foods.

● In cases of spleen and stomach weakness, differentiation of the root should be made between congenital deficiency, which is related to kidney deficiency, or postnatal disharmony, caused by damage to the spleen or deficiency of the spleen after serious disease.

● Excess patterns mostly manifest as an acute disease with a short course; typically there is pain or abdominal distention abdomen present.

● A long course of disease and relapses usually indicate deficiency, or deficiency that is extreme enough to hide concurrent excess. Deficiency is mostly indicated by a bloated abdomen in the shape of a drum, or by a hollow abdomen with a boat-like appearance.

● Wind-cold diarrhea manifests with thin watery stools without a strong odor, and heat diarrhea with foul-smelling tan-colored stools.

● A greasy white tongue coating indicates mostly dampness, and a yellow coating indicates more heat. A pale enlarged tongue shows yang deficiency, and a red spotted tongue shows yin damage.

Diarrhea can also be differentiated according to the severity of the symptoms. Light symptoms include thin sloppy stools, sometimes like egg soup, low fever or normal temperature, no vomiting, a normal appetite and good spirits. Severe symptoms include frequent stools, usually over ten times a day, with frequent vomiting, fever, low spirits or agitation, a pale complexion, red lips, and deep breathing or asthma. The most severe symptoms include sunken eye sockets, scanty urine, and a dry crimson tongue with little coating. These patients and those with no appetite should be paid much attention to. An ancient saying says that three types of patients are incurable: The first type refers to those who have an entirely cold body with deep thready pulses, constant thirst, and an inability to take in any food, including milk. The second type refers to patients in whom large masses of flesh are wasting and sagging. The third type refers to patients who have convulsions. The incurable cases, as the ancient saying advises, require more careful and complicated treatment. Common clinical patterns and their associated signs and symptoms:

● Damp-heat diarrhea manifestations include a greasy yellow tongue coating, frequent downpouring, and tan-colored stools with a strong odor and mucus in some cases.

● Wind-cold diarrhea manifestations include thin stools with many bubbles and little odor, pain in the abdomen, and other typical cold signs.

● Diarrhea due to food damage manifests with indigestion, abdominal bloating, thin stools with undigested food, and pain that is alleviated after diarrhea.

● Diarrhea due to spleen deficiency is associated with a long course of disease and other signs of spleen qi deficiency.

● Diarrhea due to spleen-kidney yang deficiency manifests with thin clear and cold stools with undigested food, and other symptoms of yang deficiency and internal cold. Diarrhea can be treated by internal medicines along with tui na, external treatments, acupuncture, and moxibustion. An experienced and greatly respected doctor of TCM, Dong Yan-yao, is an expert on diarrhea in infants and children. He took his nine treatments for diarrhea from the text, Required Readings from the Medical Ancestors (Yī Zōng Bì Dú). The treatment principles are to cool and clear, to promote free-flow, to percolate with bland medicinals, to dry the spleen, to warm the kidney, to raise clear yang, to consolidate and astringe, to contract qi with sour medicinals, and to slacken qi with sweet medicinals. Dr. Dong’s treatments are effective because of his deliberate differentiation of patterns and cautious use of medicinals. Dr. Dong points out that diarrhea cannot be cured without first removing accumulation. In cases of diarrhea due to food accumulation, rather than using methods to stop diarrhea, medicinals are applied to improve bowel movements: select qīng pí (Pericarpium Citri Reticulatae Viride), chén pí (Pericarpium Citri Reticulatae), shén qū (Massa Medicata Fermentata), guăng mù xiāng (Radix Aucklandiae), and fó shŏu (Fructus Citri Sarcodactylis) with gŭ yá (Fructus Setariae Germinatus) to rectify qi and remove stagnation, and máo zhú (Atractylodes Lancea), chuān pò (Cortex Magnoliae Officinalis), and tōng căo (Medulla Tetrapanacis) to fortify the spleen and remove stagnation.

● External heat may cause large intestine qi to descend excessively, resulting in diarrhea manifested by pouring stools, which should be treated with bitter-cold medicinals such as huáng qín (Radix Scutellariae) and huáng lián (Rhizoma Coptidis) to prevent fluid body loss of the intestine and stomach.

● Excessive dampness encumbering the spleen may cause soggy diarrhea with scanty urine, which, according to Dr. Dong, can be treated by removing dampness through the urine with Sì Líng Săn (Four Substances Powder with Poria), chē qián zĭ (Semen Plantaginis), yì yĭ rén (Semen Coicis), dàn zhú yè (Herba Lophatheri), and tōng căo. For the treatment of dampness with undigested water and grain due to spleen deficiency, Dr. Dong usually prescribes bái zhú (Rhizoma Atractylodis Macrocephalae), cāng zhú (Rhizoma Atractylodis), huò gěng (Caulis Agastachis), and chuān pò (Cortex Magnoliae Officinalis) to dry dampness, and rectify the spleen.

● For chronic diarrhea damaging yang with debilitation of fire, fire failing to generate earth, and spleen-kidney yang deficiency, Dr. Dong selects fù zĭ (Radix Aconiti Lateralis Praeparata), ròu guì (Cortex Cinnamomi), páo jiāng (Rhizoma Zingiberis Praeparatum) and wú zhū yú (Fructus Evodiae) to warm the spleen and kidney.

● For pouring diarrhea with undigested grain, Dr. Dong selects gé gēn (Radix Puerariae Lobatae), hé yè (Folium Nelumbinis), and biăn dòu huā (Flos Lablab) to raise the clear by helping qi to ascend.

CASE STUDY Female, age 2. Initial Visit: November 16th, 2001 Chief Complaint: Diarrhea for 3 days. History: The child caught a cold when playing outside 3 days ago and had a low fever that night. Her temperature was brought down with some cold granules that acted quickly. She started to vomit the next morning, but with no projectile vomiting. Then diarrhea appeared four times a day, with the consistency of the stool ranging from soft to thin like egg soup with bubbles. After taking Xiāng Lián Piàn (Costusroot and Coptis Tablets), she still had a low fever, slight sensitivity to cold, and more frequent, watery diarrhea for 10 times each day. Her parents sent her to the hospital for further treatment. Signs and Symptoms: The child had thin stools with a faint odor, bowel movements more than 10 times a day, a low fever, slight sensitivity to cold, watery nasal discharge, poor appetite, no vomiting, and scanty urine. There was a light red tongue with a white greasy coating, and purple finger venules. Past History: No history of other diseases or disorders. Physical Examination: BT 38.1℃, RR 26 times/min, clear mind, low spirits, a pale complexion, sunken eye sockets, dry lips, sore throat, and clear breath sounds in both lungs. Heart rate 118 beats/min, normal heart

beat without murmurs, flat and soft abdomen, active bowel sounds, poor skin elasticity. Laboratory Examination: Routine blood test: WBC 4.4×109/L, N 0.356, L 0.644, Hb 118 g/L, PLT 156×1012/L; routine stool test: thin, watery stool; microscopic examination: WBC 2-3/HP.

Pattern Differentiation The child caught a cold while playing outside, primarily affecting the abdomen. The cold attacked her spleen and stomach, which led to failure in transportation and transformation, and then to thin clear watery stools with bubbles. The internal obstruction of cold and dampness led to disturbance of qi movement and borborygmus. The cold pathogen attacked the weiexterior and caused external signs and symptoms of fever, a light red tongue with a greasy white coating, and purple finger venules. The frequent unformed stools with little odor, and the tongue features are both consistent with catching a cold; this suggests wind-cold diarrhea. The pouring diarrhea indicates damage to yin, in spite of the short course of disease. To prevent a transmuted pattern, treatment should combine sour and sweet medicinals to restrain yin and supplement fluids while scattering cold and removing dampness. The physiochemical examination shows that the pathogen is a non-invasive bacteria, so antibiotics are not necessary.

Diagnosis Diarrhea due to wind-cold

Clinical Treatment Principles: Scatter wind and dissipate cold, remove dampness and harmonize the middle Formula: Huò Xiāng Zhèng Qì Săn (Agastache Qi-Correcting Powder) [藿⾹正⽓散]

3 doses [Formula Analysis] Huò xiāng, zĭ sū yè, bái zhĭ, and shēng jiāng dissipate wind and cold, rectify qi, and remove dampness. Bàn xià, chén pí, hòu pò, and dà fù pí warm cold, dry dampness, rectify qi, and harmonize the stomach.

Bái zhú, fú líng, gān căo, and dà zăo fortify the spleen and harmonize the stomach. Fáng fēng is added to arrest diarrhea if stool is thin and light with bubbles. Shā rén is added to relieve pain, remove internal cold, and rectify qi. Jīng jiè is added to release the exterior and dissipate cold. Western Medical Treatment Because she was diagnosed with severe dehydration, the patient in this case was treated with rehydration therapy. The first day, in order to supply half of the total loss of water and amount of water to be lost, rehydration with a sodium solution of 120 ml/kg was taken three times; half the first time, one-third the second time, and one-sixth the last time. In the following days, along with oral Smecta (Smectite Powder), rehydration was given according to the patient’s oral fluid intake. Second Visit May 19th The diarrhea was improved, with formed stools one time per day containing little indigested food, no fever, and no sensitivity to cold, but she still had a poor appetite and abdominal distention after meals. Physical examination revealed a clear mind with normal spirits and skin elasticity, heart and lung test results the same, no pain when pressing on the abdomen, normal bowel sounds, a light red tongue with a greasy white coating, and purple finger venules reaching to the wind-pass. Principles: Harmonize spleen and stomach, activate the spleen, promote appetite

Formula: Modified Bù Huàn Jīn Zhèng Qì Săn (Priceless QiCorrecting Powder) [不换⾦正⽓散加减]

3 doses [Formula Analysis] Cāng zhú, pèi lán, and huò xiāng dry dampness and assist transportation. Chén pí and zhĭ shí rectify qi and assist transportation. Jiāo shén qū, chăo mài yá, and jiāo shān zhā promote digestion and assist transportation. Mù xiāng rectifies qi and relieves abdominal distention. External Therapy: Grind dīng xiāng (Flos Caryophylli) and ròu guì into a powder in a proportion of one to two. Mix 1-2g with shēng jiāng zhī (Succus Rhizoma Zingiberis) to make a sticky paste. Apply to the navel once per day, fixing the paste with adhesive plaster.

COMMENTARY AND DISCUSSION Infantile diarrhea is a common gastrointestinal disease that often has complicated pathological mechanisms with multiple complications. Chinese medicine has a long history of dealing with infantile diarrhea, and herbal medicine has proven very effective in the treatment of infantile diarrhea, particularly non-infectious infantile diarrhea. Chinese medicinals also show satisfactory effects in the treatment of infectious infantile diarrhea when used in combination with Western medicine. Identification and pattern diagnosis is critical. In this case, the parents first gave the child Xiāng Lián Piàn before the hospital visit, which is for the damp-heat type of diarrhea, but the symptoms and signs indicated windcold invading the middle jiao. The location of an invasion is very important. If external pathogens invade the superficial area or the upper jiao, it will cause fever, chills, headaches, cough or asthma. In this case, the wind-cold evils invaded the middle jiao and injured the spleen and stomach, resulting in retention of dampness and qi stagnation. A noted Qing Dynasty physician, Chen Nian-zu (1752-1823) explained that with this kind of invasion, the qi of the four seasons does not arrive at the proper time, and that resulting pathogenic qi enters through the mouth and nose. However, this pathogen is different from pathogenic qi which injures the channels as it enters through the skin. Thus, strong diaphoretic medicinals that release the exterior should be avoided; instead use aromatic substances that promote the proper flow of qi. Therefore, a modification of Huò Xiāng Zhèng Qì Săn was selected, to good effect.

Huò Xiāng Zhèng Qì Săn includes all of the medicinals in Píng Wèi Săn (Stomach-Calming Powder) and Èr Chén Tāng (Two Matured Substances Decoction). Therefore, its ability to transform dampness and to regulate qi is stronger than its ability to release exterior conditions. For this reason, fáng fēng and jīng jiè were added at the initial visit to remove external wind-cold evils. As there were few signs of an exterior condition at the follow-up visit, and because turbid dampness was pronounced, Bù Huàn Jīn Zhèng Qì Săn was prescribed. Jiāo shān zhā was added to improve splenic transformation and transportation and to increase the appetite. Huò Xiāng Zhèng Qì Săn is an excellent prescription for regulating qi and harmonizing the middle jiao by eliminating dampness and releasing the exterior. The spleen is the most important organ for the transformation and transportation of body fluids. Therefore, hòu pò, chén pí and huò xiāng were selected to move qi and transform dampness, where bái zhú and bàn xià act to dry dampness. Water metabolism involves the lungs in the upper jiao, as the lungs are the upper source of water, so jié gĕng (Radix Platycodonis), which is in the traditional formula, can raise qi and support lungs to regulate the water passages. As kidneys are called the lower source of water, fú líng is selected to promote urination and guide out dampness from the lower jiao. According to the famous professor Wang Mian-zhi, dà fù pí acts to eliminate water through bowel movements; in this way, the middle jiao qi is rectified while dampness is removed from the upper, middle and lower jiao. Huò Xiāng Zhèng Qì Săn is a commonly used prescription, and there are many possible modifications for varied situations: Bù Huàn Jīn Zhèng Qì Săn transforms turbid dampness and regulates qi.

Yì Jiā Jiăn Zhèng Qì Săn (First Variant Qi-Correcting Powder) treats food stagnation and dampness obstructing the middle jiao. Èr Jiā Jiăn Zhèng Qì Săn (Second Variant Qi-Correcting Powder) transforms dampness, regulates qi, and promotes the flow of qi in the channels. Sān Jiā Jiăn Zhèng Qì Săn (Third Variant Qi-Correcting Powder) regulates qi, transforms dampness, and clears damp-heat. Sì Jiā Jiăn Zhèng Qì Săn (Fourth Variant Qi-Correcting Powder), or the Fourth Modification of Qi-Correcting Powder warms the middle jiao and transforms dampness. Wŭ Jiā Jiăn Zhèng Qì Săn (Fifth Variant Qi-Correcting Powder) replenishes the spleen, benefits the stomach, and warms and transforms cold and dampness. Huò Xiāng Zhèng Qì Săn was modified into the above five modifications by Wu Tang in his Systematic Differentiation of Warm Diseases (Wēn Bìng Tiáo Biàn) (1758-1836). This shows us how most formulas can be adjusted in accordance with the different properties of the disease, whether cold or heat, or excess or deficiency.

STUDY QUESTIONS 1. How can patterns of diarrhea be differentiated according to features of the stool? 2. Please analyze the importance of dampness in diarrhea.

Answers 1. Unformed stools containing milk blocks or undigested food with a sour odor, or stools like rotten eggs indicate diarrhea due to indigestion. Clear thin light yellow odorless stools with bubbles indicate diarrhea due to wind-cold. A large amount of tan and watery egg soup-like stools with little mucus and little odor, along with occasional abdominal pain, indicates diarrhea due to heat and dampness. Thin light-colored paste-like stools without odor occurring after meals indicate diarrhea due to spleen deficiency. Thin clear light-colored stools containing undigested food and without odor indicate yang deficiency of the spleen and kidney. 2. All cases of diarrhea are caused by a disorder of spleen and stomach function. The spleen governs transportation and transformation, the failure of which causes deficiency of transportation and transformation, and failure in ascending and descending. This results in water forming internal dampness, and grain forming stagnation. The food in stomach cannot be transported or transformed and so descends. The spleen is averse to dampness, and damp encumbrance of spleen yang is a primary reason for

impaired splenic function. It has even been said that all cases of diarrhea involve dampness to some degree.

CASE SCENARIOS The following cases present variations of this condition. After familiarizing yourself with the possible common pattern presentations and appropriate formulas for treatment, use the following exercises to test your overall understanding of the condition. 1. A child, 9 months of age, has acute signs and symptoms including frequent and thin stools; testing reveals bacterial dysentery. What should the test report? A. Fat globules in microscopic examination B. Small amounts of leukocytes or erythrocytes C. Phagocytes D. Positive for rotavirus E. Positive for pathogenic Escherichia coli 2. A child of 7 months has had signs and symptoms for one day that include fever, stools 9 times per day, downpouring of thin watery stools, nausea and vomiting, occasional crying, and scanty yellow urine. What is the proper treatment formula? A. Băo Hé Wán (Harmony-Preserving Pill) B. Píng Wèi Săn (Stomach-Calming Powder) C. Shēn Líng Bái Zhú Săn (Ginseng, Poria and Atractylodes Macrocephalae Powder)

D. Huò Xiāng Zhèng Qì Săn (Agastache Qi-Correcting Powder) E. Gé Gēn Huáng Qín Huáng Lián Tāng (Pueraria, Scutellaria, and Coptis Decoction) 3. A child of 7 months has had signs and symptoms since the morning that include occasional crying, 4 occasions of thin stools with bubbles and little odor, clear bowel sounds, a pale tongue with a white coating, and pink finger venules. What is the proper diagnosis? A. Diarrhea due to wind-cold B. Diarrhea due to summerheat and dampness C. Diarrhea due to food damage D. Diarrhea due to deficiency of the spleen E. Diarrhea due to yang deficiency of the spleen and kidney 4. A child, 2 years old, since having 2 cream cakes last night, has had signs and symptoms including occasional crying in the night, vomiting 2 times and passing stool 3 times this morning, thin stools, moderate crying after passing stool, no desire for food, and a curd-like greasy tongue coating. What should be the treatment principle? A. Promote digestion and remove stagnation B. Clear the intestine and remove dampness C. Dissipate wind and cold D. Fortify the spleen and boost qi

E. Warm and supplement the spleen and kidney 5. A child of 11 months feels much better after treatment. At the onset of his disease, he had diarrhea over 10 times a day, which lasted for 2 weeks. Recently, the child still had thin stools that were light in color 3 or 4 times a day, diarrhea after meals, tiredness, and a light tongue with a thin white coating. What is the correct disease mechanism? A. Wind-cold damaging the spleen B. Summerheat and intestinal dampness C. Damage to the spleen and stomach by food D. Deficiency of spleen qi E. Yang deficiency of the spleen and kidney 6. A child, one year old, had moderate or severe diarrhea for 3 months with signs and symptoms including clear and thin odorless stools containing undigested food, occasional rectal prolapse after defecation, a cold body with especially cold limbs, poor spirits, clear urine, and light finger venules. What is the proper treatment formula? A. Yì Gōng Săn (Special Achievement Powder) and Píng Wèi Săn (Stomach-Calming Powder) B. Băo Hé Wán (Harmony-Preserving Pill) and Èr Chén Tāng (Two Matured Substances Decoction) C. Shēn Líng Bái Zhú Săn (Ginseng, Poria and Atractylodes Macrocephalae Powder) and Lĭ

Zhōng Wán (Center-Regulating Pill) D. Jīn Guì Shèn Qì Wán (Golden Cabinet’s Kidney Qi Pill) and Rén Shēn Wū Méi Tāng (Ginseng and Mume Decoction) E. Fù Zĭ Lĭ Zhōng Tāng (Aconite Center-Regulating Pill) and Sì Shén Wán (Four Spirits Pill)

Answers 1. C 2. E 3. A 4. A 5. D 6. E

Chapter 12 Aversion to Food (Anorexia) Aversion to food, or anorexia, refers to the condition of long-term lack of appetite or refusal to eat. This disease is usually caused by improper feeding, sickliness, prolonged illness, or congenital deficiency. The disease is located in the spleen and stomach. Different factors contribute to the dysfunction of the spleen and stomach, bringing about anorexia. Its pathomechanisms are spleen-stomach disharmony, dysfunction of the spleen in governing ascent of the clear, and dysfunction of the stomach in governing descent of the turbid. Common clinical patterns include loss of splenic transportation, qi deficiency of the spleen and stomach, and yin deficiency of the spleen and stomach.

CLINICAL ESSENTIALS Dysfunction of the spleen and stomach in transportation and transformation is the shared pathomechanism of all types of anorexia syndromes. Decreased appetite and declining food intake are common clinical manifestations, and the basic treatment principle is to activate the spleen in order to stimulate the appetite. Zang-fu pattern differentiation is the first priority in treating anorexia. Treatment of this disease must start with the spleen and stomach. Based on the medical history, length of disease, and clinical manifestations, it is vital to distinguish between deficiency and excess, taking care not to neglect their less obvious manifestations. Zhang Xi-jun, an experienced doctor of TCM, believes that a child’s aversion to food is caused by two primary reasons. One is milk accumulation injuring the spleen-stomach. The other is deficiency of spleen and stomach bringing about milk stagnation. The appropriate formula is Kāi Wèi Jìn Shí Tāng (Appetite-Increasing Decoction), made up of huò xiāng (Herba Agastachis), pèi lán (Herba Eupatorii), hòu pò (Cortex Magnoliae Officinalis), chén pí (Pericarpium Citri Reticulatae), gān căo (Radix et Rhizoma Glycyrrhizae), yú qiū chuàn (Kalimeris Indica), yú xīng căo (Herba Houttuyniae), dry-fried shān zhā (Fructus Crataegi), dry-fried mài yá (Fructus Hordei Germinatus), and dryfried shén qū (Massa Medicata Fermentata).

● With milk accumulation, add lái fú zĭ (Semen Raphani).

● With deficiency of the spleen and stomach, subtract hòu pò, chén pí, dry-fried shān zhā, dry-fried mài yá, and dry-fried shén qū, while adding dăng shēn (Radix Codonopsis), bái zhú (Rhizoma Atractylodis Macrocephalae) and fú líng (Poria).

● With exogenous pathogens invading the stomach and obstructed stomach digestion, use the same formula but subtract dry-fried shān zhā, dry-fried mài yá, and dry-fried shén qū, and add zĭ sū yè (Folium Perillae).

● With qi counterflow due to fright and disharmony of spleen-stomach, subtract huò xiāng, pèi lán, and chén pí, but add gōu téng (Ramulus Uncariae Cum Uncis), zhū shā (Cinnabaris), dēng xīn căo (Medulla Junci), and fú líng.

● With fright, add chán tuì (Periostracum Cicadae) and jiāng cán (Bombyx Batryticatus).

● With vomiting, add raw mŭ lì (Concha Ostreae). ● Febrile disease injures stomach yin. Use the same formula without huò xiāng, pèi lán, hòu pò and chén pí, adding shā shēn (Radix Adenophorae seu Glehniae), mài dōng (Radix Ophiopogonis), yù zhú (Rhizoma Polygonati Odorati), and shí hú (Caulis Dendrobii). Substitute fried shān zhā, fried mài yá, and fried shén qū with raw mài yá.

● With chronic disease and spleen-stomach weakness, use the same formula but without hòu pò, chén pí, and add dăng shēn, shān yào (Rhizoma Dioscoreae), biăn dòu (Semen Lablab Album), and fú líng.

● With vomiting, add zĭ sū gĕng (Caulis Perillae) and bái kòu (Fructus Amomi Kravanh).

Prof. Wang Di-jun at Chengdu University of Traditional Chinese Medicine treats anorexia with comprehensive acupuncture therapy. While emphasizing the spleen and stomach, Prof. Wang also takes liver qi into account while treating this disease. Select points precisely and appropriately, and use simple, effective manipulations. The most important points include BL 20 (pí shù), BL 21 (wèi shù), RN 12 (zhōng wăn), LV 13 (zhāng mén), ST 36 (zú sān lĭ) and hua tuo jia ji points. Point combinations must be based on the condition of the patient.

● For acid reflux and vomiting, combine such points as RN 10 (xià wăn), PC 6 (nèi guān), and SP 9 (yīn líng quán).

● For lassitude and abdominal stuffiness, combine such points as BL 18 (gān shù), LV 3 (tài chōng), and LI 4 (hé gŭ).

● For loose stool or undigested food in the stool, combine such points as DU 20 (băi huì), RN 13 (shàng wăn), ST 25 (tiān shū), and BL 25 (dà cháng shù). Also selectively apply one-finger pressing, spine-squeezing, tappingpricking, moxibustion or cupping to these points. Ten days constitute one course of treatment.

● One-finger pressing refers to using the thumb or middle finger pulp to produce gentle and circular motion with light pressure; applied to the main points for 1 or 2 min per point, once a day.

● Spine-squeezing means grasping the skin along the spine with thumbs and index fingers of both hands and rolling upward from the sacrum area to the upper back region (C 7). Repeat this operation 7 to 10 times until the skin turns red.

● Tapping-pricking refers to repeatedly tapping the points using a group of firm filiform needles (instead of dermal needles) until the skin turns red.

● Apply moxibustion on the acupoints using a moxa stick for 1 or 2 min per acupoint, or stop when the skin turns red. For excess patterns, use sparrow-pecking moxibustion. For deficiency patterns, use mild moxibustion.

● Apply cupping on the related points using small glass cups. For deficiency, use flash-fire cupping, 15 to 25 times per point or stopping when the cups are warm. For excess, use retained cupping for 5 or 10 min per point. In combining methods, use finger-pressing for the main points and spine-squeezing for the jia ji points. In combining points for excess syndromes, use tapping-pricking, sparrow-pecking moxibustion, and retained cupping. These methods regulate qi. For deficiency syndromes, use mild moxibustion and flash-fire cupping. These methods elevate yang and replenish qi, helping to rebuild the sick child’s health.

CASE STUDY Female, age 4. Initial Visit: November 8th, 2001 Chief complaint: Poor appetite for 6 months. History: The child had measles a half-year ago and has had a poor appetite ever since recovery, which was neglected by the parents. The child gradually lost her appetite. Signs and Symptoms: The child had a poor appetite, thirst, dry throat, red lips, and dry stools. She defecated one time every 3 or 4 days, with stool like sheep’s droppings. She was conscious, fidgety, and with a normal degree of activity. She had a sallow yellow complexion and a thin body. Her tongue was red with a peeled coating in the middle, and her pulse was thready. Physical Examination: BT 36.9℃, weight 14 kg, normal throat, heart and lungs normal, abdominal distention with no tenderness. Laboratory Examination: RBC 4.05 × 1012/L, Hb 104 g/L, WBC 5.8 × 109/L, L 0.38, M 0.03, G 0.59, PLT 206 × 109/L. Routine stool test: yellow, hard; microscopy: normal.

Pattern Differentiation Based on the signs and symptoms, this case can be diagnosed as aversion to food (anorexia) with a spleen and stomach yin deficiency

pattern. Measles is a yang pathogen which tends to consume body fluids, in this case also damaging qi and yin of the spleen and stomach . The lack of nourishment has led to spleen deficiency with dysfunction of the stomach in absorbing food. This explains why she had no desire for food. Yin deficiency led to stomach fire flaring upward, which was manifested by thirst, diminished food intake, dry throat, and red lips. Deficiency of spleen yin led to failure of intestinal enrichment, which made difficult stools. Impaired spleen-stomach transportation and transformation and diminished food intake resulted in a thin body. Deficiency of yin and body fluids contributed to the red lips, peeled tongue coating, and thready pulse.

Diagnosis Aversion to food due to spleen-stomach yin deficiency

Clinical Treatment Principles: Enrich the spleen and nourish the stomach to promote transportation and transformation Formula: Modified Yăng Wèi Zēng Yè Tāng (Stomach-Nourishing Humor-Increasing Decoction) [养胃增液汤加减]

7 doses [Formula Analysis] Bĕi shā shēn, mài dōng, yù zhú and shí hú nourish the stomach and promote fluid production. Wū méi, bái sháo and gān căo boost the yin with sour and sweet properties. Jiāo shān zhā and chăo mài yá improve the appetite and promote transportation and transformation. Lú gēn and hú huáng lián clear heat and promote fluid production. Huŏ má rén moistens the intestines to promote defecation. Tui Na Knead băn mén (板门), supplement wèi jīng (胃经), move nèi bā guà (内八卦), separate yin and yang (fēn yīn yáng, 分阴阳), knead èr rén shàng mă (⼆⼈上马), knead RN 12 (zhōng wăn). Acupuncture

Ear Points: Spleen (pí), stomach (wèi), kidney (shèn), shen men (shén mén), subcortex (pí zhì xià). Method: Attach wáng bù liú xíng (Semen Vaccariae) to the selected points with adhesive tape. Apply every other day, alternating ears. Press 3 to 5 times a day for 3 to 5 min until slightly painful. 10 days is considered one course of treatment. Second Visit After 7 doses, the child had increased appetite and normal defecation, but still had slight thirst, a dry throat, a red tongue with little coating, and a thready pulse. The previous formula was continued, with huŏ má rén removed due to improved defecation.

Prevention and Nursing Feed the child a proper variety of foods at regular times. Do not allow the child to eat junk food before meals. Until the appetite increases, generally encourage the child to eat appealing foods based on the stomach’s preference rather than perceived nutritional needs.

COMMENTARY AND DISCUSSION As a disease, anorexia was seldom discussed in ancient pediatric classics, but after years of theoretical and clinical research since the 1980s, it has been added to pediatric TCM texts. Anorexia is more and more common in pediatric clinics, with children in cities showing higher incidence. It mainly affects children between the ages of 1 and 6. As the spleen is the root of the postnatal organs and the source of qi and blood, long-lasting anorexia will affect growth and development, eventually resulting in malnutrition or gan syndrome, which is characterized by emaciation with a sallow complexion, withered skin and hair, listlessness, and abdominal distention or a sunken canoe-shaped abdomen. Digestion involves the transformation and transportation function of the spleen and stomach. The spleen and stomach work as a pivot for sending food-essence upwards and the digested food and water downwards. Therefore, transportation is very important for the spleen and stomach. Prof. Jiang Yu-ren, a famous pediatric expert, once said, “strengthening the spleen is not just to supplement the spleen organ, but also to support its transportation function”(脾不在于补⽽在于运). Middle jiao organs have specific characteristics. The spleen prefers dryness and is averse to dampness, while the stomach prefers dampness and is averse to dryness. If the spleen is too damp, then spleen qi cannot ascend and food essence cannot be transformed and transported. On the other hand, if the stomach qi is too dry, it cannot descend, and the initially digested

food and water cannot be moved downward to the small intestine. This causes anorexia with a prolonged poor appetite or even food refusal. This is why the child in this case has emaciation and constipation simultaneously. To treat anorexia, improving the appetite is the key. Chinese medicine has a special treatment for improving diminished transformation and transportation of the spleen, referred to as “awakening the spleen with aromatic medicinals”. “Awaken the spleen” is literal, and could be translated as “appetite-improving”. The term “awaken the spleen” was derived from Sun Si-miao in the Tang Dynasty, but the therapy was based on the Yellow Emperor’s Inner Classic (Huáng Dì Nèi Jīng). It said, “apply fragrant thoroughwort to eliminate stagnant qi” (治之以兰,除陈⽓也). Wang Bing in the Tang Dynasty explained, “the fragrant thoroughwort is orchid…it removes longterm stagnant qi caused by fatty and sweet foods, as the acrid and aromatic herbs have scattering functions” (兰除陈久⽢肥不化之⽓者,以⾟能散发 故也). In different dynasties, ancient pediatric practitioners invented many “awaken the spleen” formulas. For example, Wang Ken-tang in the Ming Dynasty (1602) invented Xíng Pí Săn (Spleen-Awakening Powder) as listed in his Standards for the Diagnosis and Treatment of Cold Damage (Shāng Hán Zhèng Zhì Zhŭn Shéng). The prescription consists of rén shēn (Radix et Rhizoma Ginseng), fú líng, huò xiāng, bái zhú, dīng xiāng (Flos Caryophylli), shā rén (Fructus Amomi), zhì gān căo (Radix et Rhizoma Glycyrrhizae Praeparata cum Melle) and zhì tiān nán xīng (Rhizoma Arisaematis Praeparatum). In the Grand Compendium of Materia Medica (Bĕn Căo Gāng Mù), Li Shi-zhen, in the Ming Dynasty, said, “the herb gān sōng (Radix et Rhizoma

Nardostachyos) is aromatic and can greatly scatter spleen qi stagnation. When a small dosage of gān sōng is added to spleen and stomach supplementing medicinals, it strongly functions to awaken the spleen and improve appetite” (⽢松芳⾹,甚开脾郁,少加脾胃药中,甚醒脾⽓). When awakening the spleen to improve the appetite, a few rules must be followed. First, the selected medicinals should be light in weight and clear in property, as lighter and clearer herbs act to raise spleen qi and support transportation. Second, the dosages should be small and light. For example, a small dosage of bò he (Herba Menthae) is applied in Xiāo Yáo Săn (Free Wanderer Powder) to soothe the liver and awaken the spleen. Third, the medicinals should be acrid and aromatic so they can scatter stagnant qi, remove middle jiao dampness, and promote the transformation and transportation of food and water. These may include hòu pò, huò xiāng, pèi lán, căo guŏ (Fructus Tsaoko), zĭ sū gĕng, bò he, and shí chāng pú (Rhizoma Acori Tatarinowii). Fourth, correct preparation is also helpful. Many medicinals applied for poor appetite and weak digestion are often fried to increase the fragrance so as to favor the spleen and improve appetite. This also makes them easier to digest and absorb. These can include dry-fried bái zhú, dryfried mài yá (Fructus Hordei Germinatus), dry-fried gŭ yá (Fructus Setariae Germinatus), dry-fried bái dòu kòu (Fructus Amomi Kravanh), dry-fried shān zhā, and dry-fried shén qū. The case discussed in this section was an aftereffect of measles, which caused qi and yin deficiency in the spleen and stomach with symptoms of thirst, a desire for drinks, a dry throat and red lips, constipation, emaciation, a peeled tongue coating, and a thready pulse. It is said that “the spleen

transforms body fluids for the stomach”. Therefore, nourishing of the spleen and stomach yin and generating body fluids are the correct principles of treatment for this case, and modified Yăng Wèi Zēng Yè Tāng was applied throughout treatment.

STUDY QUESTIONS 1. What is the pathomechanism for a child’s aversion to food (anorexia)? 2. What is the difference between anorexia caused by splenic transportation dysfunction and a case associated with spleen-stomach qi deficiency? 3. State briefly the difference between anorexia and childhood malnutrition with accumulation.

Answers 1. The main pathomechanism for childhood anorexia is poor eating habits or improper feeding. Chronic disease or other disease also injures the spleen-stomach. In the congenitally deficient, lack of proper care after birth can also cause anorexia. Other cases include summerheat-dampness fumigating and spleen yang failing to raise qi. Uncontrolled emotion, such as too much thinking, also injures the spleen, but poor eating habits and improper feeding are the most common pathomechanism in children. 2. Anorexia caused by impaired splenic transportation and anorexia due to spleen-stomach qi deficiency have the same symptom of no desire to eat. But patients with impaired transportation have no other symptoms except anorexia, or they may only have stuffiness and oppression in the chest and stomach duct. They have abdominal distention after eating but with no marked manifestations of deficiency, and they are in good spirits.

Patients with spleen-stomach qi deficiency have a sallow complexion, a thin body and low spirits, which are manifestations of qi deficiency in addition to anorexia. In terms of treating the former, it is best to harmonize the spleen and stomach, activate the spleen, and stimulate the appetite with Bù Huàn Jīn Zhèng Qì Săn (Priceless Qi-Correcting Powder). For the latter, it is best to fortify the spleen, boost qi, and assist in transportation. The appropriate formula is modified Yì Gōng Săn (Special Achievement Powder). 3. Both have no desire for breast milk, but the main manifestations of anorexia are a long-term poor and decreased appetite, generally with no abdominal distention and no foul-smelling stools. Malnutrition with accumulation has such manifestations as no desire for breast milk, eating without digesting, abdominal distention, and foul-smelling stools. Through routine stool tests, undigested food residue and fat droplets can be seen.

CASE SCENARIOS The following cases present variations of this condition. After familiarizing yourself with the possible common pattern presentations and appropriate formulas for treatment, use the following exercises to test your overall understanding of the condition. 1. A child, age 2, 11 kg, began to have a poor appetite when summer set in. Other symptoms included eating without digesting, a sallow complexion, lassitude, and slightly loose stools with undigested food. What is the proper diagnosis? A. Anorexia B. Malnutrition with accumulation C. Malnutrition D. Summer non-acclimatization E. Diarrhea 2. A child, age 4, likes to eat fried foods. Over the past 2 months, he has had no desire to eat, eats less but drinks more, has slightly dry skin, dry feces, and a red tongue with peeled coating. What is the appropriate formula? A. Zēng Yè Tāng (Humor-Increasing Decoction) B. Yăng Wèi Zēng Yè Tāng (Stomach-Nourishing Humor-Increasing Decoction)

C. Shā Shēn Mài Dōng Tāng (Adenophora and Ophiopogon Decoction) D. Yăng Yīn Qīng Fèi Tāng (Yin-Nourishing Lung-Clearing Decoction) E. Zēng Yè Chéng Qì Tāng (Humor-Increasing and Qi-Guiding Decoction) 3. A child, age 3, 13 kg, has had a poor appetite since he began kindergarten. Other signs and symptoms include a sallow complexion, abdominal distention with increased eating on rare occasions, nausea, good spirits, normal urination and defecation, and a thin greasy tongue coating. What is the proper treatment method? A. Promote digestion, guide out food stagnation, rectify qi, resolve accumulation B. Fortify the spleen, boost qi, increase the appetite, assist transportation C. Enrich spleen, supplement the stomach, promote transportation D. Soothe the liver, remove stagnation, rectify qi, promote transportation E. Harmonize spleen and stomach, activate the spleen, increase the appetite 4. A child, age 5, had pneumonia 3 months ago. After recovery, he began to have a poor appetite, ate without digesting, a sallow complexion, lassitude, fatigue, and slightly loose stools with undigested food. What is the appropriate formula?

A. Féi Ér Wán (Childhood-Malnutrition Rectifying Pill) B. Zhĭ Zhú Wán (Immature Bitter Orange and Atractylodes Macrocephala Pill) C. Yì Gōng Săn (Special Achievement Powder) D. Băo Hé Wán (Harmony-Preserving Pill) E. Sì Jūn Zĭ Tāng (Four Gentlemen Decoction) 5. A child, age 6, 11 kg, has had a poor appetite over the past 3 months. Signs and symptoms include eating with no taste, abdominal distention with increased eating, belching, nausea, good spirits, normal urination and defecation, and a light red tongue with a thin greasy coating. What is the pathomechanism? A. Spleen-stomach qi deficiency B. Spleen encumbered by dampness C. Breast milk accumulation D. Failure of the spleen to transport E. Spleen-stomach yin deficiency 6. A child, age 10 months, 7.6 kg, is milk feeding. In the recent 2 months he has been fed mince meat, but ate without digesting. He had slightly loose stools with undigested food, profuse sweating, tired extremities, fatigue, was vulnerable to cold, had a sallow complexion, a pale tongue with thin greasy white coating, and a slow weak pulse. Which of the following treatments is inappropriate for this case? A. Activate the spleen

B. Fortify the spleen C. Dry dampness D. Guide out food stagnation E. Boost qi

Answers 1. A 2. B 3. E 4. C 5. D 6. D

Chapter 13 Food Accumulation Food accumulation, also called food damage (dyspepsia), refers to injury to the spleen and stomach from breast milk or food; there is impaired transformation and transportation where food accumulates and stagnates in the middle jiao. Clinical manifestations include no desire for breast milk or food, eating without digesting, abdominal distention, belching with a fetid odor, vomiting, foul-smelling stools or constipation. It is similar to the Western medicine condition of functional dyspepsia. Food accumulation is more common in those children with innate deficiency of the spleen, and those bottle-feeding or recovering after an illness. Clinically, it can present alone or concurrently with diseases like the common cold, pneumonia, or malnutrition. The disease has a good prognosis except on rare occasions when a patient has suffered with food accumulation and spleen-stomach damage for a long time without treatment. In such a case, food accumulation changes into malnutrition. Food accumulation is caused by improper feeding which leads to breast milk or food accumulating interiorly, although childhood spleen deficiency with failure of transportation and transformation and other diseases can also be contributing factors. The disease is located in the spleen and stomach; common clinical patterns include milk and food internal accumulation, and spleen deficiency with accumulation.

CLINICAL ESSENTIALS The main points for diagnosing food accumulation include a history of breast milk or food accumulation, eating without digesting, abdominal distention, foul-smelling stools or constipation. Stool tests will show undigested food residue or fat granules. Pattern differentiation should begin with the spleen and stomach. Based on the patient’s physique and the causes and course of the disease and accompanying symptoms, a clear distinction should be drawn between deficiency or excess, cold or heat, and the severity or mildness of the disease. A short course of disease tends to be an excess pattern. However, food accumulation due to spleen deficiency may be manifested by a deficiencyexcess complex at the onset; this is known as deficiency transforming into excess; on the other hand, long-time food accumulation can also lead to deficiency. In any event, deficiency-excess complex patterns are often present. A child is unable to control his/her own consumption of milk or food, and improper feeding will injure the child’s spleen and stomach, bringing about impaired transformation and transportation with qi stagnation and impaired ascending and descending that allows food to accumulate. Clinical manifestations include no desire for food, abdominal distention, fetid vomitus, diarrhea, or stools with undigested milk or food. All of these indicate a pattern of milk or food accumulated interiorly.

When the food accumulation transforms into heat, signs and symptoms include an extremely feverish feeling in palms and soles and in the abdomen, thirst, a desire for cold drinks, constipation, dark urine with a burning sensation, and a red tongue with a greasy yellow coating. Food accumulation with internal cold is often present in a child with a weak constitution, maladjustment after disease, or a history of improper treatment with cold or cool medicinals. Manifestations include a sallow complexion, fatigue, abdominal distention, a desire to be touched on the abdomen, belching with a fetid odor, foul smelling stools, a pale tongue with a greasy coating, and a thready rapid pulse. If food accumulation remains for a long time without successful treatment, manifestations will include a sallow complexion, emaciation, fatigue, low spirits, loose foul-smelling stools with undigested food, a pale tongue with greasy white coating, a thready slippery pulse, and light finger venules. Clinically, such cases can be treated in the same way that malnutrition is dealt with at the onset. Clinically, food accumulation should be differentiated from anorexia and malnutrition. Anorexia is less severe than food accumulation, with main symptoms including a long-term poor appetite and a general decrease in appetite. In general, there is no abdominal distention, foul-smelling stools or constipation. On the other hand, malnutrition is more severe than food accumulation. Food accumulated without digestion for a long time leads to spleen-stomach dysfunction which in turn impairs the generation and transformation of qi and blood, resulting in exhaustion. Signs and symptoms of malnutrition include emaciation, a sallow complexion, dry hair, and low spirits or agitation.

Tui na therapy is very effective for the syndrome of milk and food accumulation:

● For effective general treatment, clear wèi jīng (胃经), knead băn mén (板门), move nèi bā guà (内八卦), push sì héng wén (四横纹), pressknead RN 12 (zhōng wăn) and ST 36 (zú sān lĭ), push qī jiē gǔ downward (tuī xià qī jiē gǔ, 推下七节骨), and separate-push abdomen (fēn tuī fù yīn yáng, 分推腹阴阳).

● For food accumulation with heat, also clear tiān hé shuĭ (天河⽔) and dà cháng(⼤肠).

● For fidgeting, also clear Heart (xīn jīng), pacify Liver (gān jīng), and knead LI 11 (qū chí). From experience, Dr. Liang Zong-han has found that food accumulation is often accompanied by perspiration on the forehead and a greasy turbid coating on the tongue, and other symptoms of damp-heat. Treatment priorities for food accumulation include promoting digestion and guiding out food stagnation, assisted by acrid-warm aromatics and bitter-cold medicinals to clear and direct the turbid downward.

● Commonly used medicinals include jiāo shén qū (scorch-fried Massa Medicata Fermentata), jiāo mài yá (scorch-fried Fructus Hordei Germinatus), jiāo shān zhā (scorch-fried Fructus Crataegi), lái fú zĭ (Semen Raphani), huò xiāng (Herba Agastachis), pèi lán (Herba Eupatorii), căo dòu kòu (Semen Alpiniae Katsumadai), huáng lián (Rhizoma Coptidis), and lián zĭ xīn (Plumula Nelumbinis).

● For spleen yin impaired by interior heat accumulation, treatment should promote digestion, clear heat, and reinforce spleen yin. Medicinals include gŭ yá (Fructus Setariae Germinatus), lián zĭ xīn, huáng lián, shí hú

(Caulis Dendrobii), mài dōng (Radix Ophiopogonis), and tiān huā fĕn (Radix Trichosanthis).

● For spleen yang blocked by dampness, treatment should eliminate dampness and reinforce spleen yang. Medicinals include gŭ yá, fú líng (Poria), bái zhú (Rhizoma Atractylodis Macrocephalae), dăng shēn (Radix Codonopsis), ròu dòu kòu (Semen Myristicae), and zào xīn tŭ (Terra Flava Usta). A long course of disease also leads to food accumulation and spleen deficiency. Excess internal heat impairs spleen yin, and excess internal dampness impairs spleen yang. Therefore, retention of excess pathogens becomes complicated by a deficiency of upright qi; in these cases, it is vital to eliminate the pathogenic internal excess while also reinforcing upright qi.

CASE STUDY Male, age 3. Initial Visit: October 23rd, 1998 Chief Complaint: No desire for food with a sallow complexion for over 2 months. History: The child’s appetite for food became impaired in midAugust. He felt very uncomfortable in the stomach after eating and occasionally vomited up foul-smelling, undigested food 3 to 6 hours after eating. Sometimes he had dry stools, and sometimes loose stools containing undigested food. The child was picky about food and had a smaller appetite than other children of his age. He had routine blood and stool tests many times at the provincial children’s hospital, with normal results. The case was diagnosed as functional dyspepsia. He took Xī Jiào Mǔ Piàn (Selenious Yeast Tablets) and Mā Mī Ài (Live Combined Bacillus Subtilis and Enterococcus Faecium Granules with Multivitamins), but there was no improvement in his appetite. Signs and Symptoms: No desire for food, great stomach discomfort right after eating, a sallow complexion, fatigue, lassitude, occasional dry stools, occasional foul-smelling loose stools with undigested food, normal urination, restless sleep, slightly low spirits, a pale tongue with a white greasy coating, and light-colored finger venules. Past History: No history of other diseases or disorders.

Physical Examination: Cardiopulmonary examination found no abnormality. Soft abdomen, normal bowel sounds. Laboratory Examination: Routine blood test: RBC 4.13 × 1012/L, Hb 114g/L, WBC 6.1 × 109/L, L 0.42, M 0.03, G 0.55, PLT 213 × 109/L; routine stool test: no abnormality.

Pattern Differentiation The child has a weak spleen with irregular transportation and transformation. Excessive eating enables food to accumulate in the middle jiao, which is manifested by no desire for food and abdominal distention right after eating. Accumulated food causes qi to be unable to ascend and descend. In this case, the spleen injury further brings about impaired qi, blood, and body fluid production. Therefore, the body and spirit lose nourishment which is manifested by the sallow complexion, fatigue, and low spirits. The occasional dry stools and foul-smelling loose stools with undigested food, a pale tongue with a white greasy coating, and light finger venules are all signs of spleen qi deficiency. Thus, it is a pattern of spleen deficiency with food accumulation.

Diagnosis Food accumulation and stagnation due to spleen deficiency

Clinical Treatment

Principles: Fortify the spleen to boost transportation, promote digestion, resolve food stagnation Formula: Supplemented Jiàn Pí Wán (Spleen-Fortifying Pill) [健脾丸加味]

7 doses. One dose daily, taken in 2 portions. [Formula Analysis] Dăng shēn, bái zhú, fú líng, and gān căo fortify the spleen and boost qi. Chăo mài yá, jiāo shān zhā, and jiāo shén qū promote digestion and resolve food stagnation. Chén pí, zhĭ shí, and shā rén rectify qi, resolve food stagnation, and awaken the spleen. When combined with fried shān yào, they fortify the spleen, eliminate dampness and check diarrhea. Combined with huò xiāng, they act to awaken the spleen and eliminate dampness. Umbilical Compress Therapy

Fry 100g of wine dregs (Vini Residuum) in a pot and put it into 2 bags. Then place the hot compresses on the navel for 2 or 3 hours, once per day. Tui Na: Supplement pí jīng (脾经), move nèi bā guà (内八卦), rub RN 12 (zhōng wăn), clear and supplement dà cháng (⼤肠), and press-knead ST 36 (zú sān lĭ). Second Visit All symptoms were eliminated, and the child felt much better. Another 7 doses of the original formula were prescribed to consolidate the effect.

Prevention and Nursing Proper feeding is an effective way to avoid food accumulation. A child tends to have a deficient spleen, so it is important to provide nutritious and digestible foods. When a child is already suffering from food accumulation, it is important to restrict the daily diet. Avoid overeating or overdrinking; a normal diet cannot be resumed until the disease is resolved.

COMMENTARY AND DISCUSSION The spleen and stomach play very important roles in the transformation and transportation of food and water. In clinical practice, it is critical to identify whether the food accumulation is an etiology or if it is the result of spleen-stomach deficiency. The former is likely an excess-type of food accumulation, while the latter is likely a combination of excess and deficiency. In this case, the medical history shows that the child was picky about food and had a poorer appetite than other children of his age, thus suggesting a chronic condition of spleen and stomach deficiency. The foulsmelling loose stools with undigested food are typical signs of food accumulation. Therefore, this is a mixed-type pattern with root deficiency and branch excess symptoms. Jiàn Pí Wán is a prescription from Standards for Diagnosis and Treatment (Zhèng Zhì Zhŭn Shéng) by Ming Dynasty physician Wang Kentang (1602). The formula acts to strengthen the spleen, reduce food accumulation, and check diarrhea. Wang Mian-zhi holds that Jiàn Pí Wán is derived from Yì Gōng Săn (Special Achievement Powder), which is from Key to Diagnosis and Treatment of Children’s Diseases (Xiăo Ér Yào Zhèng Zhí Jué) by Qian Yi, edited and published in 1119 by his student, Yan Ji-zhong.

STUDY QUESTIONS 1. State briefly the relationship between food accumulation and malnutrition. 2. What are the key points in diagnosing food accumulation?

Answers 1. In terms of clinical manifestations, malnutrition is different from food accumulation. Malnutrition is characterized by a thin emaciated body, a sallow complexion, dry sparse hair, poor spirits, fidgeting, poor appetite, and a thick greasy tongue coating. Food accumulation manifests with no desire for food, eating without digesting, abdominal distention, belching with a fetid odor, and foul-smelling stools or constipation. Malnutrition is closely related to food accumulation. Long-time food accumulation will injure the spleen and stomach, bringing about emaciation by influencing the transformation of essence and developing into malnutrition. But not all cases of malnutrition are the result of food accumulation; other diseases like anorexia, diarrhea, and tuberculosis with a long course of disease are more likely to transform into malnutrition. On the other hand, a child with malnutrition has very weak spleen and stomach, causing irregular transformation and transportation. With improper feeding or consumption of indigestible foods, malnutrition will likely develop as a deficiency-excess complex pattern due to food accumulation.

2. The key points in diagnosing food accumulation are as follows: First, the child must have a history of food or milk damage. Second, clinical manifestations include no desire for breast milk or food, eating without digesting, abdominal distention, and foul-smelling stools or constipation. Third, accompanying symptoms may include fidgeting, crying at night, or vomiting. Fourth, stool tests show undigested food residue or fat granules.

CASE SCENARIOS The following cases present variations of this condition. After familiarizing yourself with the possible common pattern presentations and appropriate formulas for treatment, use the following exercises to test your overall understanding of the condition. 1. A child, age 2 and whose weight is 10 kg, has had no desire for breast milk for 5 days and has abdominal distention, loose stools containing undigested food, and a light red tongue with a white coating. What is the presenting pattern? A. Anorexia due to failure/loss of splenic transportation B. Anorexia due to qi deficiency of both the spleen and stomach C. Spleen deficiency with food accumulation D. Breast milk accumulated interiorly due to food retention E. Infantile malnutrition with accumulation 2. A child, age 3, has had a poor appetite for several days, restless sleep, abdominal distention with pain, a feverish feeling in the palms and soles, profuse sweating at night, foul smelling stools, a yellow greasy tongue coating, and a rapid slippery pulse. What is the presenting pattern? A. Anorexia due to spleen-stomach yin deficiency B. Anorexia due to failure/loss of splenic transportation

C. Spleen deficiency with food accumulation D. Accumulated food transforming into heat due to food retention E. Abdominal pain due to milk or food retention 3. A child, age 4, has signs and symptoms including abdominal pain, preference for touch, no desire for food, abdominal distention after eating, loose stools containing undigested food, a sallow complexion, fatigue, and a pale tongue with a white coating. The diagnosis is food accumulation. What are the proper treatment principles? A. Fortify the spleen, resolve food stagnation, warm the middle, remove accumulation B. Nourish the spleen, warm the kidney, boost qi, resolve food stagnation C. Fortify the spleen, boost qi, rectify qi, awaken the spleen D. Resolve food stagnation, harmonize the middle E. Fortify the spleen to benefit transportation, promote digestion, resolve food stagnation 4. A child, age 2 years and 4 months, has a weak constitution, a sallow complexion, weakness, poor appetite, appetite worsened after eating too many desserts recently, abdominal distention right after eating, preference for touch on the abdomen, loose foul-smelling stools containing undigested food, a light red tongue with a greasy white coating, and stagnated finger venules. What is the preferred formula for this case? A. Băo Hé Wán (Harmony-Preserving Pill)

B. Xiāo Rǔ Wán (Infantile Stagnation-Elimination Pill) C. Jiàn Pí Wán (Spleen-Fortifying Pill) D. Bā Zhēn Tāng (Eight-Gem Decoction) E. Féi Ér Wán (Childhood Malnutrition-Rectifying Pill) 5. In promoting digestion and guiding out food stagnation, for what is shān zhā (Fructus Crataegi) primarily responsible? A. Resolving stagnation of meat and greasy food B. Resolving stagnation of grain and food C. Resolving stagnation of grain D. Resolving accumulated food E. Rectifying qi to resolve stagnation

Answers 1. D 2. D 3. D 4. C 5. A

Chapter 14 Malnutrition Malnutrition is a chronic consumptive disease that originates from improper feeding or the effects of other diseases, which lead to damage of the spleen and stomach with qi and body fluid consumption and failure to moisten and nourish the skin, sinews, bones, channels and zang-fu. Clinical manifestations include a remarkably slim body shape with a yellow complexion, dry hair, listlessness, agitation and disorder of feeding. In Chinese medicine, malnutrition in children is termed as gān (疳), which has two implications: firstly, gān means sweetness which mainly refers to its cause; this emphasizes that indulgence in greasy and sweet foods may damage the spleen and stomach, leading to malnutrition. Secondly, gān means dryness which refers to the pathogenesis and main symptoms including dryness of qi and body fluids which lead to thin body shapes and dry, withered skin. Malnutrition is primarily caused by failure of the spleen-stomach functions of reception and transformation, and by insufficient transformation of qi, blood and body fluids. The disease is located in the spleen and stomach but also involves the heart, liver, lungs and kidneys. The disease is mostly caused by improper diet and feeding, damage of the spleen and stomach from other disease or long course of disease, congenital insufficiency or acquired functional failure. Cases of mild infantile malnutrition are manifested by disharmony of the spleen and stomach and failure in reception and transformation. Cases

of infantile malnutrition with accumulation involve deficiency complicated by excess which is manifested by deficiency and damage of the spleen and stomach, impaired transportation and transformation, interior accumulation and stagnation, blockage of qi movement and stagnation of the collaterals. Serious cases of dryness-type infantile malnutrition are manifested by deficiency and debilitation of the spleen and stomach, body fluid collapse and exhaustion, consumption and damage of qi and blood, and a thin body. Severe cases usually involve more complicated patterns of deficiencyexcess complex in other organs. Clinically, there exist both common patterns and complex patterns of malnutrition. Common patterns include mild infantile malnutrition, infantile malnutrition with accumulation, and dryness infantile malnutrition, while complex patterns include infantile malnutrition involving the eyes, infantile malnutrition with aphthae, and infantile nutritional edema.

CLINICAL ESSENTIALS Malnutrition is classified into common patterns and complex patterns. Clinically, eight-principle pattern identification is the first priority in common patterns to differentiate excess from deficiency. Zang-fu pattern differentiation is the first priority in complex patterns to identify the type of malnutrition and the zang-fu involved. The common patterns can be classified into three stages, mild infantile malnutrition, infantile malnutrition with accumulation and dryness infantile malnutrition, according to the disease courses and clinical symptoms. Malnutrition is always caused by deficiency, while the excess-deficiency complex patterns mainly exist in serious cases of infantile malnutrition with accumulation and dryness infantile malnutrition. The symptoms of sores in the mouth and on the tongue, protruding tongue and waggling tongue indicate disease in the heart affected by the spleen. The symptoms of keratomalacia, dry eyes and night blindness indicate disease in the liver affected by the spleen. The symptoms of tidal fever, long course of cough, panting and sounds of whistling or sawing due to phlegm indicate disease in the lungs affected by the spleen. The symptoms of late teething, sunken fontanel, pigeon breast and tortoise back indicate the kidney affected by the spleen. The primary acupuncture treatment principles are fortifying and activating the spleen and stomach, supplementing qi and blood, and dispersing accumulation and stagnation. Treat with acupuncture and moxibustion, with even supplementation and drainage method on points

including RN 12 (zhōng wăn), ST 36 (zú sān lĭ), BL 20 (pí shù) and sì fèng (EX-UE10), points proven to be effective in the treatment of infantile malnutrition. In addition, treatment may include tui na with pinching along the spine, dermal needling and incision therapy. Fu De-hua, a senior TCM doctor, claims that one must treat the root in cases of malnutrition, which always originates from spleen deficiency. The spleen is the foundation of acquired constitution and governs transportation and transformation, the failure of which leads to insufficient generation of essence and blood from water and grain. This, further, leads to insufficient nourishing of zang-fu and then to yin deficiency with yang hyperactivity or yin exuberance with yang deficiency, manifested by intermingling patterns of cold and heat or excess and deficiency, or by symptoms of excess or deficiency in the zang-fu. In addition, the kidneys and malnutrition are closely related. The kidneys are the foundation of congenital constitution, deficiency of which increases the likelihood of malnutrition, while infantile malnutrition with accumulation will damage the kidney in return. Clinically, Dr. Fu usually treats all types of malnutrition with Qĭ Dì Yì Gōng Săn (Lycium Berry and Rehmannia Special Achievement Powder), modified according to the symptoms. The basic formula includes gŏu qĭ zĭ (Fructus Lycii) 5g, shú dì (Radix Rehmanniae Praeparata) 9g, shān yào (Rhizoma Dioscoreae) 12g, dăng shēn (Radix Codonopsis) 12g, dry-fried bái zhú (Rhizoma Atractylodis Macrocephalae) 6g, fú líng (Poria) 6g, chén pí (Pericarpium Citri Reticulatae) 6g and zhì gān căo (Radix et Rhizoma Glycyrrhizae Praeparata cum Melle) 3g.

● In cases with symptoms of heart fire flaming upward, replace shú dì huáng with shēng dì huáng (Radix Rehmanniae) and add Dăo Chì Săn

(Red-Guiding Powder) and huáng lián (Rhizoma Coptidis) into the basic formula.

● In cases with symptoms of vigorous liver fire, add xià kū căo (Spica Prunellae), mù zéi căo (Medicinala Equiseti Hiemalis) and lóng dăn căo (Radix Gentianae) into the basic formula.

● In cases with symptoms of spleen deficiency, add shā rén (Fructus Amomi), guì zhī (Ramulus Cinnamomi) or fù zĭ (Radix Aconiti Lateralis Praeparata) into the basic formula. In cases with symptoms of kidney deficiency, add shān zhū yú (Fructus Corni) and bŭ gŭ zhī (Fructus Psoraleae) into the basic formula.

● The above formulas can be accompanied by needling extra points such as sì fèng (EX-UE10).

● Also apply plum-blossom needle-tapping on acupoints on the back, located on both sides of the spine from the seventh cervical vertebra to the coccyx, also include both the du mai and the foot taiyang bladder channel: With the child lying face down, after disinfection of the points, tap lightly with a plum-blossom needle along the spine from cervical vertebrae to the coccyx, from upper to lower, then tap along foot taiyang bladder channel on both sides of the spine. Tap the points 3 to 4 times until slight redness occurs, but without injuring the skin. Treat once every other day, with 10 times considered one treatment course. Usually, satisfactory results will occur after 3 treatment courses.

CASE STUDY Female, age 1 year. Initial Visit: April 18th, 2008 Chief Complaint: The child had become thinner with a poor appetite and bloated abdomen for 2 months. History: She had been breastfed without any supplementary foods before breastfeeding was abruptly stopped due to illness of her mother. Then the child often refused feeding or cried a lot when being fed with milk or milk powder. In addition to mental fatigue and a short temper, she grew remarkably thinner, with a poor appetite and bloated abdomen for the last 2 months. Signs and Symptoms: A thin body, bloated abdomen, mental fatigue, much crying, agitation, uneasy sleep, disorder of stool, normal urination, a light red tongue with a thin greasy coating, and light stagnation of the finger venules reaching the wind-pass. Past History: The child had a weak constitution and was prone to infection. No information was available about her history of allergies to medicines or food. Normal vaccination history. Physical Examination: BT 36.8℃, pulse 118 beats/min, respiration 32 times/ min, weight 7.5 kg, withered yellow complexion, sparse hair, loose skin, light red lips, no congestion in throat, smooth oral mucosa, clear breath sounds in the lungs, normal heart beat, pathogenic murmurs not heard, bloated abdomen with blue veins distended, thin subcutaneous fat less than 0.4 cm, and a drum-like sound upon percussion. No dynamic murmur heard, liver and spleen not palpable.

Laboratory Examination: WBC 5.8 × 109/L, N 0.256, L 0.744, Hb 88 g/L, PLT 206 × 1012/L.

Pattern Differentiation After her breastfeeding was abruptly stopped, the child, with a constantly weak constitution, proved to be partial to a particular kind of food and refused feeding, which led to damage and disorder of the spleen and stomach, causing failure in transformation of water and grain into qi and blood to nourish the body. That was the reason for her thin body, withered yellow complexion and sparse hair. Spleen deficiency failing in transportation caused inhibited qi movement and interior accumulation and stagnation, further leading to blockage of qi movement and stagnation in the intestines and stomach, resulting in her bloated abdomen and disorder of stool. Stagnation of the collaterals caused the distending blue veins. Deficient earth and vigorous wood with liver fire harassing the interior caused the short temper, irritation, unease and crying. The light red tongue, thin greasy tongue coating, and light stagnation of the finger venules reaching the wind-pass were signs of spleen and stomach deficiency, insufficient qi and blood, and interior accumulation and stagnation. The disease in this case was located mainly in the spleen and stomach while also affecting the liver. The proper diagnosis is infantile malnutrition with accumulation, with complicated patterns of a deficiency-excess complex. The main disease mechanism here is spleen-stomach deficiency with impaired function in transportation and transformation, leading to interior accumulation and stagnation, and blockage of qi movement.

Diagnosis Infantile malnutrition with accumulation due to spleen-stomach deficiency with qi and blood insufficiency

Clinical Treatment Principles: Harmonize and nourish the spleen and stomach, disperse accumulation and stagnation Formula: Modified Féi Ér Wán (Childhood-Malnutrition Rectifying Pill) [肥⼉丸加减]

14 doses [Formula Analysis] Dăng shēn, bái zhú and fú líng fortify the spleen and boost qi.

Jiāo shén qū, jiāo shān zhā and chăo mài yá promote digestion and disperse stagnation. Dà fù pí and bīng láng rectify qi and remove accumulation. Huáng lián and hú huáng lián clear heart, calm the liver, clear heat and remove agitation. Gān căo acts to harmonize all formula medicinals. Prepared Medicines

● Mù Xiāng Bīng Láng Wán (Costus Root and Areca Pill): Boil dà zăo (Fructus Jujubae) 6g and shān yào (Rhizoma Dioscoreae) 30g, pulverize, and make a paste to take along with the pill. 1.5g, 2 times daily.

● Xiăo Ér Xiāng Jú Dān (Children’s Costus Root and Tangerine Peel Elixir): Take one elixir, 3 times daily. Consider reducing dosage for children less than 1 year of age. Acupuncture Main Points: LI 4 (hé gŭ), LI 11 (qū chí), RN 12 (zhōng wăn), RN 6 (qì hăi), ST 36 (zú sān lĭ), SP 6 (sān yīn jiāo). Supplementary Points: BL 20 (pí shù), BL 21 (wèi shù), pĭ gēn (EXB4). Manipulation: Even supplementation and drainage. Moderate stimulation with no retaining of the needles. 7 days constitute one course of treatment. Extra Points: After disinfection, insert a three-edged or filiform needle into sì fèng (EX-UE10) to about 1 cm, remove and then squeeze out the white or yellow mucus or drops of blood. Apply 2 times each week until no white or yellow mucus can be squeezed out.

Second Visit After receiving the above treatment for 14 days, the appetite improved and her abdomen became soft. Remaining signs and symptoms included a slim body of 10.5 kg, occasional crying at night, fatigue, irrigation, withered yellow complexion, sparse hair, slightly poor skin elasticity, thin subcutaneous fat less than 0.6 cm, disorder of stool, normal urine, a light red tongue with a thin white coating, and light finger venules. After treatment, spleen and stomach deficiency remained as the main problems, while the symptoms of interior accumulation and stagnation had been relieved. Principles: Harmonize the spleen, promote transportation Formula: Modified Zī Shēng Jiàn Pí Wán (Life-Promoting SpleenFortifying Pill) [资⽣健脾丸加减]

14 doses [Formula Analysis]

Dăng shēn, bái zhú and shān yào fortify the spleen and boost qi. Yì yĭ rén, fú líng and zé xiè fortify the spleen and percolate dampness. Huò xiāng, shā rén and biăn dòu awaken the spleen and strengthen the appetite. Mài yá, shén qū and shān zhā promote digestion and transportation. Gōu téng calms the mind and arrests convulsion

Prevention and Nursing The child should be fed a fixed amount of milk at a fixed time, with no harsh supplemental foods. According to her symptoms and digestive capability, she should be fed nutritious and easily digestible food, gradually increasing the amount, density, richness, lack of refinement and variety; foods should be fresh and rich, the diet containing both meat and vegetables. Make sure she has enough time to sleep and to play outside, as fresh air and sunshine will help enhance her physical fitness. Pay close attention to her nursing and feeding, and measure her weight and height periodically in order to control the effects of treatment.

COMMENTARY AND DISCUSSION Malnutrition is a disease with a deficient root and excess branch. In this case, the deficiency includes spleen and stomach weakness with insufficient qi and blood. The excesses are qi stagnation in the middle jiao with heat and fire in the liver organ, resulting in the liver over-checking the spleen due to the spleen deficiency. Thus, attention should be paid not only to the weakness of the spleen and stomach or the deficiency of qi and blood, but also to the excesses. Otherwise, supplementing the spleen and stomach or nourishing qi and blood could make the stagnation and accumulation worse. In this case, modified Féi Ér Wán was applied for both issues. Féi Ér Wán is from Golden Mirror of the Medical Tradition (Yī Zōng Jīn Jiàn) by Wu Qian. It contains two formulas; one is Sì Jūn Zĭ Tāng (Four Gentlemen Decoction), containing dăng shēn, bái zhú, fú líng and gān căo, which strengthen the functions of the spleen and stomach. The other is Jiāo Sān Xiān, containing jiāo shén qū, jiāo shān zhā and chăo mài yá to improve the transformation and transportation of food and water and remove stagnation. Huáng lián and hú huáng lián existed in the original formula to clear both excess heat and deficiency heat and calm the shen (spirit), as longlasting stagnation often creates heat and fire. In the original formula, lú huì (Aloe) and shĭ jūn zĭ (Fructus Quisqualis) were applied to kill parasites, strengthen the spleen and dissolve accumulation. In addition, lú huì also clears liver heat and fire, and relieves irritability. In this case, instead of the above two medicinals, bīng láng and dà fù pí were added to promote qi

movement, reduce accumulation and guide out stagnation by mildly promoting the bowels. When the transformation and transportation functions of the spleen recover, both deficiency and excess will gradually adjust. The follow-up visit showed spleen deficiency, but with much less stagnation. Therefore, the medicinals for guiding out accumulation were replaced with aromatic medicinals that awaken the spleen, including shā rén and huò xiāng, and yì yĭ rén and zé xiè for transforming dampness. Other spleen-strengthening medicinals were added, including shān yào and biăn dòu.

STUDY QUESTIONS 1. Define malnutrition involving sweetness and malnutrition involving dryness. 2. What are the differences in clinical symptoms between malnutrition and food accumulation, and how do they relate to each other? 3. What are the key points in the prevention of malnutrition?

Answers 1. Since ancient times, malnutrition has been considered to have two implications, namely, malnutrition involving sweetness and malnutrition involving dryness. Malnutrition involving sweetness refers to its etiology, meaning a child with unrestrained diet containing much fat and sugar is more likely to damage the spleen and stomach, leading to malnutrition. Malnutrition involving dryness refers to the pathomechanism and the main symptoms, including dryness of qi and body fluids, which leads to a thin body. 2. Malnutrition is manifested by signs and symptoms including a thin body, pale complexion, dry withered hair, agitation or mental fatigue, disorder of diet and a thick greasy tongue coating; while food accumulation is manifested by no appetite, indigestion, a bloated stomach cavity and abdomen, belching with a sour and rotten smell, thin and unformed stools or dry hard stools with a sour stench. A long course of food accumulation will affect transformation and generation of water and grain, which leads to

gradual slimming of the body and malnutrition. However, not all cases of malnutrition come from food accumulation. Other diseases such as aversion to food, diarrhea and lung consumption will turn also into malnutrition after a long course. 3. First, children should be fed a fixed amount of milk at fixed times, and with no harsh supplemental foods. Food should be fresh and rich to meet the demands of their growth. Second, lifestyle should be properly arranged to enhance physical fitness by allowing enough time for sleeping and playing outside in the fresh air and sunshine. Third, it is important to correct improper dietary habits such as fondness for foods containing much fat and sugar, too much snacking, and irregular diets. Finally, in the case of a worsening appetite or no weight gain, immediately ascertain the reasons and seek proper treatment.

CASE SCENARIOS The following cases present variations of this condition. After familiarizing yourself with the possible common pattern presentations and appropriate formulas for treatment, use the following exercises to test your overall understanding of the condition. 1. A child, age 18 months, weighs 9 kg and has poor appetite and digestion, a withered complexion, agitation, irascibility, disorder of stool, a light red tongue body with a thin and slightly greasy coating, and light finger venules. What is the correct diagnosis? A. Aversion to food B. Mild infantile malnutrition C. Infantile malnutrition with accumulation D. Dryness infantile malnutrition E. Food accumulation 2. A child, age 10 months, has been fed with supplementary food since he was 4 months old, causing diarrhea and poor appetite along with gradual thinning of his body, a withered yellow complexion, sparse hair, agitation, much crying, uneasy sleep, abdomen bloated like a drum, a light red tongue with a greasy coating, blue stagnation of the finger venules. There is also frequent rubbing of the eyebrows and eyes with finger-sucking and teethgrinding.

What is the proper treatment formula? A. Féi Ér Wán (Childhood Malnutrition-Rectifying Pill) B. Bā Zhēn Tāng (Eight-Gem Decoction) C. Liù Jūn Zĭ Tāng (Six Gentlemen Decoction) D. Sì Jūn Zĭ Tāng (Four Gentlemen Decoction) E. Zī Shēng Jiàn Pí Wán (Life-Promoting Spleen-Fortifying Pill) 3. A child, age 16 months, has an extremely slim body, a withered complexion, mental fatigue, lack of strength, abdomen shaped concave like a boat, edema on the face and ankles, difficult urination, a light red tender tongue with a thin white coating, and light finger venules. What abnormal change in a blood biochemical test is to be expected? A. Globulin obviously lower than normal standard B. White globulin proportion inversion C. Serum total protein reduced D. Albumin below 20 g/L E. Albumin above 20 g/L 4. A child, 2 years old, weighing 10 kg, has a withered complexion, agitation, disorder of stool, a light red tongue body with a scanty coating, and light finger venules. What is the presenting pattern? A. Mild infantile malnutrition B. Infantile malnutrition with accumulation C. Dryness infantile malnutrition

D. Infantile nutritional edema E. Infantile malnutrition involving the lung 5. A child, age one year, weighs 7.2 kg and has a withered complexion, mental fatigue, poor appetite, abdomen bloated like a drum with distended blue veins, edema on the ankles, a light red tongue with a thin white tongue coating, and light finger venules. What are the correct principles of treatment? A. Supplement and boost qi and blood, activate the spleen B. Nourish blood, soften the liver, invigorate blood and dissolve stasis C. Enrich yin, promote fluids production, nourish blood and calm the mind D. Rectify the spleen, warm yang, promote urination and remove edema E. Harmonize spleen qi transportation and promote appetite 6. A child, 2-and-a-half years old, weighs 9.8 kg and has mental fatigue, sores on the tongue and in the mouth, mucus in the oral cavity, a dark red complexion, agitation, uneasy sleep at night, scanty yellow urine, a red tongue with a thin yellow coating, and light blue finger venules. What is the pathomechanism? A. Spleen disease involving the liver, ascendant hyperactivity of liver yang B. Spleen disease involving the heart, heart fire flaming upward C. Spleen disease involving the kidneys, vigorous fire due to yin deficiency

D. Excess fire in the spleen and stomach flaming upward through the channels E. Vigorous fire in the heart and liver, fumigating and steaming the orifices 7. A child, 21 months old, has an extremely thin body with dry withered hair, a pale complexion, mental fatigue, a concave boat-shaped abdomen, thin unformed stools, a light red tongue with little coating, and light finger venules. What is the proper treatment formula? A. Féi Ér Wán (Childhood Malnutrition-Rectifying Pill) B. Bā Zhēn Tāng (Eight-Gem Decoction) C. Liù Jūn Zĭ Tāng (Six Gentlemen Decoction) D. Liù Wèi Dì Huáng Wán (Six-Ingredient Rehmannia Pill) E. Zī Shēng Jiàn Pí Wán (Life-Promoting Spleen-Fortifying Pill) 8. A child, 2 years old, weighs 7 kg and has an abdomen bloated like a drum with distended blue veins , sagging of major masses of flesh on the limbs, no appetite, mental fatigue, lack of strength, a light red tender tongue with little coating, and light finger venules. What is the pathomechanism? A. Disharmony of the spleen and stomach, failure in reception and transformation B. Deficiency and damage of the spleen and stomach, interior stagnation C. Insufficient liver blood, failure in nourishing the channels

D. Deficiency and debilitation of the spleen and stomach impairing qi and blood E. Deficiency and debilitation of kidney yang, insufficient bone marrow and essential qi

Answers 1. B 2. A 3. D 4. A 5. D 6. B 7. B 8. D

Chapter 15 Nutritional Iron Deficiency Anemia Nutritional iron deficiency anemia is a common microcytic, hypochromic anemia caused by the decline of hemoglobin biosynthesis, and rooted in iron deficiency. Nutritional anemia is a common disease in children, especially infants. Clinically, mild cases have no obvious symptoms, while moderate cases usually have signs and symptoms including pale or withered yellow skin and mucosa, dizziness, lack of strength, poor appetite and parorexia or pica. The prognosis of the disease is favorable after reasonable treatment, though severe anemia or a long course of it may affect the growth of children by lowering the body’s resistance and increasing the possibility of external pathogenic infection. The disease belongs to blood deficiency according to traditional Chinese medicine. The disease is rooted in the foundations of congenital constitution and acquired constitution. The primary reasons for the disease include congenital constitutional weakness, kidney essence insufficient to engender blood, improper feeding, and spleen deficiency. These may result in the lack of generation of qi and blood, among which, with improper feeding and deficiency of spleen and stomach being the most common reasons. The disease is mainly located in the spleen, kidney, heart and liver. The main pathological bases are deficiency of the spleen which fails to engender qi and blood, and deficiency of the kidney, which fails to supplement essence and engender blood.

The disease belongs to infantile anemia according to Western medicine. The diagnostic standard for anemia is hemoglobin (Hb) levels below 145 g/L for newly born babies less than 10 days old, below 100 g/L for babies between 10 days and 3 months old, below 110 g/L for children between 3 months and 6 years old, and below 120 g/L for children between 6 years and 14 years old. The standard for mild anemia is hemoglobin (Hb) below 110 g/L for children between 6 months and 6 years old or lower than 120 g/L for children over 6 years old, and with an erythrocyte count below 3 × 1012-4 × 1012/L. The standard for moderate anemia is hemoglobin (Hb) between 6090 g/L and erythrocytes between 2×1012-3×1012/L. The standard for severe anemia is hemoglobin (Hb) between 30-60 g/L and erythrocytes below 1 × 1012-2 × 1012/L. The standard for extremely severe anemia is hemoglobin (Hb) below 30 g/L and erythrocytes below 1 × 1012/L. Common clinical pattern types include anemia due to spleen-stomach weakness, anemia due to heart-spleen deficiency, anemia due to liverkidney yin deficiency, and anemia due to spleen-kidney yang deficiency.

CLINICAL ESSENTIALS Blood deficiency comes on slowly, with the most distinctive signs and symptoms including pale or withered yellow lips, oral mucosa and nails along with lack of strength and poor appetite. Older children with severe blood deficiency may present with dizziness, tinnitus and blurred vision; some patients have a swollen liver or spleen. Most patients have a history of deficiencies of iron reception or consumption or chronic blood loss. Modern medical lab tests are available for diagnosing anemia. In infant cases, differentiation must be made between physiologic and pathologic anemia. The latter refers to the anemia of babies between several weeks and 3 months after birth, due to several reasons. First, babies have abundant erythrocytes after birth, and the body itself destroys too many of the red blood cells too quickly. Second is the relative reduction in erythrocytes due to reduction of erythropoietin and lower medullary hematopoiesis. The third cause is the shorter erythrocyte lifespan in fetuses and the early death of erythrocytes. The fourth cause is the lack of substance due to faster than normal growth. The last reason is the lower metabolic rate and oxygen consumption in premature babies. Physiologic anemia, or the normochromic anemia, usually resolves in 3 to 4 months without medical treatment.

● Mild anemia usually has no distinct clinical symptoms except for abnormal results from lab tests. Extended courses of anemia will have distinct symptoms, showing it to be either moderate or severe anemia.

● Mild or moderate anemia cases are primarily due to deficiency of qi and blood. Cases of qi deficiency usually accompany patterns of qi deficiency in the spleen and stomach, while cases of blood deficiency usually accompany heart blood insufficiency patterns. Severe anemia mainly affects the yin and yang of the liver and kidney.

● Anemia due to spleen-stomach weakness presents with a poor appetite, disorder of stool, light red tongue, thin and white tongue coating, a thready and weak pulse and light red finger venules.

● In addition to symptoms of spleen deficiency, anemia due to heartspleen deficiency presents with palpitations or severe palpitations, uneasy sleep at night caused by depletion of heart blood and lack of nutrition of the heart, a light red tongue, a thready and weak pulse and light red finger venules, with the tongue disclosing severe blood deficiency.

● Anemia due to liver and kidney yin deficiency presents with severe signs and symptoms including a pale complexion and withered yellow hair along with the signs of liver-kidney yin deficiency. Liver yin deficiency presents with dry eyes and trembling or convulsion of the limbs, while the kidney yin deficiency presents with aching pain, weak lumbus and legs, dizziness, tinnitus, a red tongue with little or no coating, and a rapid and wiry pulse.

● Anemia due to spleen and kidney yang deficiency presents with severe signs and symptoms such as a pale complexion and lips, pale fingernails, and sparse hair along with signs of spleen-kidney yang deficiency. The spleen yang deficiency presents with poor appetite or digestion and thin unformed stools containing undigested food, while kidney yang deficiency presents with cold limbs, aversion to cold, slow

growth, an enlarged light red tongue with a white coating, a weak deep pulse, and light red finger venules. The basic principle of treatment for anemia is to treat deficiency with supplementation. Supplementation of the spleen and kidney should take the primary role because the spleen is the source of qi and blood, and the kidneys store the essence which transforms into blood. That is why it is said that essence and blood are from the same source. Clinical treatment should be applied according to differentiation of zang-fu, blood and qi, yin and yang, and damage or deficiency. Make sure to supplement along while also promoting transportation in cases of stagnation. Because some medicinals that supplement deficiency are cloying and not easy to digest, overuse may hinder spleen-stomach transportation and transformation. Use of these medicinals should be limited, or they should be taken along with other medicinals for rectifying the spleen and promoting digestion.

CASE STUDY Male, age 7 months. Initial Visit: August 14th, 2000 Chief Complaint: Withered yellow complexion for 3 months. History: This child, a premature infant, had been fed by breast-feeding and milk feeding since birth, and most frequently had diarrhea. In the last 3 months, he had a poor appetite, pale skin and mucosa, a withered yellow complexion, mental fatigue, lack of strength, sparse hair, much crying when asleep, a low and weak voice, and thin watery stools. Signs and Symptoms: A withered yellow complexion, poor appetite, occasional diarrhea, thin sloppy stools, uneasy sleep with crying, occasional weak crying, a light red tongue, and light red finger venules. Past History: A weak constitution with susceptibility to colds Physical Examination: Pale skin and mucosa, withered yellow complexion, mental fatigue, lack of strength, sparse hair, pale lips and fingers, clear breath sounds in both lungs, clear heart beat sound, slightly rapid heart beat, no murmurs heard, soft belly without pain when pressing and slightly active borborygmus. Laboratory Examination: HGB 70 g/L, MCHC 29%, MCV 76 fl/dl.

Pattern Differentiation The child, a premature infant with congenital spleen and stomach deficiency, had spleen and stomach damage due to improper mixed feeding,

which led to the lack of healthy transportation and frequent diarrhea. Depletion of the spleen and stomach led to failure in engendering qi and blood, shown by paleness of the skin, withered yellow complexion, pale skin and mucosa, sparse hair, and lack of strength and lassitude. Failure of healthy transportation led to poor appetite and digestion. Heart blood insufficiency and lack of nutrition of the heart led to his mental fatigue and uneasy sleep with much crying at night. The tongue and pulse suggested anemia due to heart-spleen deficiency with insufficient qi and blood. Test results indicated moderate microcytic hypochromic anemia. The disease in this case is located mainly in the heart and spleen, and belongs to the category of anemia due to heart-spleen deficiency.

Diagnosis Nutritional iron deficiency anemia due to heart and spleen deficiency

Clinical Treatment Principles: Supplement the spleen, nourish the heart, boost qi, engender blood Formula: Modified Guī Pí Tāng (Spleen-Restoring Decoction) [归脾汤加减]

10 doses [Formula Analysis] Huáng qí, rén shēn, bái zhú and fú líng rectify the spleen and boost qi. Dāng guī and lóng yăn ròu nourish the heart and supplement blood. Suān zăo rén pacifies the heart and calms the mind. Cāng zhú and chén pí dry dampness and harmonize the middle jiao. Jiāo shān zhā, chăo gŭ yá and chăo mài yá disperse accumulation and promote digestion and transportation. Western Medicine Treatment: Ferrous sulfate 5mg/kg, twice daily; Vitamin C 100 mg, once daily. Second Visit August 24th After treatment, the patient’s stools were well-formed, he was sleeping easily, and his complexion had improved. The same Western medicine and Chinese medicinals as prescribed previously were continued for another 10 days.

Prevention and Nursing Explanations concerning proper nutrition during pregnancy and the feeding of infants should be communicated widely. Ideal meals during pregnancy and scientific diets for infants are key points in prevention of anemia, especially chalybeate supplementation for infants under 2 months who are premature or of low birthweight.

COMMENTARY AND DISCUSSION Unlike biomedicine, Chinese medicine holds that the generation of blood relies upon the spleen and heart. The spleen and stomach, along with the intestines, provide ying qi and body fluids through the transformation and transportation of food and water. Then, the above two ingredients are sent to the heart and blood vessels where they are transformed into blood. Therefore, the heart is also involved in the blood generation. In this case, the child had both spleen and heart deficiency, thus Guī Pí Tāng was applied. From the name of the formula, Guī Pí Tāng (Spleen-Restoring Decoction), we understand that although this prescription is for both heart and spleen, strengthening the spleen is the priority as it is the source of qi and blood. Therefore, the formula is based upon another famous prescription to supplement qi and strengthen spleen function, Sì Jūn Zĭ Tāng (Four Gentlemen Decoction), composed of dăng shēn, bái zhú, fú líng and gān căo, plus huáng qí. Guī Pí Tāng was created in the Song Dynasty and recorded in Formulas to Aid the Living (Jì Shēng Fāng) by Yan Yong-he in 1253. In the original formula, there was no dāng guī or yuăn zhì (Radix Polygalae). These two herbs were added in the 16th century text, Corrections and Annotations to Fine Formulas for Women (Jiào Zhù Fù Rén Liáng Fāng) by Xue Li-zhai. Without dāng guī in the initial formula, can it still nourish blood? This relates to another important theory in Chinese medicine, which is that blood

is generated by strengthening qi, particularly the spleen qi. Qi in this case indicates internal organ function, qi also being an important ingredient of blood. In The Yellow Emperor’s Inner Classic (Huáng Dì Nèi Jīng), blood is often referred to as “blood-qi”. The next question that arises is, since there are many blood-nourishing medicinals such as shú dì huáng (Radix Rehmanniae Praeparata), gŏu qĭ zĭ (Fructus Lycii), bái sháo yào (Radix Paeoniae Alba), then why does this formula choose dāng guī and lóng yăn ròu? This is because these two medicinals are warm-natured and so work more effectively with other qisupplementing medicinals because qi is also warm by nature. The other blood-nourishing medicinals mentioned above are yin-natured and greasy, which may interfere with the generation of qi. Dāng guī and huáng qí together become another famous formula for blood deficiency, Dāng Guī Bŭ Xuè Tāng (Chinese Angelica BloodSupplementing Decoction). In this formula, the ratio of dāng guī to huáng qí is 1:5. This again demonstrates the theory of nourishing blood through supplementing qi. In this case, jiāo shān zhā, chăo gŭ yá and chăo mài yá were added to promote the transformation and transportation of food and water for better digestion. Thus, iron and other micro-substances may be absorbed more effectively.

STUDY QUESTIONS 1. How can one differentiate between nutritional iron deficiency anemia and malnutrition? 2. How should medicinals be properly used in clinic to treat nutritional iron deficiency anemia?

Answers 1. Malnutrition, a kind of chronic consumptive disease, originates from improper feeding or the effects of other diseases, which lead to spleenstomach damage, qi and fluid consumption, and failure of the moistening and nourishing of the skin, sinews and bones, channels, and zang-fu. Clinical manifestations include a markedly slim body shape with a yellow complexion, dry hair, listlessness, agitation and disorder of feeding. The presentation is clearly different from food accumulation and aversion to food due to the distinct symptoms of obvious emaciation and disorder of the mind. Severe cases of malnutrition may also show symptoms of anemia. Nutritional iron deficiency anemia is a disease stemming from a weak prenatal constitution and improper postnatal feeding which leads to insufficiency in the spleen, kidney, liver and heart, and impaired generation and transformation of qi and blood. The most distinctive signs include pale or withered yellow lips, oral mucosa and nails with a lack of strength and poor appetite.

The anemia discussed here is microcytic hypochromic anemia, indicated by the disorder of serum iron, total iron-binding capacity or transferrin saturation, or by test results below Hb 110 g/L in children from 1 to 6 years of age, or Hb 120 g/L in children above 6 years of age. These indications are valid in the differentiation from malnutrition, while severe anemia may have similar symptoms as malnutrition. 2. Chinese medicinal prescription usually starts with the spleen and stomach by promoting the transportation to increase the appetite in order to engender the source of blood to cure the anemia. With iron supplementation, we can also reduce the side effects affecting the stomach and intestines by taking medicinals to rectify and nourish the spleen and stomach. We rarely use several medicinals for supplying blood and engendering blood together, such as dāng guī, shú dì, hé shŏu wū (Radix Polygoni Multiflori), bái sháo, ē jiāo (Colla Corii Asini), huáng jīng (Rhizoma Polygonati), sāng shèn (Fructus Mori), and gŏu qĭ zĭ. Instead, because abundant qi can engender blood and fluids, we generally choose to add one to three of them to the medicinals for rectifying the spleen and supplementing qi to ensure supplementation of the blood. The long course of anemia or a severe case may impair heart, liver and kidney functions. These should be treated separately by supplementing blood and nourishing the heart, nourishing liver and kidney, and supplementing the kidney and essence. Meanwhile, we should make sure to choose supplemental medicinals carefully to address potential stagnation; the inclusion of too many supplementing medicinals can also result in cloying.

CASE SCENARIOS The following cases present variations of this condition. After familiarizing yourself with the possible common pattern presentations and appropriate formulas for treatment, use the following exercises to test your overall understanding of the condition. 1. A child, 4 years old, has a withered yellow complexion, sparse hair, a lack of strength, poor appetite, palpitations sometimes severe, uneasy sleep at night, occasional abdominal pain, bloated abdomen and unformed stools for almost 6 months. A blood test shows Hb 95 g/L; fecal occult blood is positive, and hookworm ova are present in the feces. What is the pathogenesis? A. Insufficient iron intake B. Excessive iron loss due to hookworm infection C. Deficiency and weakness of the spleen and stomach D. Deficiency of the heart and spleen E. Hookworm infection 2. A child, 9 months old, who was breast-fed without any solid food, is found with a withered yellow complexion, light red nails and lips, and a poor appetite. Tests show nothing abnormal except RBC 3.1×109/L, Hb 98 g/L, MCV 75 fl. What is the primary diagnosis? A. Physiologic anemia

B. Nutritional iron deficiency anemia C. Aplastic anemia D. Mediterranean anemia E. Non-nutritive giant young RBC anemia 3. A child, 8 months old, who was fed mainly with milk, was sent to the hospital because of a withered yellow complexion, puffiness, a lifeless facial expression, a lack of strength, lassitude, and fibrillation of his head, hands and feet. Tests show the liver extending below the ribs by 2.5 cm, and a blood test shows Hb 80 g/L, RBC 2.5 × 1012/L, and MCV 90 fl. Marrow test shows proliferation of E-CFU. What is the diagnosis? A. Nutritional iron deficiency anemia B. Nonnutritive giant young RBC anemia C. Aplastic anemia D. Brain dysplasia E. Mixed anemia 4. A child, 10 months of age, was fed with goat milk because his mother was sick. He could crawl before he was ill, but now he cannot even sit steadily by himself. He also has a withered yellow complexion, poor appetite, lifeless facial expression, lassitude, and fibrillation of his hands and feet for one month. RBC 2.9 × 1012/L, Hb 85g/L, and MCV 105 fl. What is the proper treatment? A. Vitamin C B. Intramuscular glucoferrum

C. Ferrous Fumarate D. Intramuscular Vitamin B12 E. Blood transfusion 5. A child, 4 years old, has a withered yellow complexion, poor appetite, lassitude, a lack of strength, repeated feeling of cold, and repeated petechiae and ecchymoses. Tests show RBC 2.5 × 1012/L, Hb 78 g/L, WBC 2.2 × 109/L, PLT 38 × 1012/L, and RET 0.3%. Marrow test shows a low proliferation and significant reduction in GM-CUF, E-CUF and MK-CUF. What is the diagnosis? A. Red giant nonnutritive young anemia B. Nutritional iron deficiency anemia C. Aplastic anemia D. Allergic purpura E. Idiopathic thrombocytopenic purpura 6. A child, age 1 year, was diagnosed with nutritional iron deficiency anemia based upon signs and symptoms including pale complexion, mental fatigue, lack of strength, sparse hair and slimming of his body for 4 months. When should iron supplementation be stopped in treatment? A. When hemoglobin starts to rise B. When hemoglobin returns to normal C. 2 months after hemoglobin returns to normal D. 6 months after hemoglobin returns to normal E. 4 months after hemoglobin returns to normal

7. A child, 10 months old, was fed only milk and now has poor appetite and occasional diarrhea. Over the last 5 months, he was found to have a pale complexion, light red nails and lips, sluggishness, sparse hair, cold hands and feet, a light tongue with a white coating, and light red finger venules. A blood test shows Hb 55 g/L. What is the presenting pattern? A. Anemia due to spleen-stomach weakness B. Anemia due to spleen-kidney yang deficiency C. Anemia due to liver-kidney yin deficiency D. Anemia due to heart-spleen deficiency E. Anemia due to blood-qi deficiency

Answers 1. B 2. B 3. E 4. D 5. C 6. C 7. A

Chapter 16 Vitamin D-Deficiency Rickets Vitamin D-deficiency rickets is a chronic deficiency disease due to the disorder of metabolism of calcium and phosphorus, manifested mainly by bone lesions. Deficiency of Vitamin D may lead to disordered calcium and phosphorus metabolism, which may stop calcium salt deposition in bone growth areas, resulting in skeletal malformation. According to traditional Chinese medicine, the disease is classified within the five types of developmental delay and five kinds of flaccidity, along with pigeon-breast, tortoise back, disunited skull and malnutrition. Common in infants under 3 years of age, rickets is one of the four main diseases focused upon by the Chinese Ministry of Health. The disease is mainly caused by prenatally insufficient constitution and postnatal factors regarding feeding. The basic pathomechanism is spleenkidney deficiency that can influence the heart, liver and lungs in many cases. The kidneys are the foundation of the prenatal constitution, the deficiency of which will lead to insufficient marrow and thus cause a barrier to skeletal development, mainly manifested by flaccidity of the skull, delayed closing of the fontanelle, delay of teeth growth, and even skeletal malformation. The spleen is the foundation of the postnatal constitution and the source of qi and blood production. Disorder in diet and feeding may cause failure in transportation and a lack of nutrition of the body which can result in a variety of manifestations. These include wei qi deficiency with

ying-wei disorders which lead to profuse sweating, heart qi deficiency with heart-spirit disturbance which leads to disquieted heart spirit, and spleen deficiency with hyperactive liver qi that leads to awakening with fright during the night, vexation and agitation. The disease may be caused by several factors, including insufficient prenatal constitution, lack of nutrition to the fetus, improper nursing or feeding, lack of sunlight, a long course of another disease, and damage to the spleen and stomach. The reasons may influence each other and lead to zang-fu deficiency and damage with consumption of qi and blood and result in lack of nutrition for the bones and muscles. Rickets initiates from lung-spleen qi deficiency, and is often accompanied by insufficient heart blood and heart-spirit disturbance which is then followed by liver deficiency, malnourishment of ligaments and tendons, and vigorous hyperactivity of liver-wood. A long course of the disease is manifested by kidney essence depletion and deficiency, bone from marrow malnourishment, and eventually skeletal malformation. In the initial stage, the disease is manifested mainly by spleen deficiency, which should be treated mainly by rectifying the spleen and boosting qi. In the excitatory stage, the disease is manifested mainly by spleen and kidney deficiency, which should be treated mainly by supplementing the spleen and kidney. In the recovery and sequelae stages, the disease is mainly manifested by kidney deficiency, and should be treated by supplementing the kidney and essence while also rectifying the spleen. Meanwhile, the disease should also be treated by supplementing the lung, boosting qi, calming the liver, clearing the heart and calming the mind. In summary, the key points of rickets involve spleen and kidney insufficiency, essence and blood depletion and deficiency, and

malnourishment of bones; this is a deficiency pattern. Common deficiency patterns include lung-spleen qi deficiency, spleen deficiency with liver hyperactivity, and kidney essence deficiency and depletion.

CLINICAL ESSENTIALS Zang-fu differentiation must first be determined, and the degree and location of the disease must be also considered. The basic principle in treatment is to supplement deficiency with a focus on the spleen and kidney; with restoration of vigorous spleen qi and abundant kidney qi, sufficient essence and blood may be generated to nourish the zang-fu and marrow, symptoms of the disorder can be fully eradicated

● Dr. Zhu Rui-qun first boosts qi and supplements the kidney with his empirical formula, which consists of only five medicinals: huáng qí (Radix Astragali) and tù sī zĭ (Semen Cuscutae) each 20g, and lóng gŭ (Os Draconis; Fossilia Ossis Mastodi), cāng zhú (Rhizoma Atractylodis) and gŭ yá (Fructus Setariae Germinatus) each 10g. This is a precise formula that has shown remarkable effects in the treatment of rickets. Huáng qí acts to boost qi, rectify the spleen, consolidate the exterior and check sweating. Tù sī zĭ supplements the kidney, consolidates essence and promotes growth. They act together to supplement spleen and kidney while engendering essence, qi and blood. Lóng gŭ calms the mind and supplements calcium to relieve agitation and profuse sweating. Cāng zhú rectifies the spleen and removes dampness, especially indicated for those with a greasy tongue coating. Gŭ yá promotes digestion, harmonizes the middle, rectifies the spleen and promotes transportation, all in order to promote transformation.

● Compared with acupuncture, tui na treatment is preferable at any stage of treatment; pinching along the spine acts to warm the channels, unblock the collaterals, invigorate blood and supplement deficiency.

● In clinical practice, tui na is more commonly used than acupuncture in the treatment of rickets. The “School of Three Character Classic” from Shandong Province has developed a method that has gained widespread reputation by applying simpler manipulations to fewer points. The basic principle is to supplement liver and kidney, boost the heart and spleen, and calm the mind. Method: Knead èr rén shàng mă (⼆⼈上马), supplement pí jīng (脾 经), calm gān jīng(肝经), and clear tiān hé shuĭ (天河⽔). Kneading èr rén shàng mă acts to supplement original qi and strengthen ligaments, sinews and bones. Supplementing pí jīng (脾经) can supplement postnatal qi and blood. Calming gān jīng (肝经) and clearing tiān hé shuĭ (天河⽔) can calm the mind and stabilize fright.

CASE STUDY Male, age 3 months. Initial Visit: September 11th, 2009 Chief Complaint: Agitation, disturbed sleep and susceptibility to fright for several days. History: Over the last few days, there was uneasy sleep at night, a tendency to startle easily, agitation and profuse sweating, with no symptoms of fever, cough, vomiting, diarrhea or convulsion. Signs and Symptoms: Agitation, susceptibility to fright at night, profuse sweating, a poor appetite, slightly loose stools, normal urine, a light red tongue with a thin white coating, and light finger venules reaching the wind-pass. Past History: The child had a strong constitution. No information was available about her history of allergies to medicines or food. Normal vaccination history. Physical Examination: BT:36.8 ℃, pulse 116 beats/min, respiration 38 times/min, weight 5 kg, sparse hair, pillow-area baldness, no malformation of the skull, fontanelle 2.5 cm × 2.5 cm, slight occipital bone softness when pressed. Laboratory Examination: Blood calcium 2.0 mmol/L, phosphorus 0.9 mmol/L, alkaline phosphatase (ALP) 250 U/L.

Pattern Differentiation

Children have relatively delicate zang-fu with physical weakness and qi insufficiency, especially in the lung, spleen and kidney. Lung deficiency leads to insecurity of the wei-exterior, and ying-wei disorder causes profuse sweating. Spleen deficiency leads to less transformation and engendering of blood and qi, and insufficient blood and qi causes sparse hair and pillowarea baldness. Meanwhile, blood failure to nourish the liver causes agitation, irritability and uneasy sleep. The light red tongue with a thin white tongue coating, and light finger venules reaching the wind-pass indicate lung and spleen deficiency. The disease in this case is located mostly in the lung and spleen, but also involves liver; the main pattern is lung-spleen qi deficiency. The pathogenesis involves the lung and spleen depletion and deficiency with insufficient qi and blood, and hyperactive liver qi.

Diagnosis Rickets due to lung-spleen qi deficiency

Clinical Treatment Principles: Rectify the spleen and boost qi, supplement the lung and secure the exterior Formula: Modified Rén Shēn Wŭ Wèi Zĭ Tāng (Ginseng and Schisandra Decoction) [⼈参五味⼦汤加减]

14 doses [Formula Analysis] Huáng qí supplements the lung and boosts qi. Dăng shēn, bái zhú, fú líng and gān căo rectify the spleen and boost qi. Wŭ wèi zĭ, suān zăo rén and duàn mŭ lì check sweating and calm the mind. Chén pí and shén qū harmonize the spleen and promote transportation. Prepared Medicines

● Lóng Mŭ Zhuàng Gŭ Chōng Jì (Dragon Bone and Oyster Shell Bone-Strengthening Granules) for rickets with spleen qi deficiency and a lack of strength.

● Liù Wèi Dì Huáng Wán (Six-Ingredient Rehmannia Pill) for rickets with kidney qi deficiency and kidney essence depletion, and those with developmental delay.

● Shēn Líng Bái Zhú Săn (Ginseng, Poria and Atractylodes Macrocephalae Powder) for primary-stage rickets with spleen qi deficiency, profuse sweating, crying at night, poor appetite and indigestion.

Acupuncture Points: BL 23 (shèn shù), BL 20 (pí shù), ST 36 (zú sān lĭ), DU 14 (dà zhuī), RN 4 (guān yuán), RN 6 (qì hăi). Manipulations: Perpendicular insertion; do not retain the needle. Tui Na: Push pí jīng (脾经) and sān guān (三关) on both sides 20 times. For malformation of the lower limbs, knead ST 36 (zú sān lĭ) on both sides for 20 times; for malformation of the prothorax, push RN 17 (dàn zhōng) 20 times. One treatment per day for 10 days is one course of treatment. A second effective tui na treatment involves pinching along the spine: Push and pinch from the coccyx to DU 14 (dà zhuī) along both sides of the spine, 3 to 5 times. After 3 pushes, pinch the skin, and then press BL 23 (shèn shù) on both sides several times with the thumbs. Once per day for 6 days is one course of treatment. 2 or 3 courses of treatment are advised. Second Visit September 25th The child’s agitation and palpitations were improved, with better sleep at night; the appetite, stools and urine are all normal. Remaining signs and symptoms include profuse sweating, sparse hair with pillow-area baldness, occipital bone softness when pressed, a light red tongue with a thin white coating, and light finger venules reaching the wind-pass.

Above treatment should be continued since the child improved greatly; medicinals to supplement and check sweating should also be added. Principles: Rectify the spleen, boost qi, supplement the lung, secure the exterior Formula: Modified Rén Shēn Wŭ Wèi Zĭ Tāng (Ginseng and Schisandra Decoction) [⼈参五味⼦汤加减]

14 doses [Formula Analysis] Huáng qí supplements the lung and boosts qi. Dăng shēn, bái zhú, fú líng and gān căo rectify the spleen and boost qi. Wŭ wèi zĭ, suān zăo rén, fú xiăo mài and duàn mŭ lì check sweating and calm the mind. Chén pí and shén qū harmonize the spleen and promote transportation.

Prevention and Nursing Adhere to breast-feeding, supplementing with solid foods at the right time. Ensure that the child is exposed to sunlight and supplemented with Vitamin D. Do not make the baby sit, stand or walk at too young an age in case there is some skeletal malformation. Ensure that food is clean, and improve the child’s physical constitution. Be alert to other diseases such as pneumonia and diarrhea.

COMMENTARY AND DISCUSSION Similar to the five kinds of developmental delay and the five kinds of flaccidity in infants, Vitamin D-deficiency rickets is mainly associated with spleen and kidney dysfunction, as they are the roots of prenatal and postnatal essence. In this case, the poor appetite and watery stools are signs of spleen qi deficiency, while the profuse spontaneous sweating indicates lung qi deficiency. Therefore, the differential diagnosis is deficiency of lung and spleen qi, thus Rén Shēn Wŭ Wèi Zĭ Tāng was applied. Rén Shēn Wŭ Wèi Zĭ Tāng is derived from The Grand Compendium of Pediatrics (Yòu Yòu Jí Chéng) (1750) by Chen Fu-zheng. It functions to supplement the spleen and replenish qi, strengthen the middle and harmonize the stomach, augment the kidneys and nourish essence, and strengthen ligaments, sinews and bones. Dr. Chen applied the formula initially for the chronic cough and the spleen and middle qi deficiency manifestations of a pale face and lips. The formula is composed of Sì Jūn Zĭ Tāng (Four Gentlemen Decoction) as the base formula, with wŭ wèi zĭ added to inhibit and astringe qi and body fluids. The original formula also contains mài dōng (Radix Ophiopogonis) to moisten the lung and stomach and generate fluids. In this case, because body fluid deficiency was not a major problem, qisupplementing medicinals were added, including huáng qí, while body fluid-nourishing medicinals were omitted, including mài dōng. Lóng Mŭ Zhuàng Gŭ Chōng Jì acts effectively as a prepared formula to strengthen both spleen and kidney. It contains fifteen ingredients

including dăng shēn, huáng qí, mài dōng, guī băn (Plastrum Testudinis), bái zhú, shān yào (Rhizoma Dioscoreae), wŭ wèi zĭ, lóng gŭ, fú líng, dà zăo (Fructus Jujubae), gān căo, jī nèi jīn (Endothelium Corneum Gigeriae Galli) and others. It acts in this case to supplement calcium and strengthen bone, harmonize the spleen and stomach to improve the appetite and transformation and transportation of food and water, and replenish qi.

STUDY QUESTIONS 1. How can rickets be prevented? 2. What are the main treatment principles for this disease? 3. Analyze the influence of spleen-kidney deficiency in this disease.

Answers 1. The key points for preventing rickets include paying much attention to the mother’s health in pregnancy and ensuring that she gets adequate outside exercise. Also provide careful nursing and breast feeding with a timely introduction solid foods and Vitamin D supplementation, and adequate exposure to sunlight to improve the physical constitution. 2. Supplementation of the spleen and kidney are primary treatment principles, determined according to the degree of the deficiency. In the initial stage, the disease is manifested mainly by spleen deficiency, which should be treated mainly by rectifying the spleen and boosting qi. In the excitatory stage, the disease is manifested mainly by spleen and kidney deficiency, and should be treated mainly by supplementing the spleen and kidney. In the recovery and sequelae stages, the disease is mainly manifested by kidney deficiency, and should be treated by supplementing the kidney and essence along with rectifying the spleen. 3. The basic pathomechanism involves spleen-kidney deficiency, also influencing the heart, liver and lung in many cases. The kidney is the foundation of the prenatal constitution, stores essence, governs the bones

and engenders marrow. Deficiency of kidney qi will lead to insufficient marrow and cause barriers to skeletal development and even skeletal malformation. The spleen is the foundation of the postnatal constitution and the source of qi and blood production. Spleen deficiency may cause lack of nutrition, which could further affect the heart or liver, leading to insufficient heart qi or liver hyperactivity.

CASE SCENARIOS The following cases present variations of this condition. After familiarizing yourself with the possible common pattern presentations and appropriate formulas for treatment, use the following exercises to test your overall understanding of the condition. 1. A child, 5 months old, has profuse sweating, palpitations due to fright, agitation, sparse hair, pillow-area baldness, a broad fontanelle and slight skeletal malformation. What are the proper treatment principles? A. Supplement the lungs, nourish yin, secure the exterior, check sweating B. Calm the liver, subdue fire, suppress fright, calm the mind C. Clear the heart, drain fire, calm the liver, subdue yang D. Rectify the spleen, boost qi, supplement the lung, secure the exterior E. Supplement liver and kidney, supplement essence, boost marrow 2. A child, 4 months old, has fright palpitations, profuse sweating, sparse hair, pillow-area baldness, delay of fontanelle uniting, and lack of strength when sitting and walking. What is the presenting pattern? A. Insufficient kidney yin B. Deficient spleen and hyperactive liver

C. Vigorous heart and liver fire D. Insecurity of lung qi E. Insufficient liver yin 3. A child, 11 months old, has delayed teeth growth, a large square skull, beading of the ribs, and delay in walking. What is the presenting pattern? A. Insufficient kidney yang B. Liver-kidney yin deficiency C. Insufficient kidney essence D. Spleen qi deficiency E. Insufficient lung qi 4. A child, 4 months old, has fright palpitations, profuse sweating, sparse hair, pillow-area baldness, delay of fontanelle uniting, a lack of strength when sitting and walking, and convulsions. What formula should be chosen? A. Bŭ Shèn Dì Huáng Wán (Kidney-Supplementing Rehmannia Pill) B. Yì Pí Zhèn Jīng Săn (Spleen-Boosting Fright-Suppressing Powder) C. Liù Jūn Zĭ Tāng (Six Gentlemen Decoction) D. Yù Píng Fēng Săn (Jade Wind-Barrier Powder) E. Shèn Qì Wán (Kidney Qi Pill) 5. A child, 18 months old, has been treated for rickets, but remaining signs and symptoms include pigeon-breast, legs in an X-shape, delayed walking, a light red tongue, and a thready weak pulse.

What of the following should be prescribed for a long course? A. Yù Píng Fēng Săn (Jade Wind-Barrier Powder) B. Liù Wèi Dì Huáng Wán (Six-Ingredient Rehmannia Pill) C. Shèn Qì Wán (Kidney Qi Pill) D. Jiàn Pí Wán (Spleen-Fortifying Pill) E. Shēng Mài Yĭn (Pulse-Engendering Beverage) 6. A child, 5 months old, has profuse sweating, fright at night, sparse hair, agitation, broad fontanelle, puffy body shape, repeated contraction of colds. What formula should be chosen? A. Yù Píng Fēng Săn (Jade Wind-Barrier Powder) B. Rén Shēn Wŭ Wèi Zĭ Tāng (Ginseng and Schisandra Decoction) C. Mŭ Lì Săn (Oyster Shell Powder) D. Sì Jūn Zĭ Tāng (Four Gentlemen Decoction) E. Bā Zhēn Tāng (Eight-Gem Decoction)

Answers 1. D 2. B 3. C 4. B 5. B

6. B

PART III Heart-Liver Diseases Chapter 17 Sweating Syndrome Sweating syndrome refers to excessive and abnormal sweating with sweat often dripping from the entire body or parts of body, even when a child is quiet and inactive. The two types of childhood sweating syndrome are spontaneous sweating and night sweating. Spontaneous sweating occurs whether the child is awake or asleep and is not influenced by environmental factors or physical activity, and night sweating occurs during sleep and stops upon waking. Night sweating is most often caused by yin deficiency, while spontaneous sweating most likely involves yang deficiency. Although yin-yang balance and ying and wei harmony normally retain fluids inside the body, yin-yang imbalance or disharmonies of the zang-fu organs, qi and blood, or ying and wei, as well as insecurity of wei-yang, all can contribute to abnormal sweating with failure of the striae and interstices to open or close normally. Childhood sweating syndrome is primarily due to general weakness; the main etiologies are innate deficiency and inappropriate prevention and nursing. Common clinical pattern types include lung-wei insecurity, yingwei disharmony, qi and yin depletion, and internal steaming of damp-heat.

CLINICAL ESSENTIALS Sweating syndrome generally pertains to deficiency patterns. Spontaneous sweating patterns involve deficiencies of qi and yang, whereas night sweating involves patterns of yin and blood deficiency. Profuse sweating due to lung-wei insecurity primarily occurs at the head, neck, chest and back. Disharmony of yingwei contributes to profuse sweating even when the body is not warm. Qi and yin depletion leads to sweating over the entire body, often accompanied by signs and symptoms of deficiency heat. Internal steaming of damp-heat brings about sweating with a burning sensation. Supplementing deficiency is the basic treatment principle for sweating syndromes. For those with lung-wei insecurity, it is important to boost qi to consolidate wei qi. For disharmony of ying and wei, it is important to harmonize the ying- and wei- levels. With qi and yin depletion, it is important to boost qi and nourish yin. With steaming of internal damp-heat, it is important to eliminate damp-heat. In addition to orally-administered therapies, external supplementing therapies applied to the navel are also recommended. Dr. Zhou Bai-chuan often uses modifications of the formula Gān Mài Dà Zăo Tāng (Licorice, Wheat and Jujube Decoction) to treat childhood night sweating as follows:

● For summerheat impairing original qi manifesting with vexation, thirst, a red dry tongue and a thready rapid pulse, add tài zĭ shēn (Radix

Pseudostellariae), mài dōng (Radix Ophiopogonis), wŭ wèi zĭ (Fructus Schisandrae Chinensis), and yù zhú (Rhizoma Polygonati Odorati).

● For spleen-stomach weakness with a poor appetite, low spirits, loose stools, and a light red tongue with a thin coating, add dăng shēn (Radix Codonopsis), bái zhú (Rhizoma Atractylodis Macrocephalae), fú líng (Poria), huái shān yào (Rhizoma Dioscoreae), biăn dòu (Semen Lablab Album), and hé yè (Folium Nelumbinis).

● For food accumulation and indigestion with a poor appetite, add jiāo shān zhā (scorch-fried Fructus Crataegi), shén qū (Massa Medicata Fermentata), gŭ yá (Fructus Setariae Germinatus), mài yá (Fructus Hordei Germinatus), jī shĭ téng (Paederiae Scandentis Herba et Radix) and yú xīng căo (Herba Houttuyniae).

● For blood deficiency manifesting with a pale complexion, a light red and thin tongue with a white coating, and a thready pulse, add huáng qí (Radix Astragali) and dāng guī (Radix Angelicae Sinensis).

● For stomach yin deficiency manifesting with thirst, a desire for cold drinks, poor appetite, a red tongue with a white and yellow coating, and a thready rapid pulse, add shí hú (Caulis Dendrobii), yù zhú (Rhizoma Polygonati Odorati) and bĕi shā shēn (Radix Glehniae).

● For teeth-grinding, add wū méi (Fructus Mume). ● For low fever in the afternoon, add qīng hāo (Herba Artemisiae Annuae), bái wēi (Radix et Rhizoma Cynanchi Atrati), sāng yè (Folium Mori), mŭ dān pí (Cortex Moutan), mài dōng (Radix Ophiopogonis), wū méi (Fructus Mume), gŭ yá (Fructus Setariae Germinatus), and dà zăo (Fructus Jujubae), etc.

Along with improved living conditions in many parts of the world, another pathomechanism involves the growing number of parents who over-administer supplementing medicinals to their children. This can overburden the stomach and intestines, thus bringing about retention and steaming of damp-heat in the body that forces sweat outward.

CASE STUDY Male, age 4. Initial Visit: October 19th, 1982 Chief Complaint: Profuse sweating especially during the night. History: The child began to feel ill a year ago, with profuse sweating during the day and severe night sweats. He saw several different doctors, but with no obvious effect. Over the past month he has been dripping sweat profusely. Signs and Symptoms: A pale complexion, profuse dripping sweat, emaciation, weakness, poor appetite, weight loss, stools with undigested food, scanty urine, pale and dry lips, a thin white tongue coating, and a deep weak pulse. Past History: A weak constitution with susceptibility to colds; a history of rubella, measles and chicken pox. Physical Examination: Low spirits, apathy, emaciation, malnutrition. Heart, lungs and abdomen showed no abnormality. Laboratory Examination: Routine blood test and blood sedimentation test showed normal results. Chest X-ray showed no abnormality.

Pattern Differentiation After several febrile diseases, the child was deficient in both qi and yin, resulting in a pattern of qi and yin depletion. When qi is deficient, it is

unable to preserve yin, and yin depletion contributes to deficiency fire flaming internally; in such a case, body fluids move outward in the form of sweat. This is a case of root deficiency with qi and yin depletion as the main pathomechanism.

Diagnosis Sweating syndrome due to qi and yin depletion

Clinical Treatment Principles: Boost qi, nourish yin, consolidate yin, arrest sweating Formula: Modified Shēng Mài Săn (Pulse-Engendering Powder) [⽣脉散加减]

5 doses [Formula Analysis] Dăng shēn boosts qi and promotes fluid production. Mài dōng nourishes yin and clears heat.

Wŭ wèi zĭ astringes and arrests sweating. Huáng qí boosts qi to consolidate the exterior. Bái zhú and fú líng fortify the spleen and rectify qi. Bì táo gān astringes and arrests sweating. Lóng gŭ, mŭ lì, and fú xiăo mài supplement and nourish both the heart and spleen and astringe and arrest sweating. Prepared Medicines: Shēng Mài Yĭn Kŏu Fú Yè (Pulse-Engendering Beverage Oral Liquid), 5 ml, 3 times per day.

External Therapy

● Mix 3 grams of powdered wŭ bèi zĭ (Galla Chinensis) with water or vinegar to make a paste. Apply to the navel before sleep, and affix with adhesive plaster. Apply once daily for 7 days as one course of treatment.

● Powdered lóng gŭ and mŭ lì may also be applied topically before sleep. Second Visit 3 days after treatment, the main symptoms were greatly relieved. On the 5th day, no sweating was seen. The patient still had fatigue, a poor appetite, stools with undigested food, dry pale lips, a pale tongue with a white coating, and a deep thready pulse. After treatment, the sweating stopped, showing that the yin deficiency was alleviated; however, signs of spleen qi deficiency remained.

Principles: Nourish yin, boost qi, fortify the spleen, regulate the stomach Formula: Modified Shā Shēn Mài Dōng Tāng (Adenophora and Ophiopogon Decoction) [沙参麦冬汤加减]

7 doses [Formula Analysis] Mài dōng, yù zhú, bái sháo, shí hú and shā shēn enrich and nourish fluids of the lung and stomach. Dăng shēn, cāng zhú, and shān zhā fortify the spleen and regulate the stomach.

COMMENTARY AND DISCUSSION Concerning the differential diagnosis of this case, the disease history is an important clue. Even though he was only 4 years old, he had already suffered from rubella, measles, and chickenpox, all of which are warm diseases. A warm disease is apt to cause qi and yin deficiency because heatand fire-evils consume qi and body fluids. The clinical symptoms support this inference; his weak body constitution and susceptibility to catching colds along with fatigue and poor appetite are signs of lung and spleen qi deficiency. Emaciation, dry and pale lips, weight loss and scanty urine are symptoms of yin and fluid deficiency. Here, the deficiencies of qi, fluids and yin become a vicious cycle. Qi deficiency fails to inhibit the loss of fluids, resulting in dehydration. Longterm fluid deficiency causes yin deficiency, and then the empty heat burns and consumes qi, making the qi deficiency worse. The loss of fluids itself can also cause qi deficiency, as the body fluids carry qi. Among the deficiencies of qi and yin-fluids, strengthening qi and inhibiting fluid loss are the main treatment priorities. The modified prescription contains three essential formulas: Yù Píng Fēng Săn (Jade Wind-Barrier Powder) to supplement qi, stabilize the exterior and stop sweating, Mŭ Lì Săn (Oyster Shell Powder) to inhibit sweating and stabilize the exterior, and Shēng Mài Săn to augment qi, generate fluids, and preserve yin to stop sweating. The first two formulas fall into the category of formulas that stabilize the exterior and lung to inhibit sweating. In this modification, medicinals

that boost qi include dăng shēn, huáng qí and bái zhú; these medicinals act on the root, enabling qi to effectively hold and inhibit the loss of fluids. In order to stop sweating effectively and quickly, a group of stabilizing medicinals were also applied, including wŭ wèi zĭ, lóng gŭ, mŭ lì and fú xiăo mài. Bì táo gān functions strongly to stop sweating, but is seldom used in clinic. The Grand Compendium of Materia Medica (Bĕn Căo Gāng Mù) says that bì táo gān is “bitter, slightly warm and slightly toxic… it treats infantile sweating and also miscarriage in women” (性苦,微温,有⼩ 毒……治⼩⼉虚汗,妇⼈妊娠下⾎). The only fluid- and yin-nourishing medicinal in this formula is mài dōng, which proves that nourishing yin and body fluids was not the priority treatment at the initial visit. At the follow-up, after the sweating had stopped and other symptoms were significantly improved, attention was then paid to the nourishing of yin and fluids. A number of yin- and fluid-nourishing medicinals were applied, including shí hú, yù zhú, bái sháo, shā shēn and mài dōng. Dăng shēn, shān zhā and cāng zhú were also added to strengthen the spleen and improve the transformation and transportation of food and water. This ensures that the yin- and fluid-nourishing medicinals can be absorbed effectively.

STUDY QUESTIONS 1. How does one differentiate sweating syndrome from normal profuse sweating? 2. How does one differentiate between spontaneous sweating and night sweating? 3. To what must attention be paid in prevention and nursing?

Answers 1. Sweating syndrome refers to excessive sweating sometimes with dripping sweat from the entire body, or parts of the body, even when the child is quiet. Profuse sweating due to hot weather, too many clothes or blankets, hurried feeding, or strenuous exercise with no other ailments is not an indication of morbidity. 2. Sweating during sleep that stops upon waking is known as night sweating. Sweating whether the child is awake or asleep is known as spontaneous sweating. Night sweating is more likely to be caused by yin deficiency while spontaneous sweating by yang deficiency. Both spontaneous sweating and night sweating can appear in children. 3. Clothes should be changed and washed regularly to ensure personal hygiene. Keep the skin clean and dry; dab the sweat with a soft dry towel or gauze. Avoid the use of wet cold towels, lest the child may catch a cold. Because of fluid and qi consumption due to excessive sweating, it is

important to take in more water and nutritious and easily digestible foods. Avoid foods that are rich, spicy, fried, or grilled.

CASE SCENARIOS The following cases present variations of this condition. After familiarizing yourself with the possible common pattern presentations and appropriate formulas for treatment, use the following exercises to test your overall understanding of the condition. 1. A child, age 4, has had profuse sweating, mainly on the head, shoulders and back, for the past 2 years, and movement leads to even more sweating. He is susceptible to cold, feels weak and fatigued and has a sallow complexion, a pale tongue with a thin white coating, and a thready and weak pulse. What is the presenting pattern? A. Insecurity of lung-wei B. Disharmony between ying and wei qi C. Qi-yin depletion D. Insufficiency of both the heart and spleen E. Steaming of damp-heat 2. A child, age 4, has had profuse sweating for over a year. Signs and symptoms include profuse sweating during the day, more sweating after sleep, emaciation, low spirits, occasional low fever, thirst, red lips, a feverish feeling in the palms and soles, a pale tongue with little coating, and a rapid thready pulse. What is the best treatment formula?

A. Dāng Guī Liù Huáng Tāng (Chinese Angelica Six Yellow Decoction) B. Guì Zhī Tāng (Cinnamon Twig Decoction) C. Mŭ Lì Săn (Oyster Shell Powder) D. Shēng Mài Săn (Pulse-Engendering Powder) E. Xiè Huáng Săn (Yellow-Draining Powder) 3. A child, age 3, has had profuse sweating for 2 weeks with increased sweating at night, mainly on the head. Other signs and symptoms include sweating with a hot sensation on the skin, yellow sweat, bad breath, thirst, a red tongue with a greasy yellow coating, and a rapid slippery pulse. What are the correct principles of treatment? A. Boost qi to consolidate the exterior B. Harmonize ying and wei levels C. Boost qi and nourish yin D. Supplement and nourish the heart and spleen E. Drain heat from the spleen 4. A child, age 1, has been sick for half a year with signs and symptoms including spontaneous sweating, movement contributing to increased sweating, low spirits, fatigue, susceptibility to cold, and a pale tongue with a thin white coating. What are the correct principles of treatment? A. Harmonize ying and wei levels B. Boost qi to consolidate the exterior

C. Boost qi and nourish yin D. Fortify the spleen and boost qi E. Enrich and nourish the lungs 5. A child, age 2, has had profuse sweating during the day for over a week. Other signs and symptoms include sweating though the body is not warm, sensitivity to cold, aversion to wind, occasional low fever, poor appetite, low spirits, a pale tongue with a thin white coating, and a moderate pulse. What is the pathomechanism in this case? A. Insecurity of lung-wei B. Disharmony between ying and wei qi C. Qi and yin depletion D. Insufficiency of both heart and spleen E. Steaming of damp-heat

Answers 1. A 2. D 3. E 4. B 5. B

Chapter 18 Viral Myocarditis (VMC) Viral myocarditis (VMC) is due to an infection by common viruses and is characterized by the pathological change of local or diffusive myocarditis. Its clinical manifestations include poor spirit, fatigue, a pale complexion, palpitations, shortness of breath, cold limbs and profuse sweating. This disease is triggered by the invasion of warm-heat pathogens because of deficient upright qi. With delicate lungs, wei qi insecurity, and a relatively deficient spleen, children are more likely to be attacked by windheat and damp-heat pathogens. Exogenous wind-heat pathogens often invade lung-wei from the nose and throat. Exogenous damp-heat pathogens often invade from the mouth and nose and accumulate in the stomach and intestines. Then, pathogens collect in the heart, bringing about heart vessel obstruction and blocked blood flow. When heat-toxin damages ying-yin, it leads to deficiency of both heart qi and yin. Heart qi insufficiency makes it impossible for the blood to flow freely, and unsmooth blood flow may lead to qi stagnation and blood stasis. Consumption of heart yin leads to malnutrition of the heart vessels and failure of yin in checking yang, which is manifested by palpitation. Damaged heart yang makes it impossible to boost yang and transform qi. As a result, severe palpitations may appear.

If treatment of the disease is delayed, spleen and lung will be damaged. Spleen deficiency contributes to the impairment of fluid distribution while lung deficiency leads to failure of the lung qi to purify, bringing about generation of internal turbid phlegm. Binding of the phlegm and blood stasis blocks the collaterals. At the onset, a patient with deficient yang may have such fatal symptoms and signs as heart-kidney yang deficiency or even heart yang collapse. Protracted course of the disease is often the result of improper treatment like inducing sweating to an extreme degree or yin and yang damage due to the disease or medication. Deficiency of both qi and yin and malnutrition of the heart vessels bring about a deficiency pattern, mainly manifested by palpitations, or a deficiency-excess complex accompanied by collateral blockage. Common clinical pattern types include wind-heat invading the heart, damp-heat invading the heart, dual deficiency of qi and yin, heart yang deficiency, and phlegm-stasis blocking the collaterals.

CLINICAL ESSENTIALS Pattern differentiation of deficiency and excess is the top priority:

● A short course of the disease with chest oppression and pain, shortness of breath with profuse sputum, nausea or vomiting, abdominal pain, diarrhea, and a red tongue with a yellow coating indicates an excess pattern.

● A course of the disease lasting several months with palpitations, shortness of breath, low spirits, fatigue, a pale complexion, profuse sweating, and a pale or red tongue with a smooth or little coating indicates a deficiency pattern. In general, if the disease is in the acute stage, there is an excess pattern, and if at the recovery stage or chronic stage, in most cases, there is a deficiency pattern. If at the sequelae stage, there is typically a deficiencyexcess complex pattern. Differentiation of the severity is the second priority:

● Consciousness, a look of ease, rosy cheeks, excess and forceful pulse show that it is a mild disease.

● Pale complexion, panting, dyspnea, cold limbs, purple lips, fidgeting, a faint pulse as if expiring, or an intermittent pulse indicate that it is a severe condition. Treatment principles include reinforcing upright qi and dispelling pathogens, clearing heat and resolving toxins, invigorating blood and

dissolving stasis, warming and boosting heart yang, nourishing the heart and securing the root.

● At onset when pathogens invade the heart, treatment must clear heat, resolve toxins, nourish the heart, and invigorate blood.

● With damp-heat invading the heart, treatment must clear and remove damp-heat, resolve toxins and vent pathogens.

● With qi and yin depletion, treatment must boost qi, nourish yin, tranquilize the heart and calm the mind.

● With heart yang deficiency, treatment must warm yang, invigorate blood, nourish the heart and unblock the collaterals.

● With phlegm-stasis blocking the collaterals, treatment must eliminate phlegm, invigorate blood, dissolve stasis and unblock the collaterals. In Treatise on Cold Damage (Shāng Hán Lùn) it is stated that for intermittent pulse and palpitations, Zhì Gān Căo Tāng (Honey-Fried Licorice Decoction) is the best choice. Clinical experience has shown that Zhì Gān Căo Tāng has good results for some kinds of viral myocarditis, but only with correct pattern differentiation and proper formula modification. The ingredients include zhì gān căo (Radix et Rhizoma Glycyrrhizae Praeparata cum Melle), dà zăo (Fructus Jujubae), ē jiāo (Colla Corii Asini), shēng jiāng (Rhizoma Zingiberis Recens), rén shēn (Radix et Rhizoma Ginseng), shēng dì (Radix Rehmanniae), guì zhī (Ramulus Cinnamomi), mài dōng (Radix Ophiopogonis), and má rén (Fructus Cannabis). Zhì Gān Căo Tāng is also called Fù Mài Tāng (Pulse-Restoring Decoction). If it is modified by removing dà zăo, shēng jiāng, rén shēn and guì zhī while also adding bái sháo (Radix Paeoniae Alba), it is called Jiā

Jiăn Fù Mài Tāng (Pulse-Restoring Variant Decoction). Dr. He Shi-ying states that both formulas act to nourish the heart and the blood to rectify irregularities of the heart and pulse rate. He also adds that in Zhì Gān Căo Tāng, rén shēn, guì zhī, shēng jiāng and dà zăo are included in the formula to assist yang.

● Zhì Gān Căo Tāng is applicable for those with heart yang insufficiency manifested by irregularity of the heart and pulse rate, with a slow pulse.

● Jiā Jiăn Fù Mài Tāng is applicable for those with heart yin insufficiency manifested by irregularity of the heart and pulse rate, with a racing pulse.

CASE STUDY I Male, age 12. Initial Visit: March 12th, 1975 Chief Complaint: Pale complexion for one-half month, poor spirits, poor appetite, palpitations and shortness of breath. History: Following an electrocardiogram at the Internal Medicine Department, he was diagnosed with viral myocarditis. Signs and Symptoms: The child had a pale complexion, a pale moist tongue, and a slow thready pulse that was knotted frequently. He also complained of occasional dizziness, sensitivity to cold and cold limbs. Diagnosis Viral myocarditis due to heart yang insufficiency Clinical Treatment Principles: Warm heart yang, calm the mind Formula: Modified Zhì Gān Căo Tāng (Honey-Fried Licorice Decoction) and Guī Pí Tāng (Spleen-Restoring Decoction) [炙⽢草汤合归脾汤加减]

15 doses [Formula Analysis] Zhì gān căo, dăng shēn, huáng qí and dà zăo boost heart qi and supplement spleen qi to support the source of qi and blood production. Guì zhī and shēng jiāng, acrid in flavor and warm in nature, can warm the heart yang and unblock the blood vessels. Shēng jiāng and dà zăo harmonize ying and wei levels. Shú dì huáng, yuăn zhì and băi zĭ rén enrich yin, nourish the heart, cal the mind, and relieve palpitations. Lóng chĭ calms the mind with heavy sedatives. Second Visit March 27th After taking the above formula for half a month, his palpitation and shortness of breath were relieved, he had no more dizziness, his aversion to cold was improved, he still had a pale tongue, and his pulse, although still knotted, was improved. The same prescription was continued for another month. Third Visit April 27th

After taking the modified Zhì Gān Căo Tāng and Guī Pí Tāng for a month and a half, in most cases, he had no palpitations or shortness of breath, and his pulse was regular and no longer knotted. ECG was normal. Băi Zĭ Yăng Xīn Dān (Arborvitae Seed Heart-Nourishing Elixir) was prescribed for long-term treatment. Prepared Medicines: Băi Zĭ Yăng Xīn Dān (Arborvitae Seed HeartNourishing Elixir), one-half pill, twice daily; to be taken on a long-term basis. Source: He Shi-ying’s Pediatric Medical Records

CASE STUDY II Female, age 12. Initial Visit: Feb 8th, 2009 Chief Complaint: Palpitations accompanied by shortness of breath and fatigue for over 2 months. History: The child had a cold on Dec 4th, 2008 and began to have fever, running nose, sore throat and her temperature was 39℃. She was diagnosed with an infection of the upper respiratory tract at a local community hospital and prescribed Sì Jì Kàng Bìng Dú Hé Jì (Four Seasons Antivirus Mixture) and Tylenol. After taking these medicines, her fever came down, but she still had palpitation, shortness of breath, fatigue, sweating even with the slightest movement, sore throat and poor appetite. The degree of her illness varied from time to time. She was diagnosed with viral myocarditis in another hospital and was given some Chinese medicine, but without obvious effect. So she was taken to our department for further treatment. Signs and Symptoms: Palpitation, shortness of breath, fatigue, sweating even with the slightest movement, sore throat, poor appetite, a red tongue with a thin coating, and an intermittent pulse. Past History: The child had low weight at birth and was weak in constitution and susceptible to colds. Physical Examination: Pale complexion, sore throat, red and swollen tonsils (degree Ⅲ), no purulent secretion, audible dull first heart sound at the apex of the heart, frequent premature beat, HR 116 beats/min, no pathologic murmur.

Laboratory Examination: Hemogram: WBC 12.5 × 109/L, N 61%, L 39%; AST (aspartate transaminase) 50 IU/L, α-HBDH 279 IU/L. ECG showed ST-T segment elevation in lead Ⅱ, flat T waves in leads Ⅱ and a VF, T-wave inversion in lead Ⅲ, frequent premature ventricular contractions.

Pattern Differentiation The main characteristics of the child include a low-birth weight, frequent lung deficiency, insecurity of wei qi, indicating wind-heat invading lung-wei from the exterior and harming the heart. The child has wind-heat exterior pattern as well as dizziness, lack of strength, palpitation, shortness of breath and red tongue with thin coating. When the child is affected by wind-heat invading lung-wei, at the onset, she has fever, runny nose and red tongue with thin coating. Later, the pathogens impair the heart, which is manifested by palpitation, shortness of breath, lack of strength and sweating even with the slightest movement. This disease is located in the lung, in the lung-wei and the heart. It is a pattern of wind-heat invading the heart. This pattern should be differentiated from damp-heat invading the heart. In the latter, damp-heat toxin accumulates in the spleen and stomach, which is manifested by aching muscles all over the body, nausea, vomiting, abdominal pain, diarrhea, weakness of the limbs, red tongue with yellow and greasy coating, and a soggy and rapid or intermittent pulse.

Diagnosis

Palpitation due to wind-heat invading the heart

Clinical Treatment Principles: Reinforce upright qi, dispel pathogens, clear heat, resolve toxins, nourish yin, invigorate blood Formula: Modified Yín Qiào Săn (Lonicera and Forsythia Powder) [银翘散加减]

7 doses [Formula Analysis] Jīn yín huā, bò he and dàn dòu chĭ clear heat and vent the exterior. Băn lán gēn, hŭ zhàng, xuán shēn and dān shēn clear heat and resolve toxins, cool the blood and invigorate blood. Tài zĭ shēn, huáng qí and mài dōng boost qi and nourish yin. Wŭ wèi zĭ and băi zĭ rén nourish the heart and calm the mind.

Prepared Medicines: Shēng Mài Yĭn Kŏu Fú Yè (Pulse-Engendering Beverage Oral Liquid), 5-10 ml, twice daily; or Shēng Mài Zhù Shè Yè (Pulse-Engendering Injection), 5-10 ml added in 100-250 ml 10% glucose injection for intravenous infusion, once daily. Acupuncture Main Points: BL 15 (xīn shù), RN 14 (jù quē), PC 5 (jiān shĭ), HT 7 (shén mén), SP 10 (xuè hăi). Supplementary Points: PC 7 (dà líng), BL 43 (gāo huāng), ST 40 (fēng lóng), PC 6 (nèi guān) Manipulations: Needle all points with supplementation. Retain the needles for 30 min after the arrival of qi. Treat once every other day. Ear Acupuncture Points: Heart (xīn), sympathetic (jiāo găn), shen men (shén mén), subcortex (pí zhì xià). Manipulations: Needle once every other day, or attach wáng bù liú xíng (Semen Vaccariae) with adhesive tape and press the points 2-3 times per day. Second Visit After 7 days the patient’s sore throat was much relieved. She had an improved appetite and reduced palpitation, but was still sweating with the slightest movement; the tongue was red with little coating and her pulse was intermittent. After treatment, her symptoms were alleviated, meaning that the windheat was almost cleared; however, both qi and yin of the heart still remained deficient. Principles: Nourish yin, boost qi, tranquilize the heart, calm the mind

Formula: Modified Zhì Gān Căo Tāng (Honey-Fried Licorice Decoction) and Shēng Mài Săn (Pulse-Engendering Powder) [炙⽢草汤合⽣脉散加减]

7 doses [Formula Analysis] Zhì gān căo and dăng shēn boost qi and nourish the heart. Guì zhī warms yang and promotes blood circulation. Shēng dì and ē jiāo nourish yin and blood to make vessels full. Mài dōng and wŭ wèi zĭ nourish yin and astringe yin. Suān zăo rén tranquilizes the heart and calms the mind. Dān shēn invigorates blood and dissolves stasis.

Prevention and Nursing The child should take bed-rest for at least 3-6 weeks. Overwork, mental stress, and strenuous exercise should be avoided.

COMMENTARY AND DISCUSSION When external pathogens invade human body, the lung is attacked first because the lungs have greater connections to the external environment through the nose, throat, and pores. This is why a common cold often manifests with a runny nose and sore throat followed by coughing and wheezing. In a normal situation, the heart is not often or first attacked. The lungs act as “an imperial carriage roof” to protect the heart from the invasion of external pathogens. Viral myocarditis is an exception, however. The two main factors that determine whether or not viral myocarditis occurs are the body constitution and level of defensive qi, and the condition of the heart organ. Poor body constitution and deficiency of defensive qi allow the external pathogens to run deeper to attack other internal organs. This most often occurs in the heart organ as the heart is also located in the upper jiao and is the organ next to the lungs. If the heart organ happens to be weak, the opportunity for heart illness is greatly increased as Chinese medicine believes that the weak area of the human body is where the pathogens often invade and stay. In this case, the 12 year-old had low birth weight with a weak body constitution, aversion to cold and a tendency to catch colds; this is also a predisposing factor. Therefore, she has heart qi and yin deficiency with symptoms of shortness of breath, palpitation, fatigue and spontaneous sweating, in addition to the wind-heat invasion with signs of fever, runny nose, sore throat, red tongue body and red and swollen tonsils. The treatment is not just to remove wind-heat evils, but also to boost heart qi and yin. Because of this, Yín Qiào Săn and Shēng Mài Yĭn were combined.

Shēng Mài Yĭn consists of rén shēn, mài dōng and wŭ wèi zĭ, with functions of supplementing qi, generating fluids, preserving yin and checking spontaneous sweating. It is widely used in treating various heart diseases, and is particularly useful in the treatment of palpitations when accompanied by a stifling sensation in the chest, shortness of breath, sweating, and a knotted or intermittent pulse. In this prescription, Shēng Mài Yĭn was also taken as a prepared medicine orally and through intravenous trip, and heart qi and yin were saved. Originally from Treatise on Cold Damage, Zhì Gān Căo Tāng is another famous formula for the treatment of heart diseases associated with deficiencies of yin, yang, qi and blood in the heart. This formula was also applied in He Shi-ying’s Pediatric Medical Records for another case of viral myocarditis.

STUDY QUESTIONS 1. What is the prognosis of this disease? 2. How can we differentiate the severity? 3. In terms of prevention and nursing, to what must one pay attention?

Answers 1. Most cases have a good prognosis, but in a small number there may be cardiogenic shock, heart failure, or even sudden death. Some may not recover completely, and the disease may turn into refractory arrhythmia. 2. Consciousness, a look of ease, rosy cheeks, and forceful excess pulses indicate a mild disease. A pale complexion, panting, wheezing, cold limbs, purple lips, fidgeting, and a pulse that is faint as if expiring or intermittent indicates a more severe disease. 3. At the acute stage, the child should stay in bed for at least 3-6 weeks. Children with a severe condition should take bed-rest for 6 months to a year. When the child is anxious, tranquilizers may be administered to reduce consumed oxygen and the burden of the heart. Keep a close eye on the changes in the condition; with accelerated or slower heart rates, serious arrhythmia, wheezing, panting, and a dark purple complexion, all possible measures must be taken to rescue the patient immediately.

CASE SCENARIOS The following cases present variations of this condition. After familiarizing yourself with the possible common pattern presentations and appropriate formulas for treatment, use the following exercises to test your overall understanding of the condition. 1. A child, age 4, has chest oppression, poor spirits, fatigue, and occasional pain in the precordial region. His symptoms become worse after activity. He had mumps 2 weeks ago. ECG monitored a second-degree atrioventricular block. Which of the following tests is the most significant for diagnosis? A. Routine blood test B. Blood culture C. Virus isolation from blood D. Erythrocyte Sedimentation Rate (ESR) E. Cardiac enzyme studies 2. A child, age 7, has had fatigue because of a cold for 3 weeks with occasional chest pain and labored breathing, a poor appetite, normal urination and defecation, a sore throat, coughing, a yellow tongue coating and a rapid pulse. ECG monitored Ⅱ aVF and V5 T wave inversion and CKMB elevation in the blood. What is the preferred formula for this disease? A. Yín Qiào Săn (Lonicera and Forsythia Powder)

B. Shī Xiào Săn (Sudden Smile Powder) C. Shēng Mài Săn (Pulse-Engendering Powder) D. Gé Gēn Huáng Qín Huáng Lián Tāng (Pueraria, Scutellaria, and Coptis Decoction) E. Guì Zhī Gān Căo Lóng Gŭ Mŭ Lì Tāng (Cinnamon Twig, Licorice, Dragon Bone and Oyster Shell Decoction) 3. A child, age 9, has had myocarditis for 2 weeks with alternating chills and fever, chest oppression, labored breathing, aching muscles, epigastric focal distention, diarrhea, nausea, a red tongue with greasy yellow coating, and a rapid soggy pulse. What is the pathomechanism? A. Wind-heat invading the heart B. Damp-heat invading the heart C. Excess heat binding the chest D. Heart vessel obstruction E. Phlegm-fire harassing the heart 4. A child, age 4, has had myocarditis for 4 months with fatigue, labored breathing, palpitations, symptoms worse after activity, a smooth red tongue with little coating, and a rapid thready pulse. What is the principle of treatment? A. Clear heat, resolve toxins, nourish yin, invigorate blood B. Clear heat, remove dampness, tranquilize the heart, calm the mind C. Boost qi and nourish yin, tranquilize the heart, calm the mind

D. Warm and boost heart yang, tranquilize the heart, calm the mind E. Eliminate phlegm, invigorate blood, unblock the collaterals, dissolve stasis 5. A child, age 3, has had poor spirits, fatigue and palpitations for 2 weeks. In the past 2 days he was again invaded by external pathogens with signs and symptoms including fever, cough, a sudden pale complexion, tachypnea followed by profuse sweating, extreme cold of the hands and feet, purple lips, weak breathing, and a faint thready pulse as if expiring. What is the presenting pattern? A. Wind-heat invading the heart B. Heart vessel obstruction C. Qi and yin depletion D. Heart yang deficiency E. Heart yang collapse 6. A child, age 6, has had myocarditis for 6 months with a dark gloomy complexion, chest oppression, occasional stabbing pain in the precordial region, a dusky tongue with a greasy white coating, and a slippery pulse. What is the presenting pattern? A. Wind-heat invading the heart B. Damp-heat invading the heart C. Qi and yin depletion D. Heart yang deficiency E. Accumulated phlegm obstructing the collaterals

7. A child, age 2, has had myocarditis for 5 months with a sallow complexion, emaciation, lassitude, shortness of breath, fatigue, sweating with the slightest movement, vexing heat, thirst, shallow sleep, a poor appetite, loose stools and a smooth red tongue with little coating. What is the preferred formula for this disease? A. Shī Xiào Săn (Sudden Smile Powder) B. Guā Lóu Xiè Bái Bàn Xià Tāng (Trichosanthes, Chinese Chive and Pinellia Decoction) C. Gé Gēn Huáng Qín Huáng Lián Tāng (Pueraria, Scutellaria, and Coptis Decoction) D. Guì Zhī Gān Căo Lóng Gŭ Mŭ Lì Tāng (Cinnamon Twig, Licorice, Dragon Bone and Oyster Shell Decoction) E. Zhì Gān Căo Tāng (Honey-Fried Licorice Decoction) and Shēng Mài Săn (Pulse-Engendering Powder) 8. A child, age 9, has had myocarditis for one week with a persistent low fever, runny nose, sore throat, cough with sputum, dizziness, fatigue, palpitations, shortness of breath, chest oppression and pain, a red tongue with thin yellow coating, and a rapid pulse. What is the principle of treatment? A. Clear heat, resolve toxins, nourish yin, invigorate blood B. Scatter wind, clear heat, diffuse the lung, clear the throat C. Clear heat, eliminate dampness, tranquilize the heart, calm the mind D. Eliminate phlegm, invigorate blood, unblock the collaterals, dissolve stasis

E. Boost qi, nourish yin, tranquilize the heart, calm the mind 9. A child, age 7, has had myocarditis for over one year with severe palpitations, low spirits, lassitude, shortness of breath, fatigue, aversion to cold, cold limbs, a pale complexion, dizziness, profuse sweating, an enlarged pale tongue, a weak and moderate pulse. What is the preferred formula for this disease? A. Shēng Mài Săn (Pulse-Engendering Powder) B. Guī Pí Tāng (Spleen-Restoring Decoction) C. Zhì Gān Căo Tāng (Honey-Fried Licorice Decoction) D. Bŭ Zhōng Yì Qì Tāng (Center-Supplementing and Qi-Boosting Decoction) E. Guì Zhī Gān Căo Lóng Gŭ Mŭ Lì Tāng (Cinnamon Twig, Licorice, Dragon Bones and Oyster Shell Decoction) 10. A child, age 5, has chest oppression and pain, dizziness, fatigue, palpitations, stomach cavity pĭ (sensation of blockage, painless upon pressure), profuse sputum, occasional nausea, a dusky tongue with greasy white coating, and an intermittent pulse. What is the preferred formula for this disease? A. Yín Qiào Săn (Lonicera and Forsythia Powder) B. Wēn Dăn Tāng (Gallbladder-Warming Decoction) C. Qīng Yíng Tāng (Ying Level Heat-Clearing Decoction) D. Zhì Gān Căo Tāng (Honey-Fried Licorice Decoction) and Shēng Mài Săn (Pulse-Engendering Powder)

E. Guā Lóu Xiè Bái Bàn Xià Tāng (Trichosanthes, Chinese Chive and Pinellia Decoction) and Shī Xiào Săn (Sudden Smile Powder)

Answers 1. E 2. A 3. B 4. C 5. E 6. E 7. E 8. A 9. E 10. E

Chapter 19 Anaphylactoid Purpura Anaphylactoid purpura, also known as Henoch-Schönlein purpura (HSP), refers to a syndrome of blood vessel inflammation, or vasculitis, that typically affects small arterial vessels or capillaries in the body; it manifests as purpura of the skin with arthritis, abdominal pain, and blood in the stool or urine with proteinuria. In traditional Chinese medicine, anaphylactoid purpura is related to externally contracted wind-heat, or heat generated internally sometimes associated with improper diet, and damp-heat damaging the collaterals which causes blood to extravasate from the vessels into the skin, stools and urine. Those with a long course of disease have recurrent purpura due to failure of qi to contain blood and yin deficiency with internal heat. This disease is always in the blood level, and because the blood which leaves the vessels cannot disperse immediately, blood stasis is also present. In the acute stage, this disease is often an excess or yang pattern. The pathomechanisms are mainly blood heat and blood stasis. A longer course will change the disease into yin-type deficiency pattern with pathomechanisms of qi deficiency and yin deficiency. Common clinical pattern types include wind-heat damaging collaterals, damp-heat blockage, blood-heat extravasating, failure of qi to contain blood, and yin deficiency of the liver and kidney.

This disease is a self-limiting, and most cases have good prognosis; patients with a mild condition can recover within 7-10 days, but a small number will have recurrence within a year. Cases with poor prognoses often involve renal failure or intracerebral hemorrhage.

CLINICAL ESSENTIALS There are several key points in the diagnosis of anaphylactoid purpura. Firstly, the onset, course of disease and color of the purpura are used to differentiate patterns of deficiency and excess. An acute onset, a short course and bright color suggest an excess pattern, while a slow onset with a light color, a long course and recurrence suggests deficiency. Secondly, differentiation of the severity is based on the amount of bleeding and whether it is accompanied by kidney damage or intracerebral hemorrhage. Little bleeding shows a mild disease, while large amounts of bleeding including bloody stools, bloody urine and obvious proteinuria, or headache, coma and convulsion show a severe disease. In terms of an excess pattern, treatment should clear heat and cool the blood. In terms of deficiency, treatment should boost qi to contain blood, enrich yin and subdue fire. Mutual transformation of different patterns or their simultaneous presence should also be taken into consideration in clinical practice. In terms of treatment, a clear distinction between primary and secondary priorities should be drawn; only in this way can every factor be taken into consideration.

CASE STUDY I Male, age 3. Chief Complaint: Recurrent purpura. Signs and Symptoms: There are macules and papules on the lower limbs and also a sallow complexion, emaciation, a thin yellow tongue coating, and a thready wiry pulse.

Diagnosis Purpura due to liver and kidney yin deficiency

Clinical Treatment Principles: Enrich yin, nourish the kidney, cool the ying level, dissolve macules Formula

Second Visit Half a month later, all symptoms were gone; a half-year follow-up visit showed that the purpura did not recur. Source: Qian Yu-shou’s Medical Records

CASE STUDY II Male, age 7. Initial Visit: July 12th, 2003 Chief Complaint: Skin rash and joint pain for 1 week. History: The child had shrimp a week ago and then had a bright red rash on the surface of his skin that was itchy, mostly seen in the lower limbs and buttocks, and ankle pain. He took Chlor-Trimeton with no obvious effect. Signs and Symptoms: The child had a bright red rash with no fading in color when pressed with itchiness and a presence mostly seen on the lower limbs and buttocks. There was ankle pain in both legs, a poor appetite, intermittent abdominal pain, dark urine, dry stools, a red tongue with thin yellow coating, and a floating rapid pulse. Past History: Allergy to seafood. Physical Examination: Bright red rash on the body and four limbs, no fading in color when pressed, mostly seen on the buttocks and lower limbs, no enlargement of superficial lymph nodes, clear breath sounds in both lungs, rales not heard, HR 89 beats/min, regular heart beat, no pathologic murmur, soft abdomen, tenderness around the navel, non-rebound tenderness, normal bowel sounds, ankle pain. Laboratory Examination: Hb 121 g/L, RBC 6.5 × 1012/L , WBC 7.3 × 109/L, PLT 236 × 109/L; routine urine test: occult blood (++), red blood cells (+), protein (-).

Pattern Differentiation The invasion of wind-heat pathogens contributes to a fight between qi and blood. When heat damages collaterals, it causes blood to extravasate out of the vessels and percolate under the skin, later developing into purpura. When wind-heat pathogens invade the blood level, blood heat damages the collaterals in the intestines. Blood extravasation leads to abdominal pain. When heat damages collaterals in the joints, this contributes to qi and blood movement dysfunction and ankle pain. Dry stools, dark urine, the red tongue with thin yellow coating and a floating rapid pulse are all signs of wind-heat. This disease is located in the blood level.

Diagnosis Purpura due to wind-heat damaging the collaterals

Clinical Treatment An acute onset and bright red purpura with itchiness, accompaniment by other signs and symptoms of externally contracted wind-heat are the key points of differentiation. The top priority in treatment is to dispel wind and clear heat, cool the blood and soothe the collaterals, and the secondary priority is to invigorate blood and dissolve stasis. Western medicines for anti-coagulation and glucocorticoid treatment are also appropriate. Principles: Dispel wind, clear heat, cool the blood, soothe the collaterals

Formula: Modified Yín Qiào Săn (Lonicera and Forsythia Powder) [银翘散加减]

7 doses [Formula Analysis] Jīn yín huā, bò he and niú bàng zĭ scatter wind and dissipate pathogens. Lián qiào, băn lán gēn and gān căo clear heat and resolve toxins. Chì sháo and zĭ căo clear heat and cool the blood. Dì fū zĭ and chán tuì dispel wind and relieve itching. Qín jiāo, fáng jĭ and niú xī dispel wind and unblock the collaterals. Guăng mù xiāng and yán hú suŏ move qi to relieve pain. Prepared Medicines: Hé Yè Wán (Lotus leaf Pill), children over 7 years take 4.5 g per time, 2-3 times a day. Take with warm water before

meals. Second Visit After taking the formula for 7 days, the skin rash had improved but there was still recurrent purpura, sometimes indistinct and light purple in color with persistent abdominal pain, low spirits, fatigue, a sallow complexion, poor appetite, dizziness, palpitation, a pale tongue with thin white coating, and a thready weak pulse. After treatment, the skin rash had improved and his joint pain was gone, which indicates that the wind-heat had been cleared; however, at this point there was a deficiency pattern due to failure of qi to contain blood. Principles: Fortify the spleen, boost qi, nourish and contain blood Formula: Modified Guī Pí Tāng (Spleen-Restoring Decoction) [归脾汤加减]

7 doses [Formula Analysis]

Rén shēn, bái zhú, fú líng, gān căo and huáng qí boost qi and supplement the spleen. Dāng guī, bái sháo, shēng dì and lóng yăn ròu nourish the blood and nourish the heart. Suān zăo rén and fú shén tranquilize the heart and calm the mind. Shā rén and shén qū awaken the spleen and promote digestion.

COMMENTARY AND DISCUSSION In this case, the child has a history of food allergies; he ate shrimps about a week ago and then had a bright red, itchy rash mostly seen on the lower limbs and buttocks with ankle pain. The diagnosis was wind-heat damaging collaterals. The signs and symptoms of wind-heat are obvious and include acute onset, bright red purpura, an itchy sensation, a thin and yellow tongue coating and a floating rapid pulse. But this wind-heat is different from the common sense of wind or heat evil that invades the body through the sense organs, sweat pores or wind points such as GB 20 (fēng chí), DU 16 (fēng fŭ) and BL 12 (fēng mén); this kind of invasion leads to the common cold or flu. This wind-heat is diet-related and invades the human body through the digestive organs such as the stomach, spleen and intestines. This is not a season-, climate- nor environment-related type of wind invasion. In TCM, the analysis of etiology closely relates to symptomatology. Because food allergies have symptoms of skin rashes with itching which are similar to the symptoms of wind-evil, food allergies too are recognized as invasion of wind-evil. A famous prescription for food allergies, Tòng Xiè Yào Fāng (Important Formula for Painful Diarrhea), is a good example for explanation. This prescription has fáng fēng (Radix Saposhnikoviae) as an important ingredient that is commonly used to treat common cold caused by wind evil. It looks like there was dampness evil in this case, as well. First, dampness-evil tends to attack the lower part of body, and the rashes were

mostly seen on the lower limbs and buttocks. Second, the swollen and painful ankle joints indicate lower pouring of dampness. Third, the child had a poor appetite, intermittent abdominal pain and dark urine. These all indicate damp-heat evils attacking the middle and lower jiao. In addition, at the follow-up visit, the child had persistent abdominal pain, low spirits, fatigue, a sallow complexion and poor appetite, showing spleen deficiency, as dampness is apt to attack the spleen organ. This is why qín jiāo, fáng jĭ and niú xī were applied to expel wind-dampness and unblock channels and collaterals. Although anaphylactoid purpura may cause serious complications such as kidney inflammation or nephritis, which occurs in almost half of older affected children, in this case the diagnosis and differentiation are accurate, and treatment was very prompt. Therefore, the symptoms were relieved with the use of the two prescriptions.

STUDY QUESTIONS 1. What is the prognosis of this disease? 2. What are the manifestations of the failure of qi to contain blood on the patient’s second visit? 3. In terms of prevention and nursing, to what should attention be paid?

Answers 1. This is a self-limiting disease. Most patients have good prognosis. Patients with a mild condition can get well within 7-10 days, but a small number will have recurrence. The recurrence often happens about a few weeks or even several months or a year later. Some patients with renal failure or intracerebral hemorrhage have bad prognoses. 2. Recurrent, sometimes indistinct light purple purpura with persistent abdominal pain, low spirits, fatigue, a sallow complexion, poor appetite, dizziness and palpitation indicate a case of spleen qi and heart blood deficiency. The pale tongue with a thin white coating, and thready weak pulse indicate failure of qi to contain blood. 3. Avoid exposure to dubious allergens, such as pollen and paint. Eat light foods. Avoid crabs, shrimp or lobster, or any spicy or fatty food. In the acute stage or profuse bleeding stage, it is best for the child to remain at ease and stay in bed. To ensure proper treatment is given at all times, keep a

close eye on any changes in the condition; especially abdominal pain, diarrhea, black stools, and joint pain or swelling.

CASE SCENARIOS The following cases present variations of this condition. After familiarizing yourself with the possible common pattern presentations and appropriate formulas for treatment, use the following exercises to test your overall understanding of the condition. 1. A child, age 4, has a pattern of blood-heat extravasation with purpura along with a suddenly pale complexion, cold limbs, profuse sweating and a faint pulse as if expiring. What is the pathomechanism? A. Heart qi insufficiency B. Heart blood stasis obstruction C. Qi and yang collapse D. Damaged heart yin E. Inability to nourish the heart due to blood deficiency 2. A child, age 6, has bright red stasis macules of different sizes on the surface of the skin that do not fade in color when pressed, are symmetrical, appear in turn, and are mostly seen on the lower limbs and buttocks or around the joints; also accompanied by abdominal pain, vomiting and bloody stools. Lab tests show that PLT counts, bleeding and clotting and blood clot retraction times are normal. Routine urine tests monitored bloody urine and proteinuria. What is the diagnosis?

A. Chickenpox B. Scarlet fever C. Anaphylactoid purpura D. Thrombopenic purpura E. Rubella 3. A child, age 7, has stasis spots and stasis maculae mainly on the calves and buttocks that are bright red in color, itchy, and accompanied by fever, runny nose, abdominal pain, joint pain, a red tongue with thin yellow coating, and a floating rapid pulse. What is the pathomechanism? A. Abundant heat toxin causing blood to extravasate B. Longer course of disease contributing to qi deficiency, which in turns fails to control blood C. Yin deficiency resulting in vigorous fire damaging the collaterals D. Externally contracted wind-heat invading the collaterals E. Qi stagnation due to congealing cold and blood blocking the channels 4. A child, age 8, has acute and diffusive purpura over his body, especially in his lower limbs and buttocks that are bright red in color and of different sizes. There is a red tongue with thin yellow coating, and a floating rapid pulse. What is the preferred formula for this disease? A. Lián Qiào Bài Dú Săn (Weeping Forsythia Capsule ToxinResolving Powder)

B. Xī Jiăo Dì Huáng Tāng (Rhinoceros Horn and Rehmannia Decoction) C. Dà Bŭ Yīn Wán (Major Yin-Supplementing Pill) D. Guī Pí Tāng (Spleen-Restoring Decoction) E. Yín Qiào Săn (Lonicera and Forsythia Powder) 5. A child, age 6, has been sick for 5 days. The signs and symptoms include red purpura in the lower limbs, abdominal pain, joint pain, reduced food intake, dry stools, yellow urine, shallow sleep, a red tongue with a yellow coating, and a forceful rapid pulse. What is the preferred formula for this disease? A. Lián Qiào Bài Dú Săn (Weeping Forsythia Capsule ToxinResolving Powder) B. Xī Jiăo Dì Huáng Tāng (Rhinoceros Horn and Rehmannia Decoction) C. Dà Bŭ Yīn Wán (Major Yin-Supplementing Pill) D. Guī Pí Tāng (Spleen-Restoring Decoction) E. Sān Huáng Tāng (Three Yellows Decoction) 6. A child, age 7, has been sick for 2 months with recurrent, sometimes indistinct light purple purpura. Other signs and symptoms include a sallow complexion, low spirits, fatigue, poor appetite, dizziness, palpitations, a pale tongue with thin coating, and a deficient thready pulse. What is the pattern? A. Yin deficiency resulting in vigorous fire B. Qi stagnation and blood stasis

C. Wind-heat damaging collaterals D. Blood-heat extravasation E. Failure of qi to contain blood 7. A child, age 11, has intermittent purpura, nosebleed, bleeding gums, low fever, fidgeting, night sweating, dark urine, dry stools, a red tongue with little coating and a thready rapid pulse. What is the preferred formula for this disease? A. Xī Jiăo Dì Huáng Tāng (Rhinoceros Horn and Rehmannia Decoction) B. Dà Bŭ Yīn Wán (Major Yin-Supplementing Pill) C. Sì Miào Săn (Wonderfully Effective Four Powder) D. Guī Pí Tāng (Spleen-Restoring Decoction) E. Qiàn Gēn Săn (Madder Root Powder)

Answers 1. C 2. C 3. D 4. E 5. B 6. E 7. B

Chapter 20 Attention Deficit Hyperactivity Disorder Attention deficit hyperactivity disorder (ADHD), also known as minimal brain dysfunction syndrome, is one of the most common childhood disorders. Its clinical manifestations include difficulty in staying focused and paying attention, difficulty in controlling behavior, hyperactivity or over-activity, being constantly in motion, emotional lability, impulsiveness, willfulness, and learning disorders but with normal or almost normal intelligence. The etiologies include innate weakness, improper prevention and nursing, injuries, post-disease factors, or emotional disorders. The essential qi of the five viscera is the material base of one’s emotion, which is therefore closely related to the internal organs. Dysfunctions of the five viscera thus cause one’s emotions to become abnormal. Heart qi insufficiency and heart-spirit failing to be nourished contribute to moodiness and inattention. Kidney essence insufficiency and deficiency of the sea of marrow lead to weakness of mental activity. Kidney yin deficiency and water failing to nourish wood lead to ascendant hyperactivity of liver yang characterized by hyperactivity and emotionalism. Spleen failing to be nourished due to spleen deficiency leads to difficulty in keeping calm and quiet, also characterized by fickleness, improper speech and forgetfulness. Spleen deficiency and an exuberant liver aggravate hyperactivity and impulsivity.

Common clinical pattern types include liver-kidney yin deficiency, deficiency of both heart and spleen, and phlegm-fire harassing the interior.

CLINICAL ESSENTIALS In diagnosing this disease clinically, zang-fu and yin-yang pattern differentiation are the first priority. Based on the zang-fu pattern differentiation, if the disease is located in the heart, there will be inattention, moodiness, dreaminess and fidgeting. If the disease is located in the liver, there will be impulsivity, hyperactivity, difficulty in keeping quiet, irritability, and difficulty in self-control. If the disease is located in the spleen, there will be fickleness, leaving things halfdone and poor memory. If the disease is located in the kidney, the brain will be affected, which is characterized by poor school performance, poor memory, enuresis, lumbago, and fatigue. Based on yin-yang pattern differentiation, if yin essence is deficient, there will be inattention, poor self-control, moodiness, and fidgeting. If yang is agitated and hyperactive, there will be hyperpraxia, impulsivity, willfulness, and irritability. The essence of this disease is a deficiency pattern as well as a combination of root and branch manifestations. Clinically, a deficiencyexcess complex is often present. The basic treatment principle for this disease is to harmonize yin-yang. For those with deficiency of the heart and kidney, the treatment principle is to supplement and boost the heart and kidney. For those with kidney deficiency and liver hyperactivity, the treatment principle is to nourish the kidney and calm the liver. For those with qi deficiency of the heart and spleen, the treatment principle is to supplement and boost the

heart and spleen. If accompanied by turbid phlegm, phlegm-fire or blood stasis, the assistant treatment principle is to dissolve phlegm, clear heat and dispel stasis. The patient should also receive adequate psychotherapy, behavioral adjustment and education as needed. Acupuncture shows very good therapeutic effects for this condition, and greatly relieves the symptoms. Therapeutic methods include nourishing yin, subduing yang, quieting the spirit and calming the mind. The main channels involved include the du mai, the foot shaoyang gallbladder channel, and the foot jueyin liver channel. Main Points: DU 20 (băi huì), yìn táng (EX-HN3), GB 20 (fēng chí), LV 3 (tài chōng), KI 3 (tài xī), HT 7 (shén mén).

● DU 20 (băi huì) and yìn táng (EX-HN3) calm the mind and invigorate the brain.

● KI 3 (tài xī) acts to nourish yin and subdue yang. ● LV 3 (tài chōng) and GB 20 (fēng chí) soothe the liver and subdue yang.

● HT 7 (shén mén) calms the mind. Needle GB 20 (fēng chí) and LV 3 (tài chōng) with drainage, and needle KI 3 (tài xī) with supplementation. For all other points, use the even method.

● For yin deficiency and yang hyperactivity, add SP 6 (sān yīn jiāo) and GB 43 (xiá xī).

● For deficiency of both heart and spleen, add BL 15 (xīn shù) and BL 20 (pí shù).

● For phlegm-heat harassing internally, add PC 7 (dà líng) and ST 40 (fēng lóng).

● For dysphoria, add KI 6 (zhào hăi) and DU 24 (shén tíng). ● For poor appetite, add RN 12 (zhōng wăn) and ST 36 (zú sān lĭ). ● For enuresis, add RN 3 (zhōng jí) and BL 28 (páng guāng shù). Needle points on the limbs quickly and with no needle retention, retain needles in the head for 30 min. Treat once daily or once every other day.

CASE STUDY Male, age 10. Initial Visit: April, 1993 Chief Complaint: Inattention, restlessness and fidgeting for over 3 years. History: The child began to exhibit restlessness with excessive activity 3 years ago as an elementary student. He could not finish his homework on time and he was inattentive, which was not given due attention by his parents, so he did not see a doctor nor take medication. Recently, his teacher paid a visit to his house and suggested that he should see a doctor. Signs and Symptoms: Restlessness, unable to finish school work on time, inattention, poor appetite, sallow complexion, little sleep, occasional shortness of breath, normal urination and defecation, a pale tongue with little coating, and a thready pulse. Past History: A history of being picky about food, susceptible to colds, and easily frightened. Physical Examination: Consciousness, good spirits, a puffy sallow complexion, clear breath sounds in both lungs, wet or dry rales not heard, forceful heart sounds, pathologic murmurs not heard in auscultatory valve areas, soft abdomen, no abdominal tenderness, no rebound tenderness. Auxiliary Examination: No abnormality.

Pattern Differentiation

Pickiness about food suggests injury to the spleen and stomach due to improper prevention and nursing. The inattention, restlessness, pale tongue with little coating and thready pulse are all signs of heart-spirit malnourishment and yin-yang disharmony due to deficiencies of both qi and blood. This disease is located in the heart and spleen, with spleen qi deficiency in particular. This is a case of root deficiency; it must be differentiated from patterns of liver-kidney yin deficiency, which has such signs and symptoms as irritability, impulsivity, willfulness and difficulty in self-control, inattention, difficulty in keeping quiet, enuresis, lumbago and fatigue, or vexing heat in the five centers (chest, palms and soles), night sweats, constipation, a red tongue with thin coating, and a thready and wiry pulse.

Diagnosis ADHD due to heart and spleen deficiency with malnourishment of the heart-spirit

Clinical Treatment This is a case of root deficiency with heart and spleen dual deficiency, with predominant spleen qi deficiency. The treatment is based on the principle of when there is deficiency, treat with supplementation. This condition is best treated with acupuncture and tui na combined. Principles: Nourish heart and spleen, calm the mind

Formula: Modified Guī Pí Tāng (Spleen-Restoring Decoction) and Gān Mài Dà Zăo Tāng (Licorice, Wheat and Jujube Decoction) [归脾汤合⽢麦⼤枣汤加减]

14 doses [Formula Analysis] Dăng shēn, huáng qí, bái zhú, dà zăo and zhì gān căo supplement and boost spleen qi. Fú shén, yuăn zhì, suān zăo rén, dāng guī, fú xiăo mài, shēng lóng gŭ and shēng mŭ lì nourish the heart and calm the mind. Jiāo sān xiān, including jiāo mài yá, jiāo shān zhā and jiāo shén qū, promote digestion and fortify the stomach. Shān yào and shí chāng pú fortify the spleen and dissolve phlegm to open the orifices.

Prepared Medicines

● Rén Shēn Guī Pí Wán (Ginseng Spleen-Restoring Pill), 3-5g, 2-3 times a day.

● Băi Zĭ Yăng Xīn Wán (Arborvitae Seed Heart-Nourishing Pill), 35g, 2-3 times a day. Tui Na Supplement pí jīng (脾经), knead PC 6 (nèi guān), and HT 7 (shén mén), press-knead DU 20 (băi huì), rub abdomen, press-knead ST 36 (zú sān lĭ), knead BL 15 (xīn shù), BL 23 (shèn shù) and DU 4 (mìng mén). Pinch spine, rub the du mai and first lateral line of the bladder channel. Acupuncture Main Points: PC 6 (nèi guān), LV 3 (tài chōng), DU 14 (dà zhuī), LI 11 (qū chí). Supplementary Points: DU 20 (băi huì), sì shén cōng (EX-HN1), SP 1 (yĭn bái), DU 24 (shén tíng), BL 15 (xīn shù). Manipulations: Needle with rotation and drainage, and do not retain the needles. Treat once daily. Ear Acupuncture Main Points: Heart (xīn), shen men (shén mén), sympathetic (jiāo găn), brain (năo diăn, located between the tragus and antitragic notch). Manipulations: Insert shallowly, do not retain needles. Treat once a day, or attach wáng bù liú xíng (Semen Vaccariae) to the selected points with adhesive tape. Second Visit

After taking 14 days, the child had a better appetite, thus his spleen qi deficiency had been supplemented. His complexion and sleep were also improved, but he was still inattentive in class. Principles: Nourish the heart, restrain qi, quiet the heart, calm the mind Formula: Subtract jiāo sān xiān, including jiāo shān zhā, jiāo shén qū, and jiāo mài yá from the original formula. Add wŭ wèi zĭ (Fructus Schisandrae Chinensis) 6g, mài dōng (Radix Ophiopogonis) 8g, and zhēn zhū mŭ (Concha Margaritiferae Usta) 15g. 30 doses.

Follow-up After 30 doses, the parents reported that the child was attentive in class and could actively finish his homework. Testing showed the child at a normal level.

Prevention and Nursing Make sure that the affected child always eats a well-balanced diet, including plenty of fruits and vegetables.

COMMENTARY AND DISCUSSION Mental activities of the human body are based upon nourishment and support of vital substances generated by internal organs; mental activity is yang in nature, while vital substances are yin in nature. For normal activity, there must be a balance between mind and body. The capacity of attention, concentration and memory depend on the volume of blood and essence. It is said that the heart blood controls shortterm memory, while the kidney-essence governs remote memory. The heart and kidney are the two most important organs for mental activities, as they are common roots of mind and essence. A famous Qing Dynasty physician, Chen Shi-duo, said in Records of Differentiation (Biàn Zhèng Lù): “Human beings’ intellectual power derives from the harmonized condition of the heart and kidney. Disharmony of the heart and kidney induce the loss of intelligence. The intellectual power of the human body is not generated by the heart or kidneys alone, but by the two harmonized organs acting together. Kidney water nourishes the heart, and intellectual power is endless so long as heart fire warms the kidneys” (⼼肾交⽽智慧 ⽣,⼼肾离⽽智慧失。⼈之聪明,非⽣于⼼肾,⽽⽣于⼼肾之交也。 肾⽔资于⼼,则智慧⽣⽣不息,⼼⽕资于肾,则智慧亦⽣⽣无穷). Along with the heart and kidney, the spleen and liver also play important roles in the transformation of intellectual power because the spleen is the source of qi and blood, while the liver stores blood which can transform into essence. This is the basic holistic picture of intellectual

power as associated with the vital substances and internal organs. Any dysfunctions of the internal organs or disorders of vital substances can affect the spirit and mind. In this case, heart and spleen deficiency with malnourishment of the heart-spirit is the differential diagnosis, and, therefore, Guī Pí Tāng is the correct prescription. Because of its great effect in strengthening concentration and memory, Guī Pí Tāng has earned the nickname, “ExamPreparation Pills”. In reviewing Chinese classics from all dynasties, one finds many prescriptions for boosting memory and strengthening concentration, and many can be applied for the treatment of attention deficit hyperactivity disorder. Most of these prescriptions contain shí chāng pú, yuăn zhì, fú líng and rén shēn (Radix et Rhizoma Ginseng) as the basic ingredients. A few examples follow. Zhuàng Yuán Wán (Number One Scholar Pill), from Black Pearl from Red Waters (Chì Shuĭ Xuán Zhū) by Sun Yi-kui, contains shí chāng pú, yuăn zhì, bái fú líng (Sclerotium Poriae Cocos), rén shēn, bā jĭ tiān (Radix Morindae Officinalis), and dì gŭ pí (Cortex Lycii). Lìng Rén Bù Wàng Fāng (Memory-Boosting Formula), from Sun Simiao’s Important Formulas Worth a Thousand Gold Pieces for Emergency (Bèi Jí Qiān Jīn Yào Fāng), contains shí chāng pú, fú líng, fú shén (Sclerotium Poriae Pararadicis), rén shēn and yuăn zhì. Dú Shū Wán (Learning Ability-Improving Pill), also from Black Pearl from Red Waters by Sun Yi-kui, contains shí chāng pú, tù sī zĭ (Semen Cuscutae), yuăn zhì, dì gŭ pí, wŭ wèi zĭ, shēng dì (Radix Rehmanniae) and chuān xiōng (Rhizoma Chuanxiong).

Shēng Huì Tāng (Intelligence-Engendering Decoction), from Records of Differentiation by Chen Shi-tuo, contains shú dì huáng (Radix Rehmanniae Praeparata), shān zhū yú (Fructus Corni), yuăn zhì, suān zăo rén, băi zĭ rén (Semen Platycladi), fú shén, rén shēn, shí chāng pú and bái jiè zĭ (Semen Sinapis). Cōng Míng Tāng (Intelligence Decoction), from A Compilation of Benevolent Formulas (Tĭ Rén Huì Biān) by Peng Yong-guang, contains bái fú shén (Sclerotium Poriae Cocos Pararadicis), yuăn zhì and shí chāng pú. The attention deficit and hyperactivity associated with ADHD are most likely due to kidney-essence not rooting heart-fire, resulting in the floating and scattering of deficiency heat and fire, or to liver fire or phlegm-fire. This is essentially a pattern with an excess branch and a deficient root. For this reason, heart- and spirit-calming medicinals such as fú shén, yuăn zhì, lóng gŭ and mŭ lì are often applied. In this case, the young boy with attention deficit hyperactivity disorder had a tongue with little coating and a thready pulse, perhaps indicating heart, liver and kidney yin deficiency in addition to the heart and spleen deficiency. Yin-supplementing medicinals such as shú dì huáng, huáng jīng (Rhizoma Polygonati), shān zhū yú (Fructus Corni) and gŏu qĭ zĭ (Fructus Lycii) could be applied in that case. Because this young client also had spleen qi deficiency, shēng dì is better than shú dì huáng, as it is less greasy. This could explain why, after his spleen was strengthened, only then were yin-supplementing medicinals such as wŭ wèi zĭ and mài dōng added to the prescription.

STUDY QUESTIONS 1. What is the prognosis of this disease? 2. What is the etiology of this disease? 3. In terms of prevention and nursing, to what must one pay attention?

Answers 1. This disease has good prognosis. Most children improve or get completely well during puberty. 2. The etiology involves the patient’s innate weakness, improper prevention and nursing, injury, post-disease factors, or emotional disorders. 3. The child should be well cared-for and be given timely help with step-by-step training to correct his behavior and studies. Never reprimand or punish the child physically. Any small act should be given timely praise and encouragement. The child should be trained to develop good habits of life routines like getting up, eating and studying properly. Do not overindulge the child; keep the child under control lest any offensive, destructive or risky events might occur. Ensure a well-balanced diet, including plenty of fresh fruits and vegetables; avoid stimulating food or drinks.

CASE SCENARIOS The following cases present variations of this condition. After familiarizing yourself with the possible common pattern presentations and appropriate formulas for treatment, use the following exercises to test your overall understanding of the condition. 1. A child, age 8, has restlessness, impulsivity, inattention in class and fidgeting. He is restless in class and has difficulty studying, but he has normal intelligence. What is the proper diagnosis? A. Tourette syndrome (Multiple physical tics) B. Attention deficit hyperactivity disorder C. Habitual convulsion D. Childhood schizophrenia E. Autistic disorder 2. A child, age 9, has restlessness, difficulty keeping quiet, irritability, impulsivity, willfulness, vexing heat in the five centers (chest, palms and soles), lumbago, fatigue, poor memory, a red tongue, and a thready wiry pulse. What is the proper formula? A. Guī Pí Tāng (Spleen-Restoring Decoction) B. Gān Mài Dà Zăo Tāng (Licorice, Wheat and Jujube Decoction)

C. Huáng Lián Wēn Dăn Tāng (Coptis Gallbladder-Warming Decoction) D. Qĭ Jú Dì Huáng Wán (Lycium Berry, Chrysanthemum and Rehmannia Pill) E. Shēn Líng Bái Zhú Săn (Ginseng, Poria and Atractylodes Macrocephalae Powder) 3. A child, age 7, has inattention, improper speech, restlessness, emaciation, a sallow complexion, low spirits, fatigue, and poor memory; he is not quick-tempered. There is a pale tongue with little coating, and a weak pulse. What are the treatment principles for this disease? A. Enrich and nourish the liver and kidney, calm the liver, subdue yang B. Clear heat, drain fire, dissolve phlegm, calm the heart C. Nourish the heart, calm the mind, fortify the spleen, boost qi D. Clear liver heat, drain fire, suppress fright, calm the mind E. Boost qi, nourish yin, relax the liver, rectify the spleen 4. A child, age 7, has restlessness, talkativeness, fidgeting, impulsivity, willfulness, difficulty in self-control, fickleness, inattention, vexing heat in the chest, poor appetite, a bitter taste in the mouth, a red tongue with a yellow greasy coating, and a rapid slippery pulse. What is the pathomechanism here? A. Yin deficiency with vigorous fire B. Liver-kidney yin deficiency C. Qi constraint transforming into fire

D. Deficiency of both heart and spleen E. Phlegm-fire harassing the interior 5. A child, age 10, has hyperpraxia, impulsivity, irritability, difficulty in self-control, inattention in class, difficulty in study, lumbago, fatigue, enuresis, forgetfulness, a red tongue with little coating, and a deep thready pulse. What is the proper formula? A. Jìng Líng Kǒu Fú Yè (Tincture for a Quiet Mind) B. Rén Shēn Guī Pí Tāng (Ginseng Spleen-Restoring Decoction) C. Shēn Líng Bái Zhú Wán (Ginseng, Poria and Atractylodes Macrocephalae Pill) D. Lóng Dăn Xiè Gān Wán (Gentian Liver-Draining Pill) E. Xiè Qīng Wán (Draining the Green Pill)

Answers 1. B 2. D 3. C 4. E 5. A

Chapter 21 Tourette Syndrome (Multiple Physical Tics) Tourette syndrome is characterized by multiple physical tics sometimes accompanied by involuntary utterances of obscene words or socially inappropriate remarks (coprolalia). The clinical manifestations include rapid, repeated and irregular physical tics that include eye blinking, pouting, frowning, head-shaking, facing upward, lifting the shoulders, and others. Later, these symptoms worsen into involuntarily paroxysmal movements of the limbs and body, such as twisting of the body, or throwing or kicking behaviors and so on. Physical tics worsen under mental stress and disappear when sleeping. Tourette’s does not adversely affect intelligence. The main causes of Tourette syndrome include congenital insufficiencies, birth injury, suffocation, invasion of exterior pathogens and mental disturbance. Excess of the five minds and wind-phlegm brewing internally are the contributing factors. The location of this disease is in the liver, also closely related to the heart, spleen and kidney. Mental disturbance leads to disturbance of qi movement, and longstanding depression transforms into fire with stirring of liver fire. Deficiency of the spleen causes impaired transportation, and accumulated fluids turn into phlegm. Spleen deficiency leads to liver hyperactivity bringing about liver wind with phlegm harassing the upper part of the body.

Water failing to nourish wood due to liver-kidney yin deficiency causes deficiency wind to stir internally. In summary, the tics are a result of various factors acting on the liver that lead to the stirring of liver wind. There is no particular name for this disease in Chinese medicine. Tourette syndrome falls under the TCM categories of chronic infantile convulsion, spasms, convulsion, muscular twitching and cramping. Common clinical patterns include qi constraint transforming into fire, phlegm accumulation due to spleen deficiency, and wind stirring due to yin deficiency.

CLINICAL ESSENTIALS Eight-principle pattern differentiation plays a predominant role in the diagnosis of Tourette syndrome, with emphasis on differentiation of yin, yang, deficiency and excess. Internal wind, fire, phlegm and dampness are the branches, while the liver, spleen and kidney are the root, with the liver in particular. When the disease is closely related to these three internal organs, signs of both deficiency and excess can be seen. When wind, fire, phlegm and dampness co-exist, this disease can present with many variations and symptoms.

● With qi constraint transforming into fire, there is ascendant hyperactivity of liver yang. This is an excess pattern manifested by a red face, irritability, frequent tics, and a red tongue with a yellow coating.

● Spleen deficiency and phlegm accumulation shows a root deficiency pattern with branch excess, or a deficiency-excess complex characterized by a sallow complexion, emaciation, chest oppression as if coughing, irregular tics, and a pale tongue with a white or greasy coating.

● Stirring of wind due to liver-kidney yin deficiency is manifested by emaciation, red cheeks, weak tics, and a red tongue with a scanty coating. The main principles of treatment for Tourette’s are to calm the liver and extinguish wind. For stirring of wind due to yin deficiency, it is also important to enrich yin and subdue yang while softening the liver and extinguishing wind. The disease arrives gradually and abates slowly. Therapeutic effects depend upon compliance with the formula and application rules; this

disease will not resolve without eliminating turbid phlegm, extinguishing wind, nourishing the sinews and regulating the qi of the internal organs. Merely subduing the branch manifestations is not sufficient, thus both branch and root must be considered. Dr. Liu Bi-chen stresses a combination of theories in clinical practice, with a special focus on the lung.

● Commonly used medicinals include xīn yí (Flos Magnoliae), cāng ĕr zĭ (Fructus Xanthii), xuán shēn (Radix Scrophulariae), băn lán gēn (Radix Isatidis), shān dòu gēn (Radix et Rhizoma Sophorae Tonkinensis), gū sŭn (honey-fried Zizaniae Spica Infestata), guàn zhòng (Rhizoma Cyrtomii), fă bàn xià (Rhizoma Pinelliae Praeparatum), shēn jīn téng (Caulis Tinosporae Sinensis), mù guā (Fructus Chaenomelis) and others. Xīn yí, cāng ĕr zĭ, and niú bàng zĭ (Fructus Arctii) enter the lung channel to scatter wind and unblock the orifices. Băn lán gēn, xuán shēn and guàn zhòng are often used to clear heat and resolve toxins.

● Dr. Liu also recommends Jiāo Sān Xiān composed of shān zhā (scorch-fried Fructus Crataegi), shén qū (scorch-fried Massa Medicata Fermentata) and mài yá (scorch-fried Fructus Hordei Germinatus) to fortify the spleen and harmonize the stomach. Dr. Zhu Bo-chang needles both body and scalp points: Scalp acupuncture points include MS 1 (é zhōng xiàn), MS 5 (dĭng zhōng xiàn) and MS 8 ( dĭng páng xiàn I).

● For frequent blinking, add MS 12 (zhĕn shàng zhèng zhōng xiàn) and MS 2 (é páng xiàn I).

● For tics of the limbs, add MS 6 (dĭng niè qián xié xiàn). ● For abnormal utterances, add MS 11 (niè hòu xiàn). Perpendicularly insert one 25 mm needle and obliquely insert another needle nearby. Needles are retained for 60 min. Treat once every other day, with one month constituting one course of treatment. Standard acupuncture points include yìn táng (EX-HN3), LI 11 (qū chí), LI 4 (hé gŭ), GB 34 (yáng líng quán), SP 6 (sān yīn jiāo) and LV 3 (tài chōng).

● For tics of blinking and sniffing, add BL 2 (cuán zhú) and LI 20 (yíng xiāng).

● For tics of the mouth, add ST 4 (dì cāng) and ST 6 (jiá chē). ● For strange sounds coming out of larynx (e.g. snorting, grunting, or barking) add RN 23 (lián quán) and LU 7 (liè quē). Perpendicularly insert 0.38 mm × 40 mm needles; needle insertion depth depends on the girth of the patient and how deep the points can be needled. Twirl with lifting and thrusting using the even method; after qi is obtained, retain the needles for 30 min. Treat once every other day, with one month constituting one course of treatment.

CASE STUDY Male, age 4. Initial Visit: Sept 15th, 2003 Chief Complaint: Intermittent tics of the eyes and mouth for over a month. History: The child presented with frowning, eye blinking and mouth twitching a month ago. Other signs and symptoms included chest oppression, cough, a rattling sound in the throat, shaking of the body, an ill temper, and restless sleep. He had low spirits, a sallow complexion, emaciation, normal level of activity, light red lips, a pale tongue with a white coating, and a deep slippery pulse. Past History: The child suffered from dysentery half a year ago, with poor appetite even after the disease was cured. Physical Examination: BT 36.9 ℃, PR 86 beats/min, RR 22 times/min. Weight 13 kg. Abdominal distention with no tenderness, no shifting dullness, and liver and spleen not palpable. Laboratory Examination: Routine blood test: Hb 120 g/L, WBC 8 × 109/L; routine stool test: yellow and thin stool, microscopic examination (-). EEG: Normal.

Pattern Differentiation The child had suffered from acute dysentery, which caused damage to the spleen and stomach. The spleen failed to transport, leading to water-

dampness retention and fluids condensed into phlegm. Spleen deficiency and liver hyperactivity with phlegm-turbidity stirring wind led to frowning, eye blinking, and mouth twitching. Phlegm congested in the chest and obstructed qi dynamic, resulting in chest oppression. The sallow complexion, low spirits, a poor appetite, a pale tongue with a white coating and a deep slippery pulse are all signs of spleen deficiency with phlegm accumulation.

Diagnosis Tourette syndrome (multiple tics) due to spleen deficiency and phlegm accumulation

Clinical Treatment Principles: Fortify the spleen, dissolve phlegm, calm the liver and extinguish wind Formula: Modified Shí Wèi Wēn Dăn Tāng (Gallbladder-Warming Decoction with Ten Medicinals) [⼗味温胆汤加减]

5 doses [Formula Analysis] Dăng shēn and fú líng fortify the spleen and promote transportation. Chén pí and bàn xià dry dampness and dissolve phlegm. Zhĭ shí regulates qi and dissolves phlegm. Yuăn zhì and suān zăo rén dissolve phlegm and calm the heart. Gōu téng, bái sháo and shí jué míng calm the liver and extinguish wind. Gān căo harmonizes all formula medicinals. Prepared Medicinals: Hŭ Pò Bào Lóng Wán (Amber DragonEmbracing Pill), one pill, twice daily. Acupuncture Main Points: LV 3 (tài chōng), GB 20 (fēng chí), DU 20 (băi huì). Supplementary Points: yìn táng (EX-HN3), LI 20 (yíng xiāng), ST 2 (sì bái), ST 4 (dì cāng), PC 6 (nèi guān), ST 40 (fēng lóng), HT 7 (shén mén).

Manipulations: After obtaining qi, retain all needles for 20-30 min, rotating the needles with even supplementation and drainage once every 3-5 min. Second Visit September 20th 5 days after treatment, improvement was observed in the child’s tics and consciousness. The signs and symptoms included slight lassitude, soft stools once a day, normal urination, good sleep, a light red tongue with a white coating and a slightly moderate pulse. BT 37℃, heart rate 86 beats/min. After treatment, the liver wind was relieved, but signs of spleen qi deficiency still remained. Treatment was continued to supplement deficiency and calm the liver. Principles: Fortify the spleen, dissolve phlegm, calm the liver and extinguish wind Formula: Modified Shí Wèi Wēn Dăn Tāng (Gallbladder-Warming Decoction with Ten Medicinals)

7 doses

COMMENTARY AND DISCUSSION The 4 year-old in this case had a sallow facial complexion, poor spirits, poor appetite, a pale tongue with a white coating, and a slippery pulse. The medical history shows that he had suffered from dysentery a half-year previously, which weakened his body constitution. The condition was diagnosed as spleen deficiency with phlegm accumulation. Then the symptoms of Tourette syndrome occurred with frowning, blinking, muscle twitching, body shaking and restless sleep, all signs of internal wind which could be added to the differential diagnosis. Modified Shí Wèi Wēn Dăn Tāng was applied. Shí Wèi Wēn Dăn Tāng derives from Wēn Dăn Tāng (GallbladderWarming Decoction). Now, a series of questions arise. Why was the former formula applied rather than the latter? Because this was a pattern of spleen deficiency with phlegm accumulation. Why was the gallbladder treated? Why would the gallbladder be warmed when the function of Wēn Dăn Tāng is to clear heat? Wēn Dăn Tāng is actually an extension of Èr Chén Tāng (Two Matured Substances Decoction), composed of chén pí, bàn xià, fú líng and zhì gān căo. Èr Chén Tāng is a basic formula for eliminating phlegm. Wēn Dăn Tāng functions to regulate qi, transform phlegm, clear the gallbladder and harmonize the stomach in cases of dizziness, vertigo, nausea, vomiting and insomnia. It can also treat seizures accompanied by copious sputum and focal distention in the chest. The main ingredient, zhú rú (Caulis Bambusae in Taenia) acts to clear heat and dispel phlegm from the gallbladder and stomach.

While Shí Wèi Wēn Dăn Tāng omits zhú rú, dăng shēn is added to strengthen the spleen, so the formula contains no heat-clearing medicinals but rather has a greater strengthening effect. If bái zhú (Rhizoma Atractylodis Macrocephalae) were added, the formula would become Liù Jūn Zĭ Tāng (Six Gentlemen Decoction), a formula with even stronger functions for supplementing the spleen and transforming phlegm. Shí Wèi Wēn Dăn Tāng is more appropriate for this case because, as The Yellow Emperor’s Inner Classic (Huáng Dì Nèi Jīng) says, “all eleven organs depend on the gallbladder” (凡⼗⼀脏取决于胆也). The gallbladder together with the sanjiao involves the transportation of qi and body fluids. If the gallbladder and sanjiao are adjusted, the recovery of the spleen and stomach is promoted. In addition, ingredients including dăng shēn, fú líng, chén pí and zhì gān căo act to strengthen spleen function directly. Back to the question of why the gallbladder would be warmed when the function of Wēn Dăn Tāng is to clear heat, according to Sun Si-miao’s Important Formulas Worth a Thousand Gold Pieces for Emergency (Bèi Jí Qiān Jīn Yào Fāng), Wēn Dăn Tāng was initially designed for irritability and insomnia caused by cold in the gallbladder, and the original formula contained a large dosage of the warm medicinal shēng jiāng (Rhizoma Zingiberis Recens); thus it was named Gallbladder-Warming Decoction. Other ancient formula classics omitted shēng jiāng, thus emphasizing the formula function of clearing phlegm-heat. Furthermore, Wēn Dăn Tāng becomes the fundamental formula in Hāo Qín Qīng Dăn Tāng (Sweet Wormwood and Scutellaria GallbladderClearing Decoction), which acts to clear the gallbladder. Therefore, the name and the function finally match in Hāo Qín Qīng Dăn Tāng.

Shí Wèi Wēn Dăn Tāng does not strongly expel internal wind, so gōu téng, bái sháo and shí jué míng were added to calm the liver and extinguish wind.

STUDY QUESTIONS 1. What is the prognosis of this disease? 2. What were the manifestations of spleen deficiency on the second visit? 3. To what must attention be paid for prevention and nursing of this disease?

Answers 1. This patient in this case will have a very short course of disease. The symptoms will be alleviated by fortifying the spleen and calming the liver. The case has a good prognosis. 2. Slightly low spirits, soft stools, a light red tongue with a white coating, and a slightly moderate pulse. 3. Tics are aggravated with mental tension and disappear with sleep. The child’s intelligence is not affected. This disease mostly afflicts children from age 2 to 15. It has a long course, and can be relieved spontaneously or become aggravated. The child’s mental status should be monitored, while ensuring a stable lifestyle with healthy living habits. Sufficient rest is important, and the child should avoid overly exciting television programs or computer games. Overconsumption of stimulating food is prohibited. The child should be well-cared for and well-informed and he or she should be given encouragement; scolding or physical punishment should be avoided.

CASE SCENARIOS The following cases present variations of this condition. After familiarizing yourself with the possible common pattern presentations and appropriate formulas for treatment, use the following exercises to test your overall understanding of the condition. 1. What is the preferred Chinese prepared medicine for Tourette syndrome due to spleen deficiency with phlegm accumulation ? A. Dāng Guī Lóng Huì Wán (Chinese Angelica, Gentian, and Aloe Pill) B. Xiè Qīng Wán (Green-Draining Pill) C. Hŭ Pò Bào Lóng Wán (Amber Dragon-Embracing Pill) D. Qĭ Jú Dì Huáng Wán (Lycium Berry, Chrysanthemum and Rehmannia Pill) E. Liù Wèi Dì Huáng Wán (Six-Ingredient Rehmannia Pill) 2. A child, age 4, has signs and symptoms of head-shaking, shouldershrugging, eye-blinking, pouting, abnormal voice sounds sometimes severe and sometimes mild, and tics that could be checked at will. EEG reads normal. What is the correct diagnosis? A. Epilepsy B. Tourette syndrome (multiple physical tics) C. ADHD (attention deficit hyperactivity disorder)

D. Rheumatic chorea E. Habitual convulsion / habitual jerks 3. A child, age 3, has signs and symptoms including frowning, eyeblinking, head-shaking, shoulder-shrugging, coprolalia, fidgeting, irritability, constipation, a red tongue with a yellow coating, and a rapid wiry pulse. What is the best treatment formula for this case? A. Dí Tán Tāng (Phlegm-Flushing Decoction) B. Dà Dìng Fēng Zhū (Major Wind-Stabilizing Pill) C. Qĭ Jú Dì Huáng Wán (Lycium Berry, Chrysanthemum and Rehmannia Pill) D. Shí Wèi Wēn Dăn Tāng (Gallbladder-Warming Decoction with Ten Medicinals) E. Qīng Gān Dá Yù Tāng (Liver Heat-Clearing Hepatic-StagnationFacilitating Decoction) 4. A child, age 5, has signs and symptoms of eye-blinking, twitching at the corners of the mouth, physical tics, irregular fits, a poor appetite, loose stools, a pale tongue with a white coating, and a slippery weak pulse. What are the correct principles of treatment? A. Clear liver heat, drain fire, extinguish wind, suppress fright B. Fortify the spleen, dissolve phlegm, calm the liver, extinguish wind C. Enrich yin, subdue yang, soften the liver, extinguish wind D. Warm the center, fortify the spleen, strengthen earth and inhibit wood

E. Secure the root and supplement the origin, boost yin and subdue yang 5. A child, age 7, has signs and symptoms including shouldershrugging, head-shaking, eye-blinking, physical tics, emaciation, vexing heat in the chest, palms and soles, a crimson tongue with a peeled coating, and a thready rapid pulse. What is the presenting pattern? A. Qi constraint transforming into fire B. Spleen deficiency with phlegm accumulation C. Stirring of wind due to yin deficiency D. Phlegm-heat harassing internally E. Spleen-kidney yang deficiency 6. A child, age 6, has signs and symptoms including head-shaking, shoulder-shrugging, twitching of the abdominal muscles, vocal and phonic tics, a sallow complexion, emaciation, a poor appetite, a pale tongue with a white coating, and a deep slippery pulse. What is the best treatment formula for this case? A. Dà Dìng Fēng Zhū (Major Wind-Stabilizing Pill) B. Qīng Gān Dá Yù Tāng (Liver-Heat-Clearing and HepaticStagnation-Facilitating Decoction) C. Shè Gān Má Huáng Tāng (Blackberry Lily Rhizome and Ephedra Decoction) D. Shí Wèi Wēn Dăn Tāng (Gallbladder-Warming Decoction with Ten Medicinals)

E. Liù Wèi Dì Huáng Tāng (Six-Ingredient Rehmannia Decoction)

Answers 1. C 2. B 3. E 4. B 5. C 6. D

Chapter 22 Infantile Convulsion Infantile convulsion is a disease mainly manifested by tics of muscles affecting some parts or the entire body, often accompanied by loss of consciousness. The two types of the disease are acute infantile convulsion and chronic infantile convulsion. Acute infantile convulsion has a sudden onset and a short course, often manifesting with severe convulsion, a high fever, and loss of consciousness. Chronic infantile convulsions have a slow onset and a long course of disease, often manifesting with mild convulsions, a low fever, and loss of consciousness. Acute infantile convulsions result chiefly from an invasion of external pathogenic factors, from accumulated milk and food, or from sudden fright. Common clinical patterns of acute infantile convulsion include stirring wind due to wind-heat, blazing of both qi and ying, pathogenic invasion of the heart and liver, damp-heat epidemic toxins, and convulsion due to fright. The location of the disease is in the heart and liver. The invasion of external pathogenic factors transforms heat into internal wind, inwardly invading the pericardium and stirring liver wind. It is generally attributed to pathogenic excess without an insufficiency of upright qi. Common clinical patterns of chronic infantile convulsion include weakening of the spleen and stomach, spleen-kidney yang deficiency, and stirring of wind due to liver and kidney yin deficiency. The location of the disease is mainly in the spleen, stomach, liver, and kidney. As the spleen and stomach are weakened, the liver-wood insults the spleen-earth, leading

to internal stirring of liver wind; spleen-kidney yang deficiency deprives the sinews of nourishment, when insufficiency of liver-kidney yin generates wind. It is generally attributed to upright qi deficiency with residual pathogens.

CLINICAL ESSENTIALS The main signs and symptoms of acute infantile convulsion involve heat, phlegm, fright, and wind. The main treatment principles are to clear heat, eliminate phlegm, suppress fright, and extinguish wind. Treatment should be based upon the primary and secondary symptoms, and according to the severity of the condition. It is important to remember that heat is divided into two types, exterior heat and interior heat. Phlegm also consists of two types, phlegm-fire and turbid phlegm. Wind is also of two types, external wind and internal wind. Fright has a deficiency pattern including terror and fear, while the excess fright pattern manifests with surprised jumping and howling. It should also be noted that heat-clearing, exterior-releasing and exterior-venting methods are quite different from methods that resolve toxins with bitter-cold medicinals. Eliminating phlegm and opening the orifices with aromatics is also different from heart heat-clearing and phlegm-clearing methods. Livercalming to suppress fright is also different from blood-nourishing methods that act to calm the mind. In terms of extinguishing wind, methods that dispel wind are different from those which extinguish wind. In the treatment of acute infantile convulsion, the branch is treated by extinguishing wind to suppress fright, which should also be combined with treatment methods that address the root. In most cases, chronic infantile convulsion is a deficiency pattern which can include deficiency cold and deficiency heat. The main treatment principles are to treat the root by supplementing deficiency, warm the

middle to fortify the spleen, warm yang to dispel cold, enrich yin and subdue yang, and to soften the liver to extinguish wind.

● Dr. Kong Bo-hua treats infantile convulsion with shí gāo (Gypsum Fibrosum), shí jué míng (Concha Haliotidis), zhú rú (Caulis Bambusae in Taenia), cí shí (Magnetitum), gōu téng (Ramulus Uncariae Cum Uncis), bò he (Herba Menthae), lián zĭ xīn (Plumula Nelumbinis), lóng dăn căo (Radix et Rhizoma Gentianae), zhī mŭ (Rhizoma Anemarrhenae), dì gŭ pí (Cortex Lycii), xìng rén (Semen Armeniacae), quán xiē (Scorpio), sāng jì shēng (Herba Taxilli), and others. With constipation, dà huáng (Radix et Rhizoma Rhei), lái fú zĭ (Semen Raphani) and xuán míng fĕn (Natrii Sulfas Exsiccatus) are added. There are several commonly-used Chinese prepared medicines for this disease. Tài Jí Wán (Tai Ji Pill) relieves exterior and interior, clears stagnated heat, and moves the bowels. Niú Huáng Zhèn Jìng Wăn (Bovine Bezoar Tranquilizing Pill) suppresses fright and calms the mind. Niú Huáng Bào Lóng Wán (Bovine Bezoar Dragon-Embracing Pill) suppresses fright and extinguishes wind. Zhì Băo Dìng (Supreme Jewel Lozenge) cools and removes stagnation. Cí Zhū Fĕn (Loadstones and Cinnabar Powder) suppresses fright and extinguishes wind. Zĭ Xuĕ Dān (Purple Snow Elixir) clears heat and moves the bowels. Ān Gōng Niú Huáng Wán (Peaceful Palace Bovine Bezoar Pill) clears heat, extinguishes wind, and opens the orifices. Sū Hé Xiāng Wán (Storax Pill) resolves phlegm with fragrant medicinals. Líng Yáng Jiăo Fĕn (Antelope Horn Powder) clears liver heat and extinguishes wind. Yì Yuán Săn (Original Qi Boosting Powder) clears and eliminates dampness and harmonizes the middle.

● Dr. Yan Yan-feng treats acute infantile convulsion with acupuncture, using shallow insertion with drainage. The needle is inserted quickly and

stimulated with rotation for about half a minute; the needle is then removed without retention. In a study, dozens of patients were treated in this way with positive therapeutic effects. Dr. Yan selected DU 26 (shuĭ gōu), LI 4 (hé gŭ), and KI 1 (yŏng quán) as main points. For severe cases, point-pricking was applied to shí xuān (EX-UE11) and ĕr jiān (EXHN6) bilaterally along with needling of DU 20 (băi huì), and DU 14 (dà zhuī).

CASE STUDY Male, age 3. Initial Visit: November 6th, 2009. Chief Complaint: Fever for 2 days, one convulsive episode. History: The child began to have a fever the previous evening. Other signs and symptoms included BT 38℃, a runny nose, cough with no sputum, dry stools, and no stool for 2 days. He was treated with Shuāng Huáng Lián Kŏu Fú Yè (Lonicera Japonica, Scutellaria and Forsythia Oral Liquid), but his temperature remained elevated. He was irritable in the afternoon with a temperature of 39℃, tics of the limbs, and sudden unconsciousness. Past History: Susceptibility to cold. Physical Examination: BT 40 ℃, HR 150 beats/min, regular heart rhythm, no pathologic murmurs heard in auscultatory valve areas, unconsciousness, tics of limbs, blue lips, a red tongue with a yellow coating, and a floating rapid pulse. Laboratory Examination: WBC 12 × 109/L. ECG and EEG were normal.

Pattern Differentiation The patient, age 3, has delicate skin with a deficiency of wei qi which allows for external pathogenic invasion. When pathogens invade the fleshy exterior, the contention between the pathogenic qi and the upright qi results

in fever. When pathogens enter the body from the mouth and nose, the lung fails to diffuse and govern descent, also bringing about cough and runny nose. The lung and large intestine stand in an interior-exterior relationship. When the lung fails to diffuse and govern descent, bowel-qi also becomes obstructed, as manifested by constipation. Intense pathogenic heat harasses the heart-spirit and stirs liver wind, bringing about unconsciousness, tics of limbs, and blue lips. The red tongue with yellow coating and the floating rapid pulse are signs that reflect an invasion of exterior pathogenic wind-heat.

Diagnosis Acute infantile convulsion due to wind-heat stirring wind

Clinical Treatment This is a case of branch excess and root deficiency. Wind-evil invading the lung and intense heat harassing the heart-spirit led to a stirring of liver wind. Based on the principle of searching for the root, the priority here is to eliminate heat. In urgent conditions, however, primarily treat the branch. Based on this principle, this case should be supported by methods that extinguish wind and arrest convulsions. Principles: Scatter wind, clear heat, extinguish wind, arrest convulsion Formula: Modified Yín Qiào Săn (Lonicera and Forsythia Powder) [银翘散加减]

3 doses [Formula Analysis] Jīn yín huā and lián qiào clear heat and resolve toxins. Bò he and niú bàng zĭ scatter wind and clear heat. Dàn dòu chĭ and zhī zĭ dispel wind-heat. Jīng jiè suì and fáng fēng dredge the exterior and scatter wind. Gōu téng, jiāng cán, and chán tuì dispel wind and suppress fright. Prepared Medicines

● Xiăo Ér Huí Chūn Dān (Children’s Return-of-Spring Elixir), 0.9 g, twice daily.

● Ān Gōng Niú Huáng Wán (Peaceful Palace Bovine Bezoar Pill), 1/2 pill. Acupuncture Points: DU 26 (shuĭ gōu), LI 4 (hé gŭ), LV 3 (tài chōng), shí xuān (EX-UE11), DU 14 (dà zhuī).

Manipulations: Needle all points with drainage. After obtaining qi, retain the needles for 20 to 30 min, applying rotation once every 3 to 5 min. Second Visit 3 days after the above treatment, the child no longer had convulsions. He was conscious with a temperature of 38 ℃, a heart rate 110 beats/min, and slightly dry stools. He defecated every other day and had slightly yellow urine, a runny nose, a slight cough, a slightly red tongue with a thin yellow coating, and a slightly rapid and floating pulse. After treatment, the child’s symptoms were alleviated, but some signs characteristic of residual pathogens with slight exterior heat still remained. Principles: Scatter wind and clear heat Formula: Modified Yín Qiào Săn (Lonicera and Forsythia Powder)

3 doses [Formula Analysis] Jīn yín huā and lián qiào clear heat and resolve toxins. Bò he and niú bàng zĭ scatter wind and clear heat. Dàn dòu chĭ and zhī zĭ dispel wind-heat.

Jīng jiè suì and fáng fēng dredge the exterior and scatter wind. Zhú yè dispels heat and promotes urination. Zhì gān căo clears heat.

COMMENTARY AND DISCUSSION This is a mild case of infantile convulsion due to wind-heat with signs of internal wind and a shen disorder. Manifestations included a high fever, cough, a runny nose, tics of the extremities, and unconsciousness. The internal wind symptoms were caused by external wind-heat evils, thus eliminating the external wind and releasing the exterior were the priorities of treatment. Yín Qiào Săn is an appropriate formula for this case. When applying Chinese herbal medicine, two medicinals with similar functions are often used together to increase the therapeutic result. In this modified formula, there are several medicinal pairs. Jīn yín huā and lián qiào are cool and acrid medicinals that release the exterior. Both act to clear heat and expel heat-toxins. Jīng jiè suì and fáng fēng work together to eliminate wind-evil and release the exterior. In this case, they are selected to support the first medicinal pairing. Niú bàng zĭ and jié gĕng (Radix Platycodonis) are often applied together to disperse wind-heat evils and benefit the throat. In this case, jié gĕng was replaced with bò he to more strongly eliminate wind-heat. Dàn dòu chĭ and zhī zĭ are used together to clear heat and alleviate restlessness and irritability. In Treatise on Cold Damage (Shāng Hán Lùn), these two medicinals are found paired in the formula Zhī Zĭ Dòu Chĭ Tāng (Gardenia and Fermented Soybean Decoction), which is used for yangming channel patterns with heat in the diaphragm and the symptom of irritability.

Chán tuì and jiāng cán are both animal-based medicinals that work together to extinguish wind and stop spasm and convulsion. The medicinal-pairing or dui yao theory is a very important part of Chinese medicinal prescription; when appropriate medicinals are selected and paired, the effect is greater than the sum of the parts. In this case, Yín Qiào Săn was not strong enough to treat the shen disorder. Therefore, the prepared medicines Xiăo Ér Huí Chūn Dān and Ān Gōng Niú Huáng Wán were applied to treat the shen disorder.

STUDY QUESTIONS 1. What is the prognosis of this case? 2. What were the wind-heat manifestations at the return visit? 3. In terms of nursing, to what should attention be paid?

Answers 1. This case has a short course of disease and the prognosis is good. Recovery would occur after the pathogenic heat has been cleared. 2. Slightly yellow urine, a runny nose, a slight cough, a slightly red tongue with thin yellow coating, and the slightly rapid floating pulse all indicate wind-heat. 3. Acute infantile convulsion is often caused by external contraction followed by a high fever, so seasonal pathogens should be avoided whenever possible. For a child with convulsion due to high fever, at the onset of fever, it is necessary to treat with medicinals that can help to reduce the body temperature. When necessary, medicinals that arrest convulsion should be employed. During the convulsion, do not force the child to remain flat lest fracture should occur; it is best to lay the child on his or her back with the head to one side. Place tongue depressors wrapped with gauze between the upper and lower teeth lest the child bite his or her tongue. Keep the airways unobstructed.

For a child with phlegm-drool accumulation, it is important to clear the phlegm from the airway and give oxygen. Keep the room quiet, avoiding overstimulation. Closely observe the complexion, the breath, and the pulse so as to stop the disease from changing abruptly.

CASE SCENARIOS The following cases present variations of this condition. After familiarizing yourself with the possible common pattern presentations and appropriate formulas for treatment, use the following exercises to test your overall understanding of the condition. 1. A child, age 1, had a fever for one day with a temperature of 39.5 ℃. His other signs and symptoms included headache, runny nose, cough, unconsciousness, delirious speech, convulsion once with his eyes staring into space and jerking of his limbs, which was relieved a minute later. The case was diagnosed as acute infantile convulsion. What is the presenting pattern? A. Stirring wind due to wind-heat B. Blazing of both qi and ying C. Pathogens trapped in the heart and lung D. Epidemic damp-heat toxin E. Infantile convulsion due to fright 2. A child, age 4, had fever for 2 days with a temperature of 40℃. His other signs and symptoms included unconsciousness, convulsions three times which lasted for 1-2 min, abdominal pain, vomiting, stool with pus and blood, a red tongue with a greasy yellow coating, and a rapid slippery pulse. This case was diagnosed as acute infantile convulsion due to winddampness and epidemic toxins.

What is the preferred treatment formula? A. Yín Qiào Săn (Lonicera and Forsythia Powder) B. Qīng Wēn Bài Dú Yĭn (Epidemic-Clearing Toxin-Resolving Beverage) C. Líng Jiăo Gōu Téng Tāng (Antelope Horn and Uncaria Decoction) D. Huáng Lián Jiĕ Dú Tāng (Coptis Toxin-Resolving Decoction) and Bái Tóu Wēng Tāng (Pulsatilla Decoction) E. Huò Pò Xià Líng Tāng (Agastache, Magnolia, Pinellia and Poria Decoction) and Hŭ Pò Bào Lóng Wán (Amber Dragon-Embracing Pill) 3. A child, age 2, cried at night due to fright. His other signs symptoms included shivering, unconsciousness, convulsion, dark stools, and an irregular pulse rate. What are the correct principles of treatment? A. Scatter wind, clear heat, extinguish wind, arrest convulsion B. Clear qi-level heat, cool the ying-level, extinguish wind, open the orifices C. Clear heart heat, open the orifices, calm the liver, extinguish wind D. Clear heat, eliminate dampness, resolve toxins, extinguish wind E. Arrest convulsion, calm the mind, calm the liver, extinguish wind 4. A child, age 3, had a fever over 38.5 ℃, often accompanied by convulsion. During each fever, the child would have only one convulsion rather than continuous convulsions. What is the presenting pattern? A. Stirring wind due to wind-heat

B. Blazing of both qi and ying C. Pathogens trapped in the heart and lung D. Epidemic damp-heat toxin E. Infantile convulsion due to fright 5. A child, age 2, suddenly had a high fever with a temperature of 41.3 ℃, a severe headache, frequent vomiting, unconsciousness 2 days later, repeated convulsions, a red tongue with a greasy yellow coating, and a rapid slippery pulse. What is the presenting pattern? A. Stirring wind due to wind-heat B. Blazing of both qi and ying C. Pathogens trapped in the heart and lung D. Epidemic damp-heat toxin E. Infantile convulsion due to fright 6. A child, age 4, had a persistent low fever after an acute convulsion. He looked pale and had a feverish feeling in his palms and soles with stiff rigid limbs and a dry crimson tongue. What is the pathomechanism? A. Spleen deficiency and liver hyperactivity B. Decline of yang in the spleen and kidney C. Stirring of wind due to yin deficiency D. Blood deficiency producing wind E. Internal stirring of liver wind

7. A child, age 3, had low spirits, drowsiness, and slept with eyes halfclosed. Other signs and symptoms included soft stools, intermittent mild convulsions, and a pale tongue with a white coating. What is the preferred treatment formula? A. Huăn Gān Lǐ Pí Tāng (Liver-Relaxing Spleen-Regulating Decoction) B. Zhú Hán Dàn Jīng Tāng (Cold-Dispelling Convulsion-Arresting Decoction) C. Fù Zĭ Lĭ Zhōng Wán (Aconite Center-Regulating Pill) D. Sān Jiă Fù Mài Tāng (Three Shells Pulse-Restoring Decoction) E. Dà Dìng Fēng Zhū (Major Wind-Stabilizing Pill) 8. A child, age 2, had low spirits, drowsiness, and slept with his eyes half-closed. He had a pale complexion, cold sweating on the forehead, extreme cold of the hands and feet, clear profuse urine, loose stools, and slight quivering of the hands and feet. What are the correct principles of treatment? A. Warm the middle, fortify the spleen, relax the liver, regulate the spleen B. Warm and supplement spleen and kidney, restore yang to rescue desertion C. Enrich yin and subdue yang, nourish kidney and liver D. Boost qi, nourish blood, soften the liver to extinguish wind E. Supplement and boost liver and kidney, enrich yin to extinguish wind

9. A child, age 1, was born premature at eight months. He was 2.2 kg when he was born. 7 months after birth, he started to vomit. Other signs and symptoms included loose stools, sleeping with eyes half-closed, low spirits, extreme cold of the hands and feet, cold air from the mouth and nose, a pale complexion, and slightly quivering hands and feet. What is the presenting pattern? A. Blazing of both qi and ying B. Pathogens trapped in the heart and lung C. Stirring of wind due to yin deficiency D. Spleen deficiency and liver hyperactivity E. Decline of yang in the spleen and kidney

Answers 1. A 2. D 3. E 4. A 5. B 6. C 7. A 8. B 9. E

Chapter 23 Epilepsy Epilepsy refers to recurrent seizures characterized by sudden falling to the ground, loss of consciousness, foaming at the mouth, staring into space, uncontrolled jerking of the body, screaming like a pig, abnormal buzzing sounds from the throat, and a return to normalcy after waking. Its etiologies include congenital factors such as fright of the fetus or kidney yin insufficiency, and others, such as static blood obstructing the heart orifices or infantile convulsion. Phlegm blocking the heart orifices is the main contributing factor; others include patterns of stubborn phlegm lodged in the interior, sudden fright and blood stasis due to traumatic injury. The location of the disease mainly involves the heart, liver, spleen and kidney. The kidney is the foundation of the congenital, prenatal constitution, while the spleen is the foundation of acquired, postnatal constitution. Congenital insufficiency of kidney yin and failure of the spleen to transport and transform contribute to disturbance of qi movement and improper flow of body fluids, which can result in the formation of turbid phlegm. Repeated fright causes qi to become chaotic. With qi counterflow, phlegm blocks the heart orifices, bringing about unconsciousness. When phlegm invades the channels and collaterals, liver wind stirs internally to cause convulsion. This is mainly a pattern of zheng qi deficiency with excess pathogen. Common clinical types include fright epilepsy, phlegm epilepsy, wind epilepsy, blood stasis epilepsy, profuse phlegm due to spleen deficiency, and deficiency of both spleen and kidney.

CLINICAL ESSENTIALS The pattern identification of epilepsy is based on the etiology. Sources of epilepsy are varied and include fright, wind, phlegm, and blood stasis. The patient often reports being frightened before the episode occurs, and the attack is often accompanied by such psycho-emotional symptoms of fright and pig-like screaming.

● Wind epilepsy is caused by fever from external contraction, and is manifested by uncontrolled jerking of the body and fever.

● Phlegm epilepsy manifests as an alteration in mental state with signs and symptoms including unconsciousness, slumping, and the inability to hold things.

● Blood stasis epilepsy usually results from cerebral trauma, and manifests with a headache in a relatively fixed position. Identification of a deficiency pattern is based on the location of disease. Patterns of excess phlegm due to spleen deficiency and deficiency of both spleen and kidney must be differentiated.

● For deficiency patterns, the priority is treating the root by fortifying the spleen and dissolving phlegm, softening the liver and relieving urgency.

● For excess patterns, it is advisable to treat the branch by eliminating phlegm, regulating qi, extinguishing wind, opening the orifices and arresting convulsion.

● For persistent epilepsy, drug therapy should be combined with proper rescue procedures.

● For recurrent patients who have seen minimal effects with TCM treatment, treatment should also include acupuncture, incision therapy or thread-embedding therapy. Dr. Zhu Liang-chun created the following effective prescription for epilepsy: Dí Tán Dìng Xián Wán (Phlegm-Flushing Convulsion-Settling Pill). [涤痰定痫丸]

Grind to powder, mix with water and make bean-sized pills. Take 3-5 g, twice daily. Wang Jin-cai treats idiopathic epilepsy with acupuncture to good effect as follows: ● For epilepsy due to liver wind with phlegm obstruction, select DU 14 (dà zhuī), BL 18 (gān shù), BL 15 (xīn shù), RN 14 (jù quē), LV 3 (tài chōng), ST 40 (fēng lóng) and DU 20 (băi huì).

● For phlegm-fire blocking the orifices, select DU 14 (dà zhuī), BL 18 (gān shù), BL 15 (xīn shù), RN 14 (jù quē), DU 20 (băi huì) and LV 2 (xíng jiān).

● For liver-kidney yin deficiency, select DU 14 (dà zhuī), BL 18 (gān shù), BL 23 (shèn shù), KI 3 (tài xī), LV 3 (tài chōng), HT 7 (shén mén) and

SP 6 (sān yīn jiāo).

● For spleen-kidney qi deficiency, select DU 14 (dà zhuī), BL 20 (pí shù), BL 23 (shèn shù), ST 36 (zú sān lĭ), ST 40 (fēng lóng) and PC 6 (nèi guān).

● For epileptic seizures, needle DU 26 (shuĭ gōu) and KI 1 (yŏng quán) without needle retention. For daytime seizures, add BL 62 (shēn mài). For epilepsy seizures at night, add KI 6 (zhào hăi).

● With dizziness, headache and loss of memory, add GB 20 (fēng chí). ● With deficiency of qi and lack of strength, add RN 4 (guān yuán) and RN 6 (qì hăi). Dr. Wang also utilizes specific needling methods:

● Needle DU 14 (dà zhuī) 1.2 to 1.5 cun, transmitting the needling sensation to the lumbar spine.

● Needle BL 18 (gān shù), BL 20 (pí shù), BL 15 (xīn shù) and BL 23 (shèn shù) with oblique insertion to a depth of 1.2 to 1.5 cun in the direction of the spine until strong sensation is achieved.

● Needle RN 14 (jù quē) 1.2 cun with the needle pointing slightly downward to avoid puncturing the diaphragm, with the needling sensation transmitting downward to both sides.

● Needle DU 20 (băi huì) forward, backward, left and right to create a sore or numb distending sensation. Apply drainage to all other points, with normal insertion methods. For deficiency patterns, use supplementation or the even method. Stimulate the needles every 10 min; retain them for 30 min. Treat once a day, with a 3-

day interval between courses. 10 days constitute one course of treatment, and 5 courses of treatment may be required.

CASE STUDY Female, age 6. Initial Visit: June 6th, 2008 Chief Complaint: Intermittent convulsions for 4 months. History: For the past 4 months, the patient’s symptoms have included sudden falling to the ground, loss of consciousness, uncontrolled jerking of the body, stiff neck, staring into space, clenched teeth, foaming at the mouth and abnormal buzzing sounds from the throat. The patient is normal upon waking. The episodes last for 2-4 min each time, with somnolence following. Temperature is normal at the onset of the convulsions. Phenobarbitone was taken orally, but the convulsions were not effectively controlled. Past History: History of febrile convulsions. Signs and Symptoms: White greasy tongue coating, wiry and slippery pulses. Laboratory Examination: WBC 9 × 109 /L. EEG showed spike waves.

Pattern Differentiation The signs and symptoms suggest a diagnosis of epilepsy, specifically wind-type epilepsy. Liver yang harassing the upper body pushes the windphlegm to block the orifices of the heart. Intense liver fire generates internal stirring of liver wind, which causes spasms of the sinews and ligaments.

This brings about sudden falling to the ground, uncontrolled jerking of the body, stiff neck, staring into space, clenched teeth and foaming at the mouth. The white greasy tongue coating and the wiry and slippery pulse are both manifestations of wind-type epilepsy.

Diagnosis Epilepsy (Wind-type)

Clinical Treatment This is a case with branch excess manifestations. The main pathomechanism involves liver wind stirring internally and harassing the heart-spirit with wind-phlegm congesting in the upper body. Therefore, this case should be treated primarily by extinguishing wind and arresting convulsion, assisted by acupuncture and moxibustion therapy. Principles: Extinguish wind and arrest convulsions Formula: Modified Dìng Xián Wán (Convulsion-Settling Pill) [定痫丸加减]

7 doses [Formula Analysis] Líng yáng jiăo fěn and gōu téng clear heat, cool the liver and extinguish wind. Quán xiē, chán tuì and tiān má extinguish wind and arrest convulsion. Chuān bèi mŭ and zhú lì clear heat and dissolve phlegm. Yuăn zhì, fú shén and hŭ pò tranquilize the heart and calm the mind. Dăn nán xīng and bàn xià dissolve phlegm and arrest convulsion. Shí chāng pú acts to open the orifices and calm the mind. Prepared Medicines: Yī Xián Wán (Epilepsy-Arresting Pills), 1g, 2 times daily. Acupuncture During onset: DU 26 (shuĭ gōu), LI 4 (hé gŭ), shí xuān (EX-UE11), PC 6 (nèi guān), KI 1 (yŏng quán)

During the resting period: DU 14 (dà zhuī), HT 7 (shén mén), BL 15 (xīn shù), ST 40 (fēng lóng), PC 6 (nèi guān) Manipulations: After qi is obtained, needles must be retained for 20-30 min. Apply rotation every 3-5 min. Needle with drainage during the period of onset; use the even method during resting periods. Second Visit June 9th After taking 7 doses of the above prescribed formula, the patient had no convulsion, but a poor appetite, fatigue, loose stool, a pale tongue with a thin whitish coating and a thready pulse. Principles: Fortify the spleen, dissolve phlegm and calm the mind Formula: Modified Liù Jūn Zĭ Tāng (Six Gentlemen Decoction) [六君⼦汤加减]

14 doses [Formula Analysis]

Dăng shēn, fú líng and bái zhú fortify the spleen and calm the mind. Fă bàn xià and chén pí dissolve phlegm and move qi. Huái shān yào, biăn dòu and shān zhā fortify the spleen, resolve dampness and promote appetite. Lóng yăn ròu calms the mind and supplements the blood.

COMMENTARY AND DISCUSSION According to the signs and symptoms, this case was diagnosed as wind-induced epilepsy. As noted in the introduction, no matter which type of epilepsy appears, “phlegm blocking the orifices is the essential pathological mechanism”. Therefore, expelling phlegm, descending counterflow qi, extinguishing inner wind and arresting convulsion are the main principles of treatment. For this purpose, a modification of Dìng Xián Wán was applied. Dìng Xián Wán is originally found in Medical Revelations (Yī Xué Xīn Wù) (1732) by Chen Guo-peng, which gave three focuses of treatment for the disease. These are expelling phlegm, relaxing spasms and calming the shen (spirit). Of these three, expelling phlegm is given priority. Chen said of this formula, “Although [epilepsy] derives from the five internal organs, phlegm-drool retention is the root of the problem; therefore Dìng Xián Wán is given”(虽有五脏之殊,⽽为痰涎则⼀,定痫丸主之). Within Dìng Xián Wán, medicinals that expel phlegm and unblock the orifices include dăn nán xīng, bàn xià, zhú lì, chuān bèi mŭ and shí chāng pú. Medicinals that relax spasm and eliminate inner wind include quán xiē and tiān má. Medicinals to calm shen (spirit) include yuăn zhì, hŭ pò and fú shén. The original formula also includes chén pí (Pericarpium Citri Reticulatae) to move qi and awaken the spleen, dān shēn (Radix et Rhizoma Salviae Miltiorrhizae) to promote blood circulation and mài dōng (Radix Ophiopogonis) to protect ying-yin. In this case, líng yáng jiăo fěn was

added to clear heat and cool liver in order to strengthen the effect of expelling internal wind. After 7 days of treatment, the patient had no convulsion, but she still presented with signs and symptoms of spleen-deficiency. Therefore, Liù Jūn Zĭ Tāng was applied to fortify the spleen, dissolve phlegm and calm the mind, in order to consolidate the effect.

STUDY QUESTIONS 1. What is the prognosis of this disease? 2. What were the spleen-deficiency manifestations upon the return visit? 3. In terms of nursing, to what should one pay attention?

Answers 1. This condition generally requires a long course of treatment. In most cases, when the symptoms are gone, it is important to continue treatment for another 2 or 3 years. During puberty, it is advisable to take medicinals for another one or 2 years, and then to taper off. Avoid stopping medications suddenly, as the disease may return and could be worse. For the sake of convenience and patient compliance, when the epilepsy is under control, the formula for arresting epilepsy can be changed from a decoction to pills, powders or syrup. This case has a short course of the disease. After the pathogenic internal heat has been eliminated, the body will heal; so the disease has a generally good prognosis. 2. The poor appetite, fatigue, loose stool, pale tongue with thin whitish coating and thready pulse are all signs of spleen deficiency. 3. Keep such trigger factors as high fever, fright, tension, tiredness, and emotions under control. Long periods of TV watching or computer games should be prohibited. The child should not be permitted to play near water, fire, or with sharp instruments. Do not encourage the child to attempt

to stop the convulsions, as sprains or ligament damage can result. During an attack, to avoid choking or biting of the tongue, it is best to keep the child in a lateral position and to place a tongue depressor wrapped in gauze between the teeth to free respiration and allow discharge of phlegm and saliva. After convulsions, the patient tends to be very tired and sleepy. Make sure the child has good rest so that the upright qi can be restored.

CASE SCENARIOS The following cases present variations of this condition. After familiarizing yourself with the possible common pattern presentations and appropriate formulas for treatment, use the following exercises to test your overall understanding of the condition. 1. A child, age 4, suddenly had convulsions accompanied by loss of consciousness for 5 min. The symptoms were spontaneously relieved. Before the convulsions, the child vomited. He had no fever but did have soft stools. Routine stool examination revealed no red or white blood cells. EEG showed spike wave, sharp wave and spike-slow complex waves with burst phenomenon. In the past, the child had convulsions 3 times following a high fever caused by a common cold. What is the diagnosis? A. Acute infantile convulsion B. Chronic infantile convulsion C. Epidemic toxin dysentery D. Epilepsy E. Summerheat-warmth 2. A child, age 7, has had recurrent epileptic seizures for 3 years. The main signs and symptoms include convulsions, staring into space, loss of consciousness, a white tongue coating, and a wiry pulse. The child appeared normal when symptoms were not present.

What is the preferred prescription for treatment? A. Dìng Xián Wán (Convulsion-Settling Pill) B. Dí Tán Tāng (Phlegm-Flushing Decoction) C. Yī Xián Wán (Epilepsy-Arresting Pills) D. Tōng Qiào Huó Xuè Tāng (Orifice-Unblocking Blood-Quickening Decoction) E. Hé Chē Bā Wèi Wán (Placenta Hominis Pill with Eight Other Medicinals) 3. A child, age 9, has had a recurrent stomachache for 4 years. The main signs and symptoms include severe abdominal pain, a pale complexion, profuse sweating and unconsciousness sometimes accompanied by vomiting. The child appears normal when symptoms are not present. EEG monitor shows epileptiform discharges. What is the presenting pattern? A. Wind epilepsy B. Fright epilepsy C. Phlegm epilepsy D. Blood-stasis epilepsy E. Deficiency-type epilepsy 4. A child, age 4, has signs and symptoms including staring into space, loss of consciousness, accumulation of phlegm and body fluids, abnormal buzzing sounds in the throat, a normal appearance when symptoms are not present, a white tongue coating, and a slippery pulse. What are the correct treatment principles?

A. Suppress fright and calm the mind B. Eliminate phlegm and open the orifices C. Extinguish wind and arrest convulsions D. Dissolve stasis and open the orifices E. Fortify the spleen and dissolve phlegm 5. A child, age 5, has had convulsions for 5 months, which have been accompanied by pig-like screaming, abnormal buzzing sounds in the throat, psychosomatic fears, complexion alternating between red and pale, fearfulness, restlessness, uncontrolled jerking of the body, a light red tongue with a thin white coating, and a wiry and slippery pulse. What is the presenting pattern? A. Wind epilepsy B. Phlegm epilepsy C. Fright epilepsy D. Blood-stasis epilepsy E. Deficiency epilepsy 6. A child, age 7, has had recurrent convulsions for one and a half years, accompanied by loss of consciousness, myotonia, jerking of the extremities on the left side, a red tongue, and a choppy pulse. He had cerebral trauma one month before the present disease. What is the pathogenesis? A. Obstinate phlegm blocking the orifices B. Epilepsy following fright

C. Blood blocked in the heart orifices D. Phlegm obstruction due to spleen deficiency E. Deficiency of both spleen and kidney 7. A child, age 12, has had convulsions for 10 years accompanied by carpopedal spasms, occasional vertigo, mental retardation, aching and limp lumbus and knees, fatigue, lassitude, no desire to speak, cold limbs, soft stools, a pale tongue with a white coating, and a deep fine pulse. What are the correct treatment principles? A. Eliminate phlegm and open the orifices B. Extinguish wind and arrest convulsion C. Suppress fright and calm the mind D. Fortify the spleen and dissolve phlegm E. Supplement spleen and kidney 8. A child, age 6, has suddenly begun to have convulsions, which have occurred 3 times. They were accompanied by loss of consciousness, with each episode lasting for 3-5 min. The symptoms were spontaneously relieved. Other symptoms included a light red tongue and a wiry pulse. EEG monitored epileptiform discharges. What are the correct treatment principles? A. Suppress fright and calm the mind B. Eliminate phlegm and open the orifices C. Clear liver heat and suppress fright D. Extinguish wind and arrest convulsion

E. Enrich yin and soften the liver

Answers 1. D 2. A 3. C 4. B 5. C 6. C 7. E 8. D

PART IV Kidney Diseases Chapter 24 Acute Glomerulonephritis Acute glomerulonephritis, the main symptoms of which are edema and hematuria, is caused by water-dampness and associated with metabolic disorders. Less serious cases may have palpebral and facial edema and microscopic hematuria. Serious cases may have body swelling, gross hematuria, and even heart, brain and kidney failure. Acute glomerulonephritis is caused by external contraction of wind pathogens, damp-heat and toxins, which lead to disorders of the lung, spleen and kidney, especially the lung and spleen. Wind, heat and toxins combine with water-dampness and lead to failures of regulation, transportation and transformation, and opening and closing. Water metabolism disorder leads to swelling. Heat invades the blood vessels in the lower jiao, which leads to hematuria. Under serious conditions, flooding water and pathogens may lead to the patterns of pathogens invading the heart and liver, water attacking heart and lung, and damp turbidity and toxic pathogens blocking internally. Invasion by damp-heat for a long time will cause yin to be damaged and qi

worn, leading to yin deficiency or qi deficiency with pathogens retained, make required treatment longer. With delayed treatment, pathogens invade into the collaterals and cause obstruction, which may lead to the blood stasis presenting with continuous hematuria, a somber facial complexion and a purple tongue. This disease has an acute stage and a recovery stage. The main patterns in the acute stage are contention between wind and water and damp-heat invasion. The main patterns in the recovery stage are yin deficiency with retained pathogens, and qi deficiency with retained pathogens.

CLINICAL ESSENTIALS In the acute stage of acute glomerulonephritis, upright qi is abundant and pathogenic qi is in excess; this leads to an acute onset with quickly changing symptoms, and obvious edema and hematuria. The recovery stage is characterized by reduced edema, increased urine, and the disappearance of gross hematuria, though microscopic hematuria or proteinuria and dampheat remain. The pattern of yin deficiency with pathogens retained is characterized by dizziness and weakness, heat in the hearts of the palms and soles, and a red tongue with little coating. The pattern of qi deficiency with pathogens retained is characterized by fatigue and lack of strength, poor appetite, unformed stools, spontaneous sweating and a pale tongue. There is a large disparity between the mild and serious patterns of acute glomerulonephritis. Mild presentations include patterns of wind-cold, wind-heat, heat-toxin, damp-heat and cold-damp. Edema, decreased urine output, and rising blood pressure are transient, and may be cured by Chinese medicine. Serious patterns have signs and symptoms including severe edema and continuous scant and blocked urine, and they may soon develop into acute and serious patterns such as pathogens invading the heart and liver, water attacking heart and lung, and damp-turbidity and toxic pathogens blocking internally. These are transmuted patterns, and require emergency rescue. Furthermore, changes in urine should be paid attention to when differentiating patterns. For longer cases of diminished urine and more

serious edema, it is more likely that transmuted patterns will come into being. The treatment principle for acute glomerulonephritis must focus on the disease mechanism, which in the acute stage is pathogens in excess and in the recovery stage upright qi deficiency with pathogenic qi retention. In the acute stage, the principal treatment is to dispel pathogens. Along with this, treatment may diffuse the lung and promote urination, clear heat and cool the blood, and resolve toxins and drain dampness. In the recovery stage, the principle is to reinforce upright qi while dispelling pathogenic qi. The proportion of supplementation to dispelling depends on the relative proportion of upright qi to pathogenic qi. For example, in the early stage of recovery, the principle is to dispel damp-heat and other pathogens while reinforcing upright qi by nourishing yin or boosting qi, for damp-heat has not been dispelled adequately in this period. In the later stage of recovery, the principle is to reinforce upright qi while clearing heat and removing dampness, for damp-heat has been almost completely dispelled in this period. If there is only a lack of upright qi, the suitable treatment is to supplement and boost. To prevent retaining the pathogen and delaying recovery, it is not recommended to warm and supplement in the early stage. Do not boost the pathogen when supplementing and boosting the upright qi, and do not damage the upright qi when dispelling the pathogen. For different transmuted patterns, there are different treatments, including those which calm the liver and extinguish wind, clear heart heat and promote urination, drain the lungs and expel water, warm and supplement heart yang, and dredge the bowels and direct the turbid downward. Use Western medicine emergency treatment when necessary.

Dr. Zhou Zhong-ying has effectively used Líng Guì Fú Píng Tāng (Poria, Cinnamon Twig, and Duckweed Decoction) for the disease, which includes má huáng (Herba Ephedrae), fú píng (Herba Spirodelae), fáng fēng (Radix Saposhnikoviae), sū yè (Folium Perillae), shēng jiāng pí (Cortex Zingiberis Rhizomatis, Exodermis Zingiberis Recens), jié gĕng (Radix Platycodonis) and others.

● If it is the wind-cold pattern with signs and symptoms including aversion to cold and no sweating, joint pain, a glossy tongue coating and a floating tight pulse, add guì zhī (Ramulus Cinnamomi) to diffuse the lung, unblock yang and induce sweating to release the exterior.

● If it is the wind-heat pattern with signs and symptoms including vexation thirst, rough breathing, obvious fever, a yellow tongue coating and a floating tight pulse, add shí gāo (Gypsum Fibrosum), sāng bái pí (Cortex Mori) and lú gēn (Rhizoma Phragmitis) to clear and diffuse lung-wei and free and regulate the waterways.

● If it is the wind-pathogen with dampness pattern with signs and symptoms including aching limbs, a greasy tongue coating and a floating soggy pulse, add qiāng huó (Rhizoma et Radix Notopterygii), qín jiāo (Radix Gentianae Macrophyllae), fáng jĭ (Radix Stephaniae Tetrandrae) and cāng zhú (Rhizoma Atractylodis) to diffuse the exterior and remove dampness. Dr. Wu Ping-lu uses acupuncture to treat the associated proteinuria. The main points include two sets. The first includes DU 14 (dà zhuī), LI 15 (jiān yú), LI 11 (qū chí), LI 4 (hé gŭ), PC 6 (nèi guān), ST 36 (zú sān lĭ), SP 9 (yīn líng quán), BL 13 (fèi shù) and RN 9 (shuĭ fēn). The second set includes LI 4 (hé gŭ), LV 3 (tài chōng), LI 11 (qū chí), SP 10 (xuè hăi), SP

6 (sān yīn jiāo), PC 3 (qū zé), BL 40 (wĕi zhōng), ST 36 (zú sān lĭ), SP 9 (yīn líng quán), BL 20 (pí shù) and BL 23 (shèn shù). There are many options for supporting points. Add ST 28 (shuĭ dào) and RN 4 (guān yuán) if there is blocked urine. Add DU 26 (shuĭ gōu) for facial swelling. Add LV 1 (dà dūn) for hematuria. Add LU 5 (chĭ zé) and LU 9 (tài yuān) for coughing. Add ST 25 (tiān shū) for unformed stool and abdominal distention. Add PC 6 (nèi guān) and RN 12 (zhōng wăn) for nausea and vomiting. Add HT 7 (shén mén) and PC 6 (nèi guān) for palpitations and insomnia. He also gives specific methods and manipulations. Firstly, disinfect the points. Use No. 30 filiform needles. Needle bilaterally. After qi is obtained, use the draining method according to the condition of disease. First needle the points on the back, and retain the needles for 15-30 min, adding moxibustion. Secondly, needle ST 36 (zú sān lĭ) joining to SP 9 (yīn líng quán) and retain the needles for 15-30 min. Then, needle the other points from the top of the body to bottom, one by one, with no needle retention. Needle the above two groups of points in turn, once every 1 or 2 days. 6 times is considered one treatment period. Stop for one week between two periods of treatment.

CASE STUDY Female, age 6. Initial Visit: May 8th, 2005 Chief Complaint: Facial edema and gross hematuria for 2 days. History: The patient felt throat discomfort 10 days ago and was treated with Sù Xiào Shāng Fēng Jiāo Náng (Paracetamol, Caffeine, Artificial Cow-bezoar and Chlorphenamine Maleate Capsules). She went to the hospital because facial edema appeared over the last 2 days, along with scanty urine and gross hematuria. Meanwhile, the edema got more severe and spread to the entire body. At intake, her skin was glossy and light, pitted when pressed and soon recovered. Other present signs and symptoms included scanty reddish urine, aversion to wind and cold, dry mouth, thirst, sore throat, cough, nasal congestion, a pale tongue with a thin yellow coating, and a floating pulse. Past History: Good general health with no history of infectious diseases. Physical Examination: BT 36.9℃, PR 90 times/min, RR 25 beats/min, BP 130/70 mmHg. Good spirit, slightly low heart sounds, no wet rales heard at the bottom of the two lungs, liver palpated 3.5 cm below ribs and soft. Laboratory Examination: Urea nitrogen 4.5 mmol/L, cholesterol 1.37 mmol/L, albumin 50 g/L. A routine blood test showed Hb 110 g/L, WBC 11 × 109/L, N 75%, L 25%, ESR 95 mm/h, Antistreptolysin “O” antibodies 600 μmol/L .

Pattern Differentiation The patient got the disease after external contraction. Edema and hematuria are the main symptoms. The external pathogen invades the lung and spleen, leading to the lung failing to regulate the waterways and the spleen failing to move and transform, resulting in flooding water-dampness. The bladder fails to transform qi, causing the edema and scanty or blocked urine. External pathogens attack the lung, and the lung fails to diffuse and govern descent, causing cough. Damp-heat pours down into the bladder and damages the blood vessels in the lower jiao, causing hematuria. The yellow tongue coating and the floating rapid pulse are both features of exterior wind-heat.

Diagnosis Edema due to contention between wind and water

Clinical Treatment This case is branch excess, but without a root deficiency. The windheat pathogen invades the lungs and spleen, leading to the lungs failing to regulate the waterways and the spleen failing to move and transform, causing flooding water-dampness. The priority of treatment here is to diffuse the lung, promote urination and clear wind-heat. Principles: Scatter wind, diffuse the lung, promote urination, disperse swelling

Formula: Modified Má Huáng Lián Qiào Chì Xiăo Dòu Tāng (Ephedra, Forsythia, and Rice Bean Decoction) and Wŭ Líng Săn (Five Substances Powder with Poria) [麻黄连翘⾚⼩⾖汤合五苓散加减]

5 doses [Formula Analysis] Má huáng and guì zhī dissipate wind-cold, diffuse the lung and promote urination. Lián qiào clears heat and resolves toxins. Xìng rén, fú líng, zhū líng, zé xiè and chē qián zĭ diffuse the lung and direct qi downward. Xiăo jì cools the blood and stanches bleeding. Gān căo harmonize all formula medicinals. Prepared Medicines: Shèn Yán Qīng Rè Piàn (Heat Clearing Tablets for Nephritis), 3g, twice a day. Second Visit

After 5 days treatment, the edema has been relieved and the gross hematuria disappeared. The tongue is light red, the coating is thin and white, and the pulse is slightly floating. All the symptoms have been relieved, but there is still some residual pathogen retained. Principles: Promote urination and disperse swelling Formula: Modified Má Huáng Lián Qiào Chì Xiăo Dòu Tāng (Ephedra, Forsythia, and Rice Bean Decoction) and Wŭ Líng Săn (Five Substances Powder with Poria) [麻黄连翘⾚⼩⾖汤合五苓散加减]

5 doses [Formula Analysis] Má huáng and guì zhī dissipate wind-cold, diffuse the lung and promote urination. Lián qiào clears heat and resolve toxins. Xìng rén, fú líng, zhū líng, zé xiè, and chē qián zĭ diffuse the lung and direct qi downward. Xiăo jì cools the blood and stanches bleeding. Gān căo harmonizes all formula medicinals.

COMMENTARY AND DISCUSSION The diagnosis in this case is edema. According to fundamental Chinese medicine, extra body fluids are excreted out of the human body through sweating, governed by the lungs, and urination, controlled by kidneys. In order to remove extra body fluids and maintain the water metabolism, both diaphoretics and diuretics are applied commonly in clinical treatment. Zhang Zhong-jing set up a treatment strategy for edema in his Essentials from the Golden Cabinet (Jīn Guì Yào Lüè) saying, “edema above the waist should be treated with diaphoretic therapy and the edema below the waist with diuretic therapy” (诸有⽔者,腰以下肿,当利⼩便︔腰以上肿,当 发汗乃愈). In this case, the 6 year-old female patient had facial edema and edema around her eyelids for over 2 days, and the symptoms got worse when she got to the hospital. She had edema through her whole body with difficult urination and blood in the urine. All these indicate that both diaphoretic and diuretic therapies were needed for this emergent situation. Therefore, Má Huáng Lián Qiào Chì Xiăo Dòu Tāng and Wŭ Líng Săn were combined to treat this case. Má Huáng Lián Qiào Chì Xiăo Dòu Tāng was originally designed to treat the yang type of jaundice due to internal damp-heat. The Treatise on Cold Damage (Shāng Hán Lùn) says, “a person who suffers from febrile disease with heat accumulated internally must have jaundice, and Má Huáng Lián Qiào Chì Xiăo Dòu Tāng should be served” (伤寒,热瘀在 ⾥,身必黄。麻黄连翘⾚⼩⾖汤主之). This herbal prescription has stronger functions of clearing and draining damp-heat than of spreading and

dispersing external cold and is commonly applied for wind-edema in modern times, particularly for infant edema due to accumulation of dampheat with unreleased external syndrome. The modified prescription contained Má Huáng Tāng (Ephedra Decoction), which contains má huáng, guì zhī, xìng rén and gān căo to promote perspiration, and Wŭ Líng Săn to promote urination. With both prescriptions together, body fluids are removed through the pores and urinary tract. Chì xiăo dòu (Semen Phaseoli) has a sweet taste with a neutral property. It has functions of clearing heat and promoting urination for edema, urinary difficulty, and leg qi edema. It also disperses blood stasis, reduces swelling and clears damp-heat and is commonly applied as a dietary therapy for frequent difficult and painful urination by clearing heat from the heart and small intestine. If the patient does not have hematuria, chì xiăo dòu might be kept in the herbal prescription.

STUDY QUESTIONS 1. What is the prognosis of this case? 2. To what must one pay attention when nursing this disease?

Answers 1. The course of this disease is short and can be cured after removing waterdampness. The prognosis is good. 2. In the acute period, the patient needs 2 to 3 weeks of bed rest. When the gross hematuria disappears, edema fades, and the blood pressure becomes normal, the patient can get out of bed and enjoy light activities. When ESR becomes normal, the patient can go to school, but cannot continue studying past the school day. The patient must avoid heavy physical exertion for 3 months, and when the urinary sediment cells normalize, he can participate in moderate exercise. Limit protein and water in the acute period, and limit salt and water for patients who have edema and high blood pressure. The intake of salt should be about 60 mg/kg per day. Limit the intake of protein of azotemic patients. 0.5 g/kg of highquality animal protein per day may reduce the burden of renal excretion.

CASE SCENARIOS The following cases present variations of this condition. After familiarizing yourself with the possible common pattern presentations and appropriate formulas for treatment; use the following exercises to test your overall understanding of the condition. 1. A child, age 8, caught cold a week ago and did not recover. Palpebral edema appeared one day ago. The child’s tongue coating is white, and the pulse is floating. What is the proper formula? A. Sān Rén Tāng (Three Kernels Decoction) B. Yuè Bì Tāng (Maidservant from Yue Decoction) C. Cōng Chĭ Jié Gĕng Tāng (Scallion, Fermented Soybean and Platycodon Decoction) D. Huò Xiāng Zhèng Qì Săn (Agastache Qi-Correcting Powder) E. Má Huáng Lián Qiào Chì Xiăo Dòu Tāng (Ephedra, Forsythia, and Rice Bean Decoction) 2. A patient, age 3, has experienced edema for 2 months and now has signs and symptoms including occasional edema in the face, yellow facial complexion, fatigue and lack of strength, spontaneous sweating, a pale tongue with a white coating, and a moderate pulse. What is the proper treatment? A. Fortify the spleen and boost qi.

B. Warm yang and promote urination. C. Fortify the spleen and remove dampness. D. Fortify the spleen and nourish the blood. E. Warm the kidney and promote urination. 3. A patient, age 7, has been sick for 2 weeks with slight edema, scanty yellow urine, microscopic hematuria, a red tongue, a thin white tongue coating that is a little greasy, and a slippery and rapid pulse. What is the diagnosis? A. Edema, contention between wind and water B. Edema, damp-heat invading inward C. Edema, yin deficiency and retained pathogen D. Hematuria, frenetic movement of blood heat E. Hematuria, yin deficiency with internal heat 4. What is the suitable formula for the pattern of yin deficiency and retained pathogen? A. Modified Wŭ Wèi Xiāo Dú Yĭn (Five Ingredients Toxin-Removing Beverage) and Xiăo Jì Yĭn Zĭ (Field Thistle Drink) B. Shēn Líng Bái Zhú Săn (Ginseng, Poria and Atractylodes Macrocephalae Powder) C. Lóng Dăn Xiè Gān Tāng (Gentian Liver-Draining Decoction) and Líng Jiăo Gōu Téng Tāng (Antelope Horn and Uncaria Decoction) D. Sì Jūn Zĭ Tāng (Four Gentlemen Decoction) and Yù Píng Fēng Săn (Jade Wind-Barrier Powder)

E. Zhī Băi Dì Huáng Wán (Anemarrhena, Phellodendron and Rehmannia Pill) and Èr Zhì Wán (Double Supreme Pill) 5. What is the suitable formula for the pattern of qi deficiency with retained pathogens? A. Modified Wŭ Wèi Xiāo Dú Yĭn (Five Ingredients Toxin-Removing Beverage) and Xiăo Jì Yĭn Zĭ (Field Thistle Decoction) B. Shēn Líng Bái Zhú Săn (Ginseng, Poria and Atractylodes Macrocephalae Powder) C. Lóng Dăn Xiè Gān Tāng (Gentian Liver-Draining Decoction) and Líng Jiăo Gōu Téng Tāng (Antelope Horn and Uncaria Decoction) D. Sì Jūn Zĭ Tāng (Four Gentlemen Decoction) and Yù Píng Fēng Săn (Jade Wind-Barrier Powder) E. Zhī Băi Dì Huáng Wán (Anemarrhena, Phellodendron and Rehmannia Pill) and Èr Zhì Wán (Double Supreme Pill) 6. What is the suitable formula for the pattern of pathogens invading the heart and liver? A. Zhèn Gān Xī Fēng Tāng (Liver-Sedating and Wind-Extinguishing Decoction) and Wŭ Líng Săn (Five Substances Powder with Poria) B. Líng Jiăo Gōu Téng Tāng (Antelope Horn and Uncaria Decoction) and Zhì Băo Dān (Supreme Jewel Elixir) C. Tiān Má Gōu Téng Yĭn (Gastrodia and Uncaria Decoction) and Xiè Xīn Tāng (Heart-Draining Decoction) D. Fù Zĭ Xiè Xīn Tāng (Aconite Heart-Draining Decoction) and Wēn Dăn Tāng (Gallbladder-Warming Decoction)

E. Lóng Dăn Xiè Gān Tāng (Gentian Liver-Draining Decoction) and Líng Jiăo Gōu Téng Tāng (Antelope Horn and Uncaria Decoction)

Answers 1. E 2. C 3. B 4. E 5. B 6. E

Chapter 25 Nephrotic Syndrome Nephrotic syndrome is caused by failure of the storage function of the kidney and the outward discharge of essence, which leads to a large number of plasma proteins lost through the urine. The main clinical features of the disease are albuminuria, hypoproteinemia, hypercholesterolemia, and edema to various extents. The main causes of this disease include constitutional insufficiency, weak health after a long course of illness and the inward penetration of external pathogens. Lung, spleen and kidney depletion leads to abnormal qi transformation and movement, where fluid transformation becomes disordered and fluids flood the skin to cause edema. The kidney fails to store and hold essence, where leakage of essence into urine causes albuminuria. Water retention leads to qi obstruction, and qi stagnation leads to blood stasis. Clinical pattern types include lung-spleen qi deficiency, spleen deficiency with damp encumbrance, spleen-kidney yang deficiency, liverkidney yin deficiency, and deficiency of both qi and yin.

CLNICAL ESSENTIALS The main point in pattern differentiation of this disease is to identify root and branch patterns. The root pattern of nephrotic syndrome is deficiency of upright qi; this includes lung-spleen qi deficiency, spleen deficiency leading to damp encumbrance, spleen-kidney yang deficiency, liver-kidney yin deficiency and deficiency of both qi and yin. The identification of the root pattern may be based on the history of illness, and the presence of swelling or symptoms of the whole body.

● Lung-spleen qi deficiency has signs and symptoms including recurrent colds, edema caused by external contraction most serious in the face, spontaneous sweating, poor appetite and digestion, unformed stools and lack of strength.

● Spleen deficiency leading to damp encumbrance often appears in the beginning of the disease or in cases with persistent edema. The most serious edema occurs in the limbs. It has signs and symptoms including yellow face, chest oppression, abdominal distention, poor appetite and digestion, unformed stools, and other spleen-stomach symptoms.

● Spleen-kidney yang deficiency has signs and symptoms including a high degree of edema with hydrothorax and ascites, mental fatigue, aversion to cold, lack of warmth in the limbs, and other yang deficiency cold symptoms.

● Liver-kidney yin deficiency often appears in patients who are constitutionally yin deficient, or after long-term treatment with hormones.

Edema is mild or disappears completely. Signs and symptoms include tidal reddening of the face, dizziness, vexation, and a red tongue without coating.

● Deficiency of both qi and yin often appears in patients with a long course of disease or recurrent disease, or after long-term or repeated treatment with hormones. The edema is mild or non-existent. Signs and symptoms include susceptibility to external contraction, qi deficiency, dry throat and mouth, heat in the palms and soles, a red tongue with little coating, and other yin deficiency symptoms. The branch patterns of nephrotic syndrome involve pathogenic excess, which include external contraction, water-dampness, damp-heat, blood stasis and dampturbidity. External contraction usually refers to the contraction of wind-pathogens like wind-cold or wind-heat. Signs and symptoms include fever, aversion to wind, cough, runny nose, and a sore red throat.

● Water-dampness is characterized by obvious edema or hydrothorax and ascites.

● Damp-heat stagnates in the upper jiao, and symptoms include toxic sores on the skin. Middle jiao damp-heat has signs and symptoms of sliminess and bitterness of the mouth, thirst with no desire for fluids, fullness and focal distention in the epigastric region with poor appetite, and a slimy yellow tongue coating. Lower jiao damp-heat has scanty reddish difficult urination with a burning sensation.

● Blood stasis is characterized by a somber facial complexion and a dark purple tongue with stasis maculae; this can also be identified by the hemorheology index.

● Damp-turbidity has signs and symptoms including nausea and vomiting, a heavy cumbersome body, and listlessness of essence and spirit. Treatment principles for nephrotic syndrome include remaining aware of the disease mechanism. If there is root deficiency and branch excess, give priority to supporting upright qi and banking the root, and emphasize boosting qi, fortifying the spleen, nourishing the kidney and regulating yin and yang. Treatments for dispelling pathogens such as diffusing the lung, promoting urination, clearing heat, dissolving stasis, removing dampness, or directing the turbid downward can be used to treat the branch. In different periods of the disease, treatment may be adjusted to focus on the main problem in each period. For example, if edema is serious or if external pathogens, damp-heat or other pathogens are obvious, treatment to dispel pathogens should be used to cure the branch quickly. After the edema or external pathogens have disappeared or become less serious, treatment can focus on supporting the upright qi while dispelling pathogens, replenishing deficiency and reinforcing upright qi by treating both branch and root. Generally, according to the condition of deficiency and excess of root and branch, the emphasis is on reinforcing upright qi or dispelling pathogens. If the results with Chinese medicine are inadequate, assistance with Western medicine and other integrated treatments should be sought. For serious patients with signs and symptoms of water attacking the heart and lung, pathogens invading the heart and liver, damp-turbidity and toxic pathogens blocking internally, emergency treatment with Chinese and Western medicine should be applied. The treatment period should be long, and after urine protein disappears, treatment should continue for at least

half a year, with some serious cases treated for a year or more. Hormones should be reduced gradually, as a sudden withdrawal may cause a relapse. Dr. Gao Hui-yuan holds that nephrotic syndrome is a complicated and refractory disease in which the deficiency is complicated by excess, with root deficiency combining with branch excess. Upright qi deficiency refers to deficiencies of the lung, spleen, kidney, liver and other organs, especially spleen-kidney yang deficiency. Pathogenic excess refers to the inducements of the disease such as external pathogens, damp-turbidity, internal wind, damp-heat and blood stasis, among which dampturbidity, damp-heat, toxins, and blood stasis are most common. General treatment principles are to warm yang, promote qi transformation and free the sanjiao. Dr. Gao uses a modification of Jiā Wèi Chūn Zé Tāng (Supplemented Spring Pond Decoction), comprised of shān yào (Rhizoma Dioscoreae), shēng huáng qí (raw Radix Astragali), shú dì huáng (Radix Rehmanniae Praeparata), zé xiè (Rhizoma Alismatis), zhū líng (Polyporus), bái zhú (Rhizoma Atractylodis Macrocephalae), fú líng (Poria), guì zhī (Ramulus Cinnamomi), fù zĭ (Radix Aconiti Lateralis Praeparata), and dăng shēn (Radix Codonopsis). Used together, the above medicinals act to boost qi, fortify the spleen, warm yang, transform qi, promote urination and disperse swelling. The formula can be modified according to the presenting condition. Dr. Zhang Wei-shi used acupuncture and moxibustion in the treatment of nephrotic syndrome, with an effective rate reaching 94%.

● In periods of edema, needle RN 9 (shuĭ fēn) and RN 6 (qì hăi) with drainage, and RN 4 (guān yuán) with supplementation.

● In the period of no edema, needle following two point groups alternately:

Group A: RN 6 (qì hăi), RN 4 (guān yuán), and right GB 26 (dài mài), all needled with supplementation. Group B: BL 23 (shèn shù) bilaterally and GB 26 (dài mài) on the left, needle both with supplementation. Moxibustion: Cut fresh shēng jiāng (Rhizoma Zingiberis Recens) into slices 0.1 cm thick and 0.8 cm in diameter. Puncture the middle of the ginger 3 or 4 times with a needle, and put the ginger slices onto the points. Twist moxa into cones as large as soybeans, and burn them on top of the ginger. When the moxa is about to burn out, to perform drainage, one should remove the moxa cones. To apply supplementation, one should press a matchbox (or something similar) on the moxa cones for about a halfminute, letting the heat permeate the points. Use 5 cones for each point, once every other day. 15 times constitute one course of treatment; allow 5 days before starting the next course. If the skin surrounding the points reddens and blisters after the moxibustion, cover the points with sterile gauze.

CASE STUDY Male, age 7. Initial Visit: March 18th, 2007 Chief Complaint: Edema for 3 days. History: The patient caught a cold 3 days ago, and edema appeared afterwards. Signs and symptoms included moderate edema of the whole body mainly affecting the face and eyes. There was also scanty yellow urine, shortness of breath, a lack of strength, spontaneous sweating, unformed stools, coughing, sticky and thick sputum, and a weak pulse. Past History: Weak constitution with susceptibility to colds and flu. Physical Examination: The throat was red, but tonsils not swollen. Laboratory Examination: Routine urine test: protein (+++), occult blood (++). Microscopic examination: RBC full view, few WBC. Plasma albumin 25 g/L, Bloodlipid 7.9 mmol/L.

Pattern Differentiation The patient had been susceptible to colds since he was very young; the weak resistance and lung qi deficiency led to susceptibility to cold and shortness of breath. Spleen qi deficiency led to unformed stools; the spleen also governs the four limbs, so the spleen qi deficiency also led to a lack of strength. The light red tongue and weak pulse are signs of lung-spleen qi deficiency. The

red throat, coughing and thick sputum indicate externally-contracted windheat.

Diagnosis Edema due to lung-spleen qi deficiency and external wind-heat

Clinical Treatment Principles: Boost qi, fortify the spleen, promote urination, diffuse the lung, clear heat Formula: Modified Fáng Jĭ Huáng Qí Tāng (Stephania Root and Astragalus Decoction) and Wŭ Líng Săn (Five Substances Powder with Poria) [防⼰黄芪汤合五苓散加减]

10 doses [Formula Analysis]

Huáng qí and bái zhú boost qi and fortify the spleen. Fú líng, zhū líng and zé xiè fortify the spleen and promote urination. Má huáng, guì zhī, and hàn fáng jĭ diffuse the lung, unblock yang and promote urination. Jīn yín huā and yú xīng căo clear lung heat. Xìng rén directs qi downward and relieves cough. Gān căo banks up earth and harmonizes the center. In addition, it harmonizes all the medicinals. Prepared Medicines: Léi Gōng Téng Duō Gān Piàn (Tripterygium Wilfordii Polyglycoside Tablets) 1mg/kg, 2-3 times daily. 3 months constitute one course of treatment. Second Visit After 10 days of treatment, the edema was reduced, urination increased, stools became thicker, and coughing and sputum disappeared. The patient still had shortness of breath, a lack of strength, and the pulse was slightly weak. Examination found that the throat was no longer red, and the tonsils were not swollen. Heat-toxin was released, but spleen deficiency and edema remained. Principles: Boost qi, fortify the spleen, promote urination, diffuse the lungs, clear heat Formula: Fáng Jĭ Fú Líng Tāng (Stephania Root and Poria Decoction) and Shēn Líng Bái Zhú Săn (Ginseng, Poria and Atractylodes Macrocephalae Powder) [防⼰茯苓汤合参苓⽩术散]

7 doses [Formula Analysis] Huáng qí, rén shēn, fú líng and bái zhú boost qi, fortify the spleen, drain dampness and disperse swelling. Hàn fáng jĭ, guì zhī, zhū líng, zé xiè and yì yĭ rén dispel wind and promote urination.

COMMENTARY AND DISCUSSION The medicinals applied in this case consist of three of Zhang Zhongjing’s classic medicinal prescriptions. One is Má Huáng Tāng (Ephedra Decoction), consisting of má huáng, guì zhī, xìng rén and gān căo. Another is Wŭ Líng Săn (Five Substances Powder with Poria), containing fú líng, zhū líng, zé xiè, bái zhú and guì zhī. The third is Fáng Jĭ Huáng Qí Tāng, consisting of huáng qí, hàn fáng jĭ, bái zhú, gān căo, shēng jiāng and dà zăo (Fructus Jujubae). Why were these three classic prescriptions combined in this case? As we know, edema is caused by disorder of the transformation and transportation of body fluids. The lung, spleen and kidney take responsibility for body fluid disorders, and the medicinals applied for this case treat these three organs. The Yellow Emperor’s Inner Classic (Huáng Dì Nèi Jīng) suggests that for water retention, to “open the ghost door” (sweating pores) and to clean the “clear residence”(the urinary bladder) (开鬼门,洁净府); this is because the lung and kidney control the excretion of body fluids through the pores and the bladder. Má Huáng Tāng enters the lung; it has a strong diaphoretic effect and removes extra body fluids from the upper jiao and superficial areas through sweating. In this case, Má Huáng Tāng also acts to disperse lung qi and stop cough. Wŭ Líng Săn promotes urination by qi transformation, and expels water retention from the lower jiao through urination.

Since the spleen is the most important organ for the transformation and transportation of body fluids, treatment for edema should not exclude the spleenorgan. Fáng Jĭ Huáng Qí Tāng is designed to augment qi, strengthen the spleen, promote urination and reduce edema. Zhang Jing-yue (15631640), in his classic The Complete Works of [Zhang] Jing-yue (Jĭng Yuè Quán Shū), advocated that “the root of edema is in the kidney, the symptom is in the lung, while the controlling is in the spleen” (其本在肾,其标在 肺,其制在脾). Here, Fáng Jĭ Huáng Qí Tāng can also strengthen the exterior and improve immunity, as the patient in this case was susceptible to colds due to a poor body constitution. Huáng qí, as one of the chief medicinals, is the principal substance to stabilize the exterior and strengthen the protective qi, and bái zhú is an assistant medicinal to enhance the functions of huáng qí. Combined, these two ingredients remind us of another famous medicinal prescription, Yù Píng Fēng Săn (Jade Wind-Barrier Powder), which strengthens the exterior and boosts protective qi. In this case, these medicinals also help to prevent the over-diaphoretic effect of má huáng. Even though it is said that this is a modification of Fáng Jĭ Huáng Qí Tāng, it is also similar to Fáng Jĭ Fú Líng Tāng (Stephania Root and Poria Decoction), another classic prescription from Essentials from the Golden Cabinet (Jīn Guì Yào Lüè), which is actually listed as the basic formula for the follow-up visit. The medicinals applied for this child contained all the ingredients of Fáng Jĭ Fú Líng Tāng, while Fáng Jĭ Huáng Qí Tāng contains shēng jiāng and dà zăo, which were not applied in this formula. Fáng Jĭ Fú Líng Tāng has a stronger effect in treating superficial edema, and focuses more on the exterior and muscle levels.

In this case, another two medicinals, jīn yín huā and yú xīng căo were added for the congested throat, cough and thick sputum due to wind-heat invasion. These cold-natured medicinals also balance the warm properties of má huáng and guì zhī which were omitted at the follow-up visit in order to protect the defensive qi, furthermore, it was not necessary to continue with the sweating method.

STUDY QUESTIONS 1. What is the prognosis of this case? 2. What were the signs and symptoms of spleen deficiency and edema at the patient’s second visit? 3. To what must attention be paid attention in nursing this disease?

Answers 1. Treatment of this disease is long, and the disease is often treated with integrated Chinese and Western medicine, specifically hormone treatments combined with Chinese medicine. After the urine proteins disappear, treatment should continue for at least half a year, and some serious patients should be treated for a year or more. Hormones should be reduced gradually, as a sudden withdrawal may cause a relapse. 2. The edema was reduced and urination increased, which showed that the edema had become less serious. The patient still had shortness of breath, lack of strength, stools becoming thicker, and a slightly weak pulse; these were the remaining signs of spleen deficiency. 3. In nursing the disease, attention must be paid to disease findings. If there are sores and boils or itchy papules on the skin, dental cavities, or tonsillitis, treatment should be timely. Try to avoid infections of the respiratory system, urethra and skin. During the period of edema, if the patient has high blood pressure, limit salt intake depending on the degree of

edema and blood pressure. Patients with serious edema and obvious high blood pressure should stop all salt intake and control water intake. In the period of edema, patients need mild and easily digested foods that are full of vitamins and calcium, and high-quality protein from milk, eggs, fish and lean meat. The intake of protein should be controlled at 1.5 to 2.0 g per kg of body weight per day, not higher or lower. Keep the skin clean, especially in the folds. If there is edema in the scrotum, avoid squeezing and pressing.

CASE SCENARIOS The following cases present variations of this condition. After familiarizing yourself with the possible common pattern presentations and appropriate formulas for treatment, use the following exercises to test your overall understanding of the condition. 1. A child has edema of the whole body, scanty blocked urination, poor appetite and digestion, nausea, vomiting, a greasy tongue coating, and a wiry pulse. What is the best treatment formula? A. Yù Shū Dān (Jade Pivot Elixir) B. Zĭ Xuě Dān (Purple Snow Elixir) C. Wēn Dān Tāng (Gallbladder-Warming Decoction) D. Lóng Dăn Xiè Xīn Tāng (Gentian Heart-Draining Decoction) E. Fù Zĭ Xiè Xīn Tāng(Aconite Heart-Draining Decoction) 2. A child has edema of the whole body, especially in the face and eyes, scanty urination, a pale face, shortness of breath, lack of strength, poor appetite and digestion, unformed stools, spontaneous sweating, susceptibility to cold, cough, a pale enlarged tongue, and a weak pulse. What is the best treatment formula? A. Shí Pí Yĭn (Spleen-Bolstering Decoction) B. Yì Gōng Săn (Special Achievement Powder)

C. Shēn Líng Bái Zhú Săn (Ginseng, Poria and Atractylodes Macrocephalae Powder) D. Yù Píng Fēng Săn (Jade Wind-Barrier Powder) and Sì Jūn Zĭ Tāng (Four Gentlemen Decoction) E. Fáng Jĭ Huáng Qí Tāng (Stephania and Astragalus Decoction) and Wŭ Líng Săn (Five-Ingredient Powder with Poria) 3. A child has edema, which is sometimes severe and sometimes mild, headache, dizziness, vexation, a dry throat and mouth, heat in the palms and soles, dry eyes, insomnia, a red tongue with little coating, and a wiry thready rapid pulse. What is the best treatment formula? A. Zhēn Wǔ Tāng (True Warrior Decoction) B. Yī Guàn Jiān (Effective Integration Decoction) C. Yù Píng Fēng Săn (Jade Wind-Barrier Powder) D. Liù Wèi Dì Huáng Wán (Six-Ingredient Rehmannia Pill) E. Zhī Băi Dì Huáng Wán (Anemarrhena, Phellodendron and Rehmannia Pill) 4. A child, age 6, has edema after repeatedly catching colds, and abnormal urination for one year. After treatment, the edema subsided but the urination was not normalized. At a follow-up visit, the patient has a pale face, fatigue and lack of strength, susceptibility to cold, tendency to sweat easily, a pale tongue with thin white coating, and a weak moderate pulse. The disease is nephrotic syndrome.

What is the presenting pattern? A. Contention between wind and water B. Deficiency of both qi and yin C. Lung-spleen qi deficiency D. Spleen-kidney yang deficiency E. Liver-kidney yin deficiency 5. A child, age 8, has a high degree of edema of the whole body, especially in the lower limbs. When pressed, the skin sinks deeply and is slow to rebound. Other signs and symptoms include a pale face, fatigue and lack of strength, stomach and abdominal distention, unformed stools, scanty urination, a pale and enlarged tongue with a white coating, and a deep thready pulse. What are the correct treatment principles? A. Fortify the spleen, boost qi, promote urination, disperse swelling B. Warm the kidney, fortify the spleen, warm qi to move water and promote urination C. Enrich yin, nourish the kidney, clear heat, drain dampness D. Remove dampness, direct the turbid downward, boost qi to promote urination E. Supplement and boost the spleen and lung, diffuse the lung, promote urination 6. A child has edema of the whole body, especially in the lower limbs with a pale face, cold limbs and aversion to cold, lassitude of spirit, a tendency to lie down, scanty urination, a poor appetite, unformed stools, a

pale enlarged tongue with teeth marks, a glossy white tongue coating, and a deep weak thready pulse. What is the best treatment formula? A. Zhēn Wŭ Tāng (True Warrior Decoction) and Huáng Qí Guì Zhī Wŭ Wù Tāng (Astragalus and Cinnamon Twig Five Substances Decoction) B. Yī Guàn Jiān (Effective Integration Decoction) C. Yù Píng Fēng Săn (Jade Wind-Barrier Powder) D. Liù Wèi Dì Huáng Wán (Six-Ingredient Rehmannia Pill) E. Zhī Băi Dì Huáng Wán (Anemarrhena, Phellodendron and Rehmannia Pill)

Answers 1. C 2. E 3. E 4. C 5. B 6. A

Chapter 26 Urinary Frequency Urinary frequency is a common urinary disease in children that features abnormally frequent urination. It is classified in TCM as a kind of lin (strangury) syndrome, or painful urination syndrome; in children heat lin and qi lin patterns are most common. According to Western medicine, urinary-tract infections, urinary calculi, tumors and daytime urinary frequency syndrome can all cause frequent urination. The most common are urinary-tract infections and daytime urinary frequency syndrome, also known as nervous frequent urination. Urinary frequency often occurs in preschool children, especially during infancy. The disease affects girls more often than boys, but in the newborn- or early-infant periods, incidence of the disease is higher in boys than in girls. If treated in time, the prognosis is good. The main cause of urinary frequency involves impaired bladder qi transformation caused by damp-heat brewing and binding in the lower jiao, and qi deficiency of the spleen and kidney. Yin deficiency with internal heat, caused by recurrent disease that damages kidney yin, can also lead to urinary frequency. The disease locations are the kidney and bladder. The main external cause is damp-heat, while internal causes involve spleen and kidney depletion. The pathological changes involve damp-heat brewing internally, and deficiency of the kidney and spleen. Longstanding damp-heat can damage the blood vessels in the bladder to cause bloody lin syndrome. The sediment resulted from damp-heat evaporating fluids and

condensing urine causes sandy urine, called stony lin syndrome. Damp-heat in the lower jiao consumes qi and damages yin, causing deficiency of kidney yin and yang, resulting in a complex deficiency-excess pattern. Long-lasting spleen and kidney qi deficiency damages yang qi which can also lead to edema because yang cannot transform qi, and thus qi cannot transform water. Weakened resistance may also occur; when the body is invaded by external pathogens, urinary frequency will become recurrent and more serious. The common clinical pattern types are damp-heat pouring downward, spleen and kidney qi deficiency, and yin deficiency with internal heat.

CLINICAL ESSENTIALS Urinary frequency is most commonly seen in urinary-tract infections and daytime urinary frequency syndrome. The most common symptoms of urinary-tract infection are urinary frequency, urinary urgency and painful urination; associated symptoms include fever, vexation, thirst, nausea and vomiting. Yin damage due to long-term illness will manifest with signs of internal deficiency-heat. If urinary frequency is accompanied by signs and symptoms of low fever, night sweats, red cheeks, vexing heat in the five centers (chest, palms and soles), a red tongue with little coating, and a thready rapid pulse, the pattern can be identified as yin deficiency with internal heat. The disease pattern can be identified as damp-heat pouring downward if the onset is acute and the course of disease is short, with signs and symptoms including urinary frequency and urgency, painful urination, fever, vexation, thirst, nausea, vomiting, a red tongue with a greasy coating, and a rapid pulse. The treatment is to clear heat, promote urination and free the bladder, and the formula is modified Bā Zhèng Săn (Eight Corrections Powder): Biăn xù (Herba Polygoni Avicularis), chē qián zĭ (Semen Plantaginis), qú mài (Herba Dianthi), huá shí (Talcum) and jīn qián căo (Herba Lysimachiae) clear heat and drain dampness. Dà huáng (Radix et Rhizoma Rhei) and zhī zĭ (Fructus Gardeniae) discharge heat and drain fire.

Dì jĭn căo (Herba Euphorbiae Humifusae) resolves toxins and cools blood. Gān căo (Radix et Rhizoma Glycyrrhizae) harmonizes all formula medicinals.

● With fever and aversion to cold, add chái hú (Radix Bupleuri) and huáng qín (Radix Scutellariae) to release the exterior and abate heat.

● With abdominal fullness and unformed stools, remove dà huáng and add dà fù pí (Pericarpium Arecae) and jiāo shān zhā (scorch-fried Crataegi Fructus Ustus) to move qi and promote transportation.

● With nausea and vomiting, add zhú rú (Caulis Bambusae in Taenia) and huò xiāng (Herba Agastachis) to direct counterflow downward and arrest vomiting.

● With bloody urine, urethral pain and sudden interruption of urination caused by urinary sand, more jīn qián căo should be added along with hăi jīn shā (Spora Lygodii), jī nèi jīn (Endothelium Corneum Gigeriae Galli), dà jì (Herba Cirsii Japonici), xiăo jì (Herba Cirsii) and bái máo gēn (Rhizoma Imperatae) to clear heat, promote urination, expel stones and stanch bleeding.

● With inhibited voiding of reddish urine, urethral scorching and stinging pain, thirst, vexation and a red tongue with little coating, the cause involves heat in the heart descending to the small intestine through the heart channel, and Dăo Chì Săn (Red-Guiding Powder) should be added to clear heart fire and promote urination.

● With frequent, short, red urine and lower-abdominal pain due to liver failing to ensure the smooth flow of qi, chái hú (Radix Bupleuri), xiāng fù

(Rhizoma Cyperi) and chuān liàn zĭ (Fructus Toosendan) should be added to soothe the liver and rectify qi. Acupuncture can also be effective in the treatment of urinary frequency:

● For the acute period of damp-heat pouring downward, select BL 40 (wĕi zhōng), BL 34 (xià liáo), SP 9 (yīn líng quán) and BL 65 (shù gŭ).

● With severe heat signs, add LI 11 (qū chí). ● With hematuria, add SP 10 (xuè hăi) and SP 6 (sān yīn jiāo). ● With lower-abdominal distention and pain, add LV 8 (qū quán). ● With alternating chills and fever, add PC 6 (nèi guān). ● With lumbar pain, select ear points Kidney and Lumbosacral areas.

CASE STUDY Male, age 4. Initial Visit: December 21st, 2007 Chief Complaint: Frequent urination for more than one month, more than 10 times each day. History: The child has had daily frequent urination through the winter, up to dozens of times a day with scanty dribbling urine, and normal urination at night. He was examined several times, and the results of all routine urine tests were normal. Signs and Symptoms: Frequent dribbling urination with turbid urine, lassitude of the spirit, a lack of strength, poor appetite, aversion to cold, a lack of warmth in the extremities, thin stools, a pale tongue with a thin greasy coating, and a weak thready pulse. Past History: Weak constitution with susceptibility to colds and flu. Physical Examination: Normal development, moderate nutrition, withered-yellow complexion, no facial or palpebral edema, heart and lungs normal. Soft abdomen with no pain when pressed, liver and spleen not palpable below the ribs. No percussion pain at the kidneys, no edema in lower limbs. Laboratory Examination: Routine urine test (-).

Pattern Differentiation

The child’s weak constitution led to spleen and kidney qi deficiency with true yang deficiency and bladder failure to retain, resulting in continuous frequent and dribbling urination. Deficiency of spleen qi and insufficient production of qi and blood caused lassitude of spirit, lack of strength, and a withered yellow complexion. Failure of splenic transformation caused the poor appetite, thin stools, pale tongue with a thin greasy coating, and the weak thready pulse. Kidney yang depletion caused aversion to cold and lack of warmth in the extremities. The kidney is the root of the congenital constitution and also stores essence. With congenial kidney deficiency, cold may easily invade the body and damage the kidney, leading to yang qi consumption and the failure of the bladder to ensure retention, resulting in urinary frequency. Yang moves in the daytime, so the daytime deficiency of yang caused urinary frequency. Yin moves at night, and yang is stored by yin, so at nighttime the urination was normal. In treatment, use medicinals to warm the kidney, assist yang, improve qi reception and consolidate the root. Assist with medicinals that fortify the spleen, harmonize the stomach, support earth and control water.

Diagnosis Urinary frequency due to spleen and kidney qi deficiency

Chinese Treatment Principles: Warm and supplement spleen and kidney, raise and hold qi, inhibit leakage

Formula: Supplemented Suō Quán Wán (Stream-Reducing Pill) [缩泉丸加味]

7 doses [Formula Analysis] Shān yào, yì zhì rén, bái zhú, yì yĭ rén and xiān líng pí warm and supplement the spleen and kidney and consolidate essence and qi. Wū yào moves qi, dissipates masses and promotes qi transformation. Fú líng and chē qián zĭ promote urination, percolate dampness, boost qi, supplement the kidney, fortify the spleen and promote urination. Shān yú ròu supplements the kidney, astringes essence, and stems desertion. Gān căo banks up earth and harmonizes the center. In addition, it harmonizes all the medicinals. Prepared Medicinals: Jì Shēng Shèn Qì Wán (Life-Saving Kidney Qi Pill), 3g, 2-3 times daily. Tui Na

Each afternoon, knead dān tián (丹⽥) 200 times, rub abdomen for 20 min, and knead DU 1 (guī wěi) 30 times. For older children, scrubbing can be used. Transverse scrub BL 23 (shèn shù) and BL 31-34 (bā liáo) until the points become hot. This method specifically treats spleen and kidney qi deficiency patterns. Acupuncture Points: BL 40 (wĕi zhōng), KI 10 (yīn gŭ), KI 7 (fù liū), KI 6 (zhào hăi), KI 3 (tài xī), RN 4 (guān yuán), BL 23 (shèn shù). Second Visit December 28th Having been treated with medicinals that warm the kidney and assist yang, yang qi has been consolidated, and urinary frequency has been reduced. Urination has become clear in color and longer-lasting. Another 7 doses were prescribed to consolidate the effect.

COMMENTARY AND DISCUSSION The symptoms of frequent and dribbling urination, lassitude of spirit, lack of strength and poor appetite are clear signs of spleen and kidney qi deficiency. However, the aversion to cold, lack of warmth in the extremities and thin loose stools can also indicate yang deficiency of both spleen and kidney. It is said that as qi deficiency develops, it leads to yang deficiency. Therefore, yang deficiency patterns often include most qi deficiency symptoms plus deficiency cold signs and symptoms, such as cold extremities or/and aversion to cold. Also, the symptoms of yang deficiency are more severe than qi deficiency. Although implied by the treatment principles, there is also failure to stabilize and hold fluids, evident as the child urinated dozens of times during the day. This indicates fluid leakage due to kidney qi failure to stabilize and hold. The principles of treatment in this case are to warm yang and hold body fluids to inhibit leakage, specifically; “warm and supplement spleen and kidney, raise and hold qi, inhibit leakage”. This explains why Supplemented Suō Quán Wán and the prepared medicinal formula Jì Shēng Shèn Qì Wán were prescribed. The former acts to warm the kidney, dispel cold, and stop leakage. The latter is a famous classical formula for warming yang, supplementing the kidney and strengthening the gate of vitality. Because Jì Shēng Shèn Qì Wán is a yang-warming formula, why is it named “Life-Saving Kidney Qi Pill”? The answer lies in the shào huŏ or “lesser fire” theory from The Yellow Emperor’s Inner Classic (Huáng Dì

Nèi Jīng). The “lesser fire” is a physiological motive force which controls the transformation of vital substances. Kidney yang is the representative of lesser fire; when kidney yang is strong, it transforms essence into either primary qi or kidney qi. This is why “Life-Saving Kidney Qi Pills” contain yin-and essence-nourishing medicinals such shēng dì (Radix Rehmanniae), shān yào and shān zhū yú (Fructus Corni) in addition to yang-natured medicinals such as fù zĭ (Radix Aconiti Lateralis Praeparata) and ròu guì (Cortex Cinnamomi). The warm medicinals within the formula strengthen and accelerate the transformation of essence into qi so that kidney qi is generated constantly.

STUDY QUESTIONS 1. What are the main signs and symptoms of the spleen and kidney qi deficiency pattern, and how is this differentiated from damp-heat pouring downward? 2. What is the prognosis of this case? 3. To what must one pay attention in the nursing of this disease?

Answers 1. The main symptoms of the spleen and kidney qi deficiency pattern include a long course of the disease, frequent and dribbling urination, turbid urine, lassitude of the spirit, lack of strength, a withered-yellow facial complexion, poor appetite, aversion to cold, a lack of warmth in the extremities, thin stools, palpebral edema, a pale tongue with a thin greasy tongue coating, and a weak thready pulse. The main features of damp-heat pouring downward include an acute onset with urethral burning pain, turbid dribbling urine, sagging distention in the lower abdomen and an aching lumbus. In the infants, there is also restless crying, fever, vexation, thirst, headache, body pain, nausea, vomiting, a red tongue with a thin greasy slightly yellow coating, and a strong rapid pulse. The patterns can be differentiated clearly based on these main features. 2. The disease is caused by weak constitution with spleen and kidney qi deficiency, and can be treated effectively by regulating the constitution through supplementing and boosting the spleen and kidney. If the patient’s

vitality is sufficient, the prognosis is good. However, spleen and kidney qi deficiency patterns typically last for a long time and damage yang qi. If yang cannot transform qi, and qi cannot transform water, edema may occur. The body’s resistance is also impaired, and when invaded by external pathogens, urinary frequency can become recurrent and even more serious. 3. Keep the child clean and wash the genitals frequently to avoid infection. Change diapers or underpants frequently, and be sure that they are not too tight. Treat pinworm infections, phimosis, or hymen umbrella disease in a timely fashion. Treat urinary tract malformation early, and prevent urinary tract obstructions and renal scarring. Drink more water and limit intake of spicy foods. Wash the genitals after bowel movements and before sleeping. Increase nutritional intake and provide regular exercise to build up general health.

CASE SCENARIOS The following cases present variations of this condition. After familiarizing yourself with the possible common pattern presentations and appropriate formulas for treatment, use the following exercises to test your overall understanding of the condition. 1. A child patient, age 5, has had emerging urinary frequency in the recent 2 days. His urine is scanty and yellow; other signs and symptoms include severe pain when urinating, fever, nausea, a red tongue with a greasy tongue coating, and a strong rapid pulse. What is the most suitable treatment formula? A. Liù Yī Săn (Six-to-One Powder) B. Sì Miào Wán (Wonderfully Effective Four Pill) C. Wŭ Líng Săn (Five Substances Powder with Poria) D. Èr Miào Wán (Two Mysterious Pill) E. Bā Zhèng Săn (Eight Corrections Powder) 2. A child patient, age 14, has had recurrent urinary frequency for the last half year, with signs and symptoms including dribbling urination, lassitude of the spirit, lack of strength, withered-yellow facial complexion, slight palpebral edema, poor appetite, aversion to cold, pale tongue with a thin and greasy coating, and a thready weak pulse. What is the most suitable treatment formula? A. Suō Quán Wán (Stream-Reducing Pill)

B. Bŭ Zhōng Yì Qì Tāng (Center-Supplementing Qi-Boosting Decoction) C. Yì Gōng Săn (Special Achievement Powder) D. Liù Wèi Dì Huáng Wán (Six-Ingredient Rehmannia Pill) E. Zhī Băi Dì Huáng Wán (Anemarrhena, Phellodendron and Rehmannia Pill) 3. A child patient, age 5, has had urinary frequency for the last 6 months with dribbling urine, a yellow face, fatigue, lack of warmth in the limbs, a pale tongue, and a thready pulse. What are the correct principles of treatment? A. Warm and supplement spleen and kidney, promote securing and continence B. Enrich yin, supplement the kidney, supplement middle qi C. Clear heat, drain dampness, free the bladder D. Supplement the middle, boost qi, promote securing and continence E. Warm and supplement kidney yang, enrich yin, clear heat 4. A child patient, age 7, has had urinary frequency for 3 months with a dry throat, thirst, a low fever, night sweating, vexation, dry lips, a red tongue with little coating, and a thready rapid pulse. What is the most suitable treatment formula? A. Liù Wèi Dì Huáng Wán (Six-Ingredient Rehmannia Pill) B. Jì Shēng Shèn Qì Wán (Life-Saving Kidney Qi Pill)

C. Zhī Băi Dì Huáng Wán (Anemarrhena, Phellodendron and Rehmannia Pill) D. Suō Quán Wán (Stream-Reducing Pill) E. Dăo Chì Săn (Red-Guiding Powder)

Answers 1. E 2. A 3. A 4. C

Chapter 27 Enuresis Enuresis is the involuntary discharge of urine during sleep in children over 3 years of age. As the channels become developed, qi and blood become abundant and organs grow strong, a one year-old child can control his or her urine during the day. If a child cannot control his or her urine by the age of 3 and often wets the bed, it may be enuresis. The disease most commonly affects patients under 10 years of age and affects boys more often than girls. Fewer children are affected as age increases. Occasional bedwetting due to excessive drinking before bed or fatigue is not considered pathogenic. The generation and excretion of urine involves the lung, spleen, kidney, sanjiao and bladder. Enuresis is caused by the failure of the bladder’s retentive power, primarily due to kidney qi deficiency, lung and spleen qi deficiency, or liver channel constraint heat. Furthermore, some patients wet the bed while dreaming because of heart and kidney failure to interact with a discordance of water and fire. Some wet the bed because of deep sleep due to exuberant phlegm-damp. Some patients have developed poor nighttime micturating habits, and some over 3 years of age still use diapers and have not learned to control micturition.

CLINICAL ESSENTIALS Common pattern types of enuresis include lung-spleen qi deficiency, kidney qi deficiency, and liver channel constraint-heat.

● Signs and symptoms of spleen-lung qi deficiency include enuresis during sleep, possibly several times per night, a lusterless complexion, mental fatigue, weakness, a weak voice, poor appetite, soft stools, spontaneous sweating, a tendency to catch cold, a light red tongue with a thin white coating, and a deep and weak pulse.

● Signs and symptoms of kidney qi deficiency include enuresis during sleep, possibly several times per night, profuse clear urination, a lusterless complexion, mental fatigue, weakness, a feeling of coldness and cold limbs, mental developmental disorders, a pale tongue with a white glossy coating, and a deep weak pulse. The key to differentiating this pattern is recognizing that deficiency of kidney qi, insufficiency of the mìng mén (gate of vitality) fire, and deficiency cold of the kidney result in failure to control the waterways, which leads to clear profuse urination and frequent bet-wetting. This pattern is characterized by profuse and frequent enuresis with other deficiency cold symptoms.

● Signs and symptoms of liver channel constrained heat include enuresis while dreaming, scanty yellow foul-smelling urine, impatience, much dreaming, grinding of the teeth, heat in the palms and soles, red face and lips, red eyes, a red tongue with a greasy yellow coating, and a slippery rapid pulse. The key to differentiating this pattern is recognizing that dampheat brewing internally and lying constrained in the liver channel distresses

the bladder and causes the disease. Yellow scanty urine, teeth grinding, a rash and impatient nature, and red eyes are all features of excess liver channel heat.

● Treatment for liver channel constrained heat includes modified Lóng Dăn Xiè Gān Tāng (Gentian Liver-Draining Decoction) to clear heat and promote urination, and to slow and stop the enuresis.

● Treatment for kidney qi deficiency includes a Chinese prepared medicine and tui na therapy. Administer Wŭ Zĭ Yăn Zōng Wán (Five-Seed Progeny Pill) 6g, twice daily. For tui na therapy to treat kidney qi insufficiency, knead dān tián (丹 ⽥) 200 times, rub abdomen for 20 min, and knead DU 1 (guī wěi) 30 times. With older children, scrubbing can be used. Transverse scrub BL 23 (shèn shù) and BL 31-34 (bā liáo) until they become hot to the touch. Supplement pí jīng (脾经) and shèn jīng (肾经) 800 times, push sān guān (三关) 300 times, knead dān tián (丹⽥) for 20 min, and press DU 20 (băi huì) 50 times each afternoon.

CASE STUDY Male, age 8. Initial Visit: March 8th, 2009 Chief complaint: Enuresis for more than a year, becoming worse in the recent 3 months. History: The patient often wetted the bed last year, and over the recent 3 months the bedwetting became more frequent, happening once a night or even 2 to 3 times a night, with the patient unaware until waking. He was treated with several folk prescriptions to no effect, and had taken Liù Wèi Dì Huáng Wán (Six-Ingredient Rehmannia Pill) for more than a month with no improvement. Signs and Symptoms: Enuresis during the night, frequent and profuse urination in the daytime, a lusterless complexion, fatigue, a weak voice, poor appetite, soft stools, spontaneous sweating, a light red tongue with a thin white coating, and a deep and weak pulse. Past History: A weak constitution with susceptibility to colds Physical Examination: Normal physical development, middle level of nutrition, poor spirits, normal external genitalia and urethra. Laboratory Examination: A routine urine test was negative.

Pattern Differentiation The location of disease in this case is the spleen and lung. With spleenlung qi deficiency, the failure of the lung’s governing of management and

regulation, sinking of spleen qi, the breakdown of sanjiao qi transformation, and the failure of the bladder’s retentive power and storing of fluids cause the frequent enuresis. Spleen deficiency and the failure in transportation and transformation cause the poor appetite and soft stools. Qi deficiency and failure to hold body fluids cause the spontaneous sweating. Abnormal qi transportation fails to transform food and drink into essence, so the body and mind are malnourished, causing poor spirit. A pale tongue with a thin coating and a deep and weak pulse are features of the pattern of qi and blood insufficiency and damp-turbidity brewing internally. Therefore, the pathomechanism of this disease is the spleen-lung qi deficiency, clear yang failing to ascend and the failure of the bladder’s retentive power. The previous doctor treated the disease by supplementing the kidney with Liù Wèi Dì Huáng Wán for more than a month but with no effect due to a mistake in pattern differentiation. If the enuresis was caused by kidney deficiency, there should have been signs and symptoms including a cold face and limbs, fatigued lumbus and legs, listlessness of spirit, mental retardation, and so on.

Diagnosis Enuresis due to spleen-lung qi deficiency with central qi prolapse

Clinical Treatment Principles: Boost qi and fortify the spleen, raise yang and lift the sunken, arrest enuresis with astringents

Formula: Modified Bŭ Zhōng Yì Qì Tāng (Center-Supplementing QiBoosting Decoction) and Suō Quán Wán (Stream-Reducing Pill) [补中益⽓汤合缩泉丸加减]

6 doses [Formula analysis] Huáng qí, dăng shēn, bái zhú, chén pí, shēng má, chái hú, and zhì gān căo boost qi, fortify the spleen, raise yang and eliminate dampness. Jī nèi jīn, jiāo shén qū, jiāo shān zhā, and jiāo mài yá promote digestion and appetite and support transformation. Yì zhì rén, shān yào, and wū yào warm the spleen and consolidate essence. The formula acts to promote qi and raise yang to treat the root, and consolidate the urine to treat the branch. Prepared Medicines

● Bŭ Zhōng Yì Qì Wán (Center-Supplementing Qi-Boosting Pill), 6g, twice daily.

● Suō Quán Wán (Stream-Reducing Pill), 6 g, twice daily. Body Acupuncture Main Points: HT 5 (tōng lĭ), KI 4 (dà zhōng), RN 4 (guān yuán). Supplementary Points: RN 6 (qì hăi), ST 36 (zú sān lĭ), BL 23 (shèn shù), SP 6 (sān yīn jiāo). Manipulations: Alternate between the supplemental points each treatment. Treat once daily, or once every other day. Auricular Acupuncture Points: Enuresis point (yí niào diăn) (between kidney [shèn] and endocrine [nèi fēn mì], under esophagus [shí dào]), subcortex (pí zhì xià). Auricular Plaster Therapy: Urinary bladder (páng guāng), kidney (shèn), spleen (pí), sanjiao (sān jiāo), heart (xīn), brain stem (năo gàn), shen men (shén mén). Second Visit March 14th Occurrences of enuresis decreased, diet increased, and spontaneous sweating became less obvious. 6 more doses of the same formula were prescribed. Third Visit March 20th

Enuresis ceased after 3 doses, spirit improved and the pulse became stronger after 6 doses. 10 more doses were given to consolidate the results, and there has been no relapse.

COMMENTARY AND DISCUSSION In one famous case of enuresis, a child’s parents took him to see many famous local practitioners, who diagnosed the case as kidney qi deficiency failing to hold body fluids. A few of the practitioners prescribed Jīn Guì Shèn Qì Wán (Golden Cabinet’s Kidney Qi Pill), from Essentials from the Golden Cabinet (Jīn Guì Yào Lüè), with no results. Finally the child’s parents brought him to Dr. Qin Bai-wei, a famous modern practitioner. Dr. Qin carefully checked the child’s tongue and pulse, confirmed the same diagnosis, and added only one Chinese medicinal to the previously prescribed formula, jié gĕng (Radix Platycodonis). After taking a few bags of the formula, the child’s enuresis completely disappeared. Dr. Qin explained that “The lungs are the upper source of water while the kidneys are the lower source of water. If either is out of balance, it may cause enuresis. The medicinal jié gĕng can raise qi and direct the effect of other medicinals to the upper regions of the body. This prevents the lungs from allowing body fluids to descend excessively to the urine bladder, and therefore cures the enuresis”. This story demonstrates how accurate diagnosis is essential for clinical treatment. In the current case, the child had a poor appetite, occasional loose stools, spontaneous sweating, a lusterless complexion, fatigue, weakness, a weak voice, poor appetite, etc. These are signs and symptoms of qi deficiency of the lung and spleen with middle qi prolapse, causing the bladder’s failure to hold urine. The parents initially gave him Liù Wèi Dì Huáng Wán, which did not work as the prescription is for kidney yin deficiency. The heavy and greasy yin-supplementing medicinals may even

make a case of spleen qi deficiency worse. In addition, the formula contains three draining medicinals that could promote urination and actually aggravate the symptoms. The accurate differential diagnosis for this case is spleen and lung qi deficiency with middle qi prolapse. The formula Bŭ Zhōng Yì Qì Tāng was applied to raise yang qi and supplement spleen and lung qi. In this prescription, shēng má and chái hú raise yang qi, which helps hold urine. Compared to Dr. Qin’s method of adding jié gĕng into Jīn Guì Shèn Qì Wán, this treatment is different in approach but equally effective. In this case, Suō Quán Wán was combined with Bŭ Zhōng Yì Qì Tāng. Suō Quán Wán is often added to a prescription as a modification and not commonly applied alone for adults, because it only contains three ingredients and may only be strong enough for children. Chinese Herbal Medicine: Formula and Strategies, complied and translated by Dan Bensky and Randall Barolet, states, “this combination of herbs is warm, but not drying, and eliminates cold from deficiency in the lower parts of the body. When the kidney qi is restored it will reassert its control over the functions of the bladder, and urinary frequency will cease. However, this simple formula is rather weak and therefore often inadequate by itself to resolve the problem. An exception is children, especially those with enuresis that fit this presentation, who often respond well to this formula”.

STUDY QUESTIONS 1. What are the main signs and symptoms of lung-spleen qi deficiency? How can one differentiate between this pattern and the pattern of liver channel constraint-heat? 2. How can one differentiate enuresis due to the kidney qi insufficiency? What are the treatment principles?

Answers 1. Signs and symptoms of the pattern of spleen-lung qi deficiency include enuresis during sleep, possibly several times in one night, a lusterless complexion, mental fatigue, weakness, a weak voice, a poor appetite, soft stool, spontaneous sweating, susceptibility to cold, a light red tongue with a thin white tongue coating, and a deep weak pulse. The key to pattern differentiation is recognizing that if spleen qi is deficient, transportation and transformation will fail and qi and blood will be insufficient, causing a lusterless complexion, poor appetite and soft stools. If lung qi is deficient, the defensive exterior will be insecure, causing spontaneous sweating and cold. On the other hand, damp-heat brewing internally and lying constrained in the liver channel distresses the bladder and causes enuresis. Yellow scanty urine, grinding of the teeth, a rash and impatient nature, and red eyes are all features of excessive heat in the liver channel.

2. In enuresis due to deficiency of kidney qi, insufficiency of the mìng mén fire and deficiency cold of the kidney result failure to control the waterways, which leads to clear and profuse urination and frequent bedwetting. Signs and symptoms of this pattern include profuse and frequent enuresis with other deficiency-cold symptoms. The main treatment principles are to warm and supplement kidney yang and to arrest enuresis and emission with astringents.

CASE SCENARIOS The following cases present variations of this condition. After familiarizing yourself with the possible common pattern presentations and appropriate formulas for treatment, use the following exercises to test your overall understanding of the condition. 1. A child, age 3 and a half, often catches cold and has enuresis during sleep once or twice a night with light yellow urine, poor appetite, a light red tongue with a thin white coating, and a deep weak pulse. What is the most suitable treatment formula? A. Jīn Guì Shèn Qì Wán (Golden Cabinet’s Kidney Qi Pill) and Yù Píng Fēng Săn (Jade Wind-Barrier Powder) B. Shēn Líng Bái Zhú Săn (Ginseng, Poria and Atractylodes Macrocephalae Powder) and Jiāo Tài Wán (Peaceful Interaction Pill) C. Bŭ Zhōng Yì Qì Tāng (Center-Supplementing and Qi-Boosting Decoction) and Suō Quán Wán (Stream-Reducing Pill) D. Tù Sī Zĭ Săn (Cuscuta Seed Powder) and Sì Jūn Zĭ Tāng (Four Gentlemen Decoction) E. Lóng Dăn Xiè Gān Tāng (Gentian Liver-Draining Decoction) and Dăo Chì Săn (Red-Guiding Powder) 2. A child, age 6, has signs and symptoms including bed-wetting more than twice a night, profuse clear urination, lusterless complexion, cold limbs, sensitivity to cold, a pale tongue with a white and glossy tongue coating, and a deep thready and weak pulse.

What is the most suitable treatment formula? A. Bŭ Zhōng Yì Qì Tāng (Center-Supplementing Qi-Boosting Decoction) B. Jīn Guì Shèn Qì Wán (Golden Cabinet’s Kidney Qi Pill) C. Lóng Dăn Xiè Gān Tāng (Gentian Liver-Draining Decoction) D. Tù Sī Zĭ Săn (Cuscuta Seed Powder) E. Dăo Chì Săn (Red-Guiding Powder) 3. A child, age 5, has signs and symptoms including enuresis during sleep, yellow and foul-smelling urine, a rash and impatient nature, red face and eyes, red lips, thirst, a red tongue with a yellow and greasy tongue coating, and a slippery and rapid pulse. What are the correct principles of treatment? A. Warm and supplement kidney yang, calm the mind, arrest enuresis B. Clear heat, drain dampness, relax tension, arrest enuresis with astringents C. Clear heat, soothe the liver, arrest enuresis with astringents D. Nourish the kidney, clear heat, arrest enuresis with astringents E. Clear heart heat, nourish the kidney, arrest enuresis with astringents

Answers 1. C 2. D

3. B

Chapter 28 Five Types of Developmental Delay and Five Kinds of Flaccidity The five types of developmental delay and five kinds of flaccidity are diseases of developmental disorder. The former refers to delay in a child’s reaching of the developmental milestones of standing, walking, growing hair, teething and learning to talk. The latter refers to physical softness of the head and nape, mouth, hands, feet and muscles. Both are children’s diseases of deficiency. Western disease names for these disorders include cerebral dysgenesis, rickets, cerebral palsy, mental retardation, and others. These diseases are mainly caused by prenatal insufficiency and partly by unsuitable postnatal health care, improper parenting and malnutrition. The two aspects of the pathomechanism are upright qi deficiency and pathogenic qi excess. Upright qi deficiency refers to liver, kidney, heart and spleen deficiency, qi and blood deficiency, and essence-marrow deficiency. Excess pathogenic qi includes phlegm and blood stasis blocking the heart channel and the collaterals of the brain. The heart and brain therefore fail to control shen (spirit), resulting in mental health disorders. Thus, these diseases belong to the category of root deficiency with branch excess. Such diseases that are not serious and are caused by unsuitable postnatal health care, if treated in time, can be cured. Those that have

complicated signs, long courses of disease, and are caused by prenatal insufficiency often become chronic and have bad prognosis. Common patterns of types of these diseases include liver-kidney depletion, heart and spleen deficiency, and phlegm-stasis obstruction.

CLINICAL ESSENTIALS Five types of developmental delay and five kinds of flaccidity are the clinical symptoms of degenerative encephalopathy such as in cerebral dysgenesis, mental retardation, hypothyroidism and leukodystrophy, or are caused by postnatal brain damage and weak health due to enduring illness or in the sequelae of some metabolic diseases and encephalitis. Disease identification and differentiation are based on the signs and symptoms. The main signs and symptoms of heart and spleen deficiency include language retardation, sluggish spirit, mental retardation, slowness to grow hair, sparse dry yellow hair, fatigued and soft limbs, flabby muscles, drooling from the corners of the mouth, forceless sucking and chewing, occasional protruding and waggling of the tongue, poor appetite, dry hard stools, a pale enlarged tongue, a scanty tongue coating, a thready and moderate pulse, and pale finger venules. There are several key points to pattern identification:

● First, the heart governs the spirit, and language is the voice of the heart. Heart qi deficiency causes language retardation, sluggish spirit, and mental retardation.

● Second, the heart governs the blood, spleen engenders blood, and hair is the surplus of blood. Deficiency of the heart and spleen makes the blood unable to luster the hair, so the hair grows slowly and is scant and yellow.

● Third, spleen deficiency results in loss of the source of generation and transformation, so the limbs are fatigued and soft, hands and feet are

useless, and muscles are flabby.

● Fourth, a protruding and waggling tongue is a sign of heart deficiency and mental retardation. Treat with Tiáo Yuán Săn (Original Qi-Regulating Powder) to fortify the spleen and nourish the heart, and supplement and boost qi and blood.

CASE STUDY Female, age 3. Initial Visit: April 6th, 2009 Chief Complaint: Slow development since birth, trouble speaking and walking for 2 years. History: The patient was delivered prematurely in the 7th month and fed with breast milk. She had a poor appetite and was in poor health. She teethed two teeth at one year of age and stood reluctantly at age 2. She was now over 3 years old, but could not walk steadily and was liable to fall. Signs and Symptoms: Fatigued spirit, lack of strength, a withered yellow complexion, inability to speak, 18 teeth, scant hair, flabby muscles, drooling from the corners of the mouth, poor appetite, disturbed sleep at night, a pale tongue with little coating, a deep weak thready pulse, and pale finger venules. Physical Examination: Poor development, malnourished, arms fail to hold and lift, reduced muscle tension of the limbs, normal heart and lungs. Neurological examination: decreased knee jerk. Laboratory Examination: Routine blood test: Hemoglobin 90 g/L.

Pattern Differentiation The patient is over 3 years old, but cannot walk normally and has clinical signs and symptoms of the five types of developmental delay and five kinds of flaccidity, including slowness to teethe, to grow hair and in

learning to talk, as well as softness of the hands, feet and muscles. The congenital essence deficiency due to premature birth and the acquired disharmony due to unsuitable feeding caused the developmental disorders. The liver governs the sinews, and the kidneys govern the bones. Essential qi deficiency causes the sinews and bones to lose their nourishment and so causes the delays of standing, walking, hair growing and teething. The spleen governs the muscles and limbs. Spleen deficiency impairs the transportation of essence and causes the whole body to lack nourishment; therefore, the limbs and muscles are flabby and lack strength. Spleen deficiency also results in failure to contain fluids, causing the drooling from the corners of the mouth. Spleen and kidney deficiency reduces essential qi and causes the heart and brain to lack nourishment, causing mental retardation. Therefore, the pathomechanism is liver-kidney depletion, spleenstomach weakness, and essential qi and yin-blood insufficiency, causing the organs, sinews, bones and muscles to become deprived of nourishment. This is a root deficiency pattern.

Diagnosis Five types of developmental delay and five kinds of flaccidity due to liver-kidney depletion

Clinical Treatment

Principles: Supplement liver and kidney to boost essence and strengthen sinews and bones; supplement spleen and stomach to promote the source of transformation and enrich qi and blood Formula: Modified Bŭ Shèn Dì Huáng Wán (Kidney-Supplementing Rehmannia Pill) and Bŭ Zhōng Yì Qì Tāng (Center-Supplementing QiBoosting Decoction) [补肾地黄丸合补中益⽓汤加减]

10 doses [Formula Analysis] Shú dì, shān yào, shān yú ròu, niú xī, lù róng, ròu guì and yì zhì rén,supplement the kidney, boost essence, promote brain health, and strengthen sinew and bone. Rén shēn powerfully supplements original qi. Huáng qí, fú líng, bái zhú and chén pí promote spleen-stomach transportation and enrich the source of transformation to engender qi and blood.

Lóng gŭ boosts the kidney and strengthens bone. Prepared Medicines

● Qĭ Jú Dì Huáng Wán (Lycium Berry, Chrysanthemum and Rehmannia Pill), 3g, 3 times daily. For liver-kidney depletion.

● Hé Chē Dà Zào Wán (Placenta Great Creation Pill), 3g, 3 times daily. For essence and blood insufficiency and emptiness of the sea of marrow. Tui Na Upper Extremities: DU 14 (dà zhuī), GB 21 (jiān jĭng), LI 15 (jiān yú), LI 11 (qū chí), SJ 4 (yáng chí), LI 4 (hé gŭ). Lower Extremities: BL 23 (shèn shù), DU 4 (mìng mén), DU 3(yāo yáng guān), GB 29 (jū liáo), GB 30 (huán tiào), BL 37 (yīn mén), BL 40 (wĕi zhōng), BL 57 (chéng shān), BL 60 (kūn lún), ST 41 (jiĕ xī), ST 36 (zú sān lĭ), GB 34 (yáng líng quán). Manipulations: Pushing, grasping, pressing, kneading, foulage (twisting an appendage after first applying pressure) and scrubbing. Acupuncture Points: DU 14 (dà zhuī), DU 20 (băi huì), ST 36 (zú sān lĭ), BL 23 (shèn shù), RN 4 (guān yuán).

● For mental retardation: add sì shén cōng (EX-HN1), yìn táng (EXHN3).

● For paraplegia: add GB 30 (huán tiào), BL 54 (zhì biān), GB 34 (yáng líng quán). ● For limp wrist: add SJ 5 (wài guān), SJ 4 (yáng chí). ● For varus deformity: add GB 38, 39 (jué gŭ), BL 60 (kūn lún).

● For foot eversion: add SP 6 (sān yīn jiāo), KI 3 (tài xī). Auricular Acupuncture—(for all patterns) Points: Heart (xīn), kidney (shèn), liver (gān), spleen (pí), subcortex (pí zhì xià), brain stem (năo gàn). Moxibustion Moxibustion on the ankles, 3 cones per time, treat once daily. For heart and spleen deficiency. Functional Exercise: Exercises for cerebral palsy include the training of the body, skills and speech. Orthotics can be used. Second Visit April 16th Food intake increased, drooling lessened, and spirit improved. Continue with the same formulas and tui na therapy for another month. Third Visit May 20th After one month of treatment, the patient’s appetite grew, the spirit was improved, and her limbs moved more forcefully as she walked with greater stability, her hands now held and lifted normally. Jīn Guì Shèn Qì Wán (Golden Cabinet’s Kidney Qi Pill) and Rén Shēn Jiàn Pí Wán (Ginseng Spleen-Fortifying Pill) were prescribed to supplement the spleen and kidney. Medicinal and dietary therapies were suggested to help the patient develop normally. The disease, which is chronic and due to weakness, cannot be cured in a short time. Therefore, after the illness had been moderated, the prescribed

pills were continued for over 10 months to consolidate the effect.

COMMENTARY AND DISCUSSION Chinese medicine is a holistic medicine, where all parts of the human body are interconnected. The five internal organs are connected with the five body tissues, and the vital substances are generated in different internal organs. This connection allows us to make evaluations and diagnoses according to signs and symptoms showing in the superficial areas. The Yellow Emperor’s Inner Classic (Huáng Dì Nèi Jīng) described many skills and techniques to monitor the functions of the internal organs and the abundance of vital substances through their externally correlated areas. For example, the status of essence can be evaluated by the condition of hair, teeth, bones, joints and so on. For the same reason, the growth, maturation and developmental condition of children can be monitored through the signs and symptoms showing in the superficial areas of the human body. Of all internal organs, the spleen and kidneys are the two most important organs for the growth and development as they are the roots of prenatal and postnatal essence. The signs and symptoms described in this case show the involvement of the kidney and spleen. The patient was prematurely delivered in the 7th month, which indicates insufficiency of congenital or prenatal essence with kidney deficiency. Her poor appetite, withered and yellow complexion, flabby muscles and drooling from the corners of the mouth are typical signs of spleen deficiency. Therefore, the differential diagnosis should include spleen and stomach deficiency in addition to liver and kidney depletion.

The treatment principles and medicinal prescriptions applied in this case did indeed include the spleen and stomach. The treatment principles were to supplement liver and kidney to boost essence and strengthen sinews and bones; supplement spleen and stomach to promote the source of transformation and enrich qi and blood. Modified Bŭ Shèn Dì Huáng Wán and Bŭ Zhōng Yì Qì Tāng were therefore applied to replenish these two organs. The several follow-up treatments continued with the same principles. At the third follow-up visit, Jīn Guì Shèn Qì Wán and Rén Shēn Jiàn Pí Wán were given to strengthen the kidney and the spleen. The generation of prenatal and postnatal essence is endless, and thus plays the role of governing the birth, growth, development and reproduction of the human body.

STUDY QUESTIONS 1. What are the effective TCM therapies for the five types of developmental delay and five kinds of flaccidity? 2. How effective is the treatment? What is the prognosis of this case? How should the disease be prevented and nursed?

Answers 1. Effective TCM therapies for this disease include acupuncture, moxibustion and tui na. 2. Instances of disease that are not serious and are caused by unsuitable postnatal health care, if treated in time, can be cured. Those that have complicated signs, long courses of disease and are caused by prenatal insufficiency often become chronic and have bad prognoses. To prevent occurrence of the disease, avoid marriage among close relatives and have premarital health checks to avoid genetic diseases. Pay attention to proper nursing during pregnancy, nourish and take good care of the fetus, and do not use medicines that could be harmful to the fetus. Feed the infant reasonably, and prevent various acute and chronic diseases. In nursing the disease, pay attention to functional training and intelligence training. Provide proper nourishment and nurse according to proper, scientific understandings. Massage withered and soft muscles guided by tui na therapy to avoid atrophy.

CASE SCENARIOS The following cases present variations of this condition. After familiarizing yourself with the possible common pattern presentations and appropriate formulas for treatment, use the following exercises to test your overall understanding of the condition. 1. A patient, age 3, has signs and symptoms including language retardation, sluggish spirit, mental retardation, sparse dry yellow hair, fatigued and soft limbs, drooling from the corners of the mouth, poor appetite, dry hard stools, a pale enlarged tongue with a scanty thin coating, and pale finger venules. What is the correct diagnosis? A. Five types of developmental delay and five kinds of flaccidity due to liver-kidney depletion B. Five types of developmental delay and five kinds of flaccidity due to heart and spleen deficiency C. Five types of developmental delay and five kinds of flaccidity due to phlegm-stasis obstruction D. Rickets due to heart-spleen deficiency E. Gān pattern; dry gān pattern 2. A patient, age 2, has developed slowly with poor development of standing, walking, hair growing, teething and learning to talk compared with other children of the same age. The patient is susceptible to fright

when sleeping at night and has a pale tongue with a scanty coating, and a thready deep and weak pulse. What is the proper treatment formula? A. Jiā Wèi Liù Wèi Dì Huáng Wán (Supplemented Six-Ingredient Pill with Rehmannia) B. Shí Quán Dà Bŭ Wán (Perfect Major Supplementation Pill) C. Tōng Qiào Huó Xuè Tāng (Orifices-Unblocking Blood-Quickening Decoction) D. Kŏng Shèng Zhĕn Zhōng Dān (The Sage Confucian Secret Elixir) E. Tiáo Yuán Săn (Original Qi-Regulating Powder) 3. A patient, age 2, has a sluggish spirit, mental retardation, a pale face, soft limb-joints, arms failing to hold and lift, and legs failing to walk. What are the proper treatment principles? A. Boost qi and nourish the blood B. Fortify the spleen and supplement the kidney C. Supplement the kidney and nourish the liver D. Warm and activate spleen yang E. Supplement the heart and nourish blood 4. A patient, age 28 months, has a soft head and nape with the head always leaning, drooling from the corners of the mouth, and forceless movement. What is the proper treatment formula?

A. Bŭ Zhōng Yì Qì Tāng (Center-Supplementing Qi-Boosting Decoction) B. Bŭ Shèn Dì Huáng Wán (Supplement Kidney Rehmannia Pill) C. Shèn Qì Wán (Kidney Qi Pill) D. Bā Zhēn Tāng (Eight-Gem Decoction) E. Tiáo Yuán Săn (Original Qi-Regulating Powder) 5. A patient, age 4, has slowed responses, unclear spirit and mind, difficulty in swallowing, drooling from the corners of the mouth, and phlegm rales in the throat. What is the presenting pattern? A. Liver-kidney depletion B. Heart and spleen deficiency C. Phlegm-stasis obstruction D. Heart-kidney interaction loss E. Constrained liver channel heat 6. A patient, age 5, has deafness and aphasia, slowed responses, an unclear spirit and mind, and unconscious movement. What is the proper treatment formula? A. Shèn Qì Wán (Kidney Qi Pill) B. Bŭ Shèn Dì Huáng Wán (Kidney-Supplementing Rehmannia Pill) C. Bā Zhēn Tāng (Eight-Gem Decoction) D. Tiáo Yuán Săn (Original Qi-Regulating Powder)

E. Tōng Qiào Huó Xuè Tāng (Blood-Moving Decoction for Unblocking the Orifices) and Èr Chén Tāng (Two Matured Substances Decoction)

Answers 1. B 2. A 3. A 4. E 5. C 6. E

PART V Infectious Diseases Chapter 29 Measles Measles, also known as rubeola or morbilli, is an acute infection of the respiratory system caused by the measles virus that frequently affects children. Clinical manifestations include high fever, inflammation of the eyes and the upper respiratory tract, Koplik spots, maculopapules all over the body and, after the papules vanish, peeling skin with brown pigmentation and a bran-like appearance. Following widespread use of the measles vaccine in China, the incidence rate has dropped significantly and the periodic prevalence has diminished. This disease has the same name in Chinese and Western medicine, with the cause being external contraction of the measles virus and seasonal pathogens. Caused by external contraction, the main pathological changes due to measles occur in the lung and spleen. In winter and spring, a mixture of seasonal pathogens and wind-evils invade the lung-wei, then becoming constrained in the spleen; finally the pathogen discharges externally into the skin. Standards for Diagnosis and Treatment in Pediatrics (Yòu Kē Zhèng Zhì Zhŭn Shéng) points out that, “in the initial stage of measles, the symptoms of wind-like damage, fever, cough, nasal congestion, facial

swelling, sticky and thick nasal discharge and sputum are all signs of the lung channel”, showing that the initial signs of measles are similar to the common cold” (麻疹初出,全类伤风,发热咳嗽,⿐塞⾯肿,涕唾稠 黏,全是肺经之证). The measles pathogen attacks the lung-wei from exterior to interior, and then becomes constrained in the spleen. Because the spleen governs the muscles, controls the blood, and is paired with the four limbs, the pathogen is usually discharged into the skin as upright qi and the pathogenic qi contend, thus resulting in the widespread appearance of papules all over the body. Then, as the pathogen is expelled through the papules, measles disappear gradually and the internal heat abates, although damage to body fluids will have occurred. This is the favorable-type pattern of measles. Pathogenic qi preponderance, constitutional upright qi deficiency, improper treatment or inappropriate nursing all can result in upright qi deficiency with qi failing to draw the pathogen to the exterior; when there is continued inward invasion of the toxic pathogens, the unfavorable-type pattern results. When the pathogens invade inward, the resulting scorching of fluids will bring about internal phlegm, while phlegm-heat congestion and lung qi depression will finally result in pneumonia with panting and coughing. When exuberant heat invades upward with phlegm, the phlegmheat congestion and exuberance result in toxins attacking the throat. When the pathogenic toxins attack the jueyin, cloud the pericardium and stir liver wind, transmuted patterns occur with pathogenic invasion of the heart and liver. Common clinical manifestations include both favorable and unfavorable patterns. Favorable-type patterns include lung-wei attack in the initial stage, lung and stomach attack in the peak stage, and yin-fluid

consumption and damage in the final stage. Unfavorable-type patterns include toxins blocking the lung, toxins attacking the throat, and pathogenic invasion of heart and liver.

CLINICAL ESSENTIALS For the clinical pattern differentiation of measles, the first priority is identification of favorable- or unfavorable-type patterns. Exterior-interior pattern differentiation in favorable patterns or zang-fu pattern differentiation in unfavorable patterns can help to reveal the degree of severity and the prognosis. For the favorable-type measles pattern, in its initial stage, the exterior toxic constraint is treated by diffusing the lung and promoting papule eruption through the exterior. In the peak stage, the exuberant toxin is treated by clearing heat and resolving toxins to encourage continued eruption of papules. Bitter and cold medicinals that clear heat must not be overused, as they may cause damage to upright qi and lead to an inward invasion of the measles-toxin. In the final stage, though the pathogen and toxins have disappeared, damage to upright qi is treated by nourishing yin and clearing heat. All in all, promoting the eruption of papules, expelling evil, clearing heat and resolving toxins are the essential principles in the treatment of measles. The unfavorable measles pattern is caused by exuberant pathogens combined with upright qi deficiency. During the disease, in the peak stage, possible signs and symptoms include persistent high fever, vexation and agitation, and no sweating of the limbs. Also in the peak stage there may be fulminant eruption of dense dark purple papules, or inhibited eruption of thin pale papules or with sudden disappearance, no papules on the face, or reversal cold in four limbs with a pale complexion. All of these are signs of an unfavorable-type measles pattern.

Zang-fu pattern differentiation may include toxins blocking the lungs resulting in measles mixed with pneumonia, pathogenic toxins attacking the throat resulting in measles mixed with laryngitis, or pathogens invading the heart and liver resulting in measles mixed with encephalitis. These patterns may be accompanied by debilitation of heart yang with signs and symptoms including a pale facial complexion, reversal cold of the four limbs, and a thin pulse verging on expiration, all of which belong to the category of critical measles patterns. Dr. Qian Yu-shou has created three specific treatment strategies for the three stages of measles. First is the method of diffusing the exterior and promoting eruption, suitable for the initial stage of measles:

● Chán Yī Xuān Tòu Yĭn (Cicada Molting Diffusing and Venting Beverage), containing medicinals such as chán tuì (Periostracum Cicadae), niú bàng zĭ (Fructus Arctii), jīng jiè (Medicinala Schizonepetae), fáng fēng (Radix Saposhnikoviae), zhè bèi mŭ (Bulbus Fritillariae Thunbergii), bò he (Medicinala Menthae), guā lóu (Fructus Trichosanthis) and tōng căo (Medulla Tetrapanacis).

● Gé Gēn Jiĕ Jī Tāng (Pueraria Muscle-Resolving Decoction) can alternatively be prescribed with modifications including gé gēn (Radix Puerariae Lobatae), qián hú (Radix Peucedani), niú bàng zĭ, lián qiào (Fructus Forsythiae), chán tuì, bèi mŭ, jīng jiè, chì sháo (Radix Paeoniae Rubra), sāng bái pí (Cortex Mori), mù tōng (Caulis Akebiae), gān căo (Radix et Rhizoma Glycyrrhizae) and dēng xīn căo (Medulla Junci).

● For those with scant sweating and a greasy tongue coating, dàn dòu chĭ (Semen Sojae Praeparatum) can be added.

● For those with difficult measles eruption and a white tongue coating due to cold-evil fettering the exterior, lián qiào and sāng yè (Folium Mori)

are omitted, with zĭ sū yè (Folium Perillae) and fáng fēng added.

● For inhibited papule eruption due to upright qi deficiency, rén shēn (Radix et Rhizoma Ginseng) and huáng qí (Radix Astragali) can be added to support upright qi and outthrust papules. Dr. Qian’s second method is to outthrust the exterior, clear heat and resolve toxins, suitable for the peak stage of measles:

● Exuberant heat and coughing can be treated with Supplemented Má Xìng Shí Gān Tāng (Ephedra, Apricot Kernel, Gypsum and Licorice Decoction) or with Qīng Jiĕ Săn (Clearing and Resolving Powder) which includes dòu juăn (Semen Sojae Germinatum), shān zhī (Fructus Gardeniae), niú bàng zĭ, lián qiào, zhī mŭ (Rhizoma Anemarrhenae), zhè bèi mŭ, shí gāo (Gypsum Fibrosum), huáng qín (Radix Scutellariae), huá shí (Talcum), shí chāng pú (Rhizoma Acori Tatarinowii) and lú gēn (Rhizoma Phragmitis).

● With pneumonia, panting, cough, profuse papules and rapid breathing, sāng bái pí, tíng lì zĭ (Semen Lepidii; Semen Descurainiae), bái qián (Rhizoma et Radix Cynanchi Stauntonii) and shè gān (Rhizoma Belamcandae) can be added.

● With heat-toxins forcing upwards and concurrent diarrhea or dysentery, select Gé Gēn Huáng Qín Huáng Lián Tāng (Pueraria, Scutellaria, and Coptis Decoction) or Bái Tóu Wēng Tāng (Pulsatilla Decoction) which includes bái tóu wēng (Radix Pulsatillae), qín pí (Cortex Fraxini), huáng lián (Rhizoma Coptidis) and huáng băi (Cortex Phellodendri Chinensis) to clear the intestines and discharge heat.

● With heat-toxins transforming into fire causing ying-blood damage with dark-purple papules, a high fever, clouded spirit and a crimson prickly

tongue, select Xī Jiăo Dì Huáng Tāng (Rhinoceros Horn and Rehmannia Decoction) which includes xī jiăo (Cornu Rhinocerotis), shēng dì (Radix Rehmanniae), chì sháo and mŭ dān pí (Cortex Moutan) combined with Zĭ Xuĕ Dān (Purple Snow Elixir) to clear the ying and resolve toxins.

● For those with unconsciousness and spasm, Líng Jiăo Gōu Téng Tāng (Antelope Horn and Uncaria Decoction) and Zhì Băo Dān (Supreme Jewel Elixir) can be combined to open the orifices and extinguish wind. Dr. Qian’s third method is to nourish yin and clear heat, suitable for the final stage of measles:

● Supplemented Shā Shēn Mài Dōng Tāng (Adenophora and Ophiopogon Decoction), including shā shēn (Radix Adenophorae seu Glehniae), mài dōng (Radix Ophiopogonis), zhī mŭ, raw gān căo, sāng yè (Folium Mori), biăn dòu (Semen Lablab Album) and huā fĕn (Radix Trichosanthis).

● For incessant tidal heat present while the high fever is abating, add dì gŭ pí (Cortex Lycii), huáng qín, qīng hāo (Medicinala Artemisiae Annuae) and bái wēi (Radix et Rhizoma Cynanchi Atrati).

● For continuous dry cough, add sāng bái pí, chuān bèi mŭ (Bulbus Fritillariae Cirrhosae), pí pá yè (Folium Eriobotryae) and dōng guā zĭ (Semen Benincasae).

● For predominant phlegm, add hăi gé qiào (Concha Meretricis seu Cyclinae) powder and xìng rén (Semen Armeniacae Amarum).

● For qi counterflow, add zĭ sū zĭ (Fructus Perillae) and kuăn dōng huā (Flos Farfarae).

● For poor appetite, add jú bái (Exocarpium Citri Reticulatae Album) and raw gŭ yá (Fructus Setariae Germinatus).

● For thin stools, remove mài dōng, yù zhú (Rhizoma Polygonati Odorati) and tiān huā fĕn (Radix Trichosanthis) , and add huái shān yào (Rhizoma Dioscoreae) and bái fú líng (Poria Alba) were added.

● For a red pharynx with loss of voice, add xuán shēn (Radix Scrophulariae), mù hú dié (Semen Oroxyli), shēng dì huáng and chán tuì.

● For a red swollen and eroding pharynx, shān dòu gēn (Radix et Rhizoma Sophorae Tonkinensis), băn lán gēn (Radix Isatidis) and jiāng cán (Bombyx Batryticatus) are added with Xī Lèi Săn (Tin-Like Powder) administered by laryngeal insufflation.

● For upright qi deficiency and inward invasion of pathogens with pale white papules, a pale complexion and reversal cold of the limbs, select Supplemented Băo Yuán Tāng (Original Qi-Preserving Decoction), including rén shēn (Radix et Rhizoma Ginseng), huáng qí (Radix Astragali) and zhì gān căo with Shēng Mài Săn (Pulse-Engendering Powder) including rén shēn, wŭ wèi zĭ (Fructus Schisandrae Chinensis) and mài dōng, and Fù Zĭ Lóng Mŭ Jiù Nì Tāng (Aconite, Dragon Bones and Oyster Shell Counterflow-Rescuing Decoction) including fù zĭ (Radix Aconiti Lateralis Praeparata), lóng gŭ (Os Draconis; Fossilia Ossis Mastodi), mŭ lì (Concha Ostreae), rén shēn, bái sháo (Radix Paeoniae Alba) and zhì gān căo to return yang and draw out toxins.

CASE STUDY Female, age 2. Initial Visit: March 17th, 2001 Chief Complaint: Fever for 5 days and papules for 2 days with hasty breathing and cough for a half-day. History: 5 days ago, the child had a fever of unknown origin with a temperature of 39℃, accompanied by a runny nose, sneezing, red eyes and face and tearing eyes. She received treatment in a health center. Yesterday, red papules occurred in the hairline and behind the ears. Signs and Symptoms: Fever, a runny nose, sneezing, red eyes and face, tearing eyes, red papules in the hairline and behind the ears, no papules on the palms or soles, hasty breathing, cough, no vomiting or diarrhea, a greasy yellow tongue coating, and dark-purple finger venules visible beyond the wind-gate. Past History: The patient was constitutionally weak, caught a cold easily, and was exposed to measles. No details were available about previous clinical treatment. Physical Examination: BT 40℃, PR 150 beats/min, RR 54 times/min, expressions of acute fever, hasty breathing, flaring nostrils, lips not turning blue, some secretions in both eyes, red pharynx, no red or swollen tonsils, rough breathing sounds in both lungs, audible fine wet rales, regular rhythm of the heart, forceful heart sounds, soft abdomen, and no pathological syndrome in the nervous system.

Laboratory Examination: WBC 7.7 G/L, N 25.4%, L 62.1%. Measles antibody: measles IgM (+). Chest radiography showed increased markings in both lungs and a small patchy shadow in the lower right lung.

Pattern Differentiation: The patient was exposed to measles. Externally-contracted pathogen paired with wind-evil invaded the lung-wei, became constrained in the spleen, and then discharged externally to the skin, which caused the symptoms of fever, runny nose, sneezing, red eyes and face, tearing eyes, visible rashes on the hairline and behind the ears. The initial stage lasted for 3 to 4 days, and rashes appeared on the fourth day of fever, together with the symptoms of red tearing eyes; thus the disease could be diagnosed as measles. After invading the lung, the toxic-heat scorched fluids to cause phlegm. The congestion of phlegm-heat and blockage of lung qi led to panting and coughing, hasty breathing and flaring nostrils. Because of sudden cold depressing and blocking the measles-toxin from eruption, the resulting blocked lung qi led to qi counterflow and phlegm congestion, thus pneumonia with panting and cough occurred. The greasy yellow tongue coating and dark purple finger venules visible beyond the wind-gate indicate toxins blocking the lung.

Diagnosis

Measles-toxin blocking the lung

Clinical Treatment Principles: Diffuse the lung, open blockage, clear heat, resolve toxins Formula: Modified Má Xìng Shí Gān Tāng (Ephedra, Apricot Kernel, Gypsum and Licorice Decoction) [麻杏⽯⽢汤加减]

5 doses Antibiotics were also given as supportive treatment. [Formula Analysis] Zhì má róng diffuses the lung and relieves panting. Shēng shí gāo clears heat in the lung and stomach. Xìng rén, qián hú and chuān bèi mŭ relieve cough and panting. Jīn yín huā and yú xīng căo clear heat and resolve toxins. Gān căo and lú gēn moisten the lung and resolve cough.

Prepared Medicinals: Qīng Kāi Líng Kē Lì (Clear and Disperse Heat Granules). One package, 3 times daily. External Therapy

● Decoct a suitable amount of yán suī zĭ (Fructus Coriandri) or fresh stems and leaves of yán suī zĭ with fresh scallion and yellow wine for steaming and fumigation of the whole body. Then apply as a wash, and keep the patient warm and covered until there is slight sweating. This treatment is suitable for the early-fever stage, the eruptive stage, or for noneruption.

● Boil má huáng (Herba Ephedrae) 15g, yán suī zĭ 15g, fú píng (Medicinala Spirodelae) 15g and yellow wine 60ml with water, allowing the steam to spread throughout the room. Then dip a towel into the warm solution and rub the face, chest, back and four limbs. This treatment is suitable for the initial stage, the eruptive stage, or for non-eruption.

● Decoct xī hé liŭ (Cacumen Tamaricis) 30g, jīng jiè suì (Spica Schizonepetae) 15g and yīng táo yè (Folium Pruni Pseudocerasi) 15g, steam the body and bathe with the solution. This treatment is suitable for the early-fever stage, the peak stage, or for non-eruption. Tui Na Grasp GB 20 (fēng chí), clear pí jīng (脾经), wèi jīng (胃经), and fèi jīng (肺经), “catch the moon from the bottom of the water”, clear tiān hé shuĭ (天河⽔), press-knead èr shàn mén (⼆扇门), press BL 13 (fèi shù), push BL 10 (tiān zhù). These techniques act to clear heat, remove toxins and promote eruption to expel pathogens. Second Visit

After 5 days of treatment, the measles erupted and the fever was reduced. Other signs and symptoms included some lassitude of the spirit with relieved cough, improved appetite, visible bran-like peeling with pigmentation, a dry red tongue with a thin clean coating, and a thin weak and rapid pulse. After the above treatment, the fever abated and the measles gradually disappeared. The improvement of various symptoms indicated the elimination of pathogens but with deficiency of upright qi and damage to the lung yin and stomach yin. Principles: Nourish yin, boost qi, clear remaining pathogens Formula: Modified Shā Shēng Mài Dōng Tāng (Adenophora and Ophiopogon Decoction) [沙参麦冬汤加减]

5 doses [Formula Analysis] Shā shēn, mài dōng, tiān huā fĕn and yù zhú nourish lung and stomach fluids. Sāng yè clears the remaining heat.

Lú gēn moistens the lung and suppresses cough. Biăn dòu and gān căo nourish the stomach and boost qi.

COMMENTARY AND DISCUSSION This case is a complication of measles. A typical measles contraction consists of three stages, including the stage prior to rashes, which is very much similar to the common cold and includes symptoms of fever, cough, sneezing and runny nose, the peak stage with rashes, and the final stage after rashes. The outbreak starts from the area behind the ears and then spreads to the face, head and neck and then quickly spreads to chest, back, abdomen and extremities. Finally it occurs at the nose, palms and soles. This may take about 3 days. The stage after rashes is when all maculopapules go away following the same sequence as the outbreak, fever diminishes and appetite and spirit return to normal. There was hasty breathing, flaring nostrils and a cough on the second day of maculopapules, when the skin rashes only came to the area behind her ears. She had not yet had maculopapules on the palms and soles. Then, the measles-toxin attacked the lungs, and the congestion of phlegm-heat and blockage of the lung qi led to pneumonia. Since this is toxic-heat blocking the lungs, the treatment for this complication is different from a regular treatment of measles. A typical treatment for measles is to disperse and vent the maculopapules. Shēng má (Rhizoma Cimicifugae) and gé gēn (Radix Puerariae Lobatae) are commonly applied for the purpose of raising, dispersing and venting maculopapules. Other medicinals, such as jīng jiè, fáng fēng and bò he are also applied as assistant medicinals for venting measles.

In this case, the treatment principles are opening and facilitating the movement of lung qi, clearing heat and expelling toxins. Thus, Má Xìng Shí Gān Tāng was applied. This is probably the best formula for the complication of the measles due to toxic-heat blocking lungs. Zhì má róng opens and facilitates the movement of lung qi. Since it has functions of raising and dispersing, it definitely assists the venting of maculopapules. The second medicinal, xìng rén, directs the lung qi to descend, inhibits cough and is applied as an assistant. Shēng shí gāo clears heat from the stomach and lung. Even though it’s very cold, the acrid taste of shēng shí gāo does not interfere with the dispersing and raising effect of zhì má róng. Gān căo protects the upright qi and neutralizes the harsh effects of other ingredients. This classic formula derives from the Treatise on Cold Damage (Shāng Hán Lùn) by Zhang Zhong-jing, and the original classic named it Má Xìng Shí Gān Tāng. This name shows in sequence the importance of the medicinals in the formula. Because the dispersing and venting of maculopapules is the essential treatment principle for measles, attention should also be paid to diet and personal hygiene. Overeating and cold food should be avoided. Some folk therapies and topical applications of medicinals for promoting mild sweating can be applied to induce and support a smooth and more comfortable dispersing and venting of measles.

STUDY QUESTIONS 1. Why does the unfavorable measles pattern commonly present with pathogenic toxin blocking the lung? 2. What are the correct treatment principles for this disease? 3. To what must one pay attention in the nursing of this disease?

Answers 1. The pathology mainly occurs in the lung with toxins floating upward to the pharynx and throat; the lung opens to the nose, governs the skin and body hair, and has the interior-exterior relationship with the large intestine. The favorable pattern of measles is characterized by eruption and exterior venting of the pathogenic toxin. However, with predominant pathogens and weak general health, improper nursing, lack of proper treatment or inappropriate treatment, it is possible to contract external pathogens, which also invade the lung. The pathogens and toxins will likely depress and block the lung where the toxins cannot vent to the exterior. For this reason, pathogenic toxin blocking the lung often appears as the unfavorable-type pattern of measles. 2. The treatment principles for the disease are as follows: Measles is a yang-toxin which has a tendency of outthrusting and clearing, so the basic principles of treatment involve outthrusting and cooling. The disease is caused by measles and seasonal pathogens that contend with upright qi, so

treatment aims to dispel evil and support upright qi while clearing and discharging toxins to the exterior. During the disease, treatment according to pattern differentiation is given in different stages. In the initial stage, exterior constraint of the measles-toxin is treated by resolving the exterior and promoting the eruption of papules. Before the full eruption, cooling medicinals or a mixture of acrid-warm and acrid-cool medicinals are preferred. Although acrid medicinals are used to outthrust papules, resolve toxins and discharge heat, acrid-warm medicinals should be used with caution to avoid damage to yin-humor. In the peak stage, the exuberant toxins are treated by clearing heat and resolving toxins in order to further promote the outthrusting of papules. Bitter-cold medicinals that clear heat must not be overused, as they may damage upright qi and cause inward invasion of the measles-toxin. While in the final stage, although the pathogenic toxins have disappeared, upright qi must be supported while nourishing yin and clearing any remaining heat. All in all, the essential points in treatment are to outthrust papules, clear heat and resolve toxins. 3. The nursing of measles is quite important to avoid complications and for early recovery. Keep the patient’s room well-ventilated, ensuring a proper temperature and humidity. Avoid contracting cold due to wind, and strong sunlight. The patient should rest in a comfortable bed with a soft quilt and a quiet environment. The patient must drink plenty of water and eat delicate, easily digestible foods. Greasy or spicy foods should not be consumed during the peak stage. In the final stage, some richer foods may

be introduced according to the appetite. The eyes, nasal and oral cavities and skin should be kept clean.

CASE SCENARIOS The following cases present variations of this condition. After familiarizing yourself with the possible common pattern presentations and appropriate formulas for treatment, use the following exercises to test your overall understanding of the condition. 1. A female patient, age 4, contracted seasonal wind-warmth with a fever 2 days ago. Other signs and symptoms included cough, tidal heat, sneezing, slightly red eyes, faintly red skin, a red tongue with a yellow coating, and a soft pulse. What is the best treatment formula? A. Xuān Dú Fā Biăo Tāng (Toxin-Diffusing Exterior-Releasing Decoction) B. Qīng Jiĕ Tòu Biăo Tāng (Clearing and Resolving ExteriorOutthrusting Decoction) C. Má Xìng Shí Gān Tāng (Ephedra, Apricot Kernel, Gypsum and Licorice Decoction) D. Yín Qiào Săn (Lonicera and Forsythia Powder) E. Má Huáng Tāng (Ephedra Decoction) 2. A male patient, age 6, contracted untimely seasonal qi and now has measles. Other signs and symptoms included intense heat in the lung and intestines, disappearance of papules, loss of voice, abdominal pain, cold limbs, constipation with no defecation for 8 days, scanty urine, blackening

teeth, foul breath, vexation and agitation, a crimson tongue with a parched black coating, and a moderate pulse. What is the best treatment formula? A. Xuān Dú Fā Biăo Tāng(Toxin-Diffusing Exterior-Releasing Decoction) B. Zēng Yè Chéng Qì Tāng (Humor-Increasing Qi-Guiding Decoction) C. Dà Chéng Qì Tāng (Major Purgative Decoction) D. Má Xìng Shí Gān Tāng (Ephedra, Apricot Kernel, Gypsum and Licorice Decoction) E. Qīng Jiĕ Tòu Biăo Tāng(Clearing and Resolving ExteriorOutthrusting Decoction) 3. A male patient, age 8, originally had no apparent papules. Earlier abdominal pain was treated excessively with bitter-warm medicinals to expel foulness, which resulted in damage to the lung and fluids transforming into dryness. Other signs and symptoms included eruption of papules on the faces and cheeks, upward invasion of toxic-fire, panting with no cough, red eyes covered with discharge, coma, delirious speech, vexation, agitation, complex dysentery, thirst with increased fluid intake, abdominal pain, a crimson tongue with dry cracked coating, and a surging rapid pulse. What is the best treatment formula? A. Qīng Jiĕ Tòu Biăo Tāng (Clearing and Resolving ExteriorOutthrusting Decoction) B. Zĭ Xuĕ Dān (Purple Snow Elixir) C. Dà Chéng Qì Tāng (Major Purgative Decoction)

D. Sū Hé Xiāng Wán (Storax Pill) E. Líng Jiăo Gōu Téng Tāng (Antelope Horn and Uncaria Decoction) 4. A male patient, age 5, has full eruption of papules with a gradually abating fever. Other signs and symptoms include a visible bran-like peeling with pigmentation, thin and clean tongue coating, and a fine and forceless or fine and rapid pulse. What is the best treatment formula? A. Dà Chéng Qì Tāng (Major Purgative Decoction) B. Yăng Yīn Qīng Fèi Tāng (Yin-Nourishing Lung-Clearing Decoction) C. Qīng Jiĕ Tòu Biăo Tāng(Clearing and Resolving ExteriorOutthrusting Decoction) D. Shēng Mài Săn (Pulse-Engendering Powder) E. Shā Shēn Mài Dōng Tāng (Adenophora and Ophiopogon Decoction) 5. A male patient, age 4, first had abdominal pain and then erupting papules; he also had contracted a seasonal pathogen that resulted in panting and diarrhea. Other signs and symptoms include the outthrusting of papules, tidal heat, cough, qi counterflow, hasty panting, flaring nostrils, clear and watery diarrhea, scanty urination, a red tongue with a white coating, and a rapid pulse. What are the correct principles of treatment? A. Direct the clear to ascend, resolve the exterior, transform toxins, clear heat

B. Diffuse the lungs, open blockage, clear heat, resolve toxins C. Calm the liver, extinguish wind, clear constriction, resolve toxins D. Clear cold, resolve toxins, outthrust papules and evils E. Clear heat, resolve toxins, transform dampness, check diarrhea 6. A female patient, age 8, has dense papules clustering in dark purple patches. Other signs and symptoms include a persistent high fever, agitation and delirious speech, phlegm rales in the throat, a crimson tongue with a prickly yellow coating, and a rapid forceful pulse. What is the best treatment formula? A. Qīng Jiĕ Tòu Biăo Tāng (Clearing and Resolving ExteriorOutthrusting Decoction) B. Zĭ Xuĕ Dān (Purple Snow Elixir) C. Dà Chéng Qì Tāng (Major Purgative Decoction) D. Má Xìng Shí Gān Tāng (Ephedra, Apricot Kernel, Gypsum and Licorice Decoction) E. Líng Jiăo Gōu Téng Tāng (Antelope Horn and Uncaria Decoction) 7. A female patient, age 7, has had a fever and papules for 6 days. Present signs and symptoms include full eruption of papules with gradual disappearance, gradually abating fever, relieved cough, improved appetite, visible bran-like peeling with pigmentation, a thin clean tongue coating, and a fine and forceless or fine rapid pulse. What are the correct principles of treatment? A. Clear cold, resolve toxins, outthrust papules and evils

B. Calm the liver, extinguish wind, clear heat from the ying level, resolve toxins C. Nourish yin, boost qi, clear and resolve remaining pathogens D. Diffuse the lung, open blockage, clear heat, resolve toxins E. Release the exterior with acrid-coolness, clear and diffuse lung-wei

Answers 1. A 2. B 3. B 4. E 5. A 6. E 7. C

Chapter 30 Exanthema Subitum Exanthema subitum is caused by the external contraction of the seasonal pathogen exanthema subitum (human herpes virus type 6 and 7). Clinical features include an acute high fever lasting for 3-5 days, followed by the sudden appearance of rose-colored papules all over the body at fever abatement. Because the rashes have a measles-like appearance and occur in breast-feeding infants, the disease has also been referred to as roseola infantum. The Ming Dynasty text, Teachings on the Treatment of Pox from Wan’s Family Lineage of Doctors—Summarized Verses on Treatment of Measles Toxin (Wàn Shì Jiā Chuán Dòu Zhĕn Xīn Fă: Zhèng Zhì Gē Kuò) also suggested that this disease was different from measles. On Measles and Pox (Dòu Má Dìng Lùn) states, “diseases like roseola infantum and urticaria caused by wind-heat visiting the two channels of the spleen and lung, can be treated with Jīng Jiè Fā Biăo Tāng (Schizonepeta ExteriorReleasing Decoction) to largely scatter wind, discharge heat and clear heat” (奶麻瘾疹之类,皆风热客于脾肺⼆经所致,⽤荆芥发表汤,此药⼤能 疏风泄热清热). The cause of exanthema subitum involves external contraction of a seasonal pathogen, with the main pathological changes occurring in the lung and spleen. The pathogen enters from the mouth and nose, invades lung-wei, lodges in the fleshy exterior and struggles with blood and qi. During the struggle, heat also brews in the lung and stomach. When upright

qi dominates over the evil, the pathogen is expelled from the lung-wei, papules outthrust to the skin and the pathogenic toxin discharges externally. After the eruption of papules, children can recover from the damage to qi and yin with minimal treatment. The exanthema subitum seasonal pathogen belongs to the category of wind-heat seasonal pathogens, which easily transform to heat; the pattern of depressed heat in the fleshy exterior is quickly noticed at the onset. However, when weaker pathogenic toxins invade the lung-wei, upright qi rises to contend with them, so heat-toxins in the lung and stomach discharge to the skin; this usually resolves in the wei level without invading yingblood. For a few children, their timidity of spirit-qi and heat harassing the liver channel at the beginning of high fever can lead to clouded spirit and even convulsion, which can be soon relieved. Thus, as threatening as the disease is, it does not last long, and the pathogenic heat can be resolved without causing heavy damage to qi and yin; thus the prognosis is good. Common clinical patterns include depressed heat in the fleshy exterior and toxins outthrusting to the skin.

CLINICAL ESSENTIALS In the diagnosis and treatment of this disease, four-level pattern differentiation is the first priority. The pathogenic locations are usually focused in wei and qi levels, without further invasion of the ying-blood levels. In the mild pattern, which is most common, the pathogen becomes depressed in the fleshy exterior, with signs and symptoms including a sudden high fever lasting for 3 to 5 days and other slight symptoms, with papules outthrusting while the heat abates. In this case, modified Yín Qiào Săn (Lonicera and Forsythia Powder) is commonly prescribed to resolve the exterior and clear heat.

● With fever, aversion to cold, nasal congestion and a runny nose, zĭ sū yè (Folium Perillae) and fáng fēng (Radix Saposhnikoviae) are added to resolve the exterior and dissipate cold.

● With persistent high fever and vexation, zhī zĭ (Fructus Gardeniae) and chán tuì (Periostracum Cicadae) are added to clear heat and eliminate vexation.

● With vexation and susceptibility to fright, gōu téng (Ramulus Uncariae Cum Uncis) and jiāng cán (Bombyx Batryticatus) are added to dispel wind and settle fright.

● With intermittent nausea and vomiting, zhú rú (Caulis Bambusae in Taenia) and shēng jiāng (Rhizoma Zingiberis Recens) are added to harmonize the stomach and descend counterflow.

● With poor appetite and thin stools, scorch-fried shān zhā (Fructus Crataegi) and shén qū (Massa Medicata Fermentata) are added to disperse

food stagnation and check diarrhea. The more severe pattern involves exuberant toxins or upright qi deficiency with heat harassing the heart and liver; signs and symptoms include vexation, agitation, a clouded spirit and convulsion. Exuberant heat stirring wind is treated by clearing heat and calming fright, while abating heat with erupting papules is treated by clearing heat and engendering fluids. In this case, Yín Qiào Săn is often used with dàn dòu chĭ (Semen Sojae Praeparatum) omitted and shēng dì (Radix Rehmanniae), mŭ dān pí (Cortex Moutan), dà qīng yè (Folium Isatidis) and more xuán shēn (Radix Scrophulariae) added.

● With poor appetite, jī nèi jīn (Endothelium Corneum Gigeriae Galli) and mài yá (Fructus Hordei Germinatus) are added to fortify the spleen and harmonize the stomach.

● With dry stools, huŏ má rén (Fructus Cannabis) and fēng mì (Mel) are added to moisten the intestines and free the stool. Dr. Liao Rui-quan suggests that, for those with a high fever, thirst, coughing, vomiting, frequent diarrhea, coma, fright, shivering, a red tongue with slimy yellow coating, and a rapid pulse, a modification of Gé Gēn Qín Lián Tāng (Pueraria, Scutellaria, and Coptis Decoction) should be applied. This formula acts to release the muscles and clear the intestines and stomach. The prescription includes fĕn gé (Radix Puerariae Thomsonii), huáng qín (Radix Scutellariae), huáng lián (Rhizoma Coptidis), huò xiāng (Herba Agastachis), fáng fēng (Radix Saposhnikoviae), shén qū (Massa Medicata Fermentata), fă bàn xià (Rhizoma Pinelliae Praeparatum), chén pí (Pericarpium Citri Reticulatae) and zhú rú (Caulis Bambusae in Taenia), with 2 pieces of shēng jiāng and Hóu Zăo Săn (Macaque Stone Powder) added. After administration, papules will erupt with slight coughing and a

hoarse voice; at this point, further treatment is applied to resolve phlegm, diffuse the lung, harmonize the middle, and clear residual pathogens.

CASE STUDY Male, age 6 months. Initial Visit: May 12th, 2006 Chief Complaint: Fever for 3 days, eruption of papules for 4 hours. History: The child began to have fever of unknown origin on the morning of May 9th with a body temperature of 39.5℃. Other signs and symptoms included vexation, agitation, coughing and a runny nose. Intramuscular injection of penicillin and other oral drugs had been used previously, with the result of gradually abating heat. Red papules appeared all over the body this morning, and then he visited the hospital. Signs and Symptoms: Red or rosy papules all over the body, cough with little phlegm, constipation, a pale tongue with a slimy yellow coating, and pale purple finger venules. Past History: No history of other diseases or disorders. Physical Examination: BT 37.5℃, roseola papules all over the body, particularly on the torso, waist and buttocks, no itching, no papules on the palms or soles, normal mucosa on the cheeks, slight hyperemia in the pharynx, and slight swelling of the lymph nodes behind the ears and in the neck. Laboratory Examination: WBC 4.8 × 109/L, N 30%, L 67%, M 3%.

Pattern Differentiation

The external contraction of a wind-heat seasonal pathogen caused damage to the lung-wei with symptoms of high fever, agitation, cough and a runny nose at the beginning. With the forceful resistance from the body against the pathogen, the pathogenic heat effused to the skin and discharged to the exterior after brewing in lung-wei for several days and struggling with qi and blood. So the heat abated, and the papules erupted. The slimy yellow tongue coating and the pale purple finger venules are manifestations of pathogenic heat penetrating inward.

Diagnosis Roseola infantum (eruptive stage)

Clinical Treatment Principles: Clear heat, cool the blood, discharge heat, nourish yin Formula: Modified Huà Bān Jiĕ Dú Tāng (Macule-Dissolving ToxinResolving Decoction) [化斑解毒汤加减]

2 doses [Formula Analysis] Shí gāo, zhī mŭ and xuán shēn clear qi and cool blood. Dān pí and chì sháo cool and quicken blood. Yín huā and lián qiào clear heat and resolve toxin. Jié gĕng and niú bàng zĭ diffuse the lungs and disinhibit the throat. Prepared Medicines: Băn Lán Gēn Kē Lì (Isatis Root Granules), 2.5g, 2 or 3 times per day. Second Visit After 2 doses, the cough stopped and the papules rapidly disappeared.

COMMENTARY AND DISCUSSION A 6 month-old child was brought to the clinic during the eruptive stage of the condition, with red rashes and papules over the entire body. The characteristic symptoms and the age of the child indicated that the condition was not measles (even though exanthema subitum is sometimes called pseudo-measles). The selected formula was a modification of Huà Bān Jiĕ Dú Tāng (Macule-Dissolving Toxin-Resolving Decoction). Chinese formula books contain several versions of the base formula, which are worth comparing. The Huà Bān Jiĕ Dú Tāng in the text, Explanation of Measles (Má Zhĕn Chăn Zhù) contains shí gāo, shēng má (Rhizoma Cimicifugae), zhī mŭ, niú bàng zĭ, xuán shēn, gān căo and dàn zhú yè (Herba Lophatheri), and the version in Orthodox Lineage of External Medicine (Wài Kē Zhèng Zōng) contains shí gāo, shēng má, zhī mŭ, niú bàng zĭ, xuán shēn, huáng lián, rén zhōng huáng (Pulvis Praeparatus Glycyrrhizae), lián qiào, gān căo and dàn zhú yè. The modified Huà Bān Jiĕ Dú Tāng from this case contains shí gāo, zhī mŭ, niú bàng zĭ, xuán shēn, yín huā, lián qiào, dān pí, chì sháo and jié gĕng. Compared with the first two versions of the formula Huà Bān Jiĕ Dú Tāng, the selected formula in this case applies the essential medicinals for clearing qi and cooling blood, common to these formulas. These include shí gāo, zhī mŭ and xuán shēn. Instead of applying shēng má for upraising, dispersing and venting skin rashes, the modified formula only applied yín huā, lián qiào and niú bàng zĭ to clear heat, remove toxins and relieve the

exterior. This is because the first two formulas were designed for measles, while this modification is intended for exanthema subitum, which is a much milder condition than measles. Dān pí and chì sháo were applied to cool the blood and promote circulation. Considering that this child had a greasy yellow tongue coating, the dàn zhú yè as listed in the first two formulas could have also been added to clear damp-heat and promote urination. As this case had only mild symptoms, all symptoms disappeared and the child was healed after taking only 2 doses.

STUDY QUESTIONS 1. How can this disease be differentiated from an enterovirus infection? 2. Describe the treatment principles for exanthema subitum. 3. To what must one pay attention when nursing this disease?

Answers 1. Enterovirus infection occurs most often in the summer and results in pleomorphic rashes. Fever, runny nose, pharyngeal pain, diarrhea and pharyngeal ulcers always accompany the eruption of papules. 2. Before the eruption of papules, treatment should dispel wind and dissipate heat. When the heat abates and papules erupt, light-natured medicinals like jú huā, zhú yè and shēng dì should be prescribed. 3. In nursing exanthema subitum, it is important to keep the patient in a quiet environment, allowing for good rest while avoiding wind and cold. Easy to digest foods should be eaten while avoiding greasy foods, and the patient should drink plenty of liquids. For those with a persistent high fever, physical methods can be used to abate the heat, including cold towel compresses on the head, or rubbing or bathing with 30% to 50% solution of alcohol to dissipate heat and prevent convulsions.

CASE SCENARIOS The following cases present variations of this condition. After familiarizing yourself with the possible common pattern presentations and appropriate formulas for treatment, use the following exercises to test your overall understanding of the condition. 1. A female patient, aged 11 months, has had a fever for 2 days with a temperature of 40℃. Other signs and symptoms include thirst for drinks, cough, vomiting, frequent diarrhea, coma, fright, shivering, a red tongue with yellow and slimy tongue fur and a rapid pulse. What is the most suitable formula for this case? A. Modified Gé Gēn Qín Lián Tāng(Pueraria, Scutellaria, and Coptis Decoction) B. Má Huáng Tāng (Ephedra Decoction) C. Shēn Líng Bái Zhú Săn (Ginseng, Poria and Atractylodes Macrocephalae Powder) D. Jīng Fáng Bài Dú Săn (Schizonepeta and Saposhnikovia ToxinResolving Powder) E. Zhĭ Sòu Săn (Cough-Stopping Powder) 2. A female, age 8 months, has had a fever for 3 days and papules for 2 hours. The patient had a fever after catching cold with a temperature at 39℃ and was treated with oral drugs for cold and fever at home. Present signs and symptoms include abating heat, little papules in the skin that are rose in color, rashes first appearing on the torso and then spreading all over

the body, slightly red tongue, thin, dry tongue fur and pale purple finger venules. What are the proper treatment principles? A. Clear heat and resolve toxins B. Clear heat and cool the blood C. Resolve toxins, promote the eruption of papules D. Clear heat and engender fluids E. Nourish yin and engender fluids 3. A female patient, age 10 months, had sudden high fever that has lasted for 3 to 4 days. Other signs and symptoms include aversion to cold, nasal congestion, runny nose, occasional cough, normal spirits or occasional vexation, a diminished desire for food and drink, a red pharynx, a slightly red tongue with a slimy yellow coating, and pale purple finger venules. What is the most suitable formula for this case? A. Bái Hŭ Tāng (White Tiger Decoction) B. Yín Qiào Săn (Lonicera and Forsythia Powder) C. Má Huáng Tāng (Ephedra Decoction) D. Sāng Jú Yĭn (Mulberry Leaf and Chrysanthemum Beverage) E. Jīng Fáng Bài Dú Săn (Schizonepeta and Saposhnikovia ToxinResolving Powder) 4. A male patient, age 7 months, has had a fever for 3 to 4 days with a temperature of 40℃. Other signs and symptoms include red rashes on the chest while the fever abates, occasional vexation, a diminished appetite,

normal urine and stool, a red tongue with a thin yellow coating, and pale purple finger venules. What is the proper diagnosis? A. Rubella B. Exanthema subitum C. Scarlet fever D. Measles E. Chickenpox

Answers 1. A 2. D 3. B 4. B

Chapter 31 Rubella Rubella is an acute eruptive infectious disease caused by external contraction of the seasonal rubella virus. Clinical manifestations include fever, cough, rose-colored macules and silver sand-like papules all over the body with swollen lymph nodes behind the ears and occiput. In TCM, this condition is equivalent to wind papules, dormant papules, or wind-sand. Appearing most often in children between one and 5 years old, rubella may occur during any season, especially winter and spring, and can even become epidemic. Permanent immunity may be acquired after contracting the sickness. The conditions of rubella are usually light and mild with few complications in clinic. However, if a woman contracts the disease during her early pregnancy, the rubella virus will infect the fetus through the placenta. This can cause various congenital diseases, known as congenital rubella or congenital rubella syndrome; these including congenital heart disease, deafness, cataracts and cerebral dysgenesis. Rubella is caused by the external contraction of seasonal rubella pathogen, with the main pathological changes affecting the lung-wei. Because the lung governs the skin and body hair and opens to the nose, and because the wei controls the exterior, when a seasonal pathogen invades the body through the mouth and nose, it may be discharged into the skin as the upright qi and pathogenic qi contend. Seasonal rubella pathogen attacking the lung-wei and its accumulation in the interstices of the flesh results in aversion to wind, fever, cough and

runny nose, where exterior discharge of the toxic pathogen leads to the broad, well-distributed eruption of light red papules. Obstruction of the shaoyang channel by the toxic pathogen brings about lymphadenectasis behind the ears and occiput. In some children, the pathogen is so exuberant as to invade the qi and ying levels and blaze within the lung and stomach causing high fever, vexation and thirst, constipation, reddish urine and densely distributed bright red or deep red papules. Severe transmuted patterns occasionally appear, with inward invasion to the heart and liver. Common clinical patterns include pathogenic attack of the lung-wei and pathogenic invasion of the qi and ying.

CLINICAL ESSENTIALS Four-level warm disease pattern differentiation is used to determine the pattern and its degree of severity. First, the majority of conditions involve pathogenic attack of the lung-wei, which is a mild pattern. It occurs rapidly and features a low-grade fever with small sparsely distributed lightred papules and lymphadenectasis behind the ears and in the occiput that are painful when touched. There are no severe symptoms present. Mild patterns are treated with modified Yín Qiào Săn (Lonicera and Forsythia Powder) to scatter wind, release the exterior and clear heat:

● For painful lymphadenectasis behind the ears and in the occiput, pú gōng yīng (Herba Taraxaci), xià kū căo (Spica Prunellae) and xuán shēn (Radix Scrophulariae) are used to clear heat, resolve toxins and dissipate masses.

● For a painful red swollen pharynx, jiāng cán (Bombyx Batryticatus), mù hú dié (Semen Oroxyli) and băn lán gēn (Herba Isatidis) are added to clear heat and benefit the throat.

● For itching, chán tuì (Periostracum Cicadae) and jiāng cán are added to dispel wind and relieve itching. Second, pathogenic attack of both qi and ying is a severe pattern characterized by high fever, vexation, and thirst with densely distributed bright red or purple papules. Treatment of this disease pattern, which is rare in clinic, must clear heat from the qi level, cool the ying level and resolve toxins by using modified Tòu Zhĕn Liáng Jiĕ Tāng (Papule-Outthrusting Toxin-Resolving with Clearing Coolness Decoction).

● With thirst, tiān huā fĕn (Radix Trichosanthis) and fresh lú gēn (Rhizoma Phragmitis) are added to clear heat and promote fluid production.

● With dry bound stools, add dà huáng (Radix et Rhizoma Rhei) and xuán míng fĕn (Natrii Sulfas Exsiccatus) to drain fire and unblock the bowels.

● With densely distributed purple papules, add shēng dì huáng (Radix Rehmanniae), mŭ dān pí (Cortex Moutan) and dān shēn (Radix et Rhizoma Salviae Miltiorrhizae) to clear heat and cool the blood.

CASE STUDY Male, age 4. Initial Visit: May 15th, 1992 Chief Complaint: Fever for one day, with papules for 3 hours. History: The child had a fever for one day with body temperature at about 38 ℃. Other signs and symptoms included aversion to wind, nasal congestion, sneezing and cough. Red papules appeared on his face and neck this morning, so he was sent to visit the hospital. Signs and Symptoms: Visible papules on the face and neck, lowgrade fever, nasal congestion, sneezing, cough, a red tongue with a thin yellow coating, and a rapid floating pulse. Past History: No history of other diseases or disorders. Physical Examination: BT 37.9℃, tiny light red itchy papules on face, neck, torso and upper limbs, lymphadenectasis behind the ears and in the occiput that are painful when pressed even slightly, light red pharynx, normal breathing sound in both lungs, regular heart rhythm, forceful heart sounds, no pathological murmur, soft abdomen with no pain when pressed, no edema in the lower limbs. Laboratory Examination: WBC 56 × 109/L, N 52%, L 48%.

Pattern Differentiation When the externally contracted seasonal wind-heat pathogen invaded the lung-wei, the failure of lung qi to diffuse due to the contention between

upright qi and pathogenic qi caused such lung-wei symptoms as aversion to wind, cough and runny nose. The sparsely distributed light-red papules were the manifestation of the discharge of the wind-heat pathogen, and the itching skin was due to wind exuberance. The red tongue with a thin yellow coating and a floating and rapid pulse are associated with pathogenic accumulation in the lung-wei.

Diagnosis Rubella due to lung-wei pathogenic invasion

Clinical Treatment The presenting pattern is lung-wei pathogenic invasion. The treatment should take the basic principles of scattering wind and clearing heat, also assisted by Western drugs such as interferon and antiviral medications such as ribavirin. Principles: Scatter wind, clear heat Formula: Modified Yín Qiào Săn (Lonicera and Forsythia Powder) [银翘散加减]

2 doses [Formula Analysis] Jīn yín huā and lián qiào clear heat, resolve toxins and release the exterior with acrid-coolness. Dàn dòu chĭ, bò he, jīng jiè and chán tuì scatter wind and outthrust papules. Niú bàng zĭ, jié gĕng and gān căo resolve toxins and soothe the throat. Prepared Medicines: Băn Lán Gēn Kē Lì (Isatis Root Granules), one bag, 2 to 3 times per day. Second Visit May 17th After taking the decoction, the rubella was relieved but a slight cough, copious white phlegm, a rapid floating pulse and a white tongue coating remain. Therefore, follow-up treatment should mainly purify the lung and dissolve phlegm. Principles: Scatter wind, clear heat, dissolve phlegm and relieve cough Formula: Modified Yín Qiào Săn (Lonicera and Forsythia Powder) The following two ingredients were added to the previous formula.

2 doses

[Formula Analysis] Chē qián zĭ and zĭ wăn dissolve phlegm and relieve cough.

COMMENTARY AND DISCUSSION This is a mild case of wind-warmth disease with wind-warm evils attacking the mouth and nose and then entering the lung. As Wu Ju-tong (1758-1836) said in Systematic Differentiation of Warm Diseases (Wēn Bìng Tiáo Biàn), “all warm diseases start from the upper jiao and attack the hand taiyin lung channel” (凡病温者,始于上焦,在⼿太阴). Another famous expert on warm disease, Ye Tian-shi shared the same idea in Treatise on Warm-Heat Diseases (Wēn Rè Lùn), saying, “warm evils attack the upper part of body and invade the lung” (温邪上受,⾸先犯肺). In this case, the nasal congestion, sneezing and cough are lung symptoms. The red papules appearing on the face are also due to wind-heat in the lung, as the lung controls the skin and body hair. The child had a fever and aversion to wind, which again indicates the lung and the wei level. Thus, Yín Qiào Săn was applied. Wu Ju-tong believed that this formula derived from another prescription, Qīng Xīn Liáng Gé Săn (Heart-Clearing Diaphragm-Cooling Powder), which included lián qiào, huáng qín, bò he, zhī zĭ (Fructus Gardeniae), jié gĕng and gān căo. Wu removed huáng qín, as he believed that it might interfere with the middle jiao organs. In Systematic Differentiation of Warm Diseases, he states, “the formula only clears the evils in the upper jiao and does not interfere with the middle and lower jiao; thus there is no potential risk nor is it opening a door for a thief” (纯然清肃 上焦,不犯中下,无开门揖盗之弊).

The formula medicinals applied in this case are very light in weight and mainly composed of flowers, sprout and leaves such as jīng jiè, bò he and jīn yín huā. The formulating theory is that “medicinals for treating upper jiao diseases should be as light as feathers, otherwise the qi does not rise and disperse” (治上焦如⽻,非轻不举). These medicinals are often aromatic and should not be cooked for a long time. His advice is that “the medicinal tea is ready when a strong smell comes out… overcooking may push the herbs into the middle jiao” (⾹⽓⼤ 出,即取服……过煮则味厚⽽入中焦矣). Many experts believe that warm disease is caused by the invasion of turbid evils, and aromatic herbs act to expel turbid-filth. Wu Ju-tong said that he learned this idea from herbalist Yu Jia-yan. In fact, this method dates back to some early folk therapies. For example, celebration of the Duanwu Festival (also called the Dragon Boat Festival) is a popular custom in China, particularly in the south. Many traditional rituals of the Duanwu Festival emphasize the avoidance of disease. The desire to prevent health hazards associated with the mid-summer months, when filthy evils arise, may have been the primary motive behind the holiday. The hygiene customs and activities include hanging up icons of Zhong Kui, a mythical guardian figure, hanging of mugwort and calamus, wearing perfumed medicinal bags, and drinking realgar wine to prevent the attack of filth-evils while also promoting health and well-being. Medicinal formulating theories are greatly influenced by Chinese culture and folk therapies.

STUDY QUESTIONS 1. State the clinical essentials for this case. 2. State the primary ideas for the differentiation of rubella. 3. To what must attention be paid in the nursing of this disease?

Answers 1. Several clinical essentials must be kept in mind regarding rubella. In this case, the patient must have contracted rubella during its epidemic period. In the initial stage, the symptoms were similar to a cold with light red macules and papules on the skin. Then, the papules spread all over the body and the fever abated. After the papules decrease, there may be scaling of the skin but with no pigmented scarring. In addition to the mild symptoms, there were also enlarged lymph nodes behind the ears and swelling at the occiput as well as some glomus below the left ribs. The blood test showed a decreased WBC and a relative increase of lymph cells. 2. The differentiation of rubella takes the four-level warm disease pattern differentiation to determine the severity of the pattern. First, lungwei pathogenic attack is a mild pattern, characterized by low-grade fever, well-distributed light red papules and other mild symptoms. Pathogens attacking both qi and ying is a severe pattern characterized by high fever, vexation, thirst, and densely-distributed bright red or purple papules, which is rare in clinic.

3. First, isolate the patient child for 5 days after the full papule effusion. Ask the patient to get good rest, keep warm and drink plenty of boiled water. For those with high fever, lower the body temperature physically. To avoid damaging the skin and causing infection, those with itching should avoid scratching the skin. Clothes should be soft and loose. Easily digestible foods should be eaten, with spicy fried food and fast foods avoided.

CASE SCENARIOS The following cases present variations of this condition. After familiarizing yourself with the possible common pattern presentations and appropriate formulas for treatment, use the following exercises to test your overall understanding of the condition. 1. Zhao, a 7 year-old female, has a slight fever, red papules all over the body, paroxysmal cough, lymphadenectasis behind the ears, a red tongue tip, a thin white tongue coating, and a rapid floating pulse. What is the proper treatment? A. Scatter wind, dissipate heat, relieve panting and cough B. Clear heat, resolve toxins, dissolve phlegm, relieve cough C. Scatter wind, dissipate heat, diffuse the lung, dissolve phlegm D. Clear heat, resolve toxins, diffuse the lung, dissolve phlegm E. Clear heat, dissolve phlegm, dispel wind and relieve itching 2. Li, a 3 year-old male, has a high fever, thirst, vexation, crying, dense bright red and purple papules with some patches, scanty yellow urine, constipation, a red tongue with a rough yellow coating, and a rapid surging pulse. What is the most suitable formula? A. Yín Qiào Săn (Lonicera and Forsythia Powder) B. Má Xìng Shí Gān Tāng (Ephedra, Apricot Kernel, Gypsum and Licorice Decoction)

C. Jīng Fáng Bài Dú Săn (Schizonepeta and Saposhnikovia ToxinResolving Powder) D. Xuān Dú Fā Biăo Tāng (Toxin-Diffusing Exterior-Releasing Decoction) E. Tòu Zhĕn Liáng Jiĕ Tāng (Papule-Outthrusting Toxin-Resolving with Clearing Coolness Decoction) 3. Liu, a 5 year-old female, was hospitalized because of fever and papules for 1 day. Signs and symptoms include fever, aversion to wind, well-distributed papules on the face, torso and four limbs with sparse tiny light red papules, slight itching, sneezing, runny nose, slight cough, swollen lymph nodes behind the ears with pain when touched, a red tongue with a thin white and yellow coating, and a floating rapid pulse. What is the most suitable formula? A. Tòu Zhĕn Liáng Jiĕ Tāng (Papule-Outthrusting Toxin-Resolving with Clearing Coolness Decoction) B. Yín Qiào Săn (Lonicera and Forsythia Powder) C. Liáng Yíng Qīng Qì Tāng (Construction-Cooling Qi-Clearing Decoction) D. Jīng Fáng Bài Dú Săn (Schizonepeta and Saposhnikovia ToxinResolving Powder) E. Má Huáng Tāng (Ephedra Decoction) 4. Wang, a 4 year-old female, has a fever, red eyes, densely welldistributed red papules all over the body, sparse tiny light red papules, a paroxysmal cough, a thin white tongue coating, a red-tipped tongue, and a floating rapid pulse.

What is the correct pattern differentiation? A. Lung and stomach pathogenic invasion B. Toxin discharging to the skin C. Lung-wei pathogenic attack D. Pathogenic toxins blocking the lung E. Qi and ying pathogenic invasion

Answers 1. C 2. D 3. B 4. C

Chapter 32 Chickenpox Chickenpox is a common eruptive, seasonal, epidemic disease in children that results from externally contracted seasonal pathogenic toxins; it is characterized by fever, eruption of papules in batches on the skin with co-existing macules, vesicles and scabs. In the early Song Dynasty, Discussing the Elaboration on the Subtleties of the Treatise on Children’s Health (Xiăo Ér Wèi Shēng Zŏng Wēi Lùn Fāng) gave some description of the vesicles; “the sores have thin skins as vesicles, and are likely to become dry when they are broken, thus called chickenpox” (其疮⽪薄,如⽔疱, 破即易⼲者,谓之⽔痘). Children often contract this disease because as children their zang-fu organs are tender, their physical bodies not mature, they do not yet have abundant qi, and their defending functions are not yet strong enough. Pathological changes occur mainly in the lung and spleen channels. When seasonal pathogenic toxins invade from the mouth and nose, it accumulates and constrains in the lung, which governs the skin and body hair, and the spleen, which governs the muscles. Chickenpox appears by steaming to the fleshy exterior during its contention with internal dampness. Common clinical patterns of include damage to the lung-wei, and toxins blazing in both qi and ying.

CLINICAL ESSENTIALS For clinical pattern differentiation, four-level differentiation should be combined with zang-fu pattern differentiation, and then the disease condition should be identified according to the generalized and local symptoms. For a mild pattern, to which the majority of chickenpox child patients belong, the disease lies in the wei qi, with the signs and symptoms of small red itching and sparsely distributed papules and vesicles, also accompanied by lung-spleen patterns characterized by fever, runny nose, cough and poor appetite. Treatment with modified Yín Qiào Săn (Lonicera and Forsythia Powder) mainly acts to scatter wind, clear heat and resolve toxins, while also draining dampness.

● For cough and phlegm, xìng rén (Semen Armeniacae Amarum) and zhè bèi mŭ (Bulbus Fritillariae Thunbergii) are added to diffuse the lung and dissolve phlegm.

● For sore swollen throat, băn lán gēn (Radix Isatidis) and mă bó (Lasiosphaera seu Calvatia) are added to clear heat, resolve toxin and disinhibit the throat.

● For extremely itchy vesicles, bái xiān pí (Cortex Dictamni) and dì fū zĭ (Fructus Kochiae) are added to dispel dampness and relieve itching. For severe patterns, the disease lies in the qi and ying levels with the signs and symptoms of persistent high fever, a red face, vexation, thirst, large densely distributed purple papules, turbid exudate and in some cases, visible papules appearing in the oral mucous membrane.

Treatment with modified Qīng Wèi Jiĕ Dú Tāng (Stomach-HeatClearing and Toxin-Resolving Decoction) acts to clear qi, cool ying, resolve toxins and remove dampness.

● For mouth and tongue sores with dry stools, raw dà huáng (Radix et Rhizoma Rhei) and quán guā lóu (Fructus Trichosanthis) are added to unblock the bowels and drain fire.

● For dry mouth and lips with fluid damage, mài dōng (Radix Ophiopogonis) and lú gēn (Rhizoma Phragmitis) are added to nourish yin and promote fluid production. Because the intense fire of pathogenic toxins can easily entangle other viscera, transmuted patterns may result. There may be unconsciousness and convulsions for those with pathogen invading inwards to the heart and liver, and cough and rapid breathing for those with pathogenic toxins blocking the lung.

● When fire-toxin invades inwards to jueyin and causes unconsciousness and convulsion, gōu téng (Ramulus Uncariae Cum Uncis) and líng yáng jiăo (Cornu Saigae Tataricae) are added to suppress fright and extinguish wind. Modified Qīng Wēn Bài Dú Yĭn (Epidemic-Clearing Toxin-Resolving Beverage) can also be paired with Zĭ Xuĕ Dān (Purple Snow Elixir) to clear heat, extinguish wind and open the orifices.

● With pathogenic toxin blocking the lung with high fever, cough, panting, flaring nostrils and purple lips, a modification of Má Xìng Shí Gān Tāng (Ephedra, Apricot Kernel, Gypsum and Licorice Decoction) is used to clear heat, resolve toxins, open the lung and dissolve phlegm. Dr. Sun Jin-chen holds that severe patterns greatly damage the qi, blood and fluids of a child’s zang-fu organs. Due to yin deficiency, blood

dryness and residual toxin, there can be dry lips to the point of bleeding, gum and tongue sores, bad breath and dry stools. Following the eruption, the skin continues to itch even after the scabs have completely faded.

● Appropriate amounts of jīn yín huā (Flos Lonicerae Japonicae), xuán shēn (Radix Scrophulariae), mài dōng, shēng dì huáng (Radix Rehmanniae), rén zhōng huáng (Pulvis Praeparatus Glycyrrhizae), mŭ dān pí (Cortex Moutan), dì gŭ pí (Cortex Lycii), zĭ căo (Radix Arnebiae), bái sháo (Radix Paeoniae Alba), shí hú (Caulis Dendrobii), dān shēn (Radix et Rhizoma Salviae Miltiorrhizae) and fresh zhú yè (Folium Phyllostachydis Henonis) are used to nourish yin, moisten dryness, cool the blood and eliminate toxins. Five to seven doses is usually adequate. Moreover, attention needs to be paid to oral care and cleaning, the application of sore medicines and skin protection; the dry scabs should drop naturally without scratching in order to prevent infection. Dry foods and seafood should be avoided.

CASE STUDY Male, age 6. Initial Visit: May 28th, 1996 Chief Complaint: Fever for 5 days, papules for 1 day. History: The child began to have a fever on May 23rd, with a temperature of 39.2℃. Other signs and symptoms included a runny nose with a yellow nasal discharge, sneezing and cough. After being treated with Vitamin C Yin Qiao Tablets and paracetamol at home, the patient’s body temperature was sometimes high and sometimes low. Since yesterday, densely-distributed papules and vesicles appeared all over the body and the fever was persistent, so he was sent to the hospital. Signs and Symptoms: High fever, thirst, vexation, densely-distributed purple papules, 3 days without stools, dark urine, a red tongue with a dry yellow coating, and a rapid floating pulse. Past History: No history of other diseases or disorders. Physical Examination: BT 38.8℃, red face and lips, denselydistributed macules and papules on the torso, oval in shape, with no regular size and purple in color, turbid exudate, fewer papules on the four limbs, red pharynx, no red swollen tonsils, rough breathing sound in both lungs, regular heart rhythm, forceful heart sounds, soft abdomen with liver and spleen not palpable, no pathological syndrome in the nervous system. Laboratory Examination: WBC 7.8 × 109 /L, N 46%.

Pattern Differentiation

The clinical features included fever and the appearance of papules, vesicles and scabs on the skin caused by an externally contracted seasonal pathogen together with interiorly accumulated damp-heat, with the pathogens exteriorly dispersed to the skin. In this case, inward transmission of the intense fire-toxin to the qi and ying and its exterior dispersion to the skin caused the high fever, thirst, vexation, red face and lips, denselydistributed chickenpox, and turbid exudates. Internal accumulation of toxic heat in the blood level led to purple papules. The internal gathering of heattoxins and blockage of the bowels brought about no stool for 3 days. The red tongue with a dry yellow coating and rapid pulse were manifestations of the intense heat-toxin. The signs and symptoms are branch excess manifestations, with the pathological location residing in the qi and ying.

Diagnosis Chickenpox (severe type) due to blazing heat-toxin

Clinical Treatment Principles: Clear heat, cool blood, resolve toxins Formula: Modified Qīng Wèi Jiĕ Dú Tāng (Stomach-Heat-Clearing Toxin-Resolving Decoction) [清胃解毒汤加减]

3 doses [Formula Analysis] Shēng má outthrusts papules and clears heat. Shí gāo clears qi and discharges heat. Shuĭ niú jiăo, huáng qín, huáng lián and jīn yín huā clear heat and resolve toxins. Shēng dì huáng, mŭ dān pí and chì sháo cool blood and clear heat. Dà huáng drains fire and unblocks the bowels. Lú gēn clears heat and promotes fluid production. Prepared Medicines: Qīng Kāi Líng Kē Lì (Clear and Disperse Heat Granules), one package, 2-3 times per day. External Therapy

Grind together to powder and mix well with an appropriate amount of oil. Apply to the affected area, once daily. Second Visit After 3 days of treatment, the child’s temperature was normal and stools were regular. Other symptoms included scabbing, a red tongue with a yellow coating, and a floating pulse. Principles: Clear heat, cool the blood, nourish yin, engender fluids Formula: Modified Qīng Wèi Jiĕ Dú Tāng (Stomach-Heat-Clearing Toxin-Resolving Decoction) [清胃解毒汤加减]

3 doses [Formula Analysis] The original formula was continued, but with shí gāo, shuĭ niú jiăo and dà huáng omitted and mài dōng and shí hú added to promote fluid

production and preserve yin-fluids.

Prevention and Nursing With infection, apply 1%-2% solution of gentian violet. A lú gān shí (Calamina) wash formula can be applied for extreme itchiness with unbroken papules. Antibiotics should be used to treat those with local infection.

COMMENTARY AND DISCUSSION The six year-old in this case had accumulated toxic heat in the qi and ying levels. The signs and symptoms all together indicated the interior toxic heat was very severe. This is different from the lighter type of chickenpox or rubella, which manifest with fresh red skin rashes, fever and chills due to wind-heat invasion. In this case, a modification of Qīng Wèi Jiĕ Dú Tāng was applied, ingredients including shēng má, huáng lián, dān pí, shēng dì, huáng qín and shí gāo. These medicinals act to clear heat, cool the ying level and expel toxins. Another version of Qīng Wèi Jiĕ Dú Tāng is from Personal Experience of Small Pox and Rash Diseases (Dòu Zhĕn Chuán Xīn Lù) by Zhu Huiming in 1594. The ingredients of that formula are dāng guī (Radix Angelicae Sinensis), huáng lián, shēng dì, tiān huā fĕn (Radix Trichosanthis), lián qiào (Fructus Forsythiae), shēng má, dān pí and chì sháo. The book states that if the fire is abundant, shí gāo should be added. For mouth ulcers, swelling and painful gums following chickenpox, the formula is applied to clear the stomach, cool blood and dispel toxins. Apparently the modified Qīng Wèi Jiĕ Dú Tāng in this case is an ingredient-reduced formula from Personal Experience of Small Pox and Rash Diseases. Even though it is simpler, the modified formula maintains the essential ingredients from the classic. In this case, the accumulation of toxic heat in the qi and ying levels was very severe; therefore, shuĭ niú jiăo and dà huáng were added to

improve the effects.

STUDY QUESTIONS 1. What are the features of chickenpox? How can chickenpox and impetigo be differentiated? 2. How can the common patterns and transmuted patterns of chickenpox be identified? 3. What precautions can be taken against the disease? 4. To what should attention be paid in the nursing of this disease?

Answers 1. The chickenpox papules are characterized by visible vesicles all over the body, appearing in batches especially on the torso, along with macules and scabs in oval shapes with itching and no scars after scabbing. Impetigo, on the other hand happens mostly in summer in exposed areas like the head, face, and limbs. It begins with papules, and soon develops into pustules of irregular size, the exudates of which can be infectious. 2. There are two common patterns of chickenpox. The first involves pathogens damaging the lung-wei, featuring mild fever, runny nose, sparse red papules and a cool and clear exudate with no severe body symptoms. The second is pathogenic heat blazing in both qi and ying. There is a high fever, vexation, a red face and eyes, and densely distributed purple papules with a turbid exudate.

There are also two possible transmuted patterns. The first is pathogenic toxins invading the heart and liver. In the pathogenesis of this pattern, though vesicles fade, the symptoms of persistent high fever, blurred mind, and even unconsciousness and convulsion can remain. The second is pathogenic toxins blocking the lung. With high fever, symptoms may include unproductive cough, panting, flaring nostrils and purple lips. 3. Several things can be done to prevent chickenpox. First, control the source of infection and separate the child patient until all the vesicles scab over. Examine easily infected children who have had contact history, and inject them with varicella vaccine in case of illness. Second, avoid spreading the disease. Do not go to public places when it is contagious. The blankets, appliances and room contaminated by the patient need to be disinfected by measures including ventilation, boiling and exposure to sun or ultraviolet lamp. Third, easily-infected pregnant women should avoid contact with chickenpox patients in early pregnancy. Those contacted should be injected with varicella-zoster immunoglobulin for passive immunity. Those with chickenpox should terminate pregnancy. Fourth, for those child patients taking large doses of adrenocortical hormone or immunosuppressants, decreased immune system function or malignant tumors, varicella-zoster immunoglobulin should be injected within 72 hours of contact with chickenpox in case of infection. 4. In nursing chickenpox, ventilate and refresh the indoor air, avoid wind and cold, and prevent infection. Have the patient eat delicate foods that are easy to digest, drink boiled water, and avoid spicy food. Keep the child’s skin clean, and avoid injury to the skin from scratching. Keep the underclothes soft, and change them frequently to avoid injuring the skin and

causing infection. Finally, avoid the application of hormones in children with chickenpox. Those who have been applying hormones prior to chickenpox should reduce their use to the minimum.

CASE SCENARIOS The following cases present variations of this condition. After familiarizing yourself with the possible common pattern presentations and appropriate formulas for treatment, use the following exercises to test your overall understanding of the condition. 1. A female patient, age 7, presently has a high fever, vexation, thirst for drinks, red face and lips, densely distributed purple vesicles, with turbid exudates and a sore red base, dry stools, scanty yellow urine, a crimson red tongue with a dry rough yellow coating, and a rapid forceful pulse. What is a suitable formula to treat this disease? A. Yín Qiào Săn (Lonicera and Forsythia Powder) B. Liáng Yíng Qīng Qì Tāng (Construction-Cooling and Qi-Clearing Decoction) C. Qīng Wèi Jiĕ Dú Tāng (Stomach-Heat-Clearing and ToxinResolving Decoction) D. Xiăo Chéng Qì Tāng (Minor Purgative Decoction) E. Huáng Lián Jiĕ Dú Tāng (Coptis Toxin-Resolving Decoction) 2. A female patient, age 8 months, has had chickenpox all over her body for 4 days. Other signs and symptoms include a persistent high fever, convulsion once the previous day, somnolence, lassitude of spirit, no food intake, cough, runny nose, thin sloppy stools 3-4 times a day, scanty yellow urine, and a rapid forceful pulse. It was diagnosed as chickenpox.

What treatment should be given? A. Clear qi, cool ying, resolve toxins, remove dampness B. Clear heat, resolve toxin, assisted by releasing the exterior C. Clear heat, resolve toxin, relieve panting and cough D. Diffuse the lung, open blockage, clear heat, resolve toxin E. Resolve toxins by cooling, promote eruption of papules, vent pathogens 3. A female patient, age 3, has had a fever of unknown etiology for 3 days, a cough and a runny nose. In the previous day, macules, papules and chickenpox appeared in batches on the face and torso in different sizes, some as large as yellow beans and some as small as maize, with transparent sparse exudates. Examination revealed chickenpox on the corners of the head as large as sorghum rice, on the chest and back, and dispersed on the four limbs. Other signs and symptoms include sparsely-distributed itching papules, a light red tongue tip, a thin yellow tongue coating, and a rapid slippery pulse. What treatment should be given? A. Scatter wind, clear heat, drain dampness, resolve toxins B. Diffuse the lung, open block, clear heat, resolve toxins C. Clear heat, engender fluids, cool blood, resolve toxins D. Clear heat, resolve toxins, promote eruption of papules, relieve itching E. Clear heat, dispel wind, promote eruption of papules, vent pathogens

4. A male patient, age 2, has had fever for 2 days without sweating. Other signs and symptoms include vexation, cough, runny nose, red eyes, short scanty yellow urine, thin sloppy stools, wide distribution of chickenpox over the face and torso, itchy skin, a thin white tongue coating, and a rapid floating pulse. What is the presenting pattern? A. Heat toxin accumulation B. Wind-cold constraining the exterior C. Wind-heat invading the exterior D. Pathogens damaging lung-wei E. Damp-heat accumulation 5. A male patient, age 5, has signs and symptoms including mild fever, nasal congestion, runny nose, sneezing, cough, red papules on the skin, transparent exudates, a slightly red sore base, sparse dots with itching, a thin white tongue coating, and a rapid floating pulse. What is a suitable formula for treating this disease? A. Má Huáng Tāng (Ephedra Decoction) B. Yín Qiào Săn (Lonicera and Forsythia Powder) C. Qīng Wèi Jiĕ Dú Tāng (Stomach-Heat-Clearing Toxin-Resolving Decoction) D. Liáng Yíng Qīng Qì Tāng (Construction-Cooling Qi-Clearing Decoction) E. Shēn Sū Wán (Ginseng and Perilla Pill)

6. A female patient, age 12, had a fever and headache with aversion to cold in the morning, and was treated with penicillin and dexamethasone in the afternoon. The next day, she still had a persistent high fever with visible dispersed vesicles on the face and torso. It was diagnosed as chickenpox, and the treatment was changed to virazole for another 4 days, but with no effect. She was sent to the hospital with signs and symptoms including wide distribution of papules and chickenpox all over the body that were especially large and dense on the face, chest and abdomen, merely visible in the palms and soles but made her incapable of holding things with her hands; some papules were red while most were purple, there was turbid exudates with limited suppuration, few scabs, a high fever with much sweating, vexation, aphtha, sore throat, inability to take food, no stool for 5 days, scanty reddish urine, a red tongue with a thin coating, and a thin, rapid pulse. The pattern here is inward accumulation of damp-heat and external contraction of pathogenic toxins. What treatment should be given? A. Diffuse the lungs, open blockage, clear heat, resolve toxin, unblock the bowels and discharge heat B. Clear qi, vent toxins, eliminate dampness, resolve toxins, nourish yin, unblock the bowels C. Clear heat, engender fluids, cool the blood, resolve toxins, promote eruption of papules, vent pathogens D. Clear heat, resolve toxins, promote eruption of papules, relieve itching, nourish yin, engender fluids E. Scatter wind, clear heat, drain dampness, resolve toxins, promote eruption of papules, vent pathogen

7. A female patient, age 14, has had fever accompanied by papules for 6 days. The signs and symptoms first included persistent high fever, BT soaring to around 39.5℃, cough, a sore throat and papules and scabs on the face, chest and back. Spiramycin was given for 3 days with no effect. At a hospital visit, penicillin and virazole were used with intravenous drip for 2 days. The fever did not abate, and the other symptoms all over the body became more severe. At the visit to our clinic, signs and symptoms included high fever, cough, sore throat, reduced food intake, vexation, dry stools and yellow urine. Body examination found red face, listlessness, visible papules, sores and scabs distributed on the face, chest, abdomen and back in different sizes with swollen eyelids, red eyelid margins, congestion of the conjunctiva and sclera with excessive secretion, red swollen oral mucosa and gums, many ulcers, disordered and rough breathing sounds in both lungs by auscultation, no audible dry or moist rales, no pathological syndrome in the cardiac or nervous system, a red tongue with a thick greasy yellow coating, and rapid floating pulse. What treatment should be given? A. Clear heat, drain dampness, cool the blood, resolve toxin B. Clear heat, resolve toxins, release the exterior C. Clear heat, drain dampness, relieve panting and cough D. Diffuse the lung, open blockage, clear heat, resolve toxin E. Resolve toxin by cooling, cool the blood and stanch bleeding

Answers 1. C

2. D 3. A 4. D 5. B 6. B 7. A

Chapter 33 Hand-Foot-and-Mouth Disease Hand-foot-and-mouth disease (HFMD) is an acute, eruptive, infectious disease caused by contraction of the hand-foot-and-mouth seasonal pathogen, with clinical manifestations of vesicles on the skin of the hands and feet, mouth and pharynx. Its main pathogenic zang-fu changes occur in the lung and spleen because a child’s lung is delicate and cannot easily withstand pathogens, and because the spleen is easily damaged due to its frequent insufficiency. Without proper nursing, damage to the lung and spleen occurs when the seasonal pathogenic toxin invades from the mouth and nose. The disease is caused by intense steaming of damp-heat, internal blazing of both qi and ying, and exterior scorching of the skin due to weak health, severe pathogens, and exuberance of pathogenic qi with a decline of upright qi. If damp-heat stagnates and invades inwards to the heart, qi and yin are consumed, the heart spirit is harassed, and sometimes the child’s life can be in danger because yin damage leads to yang damage and heart yang desertion. Common clinical types include pathogens attacking the lung and spleen and intense steaming of damp-heat.

CLINICAL ESSENTIALS HFMD belongs within the TCM categories of warm disease, dampwarmth and seasonal epidemic. Reporting on this disease in China since 1981 has shown increased incidence in recent years. Reports on regional epidemics and even occasional eruptive epidemics are common. In recent years, regional epidemics have appeared in Shandong, Anhui and Guangdong provinces. Reports of death have increased, mainly due to accompanying conditions like myocarditis, pulmonary edema and aseptic meningoencephalitis. This disease occurs more often in summer and autumn. TCM maintains that overwhelming summerheat pathogens easily damage the body, especially in children whose timid spirit, deficient qi and blood, and soft zang-fu organs cannot resist the summerheat. Such people will suddenly suffer from the disease when the summerheat-toxin attacks and prevails over the body’s upright qi. Since summerheat is a yang pathogen, the disease is a warm-toxin that has a tendency to transmit and change. Thus, in the initial stage, the patient child may have qi-level patterns, including severe patterns such as blazing of both qi and ying or heat invading the pericardium. In addition, in summer and autumn, summerheat easily combines with dampness, a yin pathogen with sticky and greasy features. Since their combination retains inwards in the middle jiao and clouds clear yang, patterns of prevailing pathogenic dampness are often present.

External treatment of HFMD cannot be ignored. The vesicles in the oral and pharyngeal areas cause great pain, crying and difficulty in eating, which badly affect the life of the child. Also, breakage of the vesicles in the palms and soles increases the danger of infection. Therefore, topical, external treatment of HFMD is quite essential to relieve the pain and local symptoms.

● A small amount of Bīng Péng Săn (Borneol and Borax Powder) can be applied to the vesicles in the oral and pharyngeal areas with a cotton swab to promote the fading of the vesicles and recovery from ulcers. Bīng Péng Săn includes bīng piàn (Borneolum Syntheticum), péng shā (Borax), zhū shā (Cinnabaris) and xuán míng fĕn (Natrii Sulfas Exsiccatus). It has the effects of clearing heat, resolving toxins, dispersing swelling and relieving pain, and provides the affected areas with cooling and relief.

● Also, a mouthwash of jīn yín huā (Flos Lonicerae Japonicae) 15g, yĕ jú huā (Flos Chrysanthemi Indici) 9g, băn lán gēn (Radix Isatidis) 9g and gān căo (Radix et Rhizoma Glycyrrhizae) 3g can be decocted to enhance anti-viral effects, relieve local pain and promote recovery of wounds. Watermelon Frost Spray and ribavirin aerosol are also easy to use and quite effective.

● For vesicles on the palms and soles, an external wash can be decocted from jīn yín huā 15g, yĕ jú huā 15g, shé chuáng zĭ (Fructus Cnidii) 15g, kŭ shēn (Radix Sophorae Flavescentis) 12g, dì fū zĭ (Fructus Kochiae) 12g, băi bù (Radix Stemonae) 9g and bái xiān pí (Cortex Dictamni) 9g. The wash significantly reduces the effusion and relieves local symptoms. Additionally, lú gān shí (Calamina) lotion, Qīng Dài Săn (Indigo Powder) or acyclovir ointment may be used externally on the affected areas of the palms and soles with good results.

● In addition, the treatment of HFMD can be assisted by plaster therapy on channel points. Grind 50g of wú zhū yú (Fructus Evodiae), mix with vinegar, and apply to KI 1 (yŏng quán) every night to lead the heat downward to promote early fading of vesicles, since the vesicles in HFMD are related to accumulated heat in the heart and spleen.

CASE STUDY Male, age 3. Initial Visit: April 22th, 2009 Chief Complaint: Fever for 2 days, papules on the palms and soles for 1 day. History: This child began to have a fever on April 21st. He was also nauseous, drooling, and had no desire to eat. He was treated by his parents with Kàng Bìng Dú Kŏu Fú Yè (Antiviral Oral Liquid) but with no noticeable relief of the above symptoms. He was sent to the hospital today after his parents found large rice grain-sized papules on his palms and soles. Signs and Symptoms: Mild fever, papules on the palms and soles, some sparsely distributed vesicles that are red in color with transparent papule fluid, runny nose, cough, nausea, drooling, no desire to eat, scanty reddish urine, smooth stools, a red tongue with a thin yellow coating, and a floating rapid pulse. Past History: A weak constitution with susceptibility to colds. Physical Examination: BT 39.5℃, clear mind, dispersed papules and vesicles on the hands and feet, pharyngeal congestion, swelling of both tonsils to degreeⅠ, visible red vesicles in the pharynx, isthmus and oral mucosa, and heart, lungs and abdomen (-). Laboratory Examination: Routine blood test: WBC 12.3 × 109/L, Hb 142 g/L, N 57%, L 30%, PLT 159 × 109/L.

Pattern Differentiation

A seasonal pathogenic toxin entered from the mouth and nose, and invaded the lung and spleen. The lung controls the wei and is connected to the skin and body hair. It governs diffusion, dispersion, purification and descent and is the upper source of water. The spleen controls transportation and transformation, governs the four limbs and muscles, is the reservoir of water and grain (food and drink) and opens to the mouth. In this case, in the pathogenic toxin’s first attack, the symptoms of fever, cough, runny nose, mouth pain, poor appetite, nausea and vomiting were caused by lung failure to diffuse, wei-yang obstruction, splenic failure to transport, and stomach failure to descend harmoniously. The vesicles and papules resulted from accumulation and constraint of the pathogenic toxin, breakdown of qi transformation, and waterdampness retaining internally and thrusting externally to the fleshy exterior in its contention with the toxin. The vesicles and papules in the mouth and pharynx, broken and painful ulcers, drooling and refusal to eat were brought about by fuming and scorching of the mouth. The red tongue with the thin greasy yellow tongue coating and the rapid floating pulse are manifestations of heat attacking the lung and spleen and heat stagnating in the qi-level.

Diagnosis Hand-foot-and-mouth disease (HFMD) due to damp-heat attacking lung and spleen

Clinical Treatment

Principles: Diffuse the lung, release the exterior, clear heat, and eliminate dampness Formula: Modified Gān Lù Xiāo Dú Dān (Sweet Dew ToxinRemoving Elixir) [⽢露消毒丹加减]

3 doses [Formula Analysis] Jīn yín huā, lián qiào and bò he clear heat, resolve toxins, diffuse the lung and vent the exterior. Huáng qín clears heat, dries dampness, drains fire and resolves toxins. Huá shí and yīn chén clear heat and drain dampness. Shí chāng pú and huò xiāng dispel turbidity, harmonize the middle and diffuse turbid congestion and stagnation.

Bái dòu kòu awakens the spleen with aromatics, moves qi and resolves dampness. Zhú rú harmonizes the stomach and directs counterflow downwards. Băn lán gēn, shè gān and zhè bèi mŭ resolve toxins, disinhibit the throat, dissolve phlegm and relieve cough. This combination of ingredients aims to clear heat, release the exterior, drain dampness, move qi with aromatics, resolve toxins and disinhibit the throat. With the exterior pathogen released, dampness eliminated, heat-toxin cleared, and qi movement regulated and uninhibited, the associated symptoms will also resolve. Prepared Medicines

● Shuāng Huáng Lián Kŏu Fú Yè (Lonicera Japonica, Scutellaria and Forsythia Oral Liquid), 5-10 ml, 2-3 times daily.

● Qīng Rè Jiĕ Dú Kŏu Fú Yè (Heat-Clearing Toxin-Resolving Oral Liquid), 5-10 ml, 2-3 times daily. External Therapy

● Xī Guā Shuāng (Watermelon Frost), Bīng Péng Săn (Borneol and Borax Powder) or Zhū Huáng Săn (Pearl and Bezoar Powder), apply any one of the three to affected areas in the oral cavity for oral vesicles, 3 times daily.

● Rú Yì Jīn Huáng Săn (Agreeable Golden Yellow Powder) or Qīng Dài Săn (Indigo Powder), mix any one with sesame oil and apply to affected areas of the hands or feet, 3 times daily. Acupuncture

Points: DU 14 (dà zhuī), LI 11 (qū chí), LI 4 (hé gŭ), LU 11 (shào shāng), ST 25 (tiān shū), ST 36 (zú sān lĭ), SP 10 (xuè hăi), BL 13 (fèi shù), BL 15 (xīn shù), BL 17 (gé shù). Manipulations: Select 4-5 points, do not retain the needles. Second Visit April 25th No fever, increased food intake, dispersed visible red papules and macules on the palms, soles and oral cavity, no headache, vomiting or convulsion, but the patient did have thin yellow diarrhea 3 times a day, a red tongue with a thin yellow tongue coating, and a rapid floating pulse. Treatment should continue to release the exterior and eliminate dampness, so the above formula was again given with the addition of two ingredients:

2 doses were prescribed. Third Visit April 27th Signs and symptoms now included abated papules and heat, dry throat and mouth, dry cough with scant phlegm, nonproductive cough, little food intake, heat in the palms and soles, a red tongue with little or no coating, and a rapid and deficient pulse. This was a pattern of lung-stomach yin damage. Principles: Nourish yin, clear the lung, harmonize the stomach, direct counterflow downwards

Formula: Modified Qīng Zào Jiù Fèi Tāng (Dryness-Clearing LungRescuing Decoction) and Shā Shēn Mài Dōng Tāng (Adenophora and Ophiopogon Decoction) [清燥救肺汤合沙参麦冬汤加减]

2 doses [Formula Analysis] Sāng yè clears and outthrusts dryness-heat in the lung with its light and cold nature. Mài dōng and ē jiāo nourish yin and moisten the lungs. Tiān huā fĕn, yù zhú and shā shēn nourish yin to treat the damage to the lung and stomach yin. The bitter properties of xìng rén and pí pá yè act to descend and discharge the lung qi, as it is stated in the Yellow Emperor’s Inner Classic: Basic Questions (Huáng Dì Nèi Jīng Sù Wèn) that, “the lung is distressed by counterflow ascent of lung qi, which needs discharging by immediately taking something bitter” (肺苦⽓上逆,急食苦以泄之). The patient fully recovered after 3 doses.

COMMENTARY AND DISCUSSION Based on the signs and symptoms of rice-sized papules and some red sparsely distributed vesicles on the palms and soles with transparent papule fluid, and some visible red vesicles in pharynx, isthmus and oral mucosa, the diagnosis for hand-footmouth disease is clear. The disease belongs within the category of epidemic damp-warmth disease. The transparent papule fluid with nausea, drooling, no desire to eat and the greasy yellow tongue coating are all signs of damp-warmth evil, and therefore Gān Lù Xiāo Dú Dān was applied in this case. Gān Lù Xiāo Dú Dān is an essential formula for damp-warmth evils in the sanjiao. In this case, the cough and runny nose indicate damp-warmth evils in the upper jiao; the nausea, drooling and poor appetite are signs of damp-warmth evils in the middle jiao; and the dark colored scanty urination shows dam-warmth evils in the lower jiao. Therefore, the medicinals in the formula should act to eliminate damp-warmth from the upper, middle and lower jiao. Here huáng qín, lián qiào and bò he clear heat and damp-heat from the upper jiao and disperse pathogens. Lián qiào and bò he are acrid and facilitate the transportation of lung qi as the lungs are the upper source of body fluids. Then, a few aromatic medicinals are applied to transform dampness from the middle jiao, including huò xiāng, bái dòu kòu and shí chāng pú. For dampness in the lower jiao, draining medicinals with bland taste are added in order to promote urination, including huá shí and mù tōng (Caulis Akebiae).

When dampness and heat combine, it is important to find out whether there is more dampness than heat or more heat than dampness. Gān Lù Xiāo Dú Dān is for a condition in which there is more heat than dampness; because of this, jīn yín huā, yīn chén and zhú rú were added. Because handfoot-mouth disease is an epidemic illness, lián qiào, shè gān and băn lán gēn were added for removing pestilential factors, or li qi (pestilent qi). With modification, this formula can also be applied to treat mumps and other warm diseases due to damp-warm evils. STUDY QUESTIONS 1. How can HFMD and chickenpox be differentiated? 2. What are the treatment principles of the disease? 3. Why is it necessary to emphasize the dispelling of dampness in the treatment of HFMD?

Answers 1. HFMD is an eruptive infectious disease caused by externallycontracted seasonal HFMD pathogen, with clinical features including vesicles on the skins of the palms, soles, mouth and pharynx. The papules, ranging from several to hundreds in number, are eccentrically distributed, hard in texture, and most do not break, with turbid liquid inside and a red circle surrounding. The long axis of the sores is parallel to the skin creases of the fingers and toes. In contrast, chickenpox is caused by externally contracted chickenpox virus. The vesicles are larger than those in HFMD and distributed concentrically, with many on the torso, head and face but few on the limbs.

The vesicle walls are thin and easy to break open, causing scabbing. They are mainly ellipse-shaped, with the long axis perpendicular to the direct axis of the torso. Chickenpox also features the coexistence of macules, papules, vesicles and scabs. 2. The treatment principles of the disease are to clear heat, dispel dampness and resolve toxins. For mild patterns, treatment is given to diffuse the lung, release the exterior, clear heat and eliminate dampness. For the severe pattern, the relative preponderance of dampness or of heat must be determined. For those with preponderance of dampness, treatment is given mainly to drain and eliminate dampness, assisted by clearing heat and resolving toxins. Meanwhile, dispelling dampness should not be overdone as this may cause damage to yin with impaired body fluids leading to internal dryness. For those with a preponderance of heat, treatment is given mainly to cool and clear heat and resolve toxins, with care being taken not to damage the spleen and stomach and conduct the pathogen deeper. For those with pathogenic toxins invading inward or attacking the heart, medicinals should be combined to suppress convulsion, open the orifices, boost qi, nourish yin, invigorate blood and dispel stasis. 3. Children’s lung and spleen are deficient in qi and easily damaged. The lung is the upper source of water, while the spleen is the reservoir of water and grain. When a seasonal pathogenic toxin enters from the mouth and nose and invades the lung and spleen, the toxin accumulates and constrains the lung, causing them to fail to diffuse and purify, qi transformation become disordered, and the spleen fails to transform and transport water-dampness. Dysfunction of the lung and spleen leads to interior water-dampness which thrusts outward to the fleshy exterior as

vesicles in its contention with the toxins. Dampness is the key cause of the disease, and thus special attention must be given to the dispelling of dampness. CASE SCENARIOS The following cases present variations of this condition. After familiarizing yourself with the possible common pattern presentations and appropriate formulas for treatment, use the following exercises to test your overall understanding of the condition. 1. A patient, age 6, has had a fever for 3 days with BT of 38.5℃. Other signs and symptoms include a runny nose, cough and poor appetite. Vesicles appeared in the hard palate and gums in the oral cavity at the beginning, while papules as large as rice appeared on the palms and soles one day later, among which some were hard with turbid liquid inside and a surrounding red coloration. What is the diagnosis? A. Herpangina B. Chickenpox C. Hand-foot-and-mouth disease D. Urticaria papulosa E. Impetigo 2. A patient, age 2, has had a fever for 2 days with the highest BT being 39.8℃. Other signs and symptoms include vexation, thirst, vesicles in the oral and cheek mucosa, pain, refusal to eat, papules and macules on the palms and soles as large as rice, some densely distributed purple vesicles with turbid fluid, bad breath, drooling, dry stools, a red tongue with

a thick greasy yellow tongue coating, and purple finger venules. The condition was diagnosed as HFMD. What is the presenting pattern? A. Pathogens damaging lung-wei B. Intense steaming of damp-heat C. Pathogens attacking lung and spleen D. Constrained damp-heat in the exterior E. Constrained pathogens in the fleshy exterior 3. A patient, age 4, has had fever for 2 days with the highest BT being 38.3℃. Other signs and symptoms include sneezing, a runny nose, cough, poor appetite, diarrhea, sparsely distributed vesicles on the gums and inner lips of the oral cavity, red macules on the palms and soles, a red tongue with a thin yellow coating, and floating purple finger venules. What are the treatment principles? A. Scatter the wind, resolve toxin, clear heat, eliminate dampness B. Clear heat, cool ying, resolve toxin, dispel dampness C. Diffuse the lung, release the exterior, clear heat, resolve toxin D. Release the exterior, eliminate dampness, clear heat, resolve toxin E. Diffuse the lung, release the exterior, clear heat, eliminate dampness 4. A patient, age 7, has had fever for 4 days with the highest BT being 38.7℃. Other signs and symptoms include no elevation of BT, relatively high BT in the afternoon, vesicles in the oral cavity, palms and soles, itchy limbs and buttocks, fascicularly distributed vesicles with red base and turbid liquids, a bitter taste in the mouth, thirst, poor appetite, occasional

vomiting and nausea, reddish-yellow urine, slightly dry stools, a red tongue with a greasy yellow coating, and a rapid floating pulse. After differentiation, which of the following medicinals should not be chosen in the treatment? A. Chē qián căo (Herba Plantaginis) B. Huá shí (Talcum) C. Huò xiāng (Herba Agastachis) D. Zhĭ shí (Fructus Aurantii Immaturus) E. Zhú yè (Folium Phyllostachydis Henonis) 5. A patient, age 8, has had a fever for 3 days with the highest BT being 39.1℃. Other signs and symptoms include vexation, thirst, densely distributed purple vesicles in the oral cavity, palms, soles and buttocks, turbid vesicle liquid, foul breath, pain, poor appetite, regular urine and stools, a red tongue with a thick yellow coating, and a rapid floating pulse. What is the best treatment formula? A. Qīng Wēn Bài Dú Yĭn (Epidemic-Clearing Toxin-Resolving Beverage) B. Gān Lù Xiāo Dú Dān (Sweet Dew Toxin-Removing Elixir) C. Huáng Lián Jiĕ Dú Tāng (Coptis Toxin-Resolving Decoction) D. Qīng Shŭ Yì Qì Tāng (Summerheat-Clearing Qi-Boosting Decoction) E. Huá Shí Huò Xiāng Tāng (Talcum and Agastache Decoction) 6. A patient, age 4, has had a fever for 4 days with the highest BT being 39.6℃. Other signs and symptoms include vexation, thirst, densely

distributed vesicles in the oral cavity, palms, soles and buttocks that are purple in color, turbid vesicle liquid, foul breath, pain, poor appetite, yellow and reddish urine, dry stool, a red tongue with a thick yellow tongue coating, and a rapid floating pulse. What are the treatment principles? A. Diffuse the lung, release the exterior, clear heat, eliminate dampness B. Diffuse the lung, release the exterior, clear heat, resolve toxins C. Clear heat, cool ying, resolve toxin and dispel dampness D. Release the exterior, eliminate dampness, clear heat, resolve toxins E. Scatter wind, resolve toxin, clear heat, eliminate dampness

Answers 1. C 2. B 3. E 4. D 5. A 6. C

Chapter 34 Mumps Mumps is an infectious respiratory disease caused by external contraction of the mumps toxin. Clinical manifestations include fever with non-suppurative painful swelling on the cheek below the ear. The disease can invade other glands, nerve systems and the heart, of which common concurrent symptoms include meningocephalitis, orchitis or ovaritis in adolescents. This disease is mostly caused by external contraction of wind-warmth seasonal toxins, as well as internal accumulation of heat, all of which gather in and constrain the shaoyang channel to stagnate in the cheek below the ear where it contends with qi and blood. Pathogenic wind-warmth invades from the mouth, nose, and fleshy exterior and moves to the foot shaoyang gallbladder channel, which starts at the outer canthus of the eyes, passes through the front and back of the ears, goes down both sides of the body and stops at the fourth toe. Thus, the parotid is a place that the channel is due to cross. The pathogen contracted in the shaoyang channel invades upward to the cheek by following the channel pathway where it contends with qi and blood. The symptoms of local diffuse swelling and pain are caused by the inhibited movement of qi and blood due to stagnation. The symptoms of gaping mouth and difficulty chewing derive from the disharmony in the channel area in the jaw.

When heat-toxins are intense and the upright qi weak, it may inwardly invade the foot jueyin liver channel due to its exterior-interior relationship with foot shaoyang gallbladder, leading to stirring of liver wind and clouding of the pericardium. Thus, the symptoms of persistent high fever, convulsion, stupor, etc. may occur. Because the foot jueyin liver channel skirts around the genitals and penetrates the lower abdomen, an inward transmission of the pathogenic toxin leads to accompanying symptoms of swelling or pain in the testicles or hypogastric region as toxins drain into the testicles or scurry into the abdomen. Clinical patterns include toxins attacking the shaoyang channel, and intense internal toxic-heat. Common transmuted patterns include pathogenic toxins invading the heart and liver, and toxins scurrying to the abdomen and testicles.

CLINICAL ESSENTIALS Mumps is most often caused by an external contraction concurrent with a deficiency of upright qi. As the saying goes, “with the existence of upright qi, a pathogen cannot invade”. Owing to their delicate lung and insufficiency in defense against the exterior, children are more susceptible to this disease, which when contracted is characterized by a struggle between upright qi and externally-contracted pathogenic qi. Following the transmission and change principles of warm diseases, TCM treats the disease according to four-level pattern differentiation. Fever is common in all patients.

● At the initial stage of fever with concurrent exterior patterns, the treatment principles are to scatter wind and clear heat while diffusing and venting the pathogen, so as to resolve the toxic-heat through releasing the exterior. Therefore, medicinals to dispel wind and promote ascending and dissipation of the internal pathogens should be used. Moreover, huáng qín (Radix Scutellariae) and huáng lián (Rhizoma Coptidis) should not be overused or applied too early, as higher doses of bitter-old medicinals can easily lead to latent pathogens which are more difficult to resolve.

● At the medial stage, when heat congests the shaoyang and yangming channels with exuberant heat in the qi-level, medicinals can be added judiciously to clear qi-level heat.

● At the final stage, when the pattern is binding constraint of phlegmheat characterized by abating fever and reduced pain with only hard

swelling of the cheeks remaining, dāng guī (Radix Angelicae Sinensis) and chì sháo (Radix Paeoniae Rubra) can be used to invigorate blood, dissolve stasis and dissipate masses. Many experienced TCM doctors have particular ways of treating this disease. Jiang Yu-ren classifies mumps into patterns of warmth-toxin at the exterior, heat-toxin congestion and accumulation, or transmuted patterns.

● For warm-toxins at the exterior, a modification of Yín Qiào Săn (Lonicera and Forsythia Powder) is given to scatter wind, clear heat, dissipate masses and disperse swelling.

● For heat-toxin congestion and accumulation, a modification of Pŭ Jì Xiāo Dú Yĭn (Universal Relief Toxin-Removing Beverage) is given to clear heat, resolve toxins, soften hardness and dissipate masses.

● For a transmuted pattern with pathogenic invasion of the heart and liver, Pŭ Jì Xiāo Dú Yĭn and Zĭ Xuĕ Dān (Purple Snow Elixir) are given to clear heat, resolve toxins, extinguish wind and open the orifices.

● For a transmuted pattern with pathogenic toxins scurrying to the testicles and abdomen, modifications of Lóng Dăn Xiè Gān Tāng (Gentian Liver-Draining Decoction) are given to clear liver fire, invigorate blood and relieve pain. Zhao Xin-bo suggests clearing heat, resolving toxin, dissipating masses and dispersing swelling at the initial stage, while resolving toxin and transforming turbidity at the final stage. Wang Lie emphasizes on clearing heat, resolving toxin, invigorating blood and dispersing swelling in the treatment. And Liu Yun-yuan treats this disease by using oral decoctions to clear heat and resolve toxin and topical applications to relieve pain and disperse swelling, which has achieved good curative effects.

Topical applications can be made by grinding the medicinals to powder and mixing them with vinegar, oil, egg white and fresh medicinal juices to make a paste for external application. These include Yù Lù Gāo (Jade Dew Paste), Qīng Dài Săn (Indigo Powder), Rú Yì Jīn Huáng Săn (Agreeable Golden Yellow Powder), Liù Shén Wán (Six Spirits Pill), Niú Huáng Jiĕ Dú Wán (Bovine Bezoar Toxin-Resolving Pill), Bīng Péng Săn (Borneol and Borax Powder) and Jīn Huáng Gāo (Golden Yellow Paste). Modern acupuncture treatment of the disease began in 1957; since that time many clinical case studies have been recorded. Wang Deng-qi first needled DU 14 (dà zhuī), LI 11 (qū chí), LI 4 (hé gŭ) and ST 36 (zú sān lĭ) with strong stimulation and no needle retention. Then a three-edged needle was used on LU 11 (shào shāng) and LI 1 (shāng yáng) for bloodletting. SJ 20 (jiăo sūn), ST 7 (xià guān) and SJ 17 (yì fēng) treated with medicated threadmoxibustion. The swelling on the cheek decreased immediately, and after only two treatments by the next afternoon, the fever had abated and swelling on the cheeks disappeared completely. Notable preventative effects can be achieved with ear acupuncture and acupressure. First, select ĕr jiān (EX-HN6) and prick to bleed 1-2 drops with three-edged needle. Then apply a wáng bù liú xíng (Semen Vaccariae) ear-seed to the mumps extra point located on the cheek. Press 50 times, twice daily; change the seed and adhesive once every 3 to 4 days. 7 days constitute one course of treatment.

CASE STUDY Female, age 8. Initial Visit: June 15th, 2009 Chief Complaint: Swelling and pain on both cheeks and fever for 2 days. History: The child began to have swelling and pain on both cheeks on June 14th, and a high fever followed. Other signs and symptoms included pain when opening the mouth and with chewing, hard cheeks sensitive to pressure, and a poor appetite. After home treatment with ibuprofen and amoxicillin, there was no relief, so she was sent to the hospital. Signs and Symptoms: Fever, aversion to cold, headache, swelling and pain of both cheeks, difficulty in opening the mouth and chewing, sore throat, severe dry mouth, poor appetite, occasional halitosis, no vomiting, scanty reddish urine, smooth stools, a red tongue with a thick greasy yellow coating, and a rapid and floating pulse. Past History: A weak constitution with susceptibility to colds. She also had recent contact with a child with mumps. Physical Examination: BT 39℃, clear mind, 5 cm × 4 cm soft swollen cheeks especially on the right with sensitivity to pressure, taut local skin, slightly red and swollen parotid duct openings, congested pharynx, enlargement of both tonsils to degree Ⅱ, rough breathing sound in both lungs. No audible dry or moist rales, no abnormality in the abdomen, and no pathological syndrome in the nervous system.

Laboratory Examination: WBC 11.8 × 109/L, N 45.6%, L 50.2%, PLT 278 × 109/L, HGB 126 g/ L. Serum amylase 365 IU/L, amylase in urine 700 IU/L.

Pattern Differentiation The symptoms of local diffuse swelling and pain below the cheeks were caused by an externally-contracted pathogenic toxin from recent contact with another child with mumps. This lead to constraint and disharmony of the shaoyang channel with inhibited movement of qi and blood. The pain when opening the mouth and chewing resulted from pathogens obstructing the channel and inhibiting the jaw. The aversion to cold, fever and headache owed to the breakdown of exterior and interior, because the shaoyang channel belongs within the category of half-exterior/ half-interior. The dry mouth and sore throat arose from lung-heat damaging fluids. The scanty reddish urine, red tongue with a greasy thick yellow coating and the rapid floating pulse are all symptoms of toxic-heat congestion and accumulation.

Diagnosis Mumps due to toxic-heat accumulation

Clinical Treatment

Principles: Clear heat, resolve toxins, dissipate masses, disperse swelling Formula: Modified Pŭ Jì Xiāo Dú Yĭn (Universal Relief ToxinRemoving Beverage) [普济消毒饮加减]

2 doses [Formula Analysis] Băn lán gēn, niú bàng zĭ and pú gōng yīng resolve toxins. Huáng qín and lián qiào clear constrained fire congested in the upper body. Chái hú and bò he scatter wind-heat and disinhibit the shaoyang channel.

Huáng lián and shēng má clear heat and resolve toxins. Mă bó and xuán shēn clear heat and disinhibit the throat. Jié gĕng and gān căo penetrate into the upper body to dissipate masses and eliminate congestion. Jiāng cán dispels wind and unblocks the collaterals. Xià kū căo clears heat, dissipates masses and disperses swelling. Chén pí regulates qi and resolves congestion and stagnation. Prepared Medicinals: Sāi Xiàn Yán Piàn (Mumps Tablets), 4 tablets, 3 times daily. External Therapy

● Rú Yì Jīn Huáng Săn (Agreeable Golden Yellow Powder) or Qīng Dài Săn (Indigo Powder) (proper amount): mix any one with vinegar or sesame oil and apply to the affected areas, twice daily.

● Fresh xiān rén zhăng (Radix Opuntiae et Caulis) (one piece): remove the thorns, mash or slice after washing and apply to the affected areas, once or twice daily.

● Fresh dì lóng (Pheretima) (20 pieces) and white sugar (100 g): mix until soupy and apply to the affected areas, 3 times daily. Acupuncture Main Points: LU 11 (shào shāng), LI 4 (hé gŭ), LI 1 (shāng yáng). Supplementary Points: ST 6 (jiá chē), GB 20 (fēng chí), DU 14 (dà zhuī). Manipulations: Use strong stimulation with rotation and no needle retention; treat once daily.

Spot Burning Dip dēng xīn căo (Medulla Junci) in sesame oil, ignite and scorch SJ 20 (jiăo sūn) on the affected side until the crisp “cracking” sound can be heard. Treat once a day. Second Visit June 17th Signs and symptoms included mild fever, no aversion to cold, continued swelling and pain of both cheeks, difficulty opening the mouth, constipation, and red urine. The treatment should still focus on clearing, discharging and resolving toxins. The previous formula was given with the addition of dà huáng (Radix et Rhizoma Rhei) 6g for an additional 2 doses. Third Visit June 19th Signs and symptoms included abating fever and pain; only the hard swelling on the cheeks remains. There are residual pathogens with binding constraint of phlegm-heat. Principles: Disperse scrofula, nourish yin, soften hardness, dissipate masses Formula: Modified Xiāo Luŏ Wán (Scrofula-Dispersing Pill) [消瘰丸加减]

3 doses [Formula Analysis] Xuán shēn enriches yin, subdues fire and disperses scrofula with bittersaltiness. Bèi mŭ disperses swelling, resolves constraint and dissipates masses. Hăi zăo, kūn bù and mŭ lì foster yin, subdue yang, soften hardness and disperse scrofula with salty-coldness. Xià kū căo clears heat, dissipates masses and disperses swelling. Mŭ dān pí and chì sháo cool blood, resolve toxins, invigorate blood and disperse swelling. After taking 3 doses of the prescribed formula, the patient was cured.

COMMENTARY AND DISCUSSION Mumps falls into the category of warm disease and shares the same mechanism with Dà Tóu Wēn (Swollen-Head Infection), an epidemic disease due to seasonal wind-warmth. Even though the swelling appears to be serious, the prognosis is good because this is a localized problem that seldom transmits to the ying- and blood-levels. In this case, the child had swelling and pain of both cheeks with chills, fever and headache for 2 days. She also had scanty reddish urine, a red tongue with a greasy thick yellow coating, and a rapid floating pulse. All these indicate toxic-heat and fire accumulated interiorly and exteriorly. Pŭ Jì Xiāo Dú Yĭn is an essential formula for mumps, and represents a particular style of treatment. It is said that before this formula was created, the practitioners of the time applied Chéng Qì Tāng (Qi-Guiding Decoction) plus băn lán gēn to purge the toxic-heat for mumps and other infections characterized by a swellings affecting the head. The symptoms would be released for a while, but would generally return the following day. With more purging, the symptoms would come and go, but the problem was never cured; instead, the situation became worse and then emergent. Li Dong-yuan saw this and explained, “the upper part of body belongs to heaven, while the lower part of body to earth. Now, the heat- and fireevils stay in between the heart and lungs and they flare up to attack the head and cause head swelling. Applying Chéng Qì Tāng to purge excessive heat from the stomach and intestines is to attack an area where there is no sickness”. (夫身半以上,天之⽓也,身半以下,地之⽓也,此邪热客

于⼼肺之间,上攻头⽽为肿盛,以承⽓泻胃中之实热,是为诛伐无 过。) Therefore, he created Pŭ Jì Xiāo Dú Yĭn for this kind of problem. Mumps and infections with a swollen head are due to epidemic windwarmth blocking the upper jiao and attacking the head and face. The swelling with redness and hot pain are due to qi and blood congestion, while toxic-warmth accumulating in the throat causes a sore throat. The contending of upright qi and evil qi causes the fever and chills. Therefore the wind-warmth should be dispersed, while the epidemic toxins should be cleared and scattered. The disease location is in the upper jiao, face and head. Because of this, the chief medicinals huáng lián and huáng qín are used to clear heat from the upper jiao. Xuán shēn, mă bó and băn lán gēn enhance the effect of clearing heat and removing toxins. Niú bàng zĭ, lián qiào, bò he and jiāng cán are acrid and cool or cold-natured and act to disperse the wind-heat from the head and face; these are deputy medicinals. Another group of medicinals, shēng má, chái hú and jié gĕng are also important medicinals and most interesting in that they have two functions. First, they disperse wind-heat, because the depressed fire should be raised and dispersed. Secondly, they are applied to guide the effect of the other medicinals to the head and face to achieve a stronger effect. The whole formula contains no purgative medicinals to drain heat and fire. As was discussed at the beginning of this commentary, the mumps and infection with swellings on the head are upper jiao problems, and because they are localized, they do not usually attack ying and blood levels. Treatment should not purge through the lower jiao because this may lead internal pathogens to a deeper location and result in complications.

STUDY QUESTIONS 1. Describe measures that can be taken to prevent mumps. 2. How can the swelling and pain of the cheeks be eliminated? 3. How can transmuted patterns be avoided?

Answers 1. Improve infectious disease reporting, inject attenuated live mumps vaccine, and separate patients at home or send them to the hospital. It is not necessary to keep patients under quarantine, but medical observation and nursing are needed for 3 weeks. It is also necessary to boil and disinfect the secretions from the patient’s nose and mouth from his towels and food. Keep the patient’s room well-ventilated. 2. The clinical manifestations of mumps include parotid swelling and pain. Before the appearance of transmuted patterns, treatment is given to clear heat, resolve toxins, dissipate masses and relieve pain with táo rén (Semen Persicae), mă bó (Lasiosphaera seu Calvatia), tiān huā fĕn (Radix Trichosanthis), and xià kū căo (Spica Prunellae), etc., because the patient’s major pain comes from parotid swelling. For those with hard and persistent masses, medicinals like chì sháo (Radix Paeoniae Rubra) and dān pí (Cortex Moutan) can be added to invigorate blood, also accompanied by external applications. 3. During mumps, transmuted patterns can easily result from excess heat and warmth-toxins invading deeply, also from improper or untimely

treatment. To avoid transmuted patterns, the first measure is to abate fever. For those with wind-heat, scatter wind and clear heat with yín huā (Flos Lonicerae Japonicae), lián qiào, bò he and chái hú. For those with intense heat-toxins, huáng qín, huáng lián, raw shí gāo and xuán shēn should be used in heavy doses. For those with bound stools, dà huáng and xuán míng fĕn (Natrii Sulfas Exsiccatus) can be used to unblock the bowels and discharge heat to the effect of “taking away the firewood from under the cauldron”. In addition, for timely discovery and treatment of complications, the patient must be observed closely for changes in the mind or spirit, or physically in the testicles. For those with swollen painful testicles, cold and moist applications can be given locally, and gauze can be used as a support for the swollen scrotum.

CASE SCENARIOS The following cases present variations of this condition. After familiarizing yourself with the possible common pattern presentations and appropriate formulas for treatment, use the following exercises to test your overall understanding of the condition. 1. A patient, age 3, has had a low-grade fever for 3 days. Other signs and symptoms include one-side swelling and pain on the cheek and, on the other side, an obvious edge and pain with touch, difficulty chewing, sore throat, a red tongue with a slightly yellow coating, and a rapid floating pulse. What is the best treatment formula? A. Wŭ Wèi Xiāo Dú Yĭn (Five Ingredients Toxin-Removing Beverage) B. Pŭ Jì Xiāo Dú Yĭn (Universal Relief Toxin-Removing Beverage) C. Yín Qiào Săn (Lonicera and Forsythia Powder) D. Qīng Wèi Jiĕ Dú Tāng (Stomach-Heat-Clearing Toxin-Resolving Decoction) E. Chái Hú Gé Gēn Tāng (Bupleurum and Pueraria Decoction) 2. A patient, age 5, has had a recurrent fever for 4 days. Other signs and symptoms include swollen cheeks, swelling and pain of the left testicle, pain in the lower abdomen that is sensitive to pressure, occasional vomiting, yellow urine, hard stools, a red tongue with a yellow coating, and a rapid pulse.

What is the best treatment formula? A. Wŭ Wèi Xiāo Dú Yĭn (Five Ingredients Toxin-Removing Beverage) B. Huáng Lián Jiĕ Dú Tāng (Coptis Toxin-Resolving Decoction) C. Jú Hé Wán (Tagerine Pip Pill) D. Lóng Dăn Xiè Gān Tāng (Gentian Liver-Draining Decoction) E. Yín Qiào Săn (Lonicera and Forsythia Powder) 3. A patient, age 4, has a persistent high fever, diffuse hard swelling on the cheeks below the ears that is sensitive to pressure, headache, a painful stiff nape, vexation, severe vomiting, unconsciousness, somnolence, a red tongue with a yellow coating, and a rapid wiry pulse. What is the presenting pattern? A. Pathogens attacking the shaoyang channel B. Pathogens invading heart and liver C. Toxins scurrying to testicles and abdomen D. Intense toxic-heat E. Phlegm-fire harassing inward 4. A patient, age 3, has a mild fever with aversion to cold, swelling and pain on the left cheek with no clear edge but with pain when touched, headache, a sore throat, a red tongue with a thin white coating, and a rapid floating pulse. What are the correct principles of treatment? A. Scatter wind, clear heat, dissipate masses, disperse swelling B. Clear liver heat, drain fire, invigorate blood, relieve pain

C. Clear heat, resolve toxins, extinguish wind, open the orifices D. Clear qi, cool the ying, drain fire, resolve phlegm E. Clear heat, resolve toxins, dissipate masses, disperse swelling 5. A patient, age 6, has a persistent high fever with unconsciousness, somnolence, a stiff nape, recurrent convulsions, diffuse hard swelling and pain on the cheeks below the ears that is sensitive to pressure, headache, vomiting, a red tongue with a yellow coating, and a rapid wiry pulse. What is the best treatment formula? A. Wŭ Wèi Xiāo Dú Yĭn (Five Ingredients Toxin-Removing Beverage) B. Qīng Wēn Bài Dú Yĭn (Epidemic-Clearing Toxin-Resolving Beverage) C. Yín Qiào Săn (Lonicera and Forsythia Powder) D. Pŭ Jì Xiāo Dú Yĭn (Universal Relief Toxin-Removing Beverage) E. Lóng Dăn Xiè Gān Tāng (Gentian Liver-Draining Decoction) 6. A patient, age 8, has had diffuse swelling on both cheeks centered on the ear lobes, which are sensitive to pressure, for one day. Other signs and symptoms include difficulty chewing and opening the mouth, a high fever, headache, dry mouth, thirst, bound stools, yellow urine, a red tongue with a dry yellow coating, and a rapid floating pulse. Pŭ Jì Xiāo Dú Yĭn was chosen as the main formula for treatment. Among the following, which is an improper modification? A. For intense heat, shí gāo (Gypsum Fibrosum) and zhī mŭ (Rhizoma Anemarrhenae) are added to clear heat and drain fire.

B. For swollen cheeks, sān léng (Rhizoma Sparganii) and é zhú (Rhizoma Curcumae) are added to soften hardness and dissipate masses. C. For vomiting, jiāng (Rhizoma Zingiberis Recens) Succus and zhú rú (Caulis Bambusae in Taenia) are added to clear stomach fire and arrest vomiting. D. For bound stools, dà huáng (Radix et Rhizoma Rhei) and máng xiāo (Natrii Sulfas) are added to unblock the bowels and discharge heat. E. For the dry mouth and thirst, huā fĕn (Radix Trichosanthis) and shēng dì (Radix Rehmanniae) are added to nourish yin and promote fluid production. 7. A patient, age 7, has had a high fever and diffuse swelling on the cheeks for 3 days. Yesterday, other signs and symptoms appeared including a stiff nape, unconsciousness, convulsions occurring twice, vomiting once, a red tongue with a yellow coating, and a rapid wiry pulse. What is the pathomechanism? A. Pathogenic heat invading inward and blazing of both qi and ying B. Pathogenic heat damaging yin and wind stirring due to yin deficiency C. Intense pathogenic heat and ascendant hyperactivity of liver yang D. Internal blazing of heat-toxins with liver heat attacking the stomach E. Pathogenic toxins invading inward to the heart and liver

Answers 1. E

2. D 3. B 4. A 5. B 6. B 7. E

Chapter 35 Epidemic Encephalitis B Epidemic encephalitis B refers to an acute infectious disease of the central nervous system due to the epidemic type B encephalitis virus, a seasonal pathogen, the clinical manifestations of which include high fever, disturbance of consciousness, convulsion, pathologic reflex, and meningeal irritation. It belongs in the category of summerheat epidemic warm disease, and disease names include summerheat-wind, summerheat convulsion and summerheat syncope, depending upon the clinical manifestations. This seasonal disease occurs most often in the midsummer months of July, August, and September. It may attack people of all ages, most often children under 10, and especially those between 2 and 6. It is very infectious, and exposed people are likely to become infected by the type-B encephalitis virus. However, most infections are latent, and the infection can give relatively persistent immunity. Also, antibodies transmitted from the mother can also provide a level of protection. The prognosis can be quite good if a child patient with a mild syndrome is given timely treatment. However, owing to the disease’s sudden attack and quick transmission, severe, dangerous patterns of internal block, external desertion, disordered breathing, and others are inclined to appear with high fatality rates or accompanied sequelae.

Epidemic encephalitis B is caused by the exterior contraction of the seasonal pathogen epidemic encephalitis B, an epidemic summerheatwarmth toxin. The toxin invades from the skin and body hair through mosquito bites, and, because of original qi depletion in the season of summerheat qi, the disease results. Children who are weak in spirit with insufficient qi and blood and delicate zang-fu organs may be unable to bear the “three qi” of summer, dampness, and heat. In those children, both yinfluids and yang qi are discharged by sweating, which can then easily lead to the disease. The common clinical manifestations occur in two stages. In the initial stage, critical or acute stage, the pathogen attacks the wei qi, blazes in the qi- and ying-levels and invades ying-blood. In the recovery or sequelae stage there can be patterns of yin deficiency and internal heat, ying-wei disharmony, phlegm clouding the seven orifices, phlegm-heat harassing the interior, qi deficiency and blood stasis, and wind pathogens lingering in the collaterals.

CLINICAL ESSENTIALS The manifestations of this disease can be classified into two primary stages, the acute stage and the recovery stage. For proper clinical pattern identification, one must correctly identify the presenting heat pattern. The heat syndrome in the acute stage is excess heat, which is often complicated by dampness. The initial period of the acute stage mainly features exterior heat patterns with signs and symptoms of fever, aversion to cold, and head and body pains with a stiff neck. This soon converts to an interior heat pattern, meaning the wei-level pattern converts to a qi-level pattern, with signs and symptoms including a persistent high fever, vexation, somnolence, rigidity of the neck and nape, nausea, and vomiting. The severe period of the acute stage features blazing of both qi and ying with signs and symptoms of a persistent high fever, clouded spirit, delirious speech, stiff nape, convulsion, and rapid forceful pulses. At the end of the severe period, heat invading ying-blood causes an persistent fever that is slight in the morning but severe in the evening with scorching heat in the chest and abdomen, and a dry crimson tongue. The treatment of epidemic encephalitis B requires clearing heat, eliminating phlegm, opening the orifices, and extinguishing wind. In its acute stage, resolving heat is primary. Treatment must also open the orifices, eliminate phlegm, suppress fright, and extinguish wind by combining treatments for phlegm patterns and wind patterns.

● For exterior summerheat, treatment is given to clear summerheat and vent the exterior, while eliminating dampness with aromatics.

● For summerheat inward invasion, treatment is given to clear heat with bitter-coldness while unblocking the bowels and discharging heat.

● For constrained pathogens transforming into fire and invading yingblood, treatment is given to clear ying-level heat and drain fire with bittercoldness and salty-coldness. In its recovery or sequelae stage, treatment is given to reinforce upright qi and dispel pathogens.

● For incomplete elimination of the pathogen or when deficiency heat fails to abate, treatment is given to nourish yin and clear heat or to harmonize ying and wei.

● For phlegm clouding the orifices and confused spirit, treatment is given to eliminate phlegm and open the orifices or to direct the turbid downward and revive the mind.

● For internal wind harassing and useless limbs, treatment is given to boost qi, invigorate blood, dispel wind, unblock the collaterals and relax the sinews. Meanwhile, comprehensive treatment using acupuncture, tui na, and physiotherapy can be administered to accelerate recovery.

CASE STUDY Female, age 8. Initial Visit: July 20th, 2009 Chief Complaint: Fever and headache for 2 days, convulsion for 2 hours. History: The child began to have fever and headache yesterday morning and was treated at the local clinic for a common cold with amoxicillin and Shuāng Huáng Lián (Lonicera Japonica, Scutellaria and Forsythia) without effect. This morning, the headache was worse and was accompanied by spray-like vomiting twice. Other signs and symptoms included vexation, uneasiness, clouding sleep, and convulsion of the four limbs. She was sent by emergency transport and hospitalized. Signs and Symptoms: Coma, sleep talking, a high fever with sweating, frequent convulsions, reddish-yellow urine, a red tongue with a dry yellow coating, and a rapid wiry pulse. Past History: No history of other diseases or disorders. Physical Examination: BT 39.8℃, passive body posture, coma, hasty rapid breathing, intermittent convulsion of the four limbs, both pupils of equal size, slow response to light and a rigid neck. She tested positive for Brudzinski’s, Babinski’s and Kernig’s signs. Laboratory Examination: WBC 20 × 109/L, N 80%, L 20%. Cerebrospinal fluid is slightly turbid in appearance, WBC 410 × 106/L, mainly lymph cells; positive Pandy’s test, protein 450 mg/L, sugar 2.6 mmol/L, chloride 145 mmol/L.

Pattern Differentiation This case, occurring in summer with features of a high fever, coma, and convulsion, conformed to the diagnosis of summerheat-warmth in children. Exteriorly-contracted summerheat internally accumulated in the qi-level, transformed into fire, and scurried to the ying-level, finally resulting in blazing of both qi and ying. The symptoms of high fever, vexation, sweating, and headache were caused by the intense qi-level heat, and the symptoms of neck rigidity and convulsion in the four limbs resulted from ying-yin deterioration, stirring of wind due to yin deficiency, and failure to nourish the sinews. The symptoms of coma and sleep talking result from the intense pathogenic heat scorching body fluids with phlegm clouding the seven orifices. The reddish-yellow urine and the dry yellow tongue coating were brought on by summerheat damaging the fluids. The red tongue and rapid wiry pulse were manifestations of summerheat transforming into fire with subsequent damage to ying-yin.

Diagnosis Epidemic encephalitis B due to epidemic summerheat-warmth with pathogens blazing in both qi and ying

Clinical Treatment

Principles: Clear qi-level heat, cool the blood, drain fire, resolve toxins Formula: Modified Qīng Wēn Bài Dú Yĭn (Epidemic-Clearing ToxinResolving Beverage) and Ān Gōng Niú Huáng Wán (Peaceful Palace Bovine Bezoar Pill) [清瘟败毒饮加减]

Decoct to 250 ml, nasal feed 1/4 of the solution, 4 times daily. Administer 2 decoctions in total. Mix one Ān Gōng Niú Huáng Wán with 1g powdered líng yáng jiăo (Cornu Saigae Tataricae), divide and nasal feed twice. Also use physical methods to abate fever. [Formula Analysis] Shí gāo and zhī mŭ largely clear qi-level heat. Shēng dì, chì sháo, and dān pí resolve ying-level toxins.

Yín huā, huáng lián, băn lán gēn, and zhī zĭ drain fire and resolve toxins. Gōu téng, jiāng cán, and powder of líng yáng jiăo soothe the liver, extinguish wind, and calm fright. Shí chāng pú dissolves phlegm to open the orifices. Ān Gōng Niú Huáng Wán clears heat, resolves toxins, opens the orifices, and arouses the spirit. Prepared Medicines

● Qīng Kāi Líng Zhù Shè Yè (Clear and Disperse Heat Injection): Add 10-20 ml to 100-250 ml water with 10% dextrose for intravenous infusion; administer once daily for acute-stage symptoms.

● Xĭng Năo Jìng Zhù Shè Yè (Effective Brain-Arousing Injection): Add 2-4 ml to 100-250ml water with 10% dextrose for intravenous infusion, administer once or twice daily for acute-stage high fever, vexation, coma, and convulsion.

● Xiăo Ér Líng Yáng Săn (Antelope Horn Powder for Children): 0.3 g for a one year-old, 0.375 g for a child of 2, and 0.5 g for child of 3. Administer 3 times a day for the persistent acute-stage high fever. Children in primary school can take an adult dosage.

● Hŭ Pò Zhèng Jīng Wán (Amber Fright-Settling Pill): One pill, 2 or 3 times daily, decreased dosages for children under age 3. For intense phlegm-heat symptoms, coma, and convulsion. External Therapy: Ān Gōng Niú Huáng Wán (Peaceful Palace Bovine Bezoar Pill), dissolve one pill and 6g of powdered dà huáng (Radix

et Rhizoma Rhei) in 100ml of warm water for retention enema. For symptoms of bowel excess, high fever, coma, and convulsion. Tui Na 1. For high fever and convulsion in the acute stage, suppress fright and extinguish wind: Press tiān tíng (天庭), rén zhōng (DU 26), lăo lóng (⽼龙), duān zhèng (端正), èr rén shàng mă (⼆⼈上马), jīng nìng (精宁) and wēi líng (威灵), pound xiăo tiān xīn (⼩天⼼), and grasp LI 11 (qū chí), GB 21 (jiān jĭng), BL 40 (wĕi zhōng) and BL 60 (kūn lún). Treat 1-2 times daily for 2 days. 2. For a confused spirit in the acute stage, clear heart heat, eliminate phlegm, and open the orifices: Clear xīn jīng (⼼经), fèi jīng (肺经) and gān jīng (肝经), push sān guān (三关) upward, push liù fǔ (六腑) downward, clear tiān hé shuĭ (天河 ⽔), press RN 22 (tiān tū), push BL 10 (tiān zhù) and the spine, and press ST 40 (fēng lóng). 3. For paralysis of the limbs in the recovery stage, relax the sinews, unblock the collaterals, move qi, invigorate the blood, and promote the recovery of limb function:

■ Roll from DU 14 (dà zhuī) downwards along the spine, repeat 5 times.

■ Press and knead BL 18 (gān shù), BL 17 (gé shù), BL 19 (dăn shù), BL 20 (pí shù) and BL 23 (shèn shù) until the arrival of qi.

■ Scrub from top to bottom until there is a strong sensation of heat.

■ Roll the shoulders, press and knead the inner and outer sides of the upper limbs, and press GB 21 (jiān jĭng), SI 11 (tiān zōng), LI 11 (qū chí), LI 10 (shŏu sān lĭ) and LI 4 (hé gŭ).

■ Roll the lower limbs from the buttocks downward to the knees, and press GB 30 (huán tiào), BL 12 (fēng mén), GB 34 (yáng líng quán) and BL 60 (kūn lún).

■ Lastly, shake the shoulder, elbow and wrist joints in turn, twist and extend knuckles, and shake the hip, knee, and ankle joints. Treat once a day, with 7-10 days as one course of treatment. Acupuncture 1. Acute stage Points: DU 20 (băi huì), DU 16 (fēng fŭ), GB 20 (fēng chí), PC 7 (dà líng), SI 3 (hòu xī), KI 1 (yŏng quán), RN 6 (qì hăi). Manipulations: Needle with drainage; retain all needles for 20 min to 4 hrs, accordingly.

● For high fever, add LI 11 (qū chí), DU 14 (dà zhuī) and BL 40 (wĕi zhōng). Bloodlet BL 40 (wĕi zhōng) with a three-edged needle, needle the others with drainage; retain all needles for 20 min.

● For coma, add shí xuān (EX-UE11), yìn táng (EX-HN3) and RN 6 (qì hăi). Bloodlet shí xuān (EX-UE11) and yìn táng (EX-HN3), and treat RN 6 (qì hăi) with moxa-roll sparrow-pecking until consciousness returns.

● For convulsion, add DU 26 (shuĭ gōu), DU 12 (shēn zhù), LI 4 (hé gŭ) and LV 3 (tài chōng). Needle with drainage continuously until convulsion stops. Retain needles for 2 to 4 hrs to prevent recurrence.

● For retention of urine, add RN 4 (guān yuán), RN 2 (qū gŭ) and SP 6 (sān yīn jiāo). RN 4 (guān yuán) is needled to join RN 2 (qū gŭ), manipulated with the drainage method. Needle SP 6 (sān yīn jiāo) with even supplementation and drainage until a feeling of needing to urinate is achieved. The treatment interval is determined according to the severity of disease; treat 2 to 3 times daily for a mild pattern, and every 6 hours for the severe pattern. In principle, the second acupuncture treatment can only be given after the body temperature falls following the first treatment. 2. Recovery or sequelae stages Points: DU 14 (dà zhuī), LI 11 (qū chí), ST 36 (zú sān lĭ), sì shén cōng (EX-HN1), GB 20 (fēng chí). Manipulations: Apply supplementation followed by drainage to relax the sinews, quicken the collaterals, move qi and resolve stagnation, especially in those with flaccidity of the limbs. Scalp Acupuncture Points: Motor area, Chorea Tremor area, Speech area, Sensory area.

● With impeded speech, add DU 15 (yă mén), RN 23 (lián quán) and HT 5 (tōng lĭ).

● With arched-back rigidity, add HT 7 (shén mén), DU 8 (jīn suō), PC 6 (nèi guān), PC 7 (dà líng) and BL 23 (shèn shù).

● With hypertonic muscles and paralyzed limbs, add LI 11 (qū chí) is needled to join HT 3 (shào hăi), GB 34 (yáng líng quán) is needled to join SP 9 (yīn líng quán).

● With yin deficiency and internal heat, add SP 6 (sān yīn jiāo), KI 4 (dà zhōng) and KI 5 (shuĭ quán). For excess patterns, needle with drainage; for deficiency patterns, needle with supplementation. Treat once daily with 7 days as one course of treatment; a second course of treatment may be given after an interval of 2 or 3 days. Second Visit July 22nd After 2 doses of the above formula, BT fell to 39℃, convulsion of the four limbs ceased, the patient had a clearer mind, and the vomiting was reduced. Formula: Modified Qīng Wēn Bài Dú Yĭn (Epidemic-Clearing ToxinResolving Beverage) [清瘟败毒饮加减]

2 doses. Decoct with water to 250 ml, divide into 4 portions, and take orally. Prepared Medicines: Ān Gōng Niú Huáng Wán (Peaceful Palace Bovine Bezoar Pill), taken twice (with no added líng yáng jiăo). Third Visit July 24th BT fell to 38.2℃ with a clear mind, no vomiting, dry mouth, a dry red tongue with a dry yellow coating, and a slightly rapid pulse. Formula: Modified Qīng Wēn Bài Dú Yĭn (Epidemic-Clearing ToxinResolving Beverage) [清瘟败毒饮加减]

4 doses. Decoct with water to 250 ml.

Fourth Visit July 28th BT was 36.8℃ after 4 doses of the modified formula, signs and symptoms included a much improved spirit, increased food intake, soft nape, normal pupil reaction to light, a thin dry yellow tongue coating, and a fine rapid pulse. Formula: Modified Shā Shēng Mài Dōng Tāng (Adenophora and Ophiopogon Decoction) [沙参麦冬汤加减]

6 doses were given; the patient was discharged after a full recovery. As the previous treatment had cleared the pathogenic heat but had failed to fully restore yin-fluids, Modified Shā Shēng Mài Dōng Tāng was given to clear heat and nourish yin, which finally resulted in a complete recovery with no sequelae.

COMMENTARY AND DISCUSSION Epidemic encephalitis B falls into the category of epidemic summer warm diseases. Unlike other epidemic warm diseases, epidemic summer warm diseases do not have typical symptoms in the wei-level, as the pathogens for summer warm diseases are very violent and transmit very quickly. Therefore, Warp and Woof of Warm-Heat Diseases (Wēn Rè Jīng Wĕi) quoted a famous warm disease practitioner, Ye Tian-shi, who said, “Epidemic summer warm diseases start at yangming, and ancient practitioners treated them with Bái Hŭ Tāng (White Tiger Decoction)” (夏 暑发⾃阳明,古⼈以⽩虎汤为主⽅). This means that excessive toxic-heat accumulation is the main mechanism, and that clearing heat and toxins is the key treatment principle. This 8 year-old female client was in an emergent situation, as she presented with coma, delirium, high fever with sweating and frequent convulsion in the four limbs. Therefore, attention should be paid to clearing heat, dispelling toxins, draining fire from the qi- and ying-levels, unblocking the orifices and eliminating internal wind. Therefore, Qīng Wēn Bài Dú Yĭn was administered as a decoction. As we discussed concerning this formula in the chapter on mucocutaneous lymph node syndrome, this prescription is a modification of several other formulas: Bái Hŭ Tāng, Liáng Gé Săn (Diaphragm-Cooling Powder), Huáng Lián Jiĕ Dú Tāng (Coptis Toxin-Resolving Decoction) and Xī Jiăo Dì Huáng Tāng (Rhinoceros Horn and Rehmannia Decoction). It acts to clear heat and toxins from both exterior and interior, and from the qi

and ying levels; Qīng Wēn Bài Dú Yĭn is an essential treatment formula for this case. However, the formula was not strong enough to unblock the orifices and awaken the spirit from coma, delirium and convulsion. Therefore, a famous prepared medicine, Ān Gōng Niú Huáng Wán was added to the treatment. This formula treats coma due to toxic-heat accumulating in the qi- and ying-levels with wind-phlegm blocking the orifices of the heart, consisting of several precious Chinese medicinals such as niú huáng (Calculus Bovis) to clear the heart, open the orifices, awaken the spirit and vaporize phlegm, xī jiăo (Cornu Rhinocerotis) [these days replaced by shuĭ niú jiăo (Cornu Bubali) or líng yáng jiăo (Cornu Saigae Tataricae)] to clear heat, relieve fire toxicity, cool blood and arrest tremor, shè xiāng (Moschus) to open the orifices, calm the spirit and unblock closed disorders, zhēn zhū (Margarita) to sedate the heart and liver and settle tremors, and jīng bō (Native Gold) to calm the heart, dispel toxicity and quiet the spirit. Ān Gōng Niú Huáng Wán, together with Zhì Băo Dān (Supreme Jewel Elixir) and Zĭ Xuĕ Dān (Purple Snow Elixir) are called “Three Treasures” for pediatric emergencies. After two more follow-up visits, all spiritdisorder symptoms had resolved. No specific biomedical therapy is available for epidemic encephalitis B. Supportive and symptomatic treatments for fever reduction and controlling convulsion and others are the mainstay. A vaccine is also available for prevention.

STUDY QUESTIONS 1. What are the major symptoms of pathogens invading wei qi? How can this be distinguished from pathogens blazing in the qi and ying? 2. What are the treatment principles for epidemic encephalitis B? What are the differences in treatment for the acute and the recovery stages? 3. In nursing this disease, to what must one pay attention?

Answers 1. The manifestations of wei qi invasion include sudden onset of fever, slight aversion to wind and cold, or sometimes only fever without aversion to cold, headache, rigid neck and nape, no or little sweating, and thirst with a desire for drinks often accompanied by vomiting and nausea, occasional convulsion, vexation or somnolence, a slightly red tongue with thin white or yellow coating, and a floating or forceful and rapid pulse. This pattern often occurs in the initial stage of the disease with rapid onset. Pathogens blazing in qi and ying means that the wei-level pattern has converted to a qi-and ying-level pattern, or that exuberant heat has directly invaded qi and ying. The clinical manifestations include the three major symptoms of high fever, unconsciousness, and convulsion. 2. The treatment requires clearing heat, eliminating phlegm, opening the orifices, and extinguishing wind. In the acute stage, resolving heat is primary.

For those with the exterior summerheat, treatment is given to clear summerheat and vent the exterior while eliminating dampness with aromatics. For those with inward invasion of the summerheat pathogen, treatment is given to clear heat with bitter-coldness while unblocking the bowels and discharging heat. For those with constrained pathogens transforming into fire and invading ying-blood, treatment is given to clear heat from the ying-level and drain fire with bittercold and salty-cold medicinals. Treatment must also open the orifices, eliminate phlegm, suppress fright, and extinguish wind by combining treatments for phlegm patterns and wind patterns. In the recovery or sequelae stage, treatment is given to reinforce upright qi and dispel pathogens. For those with incomplete pathogen elimination, or in whom the deficiency-heat fails to abate, treatment is given to nourish yin and clear heat or harmonize ying and wei. For those with the phlegm clouding the orifices and a confused spirit, treatment is given to eliminate phlegm and open the orifices or direct the turbid downward and revive the mind. For those with internal wind harassing and useless limbs, treatment is given to boost qi, invigorate blood, dispel or track wind, unblock the collaterals, and relax the sinews. 3. In terms of nursing, several things should be keep in mind:

First, the patient’s room must be cool and well-ventilated; it is best to keep the temperature at 30℃ or less. Keep the room quiet and have a firstaid kit on-hand including oxygen and a sputum suction apparatus. Second, keep a close eye on the patient’s BT, breath, pulse, blood pressure, complexion, pupils and changes in spirit so as to give timely treatment when necessary. Third, keep the patient’s five sensory organs and skin clean; clean the eyes, nose, and oral cavity with saline or a 1∶5000 furacillin solution. Fourth, for the patient with coma, keep turning over the body, patting the back and changing the body posture to avoid respiratory obstruction and bedsores. Fifth, feed the acute-stage patient with liquid foods and sufficient water; feed nasally when necessary. Gradually increase the intake of nutritious food during recovery. Finally, give the patient passive functional exercise as soon as possible in the recovery stage to accelerate recovery of movement function in the limbs.

CASE SCENARIOS The following cases present variations of this condition. After familiarizing yourself with the possible common pattern presentations and appropriate formulas for treatment, use the following exercises to test your overall understanding of the condition. 1. A child, age 6, has had fever for 4 hours. Other signs and symptoms include headache, vexation, nausea, and somnolence. Physical examination reveals an acute disease with BT 40.5℃, rigid neck and nape, a red tongue with a thin white coating, and a rapid floating pulse. What is the presenting pattern? A. Pathogens invading wei qi B. Pathogens blazing in the qi and ying C. Pathogens invading ying-blood D. Phlegm clouding the seven orifices E. Internal wind harassing 2. A child, age 7, has a 38.8℃ fever for 2 days with slight aversion to wind and cold, headache, no sweating, somnolence, vomiting, rigid neck and nape, a slightly red tongue with a thin white coating, rapid floating pulses, increased cerebrospinal fluid pressure, WBC 400 × 106/L in cerebrospinal fluid, and positive specific antibodies for epidemic encephalitis B. What is the best treatment formula?

A. Qīng Wēn Bài Dú Yĭn (Epidemic-Clearing Toxin-Resolving Beverage) B. Xīn Jiā Xiāng Rú Yĭn (Newly-Supplemented Mosla Beverage) C. Xiāng Rú Yĭn (Mosla Beverage) D. Yín Qiào Săn (Lonicera and Forsythia Powder) E. Bái Hŭ Tāng (White Tiger Decoction) 3. A child, age 5, has had a persistent fever for 5 days. Other signs and symptoms include constant vomiting, thirst with a desire for drinks, rigid neck and nape, coma, repeated convulsions, phlegm rales in the throat, inhibited breathing, dry stools and scanty reddish urine. He has a red tongue with a yellow coating, and a rapid forceful pulse. Physical examination found BT 39.8℃ and positive Kernig’s and Brudzinski’s signs. What is the best treatment formula? A. Xiăo Chéng Qì Tāng (Minor Purgative Decoction) B. Liáng Yíng Qīng Qì Tāng (Construction-Cooling Qi-Clearing Decoction) C. Qīng Wēn Bài Dú Yĭn (Epidemic-Clearing Toxin-Resolving Beverage) D. Ān Gōng Niú Huáng Wán (Peaceful Palace Bovine Bezoar Pill) E. Xī Jiăo Dì Huáng Tāng (Rhinoceros Horn and Rehmannia Decoction) and Zēng Yè Tāng (Humor-Increasing Decoction) 4. A child, age 2, after having fever for 24 hours, suddenly went into a coma with a rigid neck and nape, convulsion in the four limbs, phlegm gurgling in the throat, dry bound stools, scanty reddish urine, a red tongue with a dry yellow coating, a rapid forceful pulse and a BT of 40.2℃.

What is the best treatment formula? A. Xīn Jiā Xiāng Rú Yĭn (Newly-Supplemented Mosla Beverage) B. Xī Jiăo Dì Huáng Tāng (Rhinoceros Horn and Rehmannia Decoction) C. Qīng Wēn Bài Dú Yĭn (Epidemic-Clearing Toxin-Resolving Beverage) D. Dí Tán Tāng (Phlegm-Flushing Decoction) E. Zhĭ Jìng Săn (Spasm-Relieving Powder) 5. A child, age 4, has had a persistent fever for one week. Presenting signs and symptoms include an unstable fever, coma, upward staring eyes, repeated convulsions, reversal cold of the limbs, scorching heat in the chest and abdomen, fecal and urinary incontinence, a purple-crimson tongue with little saliva and a thin white coating, and a deep rapid and thready pulse. What is the presenting pattern? A. Yin deficiency with internal heat B. Disharmony of ying and wei C. Pathogens invading ying and blood D. Pathogens blazing in qi and ying E. Phlegm-heat harassing interiorly 6. A child, age 7, has had a persistent fever for 8 days. Present signs and symptoms include an unstable fever that is slight in the morning but severe in the evening, coma, upward staring eyes, occasional convulsions, cold extremities, fecal and urinary incontinence, a purple or crimson tongue with little saliva and a deep rapid thready pulse.

What is the best treatment formula? A. Bái Hŭ Tāng (White Tiger Decoction) B. Lóng Dăn Xiè Gān Tāng (Gentian Liver-Draining Decoction) C. Qīng Wēn Bài Dú Yĭn (Epidemic-Clearing Toxin-Resolving Beverage) D. Qīng Hāo Biē Jiă Tāng (Sweet Wormwood and Turtle Shell Decoction) and Qīng Luò Yĭn (Channel-Clearing Beverage) E. Xī Jiăo Dì Huáng Tāng (Rhinoceros Horn and Rehmannia Decoction) and Zēng Yè Tāng (Humor-Increasing Decoction) 7. A child, age 4, was treated for 15 days due to a summer fever. Present symptoms include an irregular low fever, vexation and agitation caused by deficiency, occasional fright, tidal reddening of the cheeks, heat in the hearts of the palms and soles, thirst with a desire to drink, dry stools, scanty urine, a red tongue with little coating, and a rapid thready pulse. What is the best treatment formula? A. Dà Dìng Fēng Zhū (Major Wind-Stabilizing Pill ) B. Qīng Shŭ Yì Qì Tāng (Summerheat-Clearing Qi-Boosting Decoction) C. Yăng Yīn Qīng Fèi Tāng (Yin-Nourishing Lung-Clearing Decoction) D. Shā Shēn Mài Dōng Tāng (Adenophora and Ophiopogon Decoction) E. Qīng Hāo Biē Jiă Tāng (Sweet Wormwood and Turtle Shell Decoction) and Qīng Luò Yĭn (Channel-Clearing Beverage)

8. A child, age 5, was treated for 30 days due to a summer fever. Present signs and symptoms include an irregular low fever, a pale complexion, mental fatigue, a lack of strength, profuse sweating, cold limbs, thin unformed stools, profuse clear urine, a pale tongue with a white coating, and a rapid thready forceless pulse. What is the presenting pattern? A. Qi deficiency fever B. Disharmony of ying and wei C. Retention of summerheat D. Spleen-kidney yang deficiency E. Lung-spleen qi deficiency 9. A child, age 5, had his fever abated after having a persistent fever for 2 weeks. Present signs and symptoms include an unstable fever, a pale complexion, mental fatigue, a lack of strength, profuse sweating with lack of warmth, cold limbs, thin stools, profuse clear urine, an enlarged pale tongue with a white coating, and a rapid thready forceless pulse. What is the best treatment formula? A. Bā Zhēn Tāng (Eight-Gem Decoction) B. Guī Pí Tāng (Spleen-Restoring Decoction) C. Guì Zhī Tāng (Cinnamon Twig Decoction) D. Guì Zhī Jiā Guì Tāng (Cinnamon Twig Decoction with Extra Cinnamon) E. Huáng Qí Guì Zhī Wŭ Wù Tāng (Astragalus and Cinnamon Twig Five Substances Decoction)

10. A child, age 3, had his temperature brought down after 2 weeks of fever. Present signs and symptoms include unconsciousness and abnormal buzzing sounds in the throat. There is a red tongue with a yellow coating, and a floating pulse. What is the correct principle of treatment? A. Track wind, unblock the collaterals, nourish the blood, relax the sinews B. Clear phlegm, drain fire, calm the spirit, stabilize the mind C. Open the orifices, direct the turbid downwards, eliminate phlegm, clear heart-heat D. Clear qi, cool ying, drain fire, clear phlegm E. Suppress fright, open the orifices, extinguish wind, dissolve phlegm 11. A child, age 6, was diagnosed with a summer fever. Present signs and symptoms include no fever, unconsciousness, impeded speech, abnormal buzzing sounds in the throat, drooling from the corners of the mouth, and an enlarged tongue with thick greasy coating. What is the best treatment formula? A. Chāng Pú Yù Jīn Tāng (Acorus and Curcuma Decoction) B. Huáng Lián Wēn Dăn Tāng (Coptis Gallbladder-Warming Decoction) C. Méng Shí Gŭn Tán Wán (Chlorite Phlegm-Removing Pill) D. Dí Tán Tāng (Phlegm-Flushing Decoction) E. Wēn Dăn Tāng (Gallbladder-Warming Decoction)

12. A child, age 6, had his temperature brought down after 2 weeks of high fever. Present signs and symptoms include continual vexation and agitation, unconsciousness, dry throat, thirst with a desire to drink, a red tongue with greasy yellow coating, and a rapid forceful pulse. What is the best treatment formula? A. Huáng Lián Wēn Dăn Tāng (Coptis Gallbladder-Warming Decoction) B. Lóng Dăn Xiè Gān Tāng (Gentian Liver-Draining Decoction) C. Dà Dìng Fēng Zhū (Major Wind-Stabilizing Pill) D. Dí Tán Tāng (Phlegm-Flushing Decoction) E. Qīng Gōng Tāng (Palace-Clearing Decoction) 13. A child, age 13, after his temperature was brought down from a summer fever, has rigid and paralyzed limbs, flaccid and useless muscles, mental fatigue, a pale tongue with a thin white coating and a weak thready pulse. What is the correct principle of treatment? A. Boost qi, nourish blood, track wind, unblock the collaterals B. Boost qi, nourish blood, invigorate blood, unblock the collaterals C. Track wind, unblock the collaterals, nourish blood, relax the sinews D. Nourish blood, relax the sinews, invigorate blood, unblock the collaterals E. Boost qi, nourish blood, enrich yin, dispel wind 14. A child, age 6, had a summer fever 2 months ago. Present symptoms include rigid and paralyzed limbs, stiff joints, occasional arch-

like spasticity, a thin white tongue coating, and a thready pulse. What is the best treatment formula? A. Bŭ Yáng Huán Wŭ Tāng (Yang-Supplementing Five-Returning Decoction) B. Huà Tán Tōng Luò Tāng (Phlegm-Transforming CollateralUnblocking Decoction) C. Xuān Bì Dá Jīng Tāng (Impediment-Diffusing ChannelOutthrusting Decoction) D. Cuō Fēng Săn (Pursed Mouth Powder) E. Zhĭ Jìng Săn (Spasm-Relieving Powder)

Answers 1. A 2. B 3. C 4. C 5. C 6. E 7. E 8. B 9. E 10. C

11. D 12. B 13. B 14. E

Chapter 36 Infectious Mononucleosis Infectious mononucleosis (IM), is an acute, self-limiting, lymphoproliferative infectious disease caused by seasonal pathogens (EBV). It is characterized by irregular fever, sore throat, lymphadenectasis, hepatomegaly, splenomegaly, a significant increase of lymphocytes in the blood, and the growth of atypical lymphocytes. The seasonal pathogen may damage the heart, lung, liver and kidney. Children are not able to tolerate the invasion of epidemic heat-toxins because their viscera and bowels are delicate, and their wei-exterior is not secure. Meanwhile, as the child constitution is “pure yang”, the externallycontracted pathogens tend to transform into heat and fire. The epidemic pathogens fill the exterior and interior, scurry into the ying-blood and sink into the heart and liver. Thus, the disease develops with the main pathological manifestations of heat, toxins, phlegm and stasis. Common clinical patterns include pathogens constrained in the lungwei, heat-toxins blazing, phlegm-heat blocking the lung, phlegm-heat obstructing the channels, liver-gallbladder heat and stasis, static toxins obstructing collaterals, and upright qi deficiency with lingering pathogens.

CLINICAL ESSENTIALS The attack, development and recovery of this disease exhibit the features of a warm disease with the rules of transmission and change of a wei-qi-ying-blood four-level pattern. In the beginning period, epidemic pathogens attack externally, and the pathogens stay in the qi level. Therefore, it should be treated by diffusing and venting the pathogens with acrid-cool medicinals. It must be noticed that the wei-level pattern lasts for only a very short time or does not appear at all in this period. Once the disease attacks, the interior-heat pattern appears immediately. During the most severe phase, the epidemic toxins transform into fire and heat. And according to Wu You-ke’s theory in the Treatise on WarmHeat Pestilence (Wēn Yì Lùn), when pathogens invade from the nose and mouth, they do not stay interiorly in the viscera and bowels or exteriorly in the channels and collaterals, but rather stay inside the area bilateral to the spine and close to the diaphragm, a place that is considered as halfinterior/half-exterior. At this point, the yang heat is so intense that there is no exit for the pathogens; therefore, sweat promotion, emetic therapy and purgative methods must be applied in order to “open the door” and expel the pathogens because, as Wu’s principles suggest, intruding pathogens should be dispelled as early as possible. Until the pathogens are dispelled, the disease cannot be cured.

● Generally, IM should be treated by clearing heat and resolving toxins. Clearing heat without dispelling the pathogens at the same time is like detaining enemies behind a closed door. The treatment should therefore be accompanied by venting the pathogens, thus providing an exit for the

pathogens while clearing heat at the same time. Dispelling the pathogens will also help bring the pathogenic internal heat under control.

● Hasty application of bitter-cold medicinals is not suggested; instead, cold medicinals and warm medicinals should be applied in combination. The cold act to reduce heat and small amounts of mild, acrid-warm medicinals act to open and unblock. The warm medicinals will therefore assist the cold medicinals and simultaneously help to prevent possible negative effects that the cold medicinals may bring due to their being too cold. For example, jīng jiè (Herba Schizonepetae), fáng fēng (Radix Saposhnikoviae) and gé gēn (Radix Puerariae Lobatae) are added to open the interstitial space while dredging the skin and body hair to help dispel the pathogens.

● During the recovery period, because the long-lingering pathogenic heat has consumed qi and damaged yin, it is more of a deficiency pattern than a pattern of pathogenic excess with pathogens lodged in the yin-level. Therefore, upright qi must be reinforced and the root fortified while the remaining pathogens are dispelled. IM should be treated on the basis of wei-qi-ying-blood pattern identification. Prof. Yuan Mei-feng, when considering the pathogenic features, the disease course, and the variety of symptoms, believes that this disease involves complicated and varied patterns that develop quickly. Therefore, treatment should not be confined to the adoption of any one particular formula or medicinal. On the contrary, a modified formula should be adopted for the specific pattern to prevent complications, the most common of which include palpitation, jaundice, pneumonia with panting and cough, and the inward invasion of pathogens into jueyin. Close attention must be paid to the pathogenesis of the disease, and the treatment

should also be modified according to different patterns. Similarly, an integrative treatment approach is suggested for those who are critically ill.

CASE STUDY Female, age 3. Initial Visit: July 8th, 2009 Chief Complaint: Persistent high fever for more than 10 days. History: The child began to suffer from a fever and sore throat some 10 days ago. She was given amoxicillin, Kàng Bìng Dú Kŏu Fú Yè (Antiviral Oral Liquid) and antipyretics, but her symptoms were not relieved. She was then brought to the hospital. Signs and Symptoms: A persistent fever, cough, intense congestion of phlegm, uneasiness, sore throat, lack of sweating, yellow urine, a red tongue with a yellow greasy coating, and a rapid pulse. Past History: A weak constitution with susceptibility to colds. Physical Examination: BT 39.6℃, full consciousness, superficial lymphadenectasis found all over the body, particularly at both sides of the neck where 5 to 6 movable broad bean-sized lymphonodi were found without tenderness, a congested pharynx, swelling of the tonsils on both sides of the throat (Level Ⅰ), heavy breath sounds auscultated in both lungs, dry or wet rales not heard, a flat soft abdomen, liver palpable 2.5 cm below the ribs, spleen palpable 1 cm below the ribs, normal nervous system responses. Laboratory Examination: Routine blood test: WBC 20 × 109/L, N 25%, L 50%, M 25%, Hb 105 g/L. Atypical lymphocytes in peripheral blood 15%.

Pattern Differentiation The patient was susceptible to infection as a result of a weak constitution. External pathogens invaded the lung and stomach first, and lung-wei was attacked; the pathogens accumulated in the throat and transformed to heat. When the heat steamed and burned the throat, such symptoms as fever, sore throat, red tonsils and coughing with large expectoration of phlegm developed. When heat-toxins were intense, the heat condensed fluids to phlegm. Stasis of phlegm and fire filled the viscera and bowels and flowed in the channels and collaterals, causing lymphadenectasis to develop. When the heat-toxins burned, qi and blood also stagnated, leading to hepatomegaly and splenomegaly. When phlegmheat blocked the lung, coughing and phlegm-panting developed. The red tongue with a greasy yellow coating and the rapid pulse suggested intense heat in the lung-wei with retention of turbid phlegm. The main presenting pattern is phlegm-heat blocking the lung.

Diagnosis Infectious mononucleosis due to phlegm-heat blocking the lung

Clinical Treatment Principles: Clear heat, resolve toxins, diffuse the lung, eliminate phlegm Formula: Modified Má Xìng Shí Gān Tāng (Ephedra, Apricot Kernel, Gypsum and Licorice Decoction) and Qīng Níng Săn (Clearing and

Calming Powder) [麻杏⽯⽢汤合清宁散加减]

6 doses [Formula Analysis] Má huáng and shēng shí gāo diffuse the lung and clear heat. Huáng qín and yú xīng căo clear heat and resolve toxins. Tíng lì zĭ, sāng bái pí and xìng rén drain the lung and clear phlegm. Tiān huā fĕn and zhī zĭ clear constrained qi-level heat. Zhú lì clears heat and eliminates phlegm. Xià kū căo and pú gōng yīng clear heat and dissipate masses. Mă bó and jiāng cán clear heat, resolve toxins and soothe the throat.

Gān căo clears heat, resolves toxins and harmonizes all the formula medicinals. Prepared Medicines

● Kàng Bìng Dú Kē Lì (Antiviral Granules). 5-10g, 3 times daily. ● Lìu Shén Wán (Six Spirits Pill). One pill each time for patients under age year, 2-4 pills for patients 1-3 years old, 5-6 pills for ages 4-8, and 8 pills ages 9-15. Take twice a day for sore throat.

● Chái Hú Zhù Shè Yè (Bupleurum Injection). 1 to 2 ml, 3 times daily, for fever reduction. External Therapy

● Shuāng Liào Hóu Fēng Săn (Superior Sore Throat Powder), Xī Lèi Săn (Tin-Like Powder), or Bīng Péng Săn (Borneol and Borax Powder). Spray an appropriate amount of either powder into the throat, 3 times each day, for a swollen and sore throat.

● Fresh pú gōng yīng (Herba Taraxaci). Mash an appropriate amount and apply to the affected area.

● Sān Huáng Èr Xiāng Săn (Three Yellows and Two Fragrance Powder), composed of 30g each of huáng băi (Cortex Phellodendri Chinensis), huáng lián (Rhizoma Coptidis) and shēng dà huáng (Radix et Rhizoma Rhei), and 15g each of rŭ xiāng (Olibanum) and mò yào (Myrrha). Grind the above medicinals into powder. For the first application, mix the powder with strong tea and apply the mixture onto the swollen lymph nodes, then remove it after it becomes dry. Thereafter, mix the powder with sesame oil and apply the mixture in the same manner. This therapy is used for lymphadenectasis, and should be applied 2 times each day until the swellings disappear.

Acupuncture Method Ⅰ Points: DU 14 (dà zhuī), LI 11 (qū chí), LI 4 (hé gŭ), LU 11 (shào shāng) Manipulations: Needle with draining method with strong stimulation; retain all needles for 30 min.

● For patients with persistent high fever and heat entering ying-blood, needle the twelve jĭng-well points from the regular channels: LU 11 (shào shāng), LI 1 (shāng yáng), PC 9 (zhōng chōng), SJ 1 (guān chōng), HT 9 (shào chōng), SI 1 (shào zé), ST 45 (lì duì), SP 1 (yĭn bái), BL 67 (zhì yīn), KI 1 (yŏng quán), GB 44 (zú qiào yīn), LV 1 (dà dūn) and shí xuān (EX-UE 11), and blood-letting DU 14 (dà zhuī).

● For patients with dysphoria, add yìn táng (EX-HN3) and HT 7 (shén mén). Method Ⅱ Points: LU 5 (chĭ zé), LI 4 (hé gŭ), ST 43 (xiàn gŭ) Manipulations: Needle with drainage to clear heat and soothe the throat.

● For patients with intense lung-stomach heat, apply blood-letting at LU 11 (shào shāng) and SJ 1 (guān chōng). Method Ⅲ Point Selection: LV 14 (qī mén), LV 3 (tài chōng), GB 34 (yáng líng quán), ST 36 (zú sān lĭ)

Manipulations: Needle with drainage; for distinct swellings below the flanks to soothe the liver, rectify the spleen, dissolve stasis and dissipate masses. Method Ⅳ Points: SJ 17 (yì fēng), SJ 10 (tiān jĭng), GB 41 (zú lín qì), LI 10 (shŏu sān lĭ), ST 9 (rén yíng) Manipulations: Needle with drainage; select these points for a swollen neck with multiple nodules formed by phlegm and stagnation. Ear Acupuncture Points: Lung (fèi), pharynx larynx (yān hóu), neck (jĭng). Manipulations: Apply rotation with moderate or strong stimulation; retain the needles for 20 to 30 min. Apply once daily for 3-5 days. Tui Na Clear fèi jīng (肺经), clear tiān hé shuĭ (天河⽔), open tiān mén (天 门), activate tài yáng (太阳), push kăn gōng (坎宫), knead ěr hòu gāo gǔ (⽿后⾼骨), and push liù fǔ (六腑) downward.

● For swollen liver and spleen, also clear gān jīng (肝经) and

separating-pushing abdomen (fēn tuī fù yīn yáng, 分推腹阴阳).

● For sore throat, perform pinching at LU 11 (shào shāng) and kneading at jīn jīn (EX-HN12) and yù yè (EX-HN13).

● For high fever and unconsciousness, perform pinching at the philtrum, shí wáng (Ten Kings) and LU 11 (shào shāng) and grasping at BL 40 (wĕi zhōng). Second Visit

July 11th After taking 3 doses, the temperature was reduced and cough was relieved, but a low fever persisted. After an additional 3 doses, her cervical lymph nodes became smaller. The liver and spleen were still palpable 1 cm below the ribs. Laboratory Examination: WBC 9.5 × 109/L, N 20%, L 50%, M 30%, Peripheral atypical lymphocytes 5%. Principles: Clear heat, resolve toxins, soften hard masses, dissolve phlegm, invigorate blood, dissolve stasis, assisted by enriching yin and clearing heat. Formula: Modified Qīng Gān Huà Tán Tāng (Liver-Heat Clearing Phlegm-Resolving Decoction) and Qīng Hāo Biē Jiă Tāng (Sweet Wormwood and Turtle Shell Decoction) [清肝化痰汤合青蒿鳖甲汤加减]

5 doses [Formula Analysis] Qīng hāo discharges heat and vent heat from the collaterals to bring out the pathogens. Bái wēi and biē jiă enrich yin and reduce heat. Jīn yín huā, huáng qín, jiāng cán and xuán shēn clear heat and resolve toxins. Shān cí gū and táo rén dissipate masses and dissolve stasis. Mŭ dān pí cools the blood and invigorates blood. Dāng guī invigorates and supplements blood. Dān shēn invigorates blood and moves qi. Hăi zăo and kūn bù soften hardness and dissolve phlegm. After 5 doses, the fever was eliminated and her lymph nodes became even smaller. The axillary and inguinal lymph nodes disappeared, and only 4-5 bean-sized cervical lymph nodes remained. Further modified decoctions of the above formula were prescribed. Third Visit August 12th The lymph nodes have disappeared, a routine blood test showed a normal hemogram, and her liver and spleen were no longer palpable. The patient was fully recovered.

COMMENTARY AND DISCUSSION This case has all the TCM pathological characteristics for infectious mononucleosis, including heat, toxin, phlegm, and hardness with stasis. Heat was manifested by a persistent fever for over 10 days. Toxins were manifested by the congested pharynx and swelling of the tonsils on both sides of the throat. Phlegm was manifested by cough and intense congestion of phlegm. Hardness and stasis were manifested by superficial lymphadenectasis found all over the body, particularly at both sides of the neck, and by hepatomegaly and splenomegaly. Although the four pathological characteristics were combined, the primary and secondary treatments were still very clear. At the initial visit, attention was paid to clearing heat, expelling toxins and eliminating phlegm, thus a modification Má Xìng Shí Gān Tāng and Qīng Níng Săn was applied. Má Xìng Shí Gān Tāng is from the Treatise on Cold Damage (Shāng Hán Lùn), and is an essential formula for cough and wheezing due to heatevil accumulating in the lung with unreleased external patterns. It was initially applied for taiyang disease for when wind-cold had transformed into internal heat with symptoms of wheezing and sweating. It is commonly applied for fever with cough or wheezing, thirst, sweating or no sweating, and a rapid pulse due to excessive heat accumulated in the lung. The excessive heat could be transformed from wind-cold, wind-heat or from a condition of internal heat with external cold, and so on. It is also applied for measles with irritability, restlessness, cough and wheezing.

Qīng Níng Săn is from the Straight Direction of Pediatric Formulas (Zhí Zhĭ Xiăo Ér Fāng). It contains sāng bái pí, tíng lì zĭ, fú líng (Poria), chē qián zĭ (Semen Plantaginis), zhī zĭ and gān căo and is used for cough, convulsions, and infant spasm and seizure due to accumulation of heat in the lung. In this case, other medicinals such as huáng qín, xià kū căo, yú xīng căo, zhú lì and pú gōng yīng were added to increase the effects of clearing heat, expelling toxins and eliminating phlegm. As extreme heat may cause convulsion and seizure, jiāng cán and pú gōng yīng were added to dispel internal wind and prevent convulsion. 3 days later, the child went back for a follow-up visit. The toxic-heat and phlegm were almost gone, and what remained was enlargement of the spleen and liver with lymph node swelling. At this point the focus of treatment was on promoting blood circulation, removing blood stasis, softening hardness and scattering nodules. Several medicinals were added for this purpose, including dāng guī, táo rén and dān shēn for blood stasis, and biē jiă, shān cí gū, kūn bù, hăi zăo and xuán shēn for the nodules and swelling. Softening hardness and scattering nodules takes a longer time, which is why they were treated with a secondary application of the formula.

STUDY QUESTIONS 1. How can infectious mononucleosis be prevented? 2. What are the treatment principles for this disease? 3. What is the prognosis for infectious mononucleosis?

Answers 1. Infectious mononucleosis is a systemic immunological disease caused by the seasonal pathogen EBV. EBV is transmitted through saliva, exposure to contaminated substances or occasionally through blood transfusion. For patients with normal immunities, the disease is most often self-limiting. However, when children with immunodeficiency are infected, EBV will grow and spread all over the body and threaten their lives. TCM doctors believe that when upright qi is retained in the interior, pathogens will not invade. On the basis of pattern differentiation, patients should be treated in different ways to boost qi, supplement the lung, fortify the spleen and consolidate the kidney. For patients susceptible to infection, the general prevention and treatment principle is to reinforce upright qi and consolidate the root. Chinese prepared medicines such as Shēn Mài Yè (Ginseng and Ophiopogon Liquid), Shēng Mài Yè (Pulse-Engendering Liquid) and Tóng Kāng Piàn (Children’s Health Pill) are effective for improving immunity. 2. The seasonal pathogen EBV is the main pathogenic factor, heattoxins and phlegm-stasis are the basic pathologic features, and clearing heat, resolving toxins, dissolving stasis and dispelling phlegm are the basic

treatment principles. The disease should be treated according to the different locations of the pathogens. When the pathogens stay in the weilevel, it should be treated by releasing the exterior with acrid-cool medicinals. When the pathogens stay in the qi-level, it should be treated by clearing qi and draining heat. When the pathogens stay in the ying-blood level, however, it should be treated by clearing heat from the ying level and cooling the blood. In the later stage of the disease, qi and yin are consumed and damaged. Therefore, it should then be treated by boosting qi, nourishing yin and dispelling the remaining pathogens. When accompanied by pathogenic dampness, aromatic medicinals should be used to eliminate dampness, or the disease should be treated by clearing heat, draining dampness, unblocking the collaterals and venting pathogens. 3. The disease usually has favorable prognosis. It normally lasts 1 to 2 weeks, but it may recur. Some of the symptoms including low fever, lymphadenectasis, or weakness and deficiency after the illness may last several weeks, or even several months. Very rarely, some cases may last a few years. The mortality rate for this disease is between 1% and 2%; common causes of the death are rupture of the splenic artery, meningitis, or myocarditis. Once children with immunodeficiency are infected, they may die due to their quickly deteriorating conditions.

CASE SCENARIOS The following cases present variations of this condition. After familiarizing yourself with the possible common pattern presentations and appropriate formulas for treatment, use the following exercises to test your overall understanding of the condition. 1. A patient, age 3, suffers from a high fever with a temperature of 39℃. Other signs and symptoms include uneasiness, thirst, swollen sore throat, a flaming red face and lips, red rashes on the skin, constipation, dark yellow urine, a red tongue with a yellow and dry coating, and a rapid pulse. He is diagnosed with infectious mononucleosis. What is the presenting pattern? A. Pathogens constraining the lung-wei B. Phlegm-heat blocking the lung C. Intense heat toxins D. Heat-stasis in the liver and gallbladder E. Toxin-stasis obstructing the channels 2. A patient, age 8, suffers from a high fever with uneasiness, thirst, a red pharynx, swollen tonsils, aphthous ulcer, halitosis, a flaming red face and lips, lymphadenectasis, constipation, dark yellow urine, a red tongue with a yellow coating, and a rapid pulse. He is diagnosed with infectious mononucleosis. What is the best formula for this case?

A. Yín Qiào Săn (Lonicera and Forsythia Powder) B. Pŭ Jì Xiāo Dú Yĭn (Universal Relief Toxin-Removing Beverage) C. Má Xìng Shí Gān Tāng (Ephedra, Apricot Kernel, Gypsum and Licorice Decoction) and Qīng Níng Săn (Clearing and Calming Powder) D. Qīng Gān Huà Tán Wán (Liver-Clearing Phlegm-Transforming Pill) E. Xī Jiăo Qīng Luò Yĭn (Rhinoceros Horn and Channel-Clearing Beverage) 3. A patient, age 4, suffers form high fever with yellowish skin and eyes, flank tumescence, poor appetite, sickness, hepatomegaly, splenomegaly, constipation, dark yellow urine, a red tongue with a greasy and yellow coating, and a rapid pulse. He is diagnosed with infectious mononucleosis. What is the best formula for this case? A. Yīn Chén Hāo Tāng (Virgate Wormwood Decoction) B. Pŭ Jì Xiāo Dú Yĭn (Universal Relief Toxin-Removing Beverage) C. Má Xìng Shí Gān Tāng (Ephedra, Apricot Kernel, Gypsum and Licorice Decoction) and Qīng Níng Săn (Clearing and Calming Powder) D. Qīng Gān Huà Tán Wán (Liver-Clearing Phlegm-Transforming Pill) E. Xī Jiăo Qīng Luò Yĭn (Rhinoceros Horn and Channel-Clearing Beverage) 4. A patient, age 11, suffers from a fever with sore throat, lymphadenectasis, hepatomegaly, splenomegaly, facial and limb paralysis,

dysphagia, and a red tongue with a greasy thin coating. He is diagnosed with infectious mononucleosis. What is the presenting pattern? A. Pathogens constraining lung-wei B. Intense heat toxins C. Phlegm-heat blocking the lung D. Heat-stasis in the liver and gallbladder E. Toxin-stasis obstructing the channels 5. A patient, age 10, is diagnosed with infectious mononucleosis with such signs and symptoms as a gradually decreasing fever, mental fatigue, lack of strength, sweating with activity, occasional dizziness, a fluctuating low fever, a light red pharynx, loose stools, smaller swollen lymph nodes, a tongue that is light in color with a thin coating, and a weak pulse. What is the best formula for this case? A. Yīn Chén Hāo Tāng (Virgate Wormwood Decoction) B. Zhú Yè Shí Gāo Tāng (Lophatherum and Gypsum Decoction) C. Má Xìng Shí Gān Tāng (Ephedra, Apricot Kernel, Gypsum and Licorice Decoction) D. Qīng Hāo Biē Jiă Tāng (Sweet Wormwood and Turtle Shell Decoction) E. Xī Jiăo Qīng Luò Yĭn (Rhinoceros Horn Channel-Clearing Beverage) 6. A patient, age 8, is diagnosed with infectious mononucleosis with such signs and symptoms as persistent fever, slight sweating, a red pharynx,

sore throat, a stuffy nose, cough, headache, body aches, cervical lymphadenopathy, a red tongue tip, a thin yellow tongue coating, and a floating rapid pulse. What is the best formula for this case? A. Yīn Chén Hāo Tāng (Virgate Wormwood Decoction) B. Pŭ Jì Xiāo Dú Yĭn (Universal Relief Toxin-Removing Beverage) C. Zhú Yè Shí Gāo Tāng (Lophatherum and Gypsum Decoction) D. Qīng Gān Huà Tán Wán (Liver-Clearing Phlegm-Transforming Pill) E. Xī Jiăo Qīng Luò Yĭn (Rhinoceros Horn Channel-Clearing Beverage) 7. A patient, age 7, suffers from a persistent fever of an uncertain origin. His cervical, axillary and inguinal lymph nodes are swollen, especially the cervical lymph nodes. Other signs and symptoms include splenomegaly, a red tongue with a greasy yellow coating, and a strong and rapid pulse. What is the best formula for this case? A. Pŭ Jì Xiāo Dú Yĭn (Universal Relief Toxin-Removing Beverage) B. Zhú Yè Shí Gāo Tāng (Lophatherum and Gypsum Decoction) C. Dài Gé Săn (Indigo and Clamshell Powder) and Qīng Gān Huà Tán Wán (Liver-Clearing and Phlegm-Transforming Pill) D. Qīng Hāo Biē Jiă Tāng (Sweet Wormwood and Turtle Shell Decoction)

E. Xī Jiăo Qīng Luò Yĭn (Rhinoceros Horn Channel-Clearing Beverage)

Answers 1. C 2. B 3. A 4. E 5. B 6. A 7. C

Chapter 37 Pertussis Pertussis is an acute infectious respiratory disease caused by external contraction of seasonal pertussis pathogens (bordetella pertussis), manifested clinically by paroxysmal spastic cough with cock-crow-like breaths at the end. The pathogenesis of this disease is the external contraction of seasonal pertussis pathogens, which invade the lung system. The pathogens and phlegm combine and accumulate in the air passage, causing failure of the lungs to purify and descend, and lung qi counterflow. Pattern differentiation of pertussis should be conducted in three periods respectively, the incipient period, the spastic cough period and the recovery period. In the incipient period, the pathogens invade the lung-wei. Therefore, patterns of wind-cold and wind-heat should be differentiated. In the spastic cough period, phlegm obstructs the lung collaterals. Phlegm-heat and turbid phlegm patterns should therefore be differentiated. In the recovery period, the pathogens have been dispelled and upright qi damaged. Yin deficiency and qi deficiency patterns should therefore be differentiated. The disease is located in the lung, but it may also invade the heart and liver in severe cases. Common clinical patterns include pathogens invading the lung-wei, phlegm-fire obstructing the lung, damage to qi and yin.

CLINICAL ESSENTIALS The main pathogenesis of this disease involves combined phlegm and qi obstructing the airway, causing the counterflow of lung qi. The disease should be treated by eliminating phlegm, clearing fire, draining the lung and directing counterflow qi downward.

● In the incipient period, it should be treated by warming the lung, dissipating cold and diffusing the lung or by scattering wind, clearing heat and diffusing the lung.

● In the spastic cough period, it should be treated by clearing phlegm, directing qi downward, draining the lung and clearing heat.

● In the recovery period, it should be treated by nourishing yin, moistening the lung, boosting qi and fortifying the spleen. The main symptom of this disease is the persistent choking cough. Hasty application of those medicinals that arrest and astringe is not recommended, however, in case of pathogenic lingering. Because the phlegm-heat pattern is most common in the spastic cough period, early application of those medicinals that nourish yin and moisten the lung is not recommended; if the phlegm-heat has not yet been cleared, the course of disease may become prolonged and treatment will become more difficult. Dr. Yang Yi-jie treats pertussis by clearing lung heat and relieving cough with a modification of Xiè Bái Săn (White-Draining Powder). Xìng rén (Semen Armeniacae Amarum) and jié gĕng (Radix Platycodonis) are added to diffuse the lung. Guā lóu pí (Pericarpium Trichosanthis) and zĭ wăn (Radix et Rhizoma Asteris) are added to dissolve phlegm. Bái qián

(Rhizoma et Radix Cynanchi Stauntonii) is added to direct qi downward and relieve cough. Hăi fú shí (Pumex) is additionally added to clear lung heat and dissolve the long accumulated phlegm. This modification has been shown quite effective for spastic cough in the incipient period. Dr. Zhang Xiao-e holds that pertussis can also be treated effectively with daily acupuncture treatment, as follows: Select LU 5 (chĭ zé), LU 8 (jīng qú) and LU 6 (kŏng zuì) as the main points.

● With internal heat, add LI 11 (qū chí) and LI 4 (hé gŭ). ● With epistaxis, add LI 6 (piān lì) and LU 11 (shào shāng). ● With vomiting, add LU 4 (xiá bái).

CASE STUDY Male, age 4. Initial Visit: May 18th, 2007 Chief Complaint: Paroxysmal cough for 10 days. History: The patient began to cough constantly in early May, and the cough became gradually worse. The paroxysmal cough attacked him at 30 min intervals, which usually lasted 10 min. When coughing severely, he would vomit. He was given Western medicines orally, but the symptoms were not relieved. Signs and Symptoms: A constant and spastic cough, particularly serious in the evening. Every time he began to cough, he shed tears and his nose ran. Yellow sticky phlegm was expectorated, and cock-crow-like echoes were heard in his throat. His cough was not relieved until he expectorated the phlegm and the food he had taken earlier. Other signs and symptoms included thirst, poor appetite, yellow dripping urine, a red tongue with a yellow greasy coating, and a slippery rapid pulse. Past History: No record of vaccination against pertussis. Physical Examination: Low spirits, slightly puffy eyelids, normal heart and lungs. Laboratory Examination: WBC 18.6 × 109/L, N 34%, L 66%.

Pattern Differentiation

Children are susceptible to pathogens due to their relatively insufficient lung, and the patient in this case was unvaccinated. Therefore, the seasonal pertussis pathogens invaded the mouth and nose to attack the lung-wei and lung qi failed to purify, causing cough. The pathogens were then transformed into fire which scorched the lung and condensed the fluids into phlegm, where phlegm and fire combined to obstruct the airway. Lung qi counterflow caused the cough to become convulsive and persistent. The cough was not relieved until he expectorated the accumulated phlegm from his throat. The lung functions to free and regulate the waterways; therefore, when the lung failed to diffuse and purify, the upwelling water caused puffy eyelids. The red tongue with a yellow greasy coating and the slippery rapid pulses are both manifestations of internal phlegm-heat obstruction. The main pathogenesis involves phlegm and qi obstruction with lung qi counterflow. The treatment principles are to eliminate phlegm, clear fire, drain the lung and direct counterflow downward.

Diagnosis Pertussis due to phlegm-heat obstructing the lung

Clinical Treatment Principles: Drain the lung, clear heat, eliminate phlegm and relieve cough Formula: Modified Sāng Bái Pí Tāng (Cortex Mori Decoction) and Tíng Lì Dà Zăo Xiè Fèi Tāng (Lepidium/Descurainiae and Jujube LungDraining Decoction)

[桑⽩⽪汤合葶苈⼤枣泻肺汤加减]

7 doses [Formula Analysis] Sāng bái pí, huáng qín, yú xīng căo and zhè bèi mŭ clear and drain lung heat, dissolve phlegm and relieve cough. Tíng lì zĭ, zĭ sū zĭ and dăn nán xīng direct counterflow downward and dissolve phlegm. Qián hú, xìng rén and băi bù purify the lung and relieve cough. Huáng lián and zhī zĭ drain fire and discharge heat. Jiāng cán and wú gōng relieve spasm and cough. Dà zăo, which is sweet in flavor and warm in property, eases the center and moderates the power of medicinals. Prepared Medicines: Lù Sī Ké Wán (Egret Cough Pills). One pill, 2-3 times daily. Acupuncture

Points: LI 4 (hé gŭ), LU 5 (chĭ zé), BL 13 (fèi shù), LI 11 (qū chí), ST 40 (fēng lóng), PC 6 (nèi guān). Manipulations: Needle with drainage; do not retain the needles. Treat once daily; 5 days constitute one treatment course. Tui Na Circularly push bā guà (八卦) counterclockwise, push liù fǔ (六腑) downward, push pí jīng (脾经), and knead xiăo héng wén (⼩横纹). Treat once daily; 10 days constitute one treatment course. Second Visit After 7 doses of the decoction, the spastic cough was relieved. Remaining signs and symptoms included scant sticky phlegm, spastic coughing, night sweats, thirst, a red tongue with little or no coating, and a thready rapid pulse. Principles: Nourish yin, moisten the lung, boost qi, fortify the spleen Formula: Modified Shā Shēn Mài Dōng Tāng (Adenophora and Ophiopogon Decoction) [沙参麦冬汤加减]

4 doses [Formula Analysis] Shā shēn, mài dōng, yù zhú, shí hú and tiān huā fĕn enrich yin to moisten the lung, and nourish yin to promote fluid production. Lú gēn clears heat and promotes fluid production. Sāng yè, kuăn dōng huā and chuān bèi mŭ clear heat and dissolve phlegm to relieve cough. Gān căo acts to harmonize all formula medicinals. After taking 4 doses of the prescribed formula, the patient was cured.

Prevention and Nursing Good ventilation should be maintained, and the patients should be kept away from cold and such stimulants as smoke, dust, odor and spicy foods. They should have good rest and adequate sleep and avoid stress; stimulating or unstable emotions should be avoided. They should be fed nutritious and digestible foods while avoiding stimulating foods such as fried, spicy, sour or salty foods. They should also have more meals in a day but with less food at each meal so as to avoid vomiting when coughing. Special attention should be paid to infant patients to prevent choking when vomiting.

COMMENTARY AND DISCUSSION The 4-year-old with pertussis had several lung issues including phlegm and fire retention with qi and water rebelling, which have reciprocal causation relationships. Because the child was unvaccinated, the seasonal pertussis pathogen invaded the lung through the mouth and nose, phlegm and fire blocked the chest, and the lung failed to control qi. Thus, rebellious lung qi caused the cough and wheezing. As the lung is the upper source of water, the rebelling qi pushed water to the upper jiao to cause the puffy eyelids. The rebelling lung qi passed through the retention of phlegm and water, which induced the cock-crow-like breaths at the end of each cough. Therefore, attention should be paid to clear fire, remove phlegm, descend lung qi, and expel water. Thus, Sāng Bái Pí Tāng and Tíng Lì Dà Zăo Xiè Fèi Tāng were prescribed at the initial visit. Tíng Lì Dà Zăo Xiè Fèi Tāng is a classic formula from Essentials from the Golden Cabinet (Jīn Guì Yào Lüè). It was designed to treat cough, wheezing, chest fullness, dyspnea and inability to lie down due to water and phlegm blocking the lung. Tíng lì zĭ acts to drain the lung, reduce phlegm, calm wheezing, move water and reduce edema. It is a harsh medicinal that should be monitored during treatment. With dà zăo in the formula, the harsh effect is harmonized and modulated. Li Shi-zheng advocated the application of tíng lì zĭ in The Grand Compendium of Materia Medica (Bĕn Căo Gāng Mù). He stated, “If the accumulation and blockage of water and qi in the lung is severe, then tíng lì zĭ should be applied for this emergent situation; the application of tíng lì zĭ will cease when the water retention is eliminated. Because it is not used for

a long time, one should not worry too much about the side effects” (然肺中 ⽔⽓满急者,非此不能除,但⽔去则⽌,不可过剂尔。 既不久服, 何 致伤⼈). Sāng Bái Pí Tāng is from The Complete Compendium of Ancient and Modern Medical Works (Gŭ Jīn Yī Tŏng Dà Quán), and contains sāng bái pí, bàn xià (Rhizoma Pinelliae), zĭ sū zĭ, xìng rén, zhè bèi mŭ, zhī zĭ, huáng qín and huáng lián. It acts to clear heat and fire, eliminate phlegm and descend lung qi, and was therefore combined with Tíng Lì Dà Zăo Xiè Fèi Tāng. In this case, yú xīng căo and dăn nán xīng were added to drain phlegm-heat. Qián hú and băi bù were added for clearing the lung and stopping cough. Two animal-based medicinals, jiāng cán and wú gōng were added to relieve spasm of airways and stop cough. Because chronic coughing injured the qi and yin of the lung, modified Shā Shēn Mài Dōng Tāng was applied at the follow-up visit to augment qi, nourish yin and moisten the lung.

STUDY QUESTIONS 1. How are patterns of phlegm-heat obstructing the lung in pertussis transmuted? Explain the mechanism. 2. Summarize the key points for the diagnosis of pertussis.

Answers 1. The phlegm-heat obstructing the lung pattern in pertussis is usually found in the spastic cough period. This pattern is easily transmuted when pathogenic toxins and phlegm-heat accumulate inside, or when there is upright qi deficiency due to insufficient endowment. If the phlegm-heat is intense and obstructs the lung, and the lung fails to diffuse and govern descent, pneumonia with panting and cough will occur as a transmuted pattern with symptoms of panting, breathlessness and nasal flaring. If the upright qi is deficient, phlegm-heat invades inward, the spirit is low, and liver wind stirs internally; pathogens invading the heart and liver will occur as a transmuted pattern with such symptoms as coma and convulsion. 2. Key points include being unvaccinated, having previous contact with other pertussis patients, the presence of paroxysmal spastic coughing with cock-crow-like echoes at the end, cough that is light in the day and heavy in the evening, a puffy face, congested eyes, and tongue frenulum ulcer. At the end of the incipient and spastic periods, WBC may rise to 2040 × 109/L and lymphocytes may increase to 60% to 80%. When coughplate method or nasopharyngeal swab method is applied for bacterial

cultures, bordetella pertussis is found. Nasopharyngeal swab smear will show DFA (+), serum ELISA will show a rise in PT and FHA.

CASE SCENARIOS The following cases present variations of this condition. After familiarizing yourself with the possible common pattern presentations and appropriate formulas for treatment, use the following exercises to test your overall understanding of the condition. 1. A patient, age 3, has a cough and a runny nose. 1 week later the cough becomes worse, with scanty production of thin white phlegm. The cough is light, but becomes heavy during sleep. His tongue is light red with thin white coating, and the pulse is floating. Which is the best treatment formula? A. Shā Shēn Mài Dōng Tāng (Adenophora and Ophiopogon Decoction) B. Jīn Fèi Căo Săn (Inula Powder) C. Sāng Bái Pí Tāng (Mulberry Root Bark Decoction) D. Yín Qiào Săn (Lonicera and Forsythia Powder) E. Sān Ào Tāng (Rough and Ready Three Decoction) 2. A patient, age 4, has a cough that has become increasingly aggravated over the past 2 weeks. The cough is paroxysmal and spastic, and accompanied by cock-crow-like breathing when coughing; it is relieved temporarily when the phlegm is expectorated. He coughs dozens of times a day, lightly in the day and heavily in the evening. His tongue is red with a thin yellow coating, and the pulse is rapid. A routine blood test shows WBC 30 × 109/L, N 30%, L 70%. He is unvaccinated.

What is the presenting pattern in this case? A. Pathogens invading the lungs B. Phlegm-heat obstructing the lungs C. Lung-spleen qi deficiency D. Phlegm-heat blocking the lungs E. Lung yin depletion 3. A patient, age 1, has been coughing persistently for 3 weeks, first softly and then heavily. When he coughs strongly, he vomits a great amount of sticky phlegm along with food, and then the cough stops temporarily. He has coughed even worse over the past few days with cock-crowlike echoes at the end. The coughing is light in the day and heavy in the evening. Other signs and symptoms include red eyes, pain in the upper part of both sides of his body, tongue frenulum ulcer, a red tongue with a thin yellow coating, and a rapid pulse. This case was treated with Sāng Bái Pí Tāng combined with Tíng Lì Dà Zăo Xiè Fèi Tāng. Which one of the following formula modifications is not appropriate? A. Add shā shēn (Radix Adenophorae seu Glehniae) and mài dōng (Radix Ophiopogonis) to moisten the lung and relieve cough B. Add lóng dăn căo (Radix et Rhizoma Gentianae) to clear and drain liver fire C. Add jiāng cán (Bombyx Batryticatus) and wú gōng (Scolopendra) to relieve spasm and cough D. Add chái hú (Radix Bupleuri) and yù jīn (Radix Curcumae) to soothe the liver and invigorate blood

E. Add Lù Sī Ké Wán (Egret Cough Pills), one pill each morning and evening 4. A patient, age 3, has been suffering from spastic cough for more than 1 month, which attacked him less frequently after some treatment. He coughs lightly with some production of thin phlegm. His limbs are slightly cold, and he is in low spirits. Other signs and symptoms include spontaneous sweating, poor appetite and digestion, loose stools, a pale tongue with thin white coating, and a thready weak pulse. What is the best formula for this case? A. Shēng Mài Săn (Pulse-Engendering Powder) B. Dú Shēn Tāng (Radix et Rhizoma Ginseng Decoction) C. Shā Shēn Mài Dōng Tāng (Adenophora and Ophiopogon Decoction) D. Rén Shēn Wŭ Wèi Zĭ Tāng (Ginseng and Schisandrae Decoction) E. Sì Nì Tāng Jiā Rén Shēn Fāng (Supplemented Frigid Extremities Decoction with Ginseng) 5. A patient, one year of age, has been suffering from paroxysmal spastic cough for a half-month with cock-crow-like breaths after coughing. His cough is light in the day but heavy in the evening, and is relieved after phlegm is expectorated. Over the past 2 days he has developed such signs and symptoms as panting, gurgling phlegm in the throat, nasal flaring, feeling suffocated with blue lips, constant vomiting, poor appetite, a red tongue with a greasy yellow coating, and a floating rapid pulse. What are the treatment principles for this case? A. Scatter wind, subdue pathogens, diffuse the lung, relieve cough

B. Dry dampness, dissolve phlegm, boost qi, fortify the spleen C. Unblock the lungs, clear heat, dissolve phlegm, relieve panting D. Relieve spasm and cough, direct counterflow qi downward E. Moisten the lungs, relieving cough, fortify the spleen, improve appetite

Answers 1. E 2. B 3. A 4. D 5. D

Chapter 38 Scarlatina Scarlatina is an acute infectious disease caused by external contraction of the seasonal scarlatina pathogen (beta-hemolytic streptococcus). Manifestations include fever, angina and a diffusive bright red rash all over the body followed by desquamation. The pathogen invades the mouth and nose to accumulate in the lung and stomach channels usually in individuals with a weak constitution. Scarlatina is categorized in TCM as a warm disease, thus it should be differentiated on the basis of four-level pattern differentiation. The course of scarlatina is, to some degree, related to its presenting pattern:

● During the prodromal period, pathogens invade lung-wei, with main signs and symptoms of fever, aversion to cold, sore throat and indistinct rashes.

● During the eruptive period, intense toxins affect the qi and ying, with main signs and symptoms of high fever, thirst, ulcers in the throat, cinnabarred or purple rashes, and a red tongue.

● During the recovery period after rash eruption, there is yin damaged, with main signs and symptoms of thirst, dry lips, desquamation, a red tongue and inadequate saliva. Common clinical patterns include pathogens invading lung-wei, intense toxins affecting qi and ying, and yin damaged after rash eruption.

CLINICAL ESSENTIALS Scarlatina should be treated primarily by clearing heat, resolving toxins and clearing and soothing the throat. This basic approach to treatment must also be combined with treatment based upon differentiation of pathogens in different locations.

● In the prodromal period, the pathogens are located at the exterior. Therefore, treatment should diffuse and vent pathogens from the exterior with acrid-cool medicinals, clear heat and soothe the throat.

● In the eruptive period, the pathogens are located in the qi and ying, so treatment should clear heat from the qi level, cool the ying level, drain fire and resolve toxins.

● In the recovery period after rash eruption, treatment should nourish yin and promote fluids.

● With accompanying bi syndrome, edema, or palpitations, also treat the corresponding symptoms. Dr. He Shi-ying treats scarlatina mainly with the formula Qīng Jiàng Wán (Clearing and Descending Pill). The medicinals in his formula cooperate with each other to clear heat, cool the ying level, resolve toxins and reduce swellings:

● Shēng dì (Radix Rehmanniae), xuán shēn (Radix Scrophulariae), mài dōng (Radix Ophiopogonis), mŭ dān pí (Cortex Moutan), chì sháo (Radix Paeoniae Rubra) and bái máo gēn (Rhizoma Imperatae) cool the ying level and protect yin.

● Lián qiào (Fructus Forsythiae), băn lán gēn (Radix Isatidis), jīn yín huā (Flos Lonicerae Japonicae) and qīng dài (Indigo Naturalis) clear heat and resolve toxins.

● Dà huáng (Radix et Rhizoma Rhei), as a bitter-cold medicinal, unblocks the bowels and promotes the flow of fluids and defecation together with shēng dì, xuán shēn and mài dōng.

● Chuān bèi mŭ (Bulbus Fritillariae Cirrhosae), cán shā (Faeces Bombycis) and zào jiá zĭ (Semen Gleditsiae) clear heat, soothe the throat, dissolve phlegm, dissipate masses, reduce swellings and expel pus.

● Bò he (Herba Menthae) diffuses and scatters wind-heat, and gān căo (Radix et Rhizoma Glycyrrhizae) resolves toxins and harmonizes all the medicinals.

CASE STUDY Male, age 6. Initial Visit: July 5th, 2008 Chief Complaint: Fever and sore throat for 3 days, rash eruption for 3 hours. History: This patient began to suffer from a sudden fever, sore throat and headache on the afternoon of July 2nd. This morning, red rashes appeared all over his body. Signs and Symptoms: A high fever, a flaming red face, thirst, uneasiness, rashes densely covering the whole body, a swollen and sore throat, dark yellow urine, dry stools, a dark red strawberry-like tongue with burs on the surface, no tongue coating, and a strong rapid pulse. Physical Examination: BT 39.4℃, dry red lips, swollen sore throat with pus, red rashes densely covering the whole body that were dim when pressed, normal heart, lung and abdomen. Laboratory Examination: Routine blood test: WBC 14 × 109/L, N 84%, L 16%.

Pattern Differentiation Contact with other scarlatina patients should be determined. To secure the correct diagnosis, a throat swab culture, an ASO test and an ERS test should also be conducted.

At the beginning of his disease, the seasonal pathogens invade the patient’s lung and become constrained in the fleshy exterior. As the pathogenic qi and upright qi contend with one another, it results in fever, a symptom of a lung-wei exterior pattern. The pathogenic toxins then invade the interior to lurk within the lung and stomach. With pathogenic heat steaming and scorching the throat, swollen and sore ulcers resulted; as heat-toxins burn and damage the surface membranes, pus developed. As toxins scurry outward to the fleshy exterior along the channels, the red rashes appeared. When the toxins transformed into fire and penetrated inward to the qi and ying, symptoms of high fever, uneasiness and thirst developed. As the toxins scorched the interior, and heart fire flamed upward to consume and damage yin-fluids, the tongue became red and strawberry-like without coating. This kind of tongue manifestation is termed “strawberry tongue”. The tongue and strong rapid pulse are both manifestations of intense interior heat-toxins.

Diagnosis Scarlatina due to intense toxins in the qi and ying levels

Clinical Treatment Principles: Clear qi, cool ying, drain fire and resolve toxins. Formula: Modified Liáng Yíng Qīng Qì Tāng (Ying-Cooling QiClearing Decoction)

[凉营清⽓汤加减]

5 doses [Formula Analysis] Shuĭ niú jiăo, chì sháo, mŭ dān pí and shēng shí gāo clear qi and cool the ying. Huáng lián, huáng qín, lián qiào and băn lán gēn drain fire and resolve toxins. Shēng dì huáng, shí hú, lú gēn and xuán shēn clear heat, protect yin and promote fluids. Prepared Medicines: Sān Huáng Piàn (Three Yellows Pill), 2 to 3 pills, 3 times daily. Acupuncture Points:GB 20 (fēng chí), BL 10 (tiān zhù), LI 4 (hé gŭ), LI 11 (qū chí), LU 11 (shào shāng), BL 17 (gé shù), SP 10 (xuè hăi), SP 6 (sān yīn jiāo). Manipulations: Needle with drainage; treat once daily. Second Visit

After 5 doses, the patient’s temperature was brought down, and his sore and swollen throat was relieved. But signs and symptoms included dry lips, thirst, inadequate saliva, dry cough, a red tongue, and a thready rapid pulse remained. Principles: Nourish yin, promote fluids, clear heat, moisten the throat Formula: Modified Shā Shēn Mài Dōng Tāng (Adenophora and Ophiopogon Decoction) [沙参麦冬汤加减]

3 doses [Formula Analysis] Shā shēn, mài dōng, yù zhú and tiān huā fĕn enrich yin to moisten the lung, and nourish yin to promote fluid production. Lú gēn clears heat and promotes fluid production. Sāng yè clears the heat in the lung. Biăn dòu and gān căo regulate and harmonize the spleen and stomach, and promote fluid production. After taking 3 doses of the prescribed formula, the patient was cured.

Prevention and Nursing Maintain good ventilation to avoid secondary infection. Provide patients with adequate nutrition and water. Feed mainly with liquid or halfliquid foods that are light and digestible. Keep the skin and mouth clean, diluted salt water may be used as a gargle. Prevent scratching or tearing of the skin when it is peeling.

COMMENTARY AND DISCUSSION The high fever, sore throat with pus, and the densely spread cinnabar red rashes are the three typical symptoms and signs of scarlatina. The restlessness, dark red tongue, rapid forceful pulses, constipation, and scanty dark urine indicate the severity of toxic-heat and fire in the qi and ying levels. Since this section discusses several skin rash problems including measles, rubella, scarlatina and exanthema subitum, it is necessary to compare their clinical symptoms for a differential diagnosis. The following chart is revised and translated from TCM Pediatrics by Jiang Yu-ren and Wang Yu-run (Shanghai Science and Technology Press, 1984).

Treatment should be timely in order to prevent complications like palpitation, edema and bi syndrome. In this case, the selected formula was a modification of Liáng Yíng Qīng Qì Tāng, from Outline of the Patterns and Treatment of Throat Granular Disorder (Hóu Shā Zhèng Zhì Gài Yào). The formula contains three groups of medicinals. Medicinals for clearing qi and cooling the ying level include shuĭ niú jiăo, chì sháo, mŭ dān pí, shēng dì and xuán shēn. Here, shēng dì huáng and xuán shēn share similar functions. Medicinals for draining fire and expelling toxins include shí gāo, huáng lián, huáng qín, lián qiào and băn lán gēn. Medicinals for generating body fluids and protecting yin include shēng shí hú and lú gēn. At the follow-up visit, modified Shā Shēn Mài Dōng Tāng was given to nourish yin and moisten the throat.

STUDY QUESTIONS 1. Summarize the symptoms, treatment principles, main formulas and commonly used medicinals for intense heat-toxins in the qi and ying. 2. What complications are common with scarlatina?

Answers 1. Symptoms of scarlatina usually include a persistent high fever, uneasiness, thirst, sore throat with ulcers and cinnabar-red or purple rashes densely covering the whole body. The rashes, which are dim when pressed, begin to erupt from the patient’s neck and chest, and then spread to the whole body. 1 or 2 days after the eruption, the tongue of the patient is strawberry-like and red with a rough yellow coating. 3 to 4 days after eruption the tongue coating begins to peel. The pulse is strong and rapid. The basic treatment principles for scarlatina are to clear qi, cool ying, drain fire and resolve toxins. The best formula for treating scarlatina is modified Liáng Yíng Qīng Qì Tāng (Ying-Cooling Qi-Clearing Decoction). Commonly used medicinals for treating scarlatina include shuĭ niú jiăo (Cornu Bubali), chì sháo (Radix Paeoniae Rubra), mŭ dān pí (Cortex Moutan), huáng lián (Rhizoma Coptidis), huáng qín (Radix Scutellariae), lián qiào (Fructus Forsythiae), băn lán gēn (Radix Isatidis), shēng dì huáng (Radix Rehmanniae), shí hú (Caulis Dendrobii), xuán shēn (Radix Scrophulariae) and lú gēn (Rhizoma Phragmitis).

2. Scarlatina is caused by a beta hemolytic streptococcus infection. If treatment is not timely, or if the infection is poorly controlled, the bacterium may spread inside the body causing lymphadenitis colli, posterior pharyngeal abscess, tympanitis, pneumonia, or even more severe complications like toxic myocarditis, infectious shock, septicemia and meningitis. In the recovery period, such allergic diseases as acute glomerulonephritis and rheumatism may occur.

CASE SCENARIOS The following cases present variations of this condition. After familiarizing yourself with the possible common pattern presentations and appropriate formulas for treatment, use the following exercises to test your overall understanding of the condition. 1. A patient, age 3, suffers from a sudden fever with headache, aversion to cold, no sweating, swollen sore throat, dysphagia, flushed skin, indistinct rashes, a red tongue with a thin yellow coating, and a floating strong pulse. What is the proper treatment formula? A. Jiĕ Jī Tòu Shā Tāng (Muscle-Resolving Rash-Expelling Decoction) B. Liáng Yíng Qīng Qì Tāng (Ying-Cooling Qi-Clearing Decoction) C. Shā Shēn Mài Dōng Tāng (Adenophora and Ophiopogon Decoction) D. Xī Jiăo Dì Huáng Tāng (Rhinoceros Horn and Rehmannia Decoction) E. Líng Jiăo Gōu Téng Tāng (Antelope Horn and Uncaria Decoction) 2. A patient, age 2, suffers from high fever with uneasiness, thirst, sore throat with ulcers, red to purple rashes densely covering the body. His rashes began to erupt from his neck and chest, and then spread to his whole body, and they were dim when pressed. His tongue is red with burs on the surface and a rough yellow coating; the pulse was strong and rapid.

What is the presenting pattern? A. Pathogens invading lung-wei B. Intense toxins in qi and ying C. Yin damage after eruption of rashes D. Pathogens penetrating the lungs and stomach E. Pathogenic toxins blocking the lungs 3. A patient, age 2, suffers from scarlatina with rashes covering his body and all the skin areas on his body peeling. His low fever is persistent and accompanied by a dry cough. He also has a poor appetite, a red peeled tongue with little saliva, and a thready rapid pulse. What are the proper treatment principles? A. Nourish yin, promote the production of liquid, clear heat, moisten the throat B. Clear qi, cool ying, drain fire, resolve toxins C. Clear heat from the qi level, cool the ying level, clear heat, soothe the throat D. Scatter wind, release the exterior, clear heat, resolve toxins E. Enrich yin, clear heat, promote urination 4. A patient, age 4, suffers from persistent low fever with cinnabar-red rashes densely covering the body, flaming red lips, thirst, a dry cough, poor appetite, a peeling red tongue with little saliva, and a thready rapid pulse. What is the presenting pattern? A. Yin damage after the eruption of rashes

B. Pathogens invading lung-wei C. Pathogens invading the lung and stomach D. Pathogens invading the lung and spleen E. Pathogens sinking into the heart and liver 5. A patient, age 3, suffers from a sudden fever with headache, aversion to cold, no sweating, a swollen sore throat, constant dysphagia, flushed skin, indistinct rashes, a red tongue with thin yellow coating, and a strong floating pulse. What is the presenting pattern? A. Pathogens invading the lung and stomach B. Pathogens invading the lung and spleen C. Pathogens invading the lung-wei D. Pathogens invading qi and ying E. Pathogens sinking into the heart and liver 6. A patient, age 3, suffers from a sudden fever with headache, aversion to cold, no sweating, a swollen sore throat, dysphagia, flushed skin, indistinct rashes, a red tongue with a thin yellow coating, and a strong floating pulse. What are the proper treatment principles? A. Diffuse and vent pathogens with acrid-cool medicinals, clear heat, soothe the throat B. Clear heat, resolve toxins, dissolve phlegm, soothe the throat C. Dry dampness, dissolve phlegm, clear and soothe the throat

D. Dissolve phlegm to open the orifices, clear and soothe the throat E. Moisten the lungs, dissolve phlegm, clear and soothe the throat

Answers 1. A 2. B 3. A 4. A 5. C 6. A

PART VI Neonatal Diseases Chapter 39 Scleroderma Neonatorum Scleroderma neonatorum is a disease in newborns with clinical manifestations that include whole-body or local coldness and congealing with stiffening of the hands, feet, lips, head, neck, skin and subcutaneous fat, sometimes accompanied by edema or damage to organ function. Newborn infants in the week after birth, or those immature and weak are the most likely to be affected. The rate of incidence of the disease is higher in colder seasons. The Chinese name of the disease is “five stiffnesses”, or fetal cold. Scleroderma neonatorum mostly appears in the cold seasons of winter and spring, and sometimes in the summer when caused by premature delivery or infection. The disease usually appears within 7 to 10 days after birth, and those with congenital weakness are more likely to be attacked. Causes of the disease include cold, premature delivery, infection, asphyxiation and others. Serious cases have poor prognoses, and are likely to be complicated by infection with pneumonia or neonatal sepsis. The worst result is death due to complications from lung bleeding.

Newborns are most likely to be affected because their constitutions are “tender yin and tender yang”. Constitutional insufficiency, yuan-source qi insufficiency, insufficiency of yang qi, cold congealing, and qi stagnation with blood stasis are the internal causes of the disease. Newborns, especially those with congenital insufficiency or retarded development, or those born during cold winters who are not well-protected and kept warm are easily attacked by pathogenic cold or other pathogens which cause irregular movement of qi and blood. The disease is located in the spleen and kidney, and the main clinical patterns are congealing cold blocking blood, and debilitation of yang qi.

CLINICAL ESSENTIALS There are records of the symptoms and the prognosis of scleroderma neonatorum in the classics. Lu Bo-si’s A Hundred Questions on Infants and Children states, “For the patient who suffers the five stiffnesses, it is hard to breathe and move. …if the feet and palms are cold and stiff, it is wind disease, which is hard to treat…if the face is pale and the abdomen is stiff, it may be life-threatening” (五硬者,仰头取⽓,难以动摇……脚⼿⼼冰 冷⽽硬者,此为风症难治……恐⾯青腹硬者,性命难保). When an infant is born, it is weak and unable to endure exposure to wind and cold which can injure the skin and block the movement of qi and blood. As nourishment is lost and cold congeals, the body becomes stiff and swollen. Attention should be paid to identifying signs of deficiency, excess, cold and stasis.

● Signs and symptoms of the cold pattern include a low body temperature, lying stiffly and lacking movement, and stiff swollen skin.

● The excess pattern is due to externally-contracted cold. Patients have a history of disease caused by not keeping sufficiently warm, and they have a less extreme decline of body temperature and smaller areas of stiffness.

● The deficiency pattern includes debilitation of yang qi often accompanied by congenital insufficiency, the body temperature is difficult to rise, and there are large areas of stiffness.

● Blood stasis is also common in the disease, and characterized by stiff and dark purple skin.

● Less serious cases are always caused by congealing cold blocking blood, while severe cases are characterized by debilitation of yang qi. It is important that treatment address the static blood by dissipating stasis and engendering new blood.

● If only warm and sweet medicinals such as fù zĭ (Radix Aconiti Lateralis Praeparata) and ròu guì (Cortex Cinnamomi) are used, the effect will not always be good. To achieve an ideal curative effect, warm and sweet medicinals should be employed along with others that invigorate blood and dissolve stasis when appropriate.

● Topical applications, medicinal bathing and pastes can be used to free the interstices and course yang qi; internal and external treatments combined will generally yield the best results.

CASE STUDY Female, age 8 days. Initial Visit: Dec 5th, 2009 Chief Complaint: Cold and stiff skin for 3 days. History: The child patient was born premature, with body weight of 1.55 kg, BT 35.5 ℃, with weak crying and weak suckling. 4 days after birth, the skin of the lower leg started to stiffen, which extended to all four limbs. The buttocks and cheeks were stiff, swollen and cold, and the lips were a dull purple. The child could not suckle, and after having been fed milk through her nose and injected with antibiotics and glucose infusion, her body temperature did not rise, and the stiff swollen areas extended to 50% of the body. Signs and symptoms: Somnolence, weak crying, cold and stiff limbs, buttocks and cheeks, dull purple skin, dark red lips and tongue, and stagnant purple finger venules. Physical Examination: BT 34.8℃, HR 110 beats/min, dull heart sounds, no obvious abnormity in the two lungs or abdomen. Laboratory Examination: WBC 11 × 109/L, N 66%, L 32%, M 2%.

Pattern Differentiation The child in this case was immature and thus had constitutional insufficiency, weak qi and physique, and she had received improper care. The attack of pathogenic cold worsened the deficiency of yang qi and cold

congealed within the skin. Internal cold leads to inhibited qi and blood flow with inability to warm the limbs, which caused coldness of the whole body with stiffness and swelling of the skin, limbs, and buttocks. Cold causes blood to stagnate and obstruct the blood vessels, resulting in the dull purple lips and skin and sluggish finger venules.

Diagnosis Fetal cold due to congealing cold blocking blood

Clinical Treatment Principles: Warm yang, dissipate cold, quicken the blood, transform stasis Formula: Modified Dāng Guī Sì Nì Tāng (Chinese Angelica Frigid Extremities Decoction) [当归四逆汤加减]

2 doses were prescribed. One decocted daily dose, fed through the nose. [Formula Analysis] Zhì fù piàn, guì zhī, and xì xīn dissipate interior-exterior cold, unblock blood and free the vessels. Dāng guī, dān shēn, and chuān xiōng nourish and invigorate blood. Chì sháo and hóng huā dispel stasis and unblock the collaterals. Huáng qí supplements qi and raises yang. Wú zhū yú and gān jiāng dispel cold and warm the channels. Prepared Medicines

● Fù Fāng Dān Shēn Zhù Shè Yè (Compound Salvia Injection) 2 ml with 10% glucose solution 30 ml for intravenous infusion. Administer once daily for 5-7 days as one course of treatment.

● Yán Suān Chuān Xiōng Qín Zhù Shè Yè (Ligustrazine Hydrochloride Injection), 2-4 mg/kg with 10% glucose solution for intravenous infusion. Administer once or twice daily.

● Lù Róng Jīng Zhù Shè Yè (Pantocrine Injection) 1 ml for intramuscular injection. Administer once daily. Applicable for patients with yang qi debilitation. External Therapy

● Fresh cōng bái (Bulbus Allii Fistulosi) 30g, shēng jiāng (Rhizoma Zingiberis Recens) 30g, dàn dòu chĭ (Semen Sojae Praeparatum) 30g. Mash and mix, stir with wine and heat; apply as a compress.

● Dāng guī 15g, hóng huā 15g, chuān xiōng 15g, chì shāo (Radix Paeoniae Rubra) 15g, tòu gǔ căo (Speranskia Tuberculata) 15g, dīng xiāng (Flos Caryophylli) 9g, chuān wū (Radix Aconiti) 7.5g, căo wū (Radix Aconiti Kusnezoffii) 7.5g, rǔ xiāng (Olibanum) 7.5g, mò yào (Myrrha) 7.5g and ròu guì (Cortex Cinnamomi) 6g. Grind into powder, add lanolin 100g and vaseline900g, and mix into a paste. Smear it onto a piece of gauze, warm it, and then apply to the affected parts, once daily. Most suitable for those patients with yang qi debilitation.

● Cōng bái 50g, shēng jiāng 30g, hóng huā 15g, ài yè (Folium Artemisiae Argyi) 15g, má huáng (Herba Ephedrae) 10g. Heat the medicinals, and pack them into a piece of cloth. Place the pack over the affected areas as a compress for 10 min, 3-4 times per day. Tui Na Wàn Huā Yóu (Wanhua Oil) is made up of 20 medicinals including hóng huā, dú huó (Radix Angelicae Pubescentis) and sān léng (Rhizoma Sparganii) which act to remove swelling, resolve blood stasis, relax the sinews and quicken the collaterals. Massage the oil into the hard swollen areas with the palms, gently rubbing the hard and swollen areas, especially on the lower extremities. Gently massage the lower limbs. Moxibustion Apply a warm moxa stick locally to the hard swollen areas. Re-warming Therapy: For children with low body temperature.

● Mild or moderate hard and swollen body: Put the patient in a room with a temperature of 26℃ to 28℃. Hot water bags, heat cradles, electric

blankets or holding the infant are ways to warm them gradually. If available, a 30℃ to 34℃ incubator can return the temperature to normal within 6-12 hrs.

● Severe hard and swollen body: First warm the infant in an incubator 1℃ to 2℃higher than the body temperature. Increase temperature of the incubator 0.5℃ to 1℃every hour, generally not to exceed 34℃. The body temperature can return to normal within 12 to 24 hrs. Second Visit December 7th The baby cried louder, and the hard swollen cheeks had become softer than before. The original prescription was again prescribed. Third Visit December 9th Having been treated for 2 days, the body temperature returned to 36℃, the hardness and swelling became less severe, the lips returned to red, and the infant could now be bottle-fed. However, the child was still in a weak condition. Principles: Warm yang, boost qi, invigorate blood, unblock the vessels Formula: Modified Rén Shēn Fù Zĭ Tāng (Chinese Angelica Frigid Extremities Decoction) [⼈参附⼦汤加减]

3 doses. [Formula Analysis] Fù zĭ and guì zhī warm yang and dissipate cold. Rén shēn, huāng qí, shān yào, bá shù and fú líng supplement qi and fortify the spleen. Dān shēn and hóng huā quicken the blood and dissipate stasis.

Fourth Visit The hardness and swelling was alleviated, the child could suckle with strength, and her body was warm with a body temperature of 37℃. After 3 days of observation, the patient was recovered and discharged from the hospital.

COMMENTARY AND DISCUSSION Even though the scleroderma neonatorum includes two main patterns, debilitation of yang qi, mainly of the spleen and kidney, and coagulation and stagnation of blood due to cold, there is no strict boundary line between these two types. Instead, they affect each other and together become the main pathological mechanism of scleroderma neonatorum. Qi and yang deficiency is the cause, and blood coagulation and stagnation is the result. Severe qi and yang deficiency induces more severe coagulation and stagnation, and vice versa. This 8-day-old child is such an example. 4 days after birth, the skin of her lower leg started to stiffen, which then extended to all four limbs. The buttocks and cheeks were stiff, swollen and cold, and her lips were dull purple. Her body temperature did not rise, and the stiff swollen areas extended to cover half of the whole body. Obviously, this is yang deficiency with internal cold. According to the formulating theory, there are three types of yangwarming prescriptions. First are formulas that warm the middle and dispel cold such as Lĭ Zhōng Wán (Center-Regulating Pill). Second are formulas that rescue devastated yang, including Sì Nì Tāng (Frigid Extremities Decoction). Third are formulas that warm the channels and disperse cold such as Dāng Guī Sì Nì Tāng (Chinese Angelica Frigid Extremities Decoction). As a variation of Guì Zhī Tāng (Cinnamon Twig Decoction) from the Treatise on Cold Damage (Shāng Hán Lùn), Dāng Guī Sì Nì Tāng acts to

warm the channels, disperse cold, nourish blood, and unblock the blood vessels; it is commonly used for Raynaud’s disease, fibromyalgia and other diseases. In this case, Dāng Guī Sì Nì Tāng was applied to unblock the channels so that the yang qi and blood could move freely through the whole body. In order to warm yang and dispel cold, zhì fù zĭ and wú zhū yú were also added. Wú zhū yú disperses cold, relieves constraint in the liver channel and alleviates pain. In another of Zhang Zhong-jing’s prescriptions, Dāng Guī Sì Nì Jiā Wú Zhū Yú Shēng Jiāng Tāng (Chinese Angelica Frigid Extremities Decoction Plus Evodia and Fresh Ginger), wú zhū yú was combined with shēng jiāng for a similar presentation with chronic cold. To promote blood circulation, chì sháo, chuān xiōng, dān shēn and hóng huā were also added to strengthen the functions of dāng guī, guì zhī and xì xīn. Thus, both the root yang deficiency and the symptoms of blood coagulation and stagnation were treated with a modification of Dāng Guī Sì Nì Tāng. When yang qi and blood were again distributed to the whole body through the channels and blood vessels, warmth returned. With a very dynamic character, Dāng Guī Sì Nì Tāng worked as a vehicle to connect the interior with the exterior. Physical therapy and topical Chinese medicinals should not be ignored for the treatment of scleroderma neonatorum, as these therapies can promote qi and blood circulation in peripheral areas of the body directly and effectively.

STUDY QUESTIONS 1. Into how many pattern types can this disease be divided? What are they and how are they identified? 2. What is the principle of treatment for this disease? Why should one emphasize the treatment of stasis? 3. In preventing and nursing, to what must one pay attention?

Answers 1. Scleroderma neonatorum includes two main patterns, debilitation of yang qi, mainly of the spleen and kidney, and congealing cold blocking blood. The disease can be identified from the following aspects: First, less serious cases are always caused by congealing cold blocking blood; this often appears in babies with weak constitutions who are attacked by cold. The pathomechanism includes congenital weakness, yang qi weakness, and attack of cold. The main clinical manifestations are a cold body with qi stagnation and blood stasis, limited stiff swollen areas, and weakened yang qi, but not to the point of collapse. Second, yang qi deficiency collapse may occur in severe cases, most common in underdeveloped fetuses. Yang qi is debilitated and yin qi is abundant, causing stagnation and stasis of blood with larger areas of stiffness and swelling. This pattern of disease is more likely to change; for instance, pneumonia may occur as a result of externally-contracted

pathogens, and lung bleeding may result from deficiency cold with failure to control blood. 2. The principle of treatment is to warm yang, dispel cold, invigorate blood and dissolve stasis. After being attacked by cold, the movement of qi and blood is blocked so that the skin and limbs fail to be warmed; this is characterized by swelling. Cold congeals the blood and vessels become blocked; this is marked by dark purple lips and skin, and red finger venules. Blood stasis is also commonly present, so treatment should also aim at dissipating stasis and engendering new blood. 3. Several points must be minded in prevention and nursing of scleroderma neonatorum. First, prenatal care is important to avoid premature labor, reduce the possibility of low birth weight, and avoid maternal injury. Infants born in winter should be kept warm, especially premature or of low birth weight; the delivery room should be as warm as 20℃. Check the infant’s skin and subcutaneous fat within the first week for stiffness or swelling. Disinfect and isolate newborns to avoid infection. When nursing the disease, keep the infant warm, especially those underdeveloped and those with hard swellings. Breastfeeding can supply heat energy. Feed those unable to suckle with a dropper or through the nose if necessary, or inject them with glucose, plasma, or others intravenous treatments. Give hypoxic babies timely oxygen. Clothing and wrappings must be soft, dry and clean. Change the sleeping position of the affected infant frequently. Disinfect and isolate the newborn to avoid cross-infection.

CASE SCENARIOS The following cases present variations of this condition. After familiarizing yourself with the possible common pattern presentations and appropriate formulas for treatment, use the following exercises to test your overall understanding of the condition. 1. A 7 day-old infant had weak breathing, low crying, a stiff swollen body and skin, dark red skin, bleeding fresh blood from the nose and mouth, and irregular breathing. The infant died after one day of treatment. What complication led to his death? A. Pneumonia B. Neonatal sepsis C. Pulmonary hemorrhage D. Heart failure E. Renal failure 2. A 7 day-old baby has a coldness of the body and four limbs, good responsiveness, low crying, stiff swollen skin that is hard to pinch, stiff swollen dark red skin of the buttocks and lower legs, local skin that is red and swollen as if frostbitten, and dull purple finger venules. What is the presenting pattern? A. Accumulation of heat-toxin B. Congealed cold blocking blood

C. Weak breath D. Deficiency of spleen and kidney E. Debilitation of yang qi 3. A 9 day-old has coldness of the body and limbs, good responsiveness, low crying, stiff swollen skin that is hard to pinch, stiff swollen dark red skin of the buttocks and lower legs, red swollen skin as if frostbitten, and dull purple finger venules. What is the best treatment formula? A. Sì Nì Tāng (Frigid Extremities Decoction) B. Shēn Fù Tāng (Ginseng and Aconite Decoction) C. Jīn Guì Shèn Qì Wán (Golden Cabinet’s Kidney Qi Pill) D. Dāng Guī Sì Nì Tāng (Chinese Angelica Frigid Extremities Decoction) E. Xuè Fŭ Zhú Yū Tāng (Blood Mansion Stasis-Expelling Decoction) 4. An 8 day-old infant, born weighing 2000g, has weak crying, a cold stiff body, a pale complexion, stiff swollen muscles, swollen areas over 60% of the body, dark red skin, scanty urine, and slightly red finger venules. What are the correct principles of treatment? A. Boost qi and nourish the heart B. Unblock the channels, quicken the collaterals, promote urination, disperse swelling C. Boost qi, warm yang, unblock the channels, invigorate blood D. Boost qi, warm yang, promote urination, disperse swelling

E. Warm the channels, dissipate cold, boost qi, invigorate blood 5. A 7 day-old has weak breathing, low crying, stiffness and swelling of the whole body, dark red skin, foaming at the mouth and irregular breathing. Which of the following should be added to the formula for the treatment of unblocking the channels and invigorating blood? A. Sāng yè (Folium Mori), qián hú (Radix Peucedani) and jié gĕng (Radix Platycodonis) B. Má huáng (Herba Ephedrae), xìng rén (Semen Armeniacae Amarum) and shēng shí gāo (Gypsum Fibrosum) C. Táo rén (Semen Persicae), hóng huā (Flos Carthami), and chì sháo (Radix Paeoniae Rubra) D. Líng yáng jiăo (Cornu Saigae Tataricae), gōu téng (Ramulus Uncariae Cum Uncis) and jú huā (Flos Chrysanthemi) E. Jiāng cán (Bombyx Batryticatus), shí chāng pú (Rhizoma Acori Tatarinowii) and tiān nán xīng (Rhizoma Arisaematis) 6. A 10 day-old baby has a cold stiff body with poor responsiveness, weak breathing, low crying, difficulty in suckling, a pale face, stiffness and swelling of the whole body, dark red skin, difficulty in urination or even no urine, pale lips and slightly red finger venules. Which of the following should be added to the formula to boost qi, warm yang, unblock the channels and invigorate blood? A. Yì yĭ rén (Semen Coicis), shān yào (Rhizoma Dioscoreae) and chē qián zĭ (Semen Plantaginis) B. Zé xiè (Rhizoma Alismatis), bái zhú (Rhizoma Atractylodis Macrocephalae) and chén pí (Pericarpium Citri Reticulatae)

C. Sāng bái pí (Cortex Mori), dà fù pí (Pericarpium Arecae) and dì gŭ pí (Cortex Lycii) D. Sān léng (Rhizoma Sparganii), é zhú (Rhizoma Curcumae) and hóng huā (Flos Carthami) E. Fú líng (Poria), zhū líng (Polyporus) and shēng jiāng pí (Cortex Zingiberis Rhizomatis) 7. A 12 day-old has weak breathing, low crying, stiff swollen dark red skin extending to the lower limbs and buttocks. Which is the correct method for re-warming therapy? A. Incubator at 28℃ B. Incubator at 30℃ C. A room at 24℃ to 26℃ D. A room at 26℃ to 28℃ E. A room at 30℃ 8. A 6 day-old has a cold stiff body with poor responsiveness, weak breathing, low crying, difficulty suckling, a pale complexion, stiffness and swelling that extends to 50% of the body, dark red skin, scanty urine, pale lips, and no red finger venules. With re-warming therapy, what is the time required to normalize the body temperature? A. 3 hrs B. 5 to 6 hrs C. 6 to 10 hrs

D. 12 to 24 hrs E. More than 24 hrs

Answers 1. C 2. B 3. D 4. C 5. E 6. E 7. D 8. D

Chapter 40 Fetal Jaundice Fetal jaundice, a disease related to fetal endowment, is characterized by an abnormal yellow coloration of the skin, mucosa and sclera. Fetal jaundice is primarily caused by fetal damp-heat, and the disease is located in the liver, gallbladder, spleen and stomach. The main pathogenesis are fetal damp-heat or cold-damp, which brew in the spleen and stomach, or long-term qi stagnation and blood stasis, which causes failure of liver qi to move freely and the outward discharge of bile. Thus, jaundice results. Clinical pattern types of fetal jaundice include depressed steaming damp-heat, cold-damp obstruction, and qi stagnation and blood stasis. Serious cases of jaundice may have transmuted patterns, such as fetal jaundice generating wind and fetal jaundice desertion.

CLINICAL ESSENTIALS The pathogenesis of fetal jaundice is pathogenic dampness, and the first step in treatment is identifying whether it is physiological or pathological. If it is pathological, treatment should be based on eightprinciple pattern differentiation and identification, firstly on differentiation of yin and yang.

● If the jaundice has an acute onset, a short course, the color of the jaundice is bright yellow, and the tongue coating is yellow greasy, this is yang jaundice.

● If the onset is slow, the course is long, the color of the jaundice is dim yellow, and the tongue coating is pale white, this is yin jaundice.

● If the course of disease is long, the condition becomes worse, the color of the jaundice is dim yellow and is gradually becoming green, and the liver and spleen are swollen and hard, this is jaundice caused by stasis accumulation. Dr. Diao Ben-shu believes the main cause of fetal jaundice involves damp-heat fetal-toxin originating with the mother. He describes four main points for effective clinical treatment: First, divert damp-heat to the stool and urine. Second, open the sweating ducts to dispel the pathogen from the striae and interstices. Third, supplement the spleen and stomach to strengthen their function of dispelling damp-heat from the middle jiao.

Lastly, regulate the spleen and stomach. To dispel damp-heat, Dr. Diao uses modifications of Yīn Chén Hāo Tāng (Virgate Wormwood Decoction). Yīn chén (Herba Artemisiae Scopariae) dispels damp-heat through the urine. Jīn qián căo (Herba Lysimachiae), măn tiān xīng (Herba Sphagni), huā bān zhú (Rhizoma Polygoni Cuspidati), zé xiè (Rhizoma Alismatis), chē qián căo (Herba Plantaginis) and other medicinals are often added to the formula to reinforce the function of clearing heat and eliminating dampness. Among these, Dr. Diao uses jīn qián căo, măn tiān xīng and huā bān zhú as the main medicinals to abate jaundice. Damp-heat spreads to the skin, striae and interstices, causing jaundice of the skin. Therefore, inducing sweating to release jaundice is convenient and effective. Dr. Diao often makes a wash formula for child patients, comprised of má huáng (Herba Ephedrae), guì zhī (Ramulus Cinnamomi), xì xīn (Radix et Rhizoma Asari), ài yè (Folium Artemisiae Argyi), shí chāng pǔ (Rhizoma Acori Tatarinowii), zĭ sū yè (Folium Perillae) and jīng jiè (Herba Schizonepetae) to induce sweating. Bitter-cold medicinals such as yīn chén, jīn qián căo, huā bān zhú, băn lán gēn (Radix Isatidis), dà qīng yè (Folium Isatidis), dà huáng (Radix et Rhizoma Rhei), hán shuǐ shí (Glauberitum) and others change acrid-warm to acrid-cold to clear heat, dispel dampness, dissipate pathogens and abate jaundice. Dr. Guo Qian uses massage therapy, choosing points on the patients’ hands. In the early period of the disease, supplement the spleen, calm the liver, and clear the stomach. Later, rub wài láo gōng (EX-UE8), supplement the spleen, calm the liver, and clear the stomach. Dip the hands in talcum powder when massaging, rub each point for about 5 min, with 3 days

constituting one course of treatment. This method is known to be very effective.

CASE STUDY Male, age 26 days. Initial Visit: March 2nd, 2009 Chief Complaint: Face, eyes and skin of the whole body have been yellow for 23 days. History: The child’s skin and eyes became yellow 3 days after birth, which grew more serious over the following 10 days. Chinese-Western medical treatments had little effect. Signs and Symptoms: Yellow coloration of the body and eyes, with the color as light as the color of an orange, fever, somnolence, abdominal distention and fullness, vomiting, diminished desire for milk, dry light yellow stools, deep yellow urine that dyes the diaper, a red tongue with a yellow coating, and purple finger venules reaching the wind-pass. Past History: The baby was born premature with a birth weight of 2.4 kg with no intrauterine distress. Physical Examination: BT 37.3℃, PR 131 beats/min, RR 42 times/min, body weight 2.9 kg, listless spirit, skin of the entire body and the sclera appear very yellow, the lower edge of the liver is palpable 2 cm below the ribs and is soft. Laboratory Examination: WBC 10.2 × 109/L, N 63%, L 25%, Hb 120 g/L, PLT 206 × 109/L, serum total bilirubin 260μmol/L, serum bilirubin 85μmol/L.

Pattern Differentiation

The child patient has congenital constitutional insufficiency. His spleen fails to move and transform, and so fails to clear the fetal-toxin and dampheat. Also, the mother consumed excessive hot, spicy, acrid, greasy and rich foods, causing damp-heat to arise and brew internally. Damp-heat swelters and transfers to the fetus, causing damp-heat to brew in the liver and gallbladder of the fetus. Liver qi cannot move freely, and qi stagnation causes blood stasis. The blood vessels are obstructed, so bile discharges outward to the skin and causes the skin and eyes to become bright yellow in color. Damp-heat internally brewing with obstruction of the qi dynamic and impairment of the harmonious descending of stomach qi cause abdominal distention and fullness, vomiting, and reduced milk intake. Damp-heat brews and binds in the intestines, causing the stool to become dry. Dampheat pours downward to the bladder and causes the urine to become yellow. Fever, red tongue with yellow tongue coating and purple finger venules reaching the wind-pass are all caused by sweltering damp-heat. The disease is located in the liver, gallbladder, spleen and stomach, and belongs to the category of yang jaundice. The disease mechanisms are damp-heat accumulation in the spleen, liver qi failing to move freely, and the outward discharge of bile.

Diagnosis Fetal jaundice due to damp-heat fuming and steaming

Clinical Treatment Principles: Clear heat, drain dampness, abate jaundice

Formula: Modified Yīn Chén Hāo Tāng (Virgate Wormwood Decoction) [茵陈蒿汤加减]

7 doses [Formula Analysis] Yīn chén clears heat and transforms dampness, drains damp-heat from the gallbladder and abates jaundice; this is a key medicinal for the treatment of jaundice. Zhī zĭ clears sanjiao damp-heat. Dà huáng drains accumulated heat, removes stasis and moves blood. It can accelerate the abating of jaundice. Fú líng and chē qián zĭ clear heat, drain dampness downward and promote urination. Dān shēn moves blood and transforms stasis. It also promotes improved liver function and reduces hepatomegaly. All of the above used together reinforce the functions of clearing heart, dispelling dampness, draining the gallbladder, quickening the blood, transforming stasis and abating jaundice. Prepared Medicines: Yīn Zhī Huáng Zhù Shè Yè (Virgate Wormwood, Gardenia and Rhubarb Injection), 5-20ml by intravenous

injection with an equal volume of 10% glucose solution; administer once or twice daily. Acupuncture Points: LV 13 (zhāng mén), LV 3 (tài chōng), BL 20 (pí shù), BL 18 (gān shù), PC 8 (láo gōng), DU 6 (jĭ zhōng). Manipulations: Insert the needles quickly, then twirl or lift and thrust without retention. Second Visit 7 days later, the jaundice of the face and body abated and the child’s spirit was improved. Other signs and symptoms included abdominal distention, restless sleep, unbound stools, and pale yellow urine; the tongue and pulse were the same as the previous visit. Principles: Dispel dampness, abate jaundice, move qi, release focal distention Formula: Modified Yīn Chén Hāo Tāng (Virgate Wormwood Decoction) [茵陈蒿汤加减]

5 doses [Formula Analysis]

Dà huáng has been removed from the formula. Yīn chén benefits the gallbladder, abates jaundice, and frees the stool. Chăo mài yá moves the spleen and supports the digestion of milk. Hòu pò moves qi and disperses pĭ. Fú líng and chē qián zĭ clear heat, drain dampness, and promote urination. Dān shēn moves blood and dissolves stasis.

Prevention and Nursing Pregnant women should pay close attention to diet and hygiene, also avoiding wine, spicy foods and drug abuse. Observe the skin coloration of newborns so as to notice if jaundice comes into being and when it fades. Keep newborns warm. If necessary, treat with regular oxygen therapy. Feed with milk soon after birth, and urge defecation of the meconium.

COMMENTARY AND DISCUSSION In this case, the child suffered from yang-type or damp-heat jaundice due to dampness and static heat accumulating interiorly. This pattern led to a “steaming”effect that caused the entire body to turn a bright-orange color. Yīn Chén Hāo Tāng, applied in this case, is a famous prescription from the Treatise on Cold Damage (Shāng Hán Lùn). The indications include whole body jaundice with a color that resembles a fresh tangerine, slight abdominal distention, urinary difficulty, thirst, a greasy yellow tongue coating and a slippery rapid pulse. This prescription only contains three ingredients, yīn chén, zhī zĭ and dà huáng, but has great effect in reducing jaundice. Zhang Zhong-jing wrote that after taking this prescription, “Urination should become normal and smooth, the urine being like the juice of a Chinese honey locust with a fresh red color. The mild abdominal distention will be reduced overnight and the jaundice released through urination” (⼩便当利,尿如皂角汁状,⾊正⾚。⼀宿腹减,湿从⼩便 去矣). How can treatment guide damp-heat out of the body? Zhang Zhongjing designed this formula to drain and remove damp-heat through the urine, and the first two ingredients are included for this purpose. The practitioner in this case noticed that the effect of promoting urination was inadequate, and, therefore, fú líng and chē qián zĭ were added to increase this function. This was a wise modification, indicating that the practitioner thoroughly understood the formulating theory of this prescription.

The importance of dà huáng in this formula should not be overlooked, even though it is applied as an assistant in this prescription and is not included for its strong purgative purpose. Actually, when dampness and heat are combined, sticky stools or diarrhea often results, where more dà huáng could make the stool drier. Here, dà huáng is bitter and cold and acts to clear heat and damp-heat, which also strengthens the effects of yīn chén and zhī zĭ to clear heat. Secondly, dà huáng benefits the gallbladder and unblocks the fu organs, maintaining the smoothness of the bile ducts to keep them unblocked so as to reduce jaundice. In addition, dà huáng promotes blood circulation and removes blood stasis, which assists the liver organ, as the liver stores blood and regulates blood distribution. When blood circulation is smooth, recovery of the liver organ is promoted and enlargement of the liver is prevented. For this same purpose, dān shēn was also added. Even though there were only six ingredients in the formula, it was very effective. 7 days later, the jaundice was much improved. Dà huáng was then removed and medicinals that strengthen the spleen and improve transformation and transportation of food and water were added, including hòu pò and chǎo mài yá, because long-term application of dà huáng can damage spleen-stomach qi. If dà huáng leads to rebound constipation, it can be replaced with a mild laxative such as qín jiāo (Radix Gentianae Macrophyllae) or dōng guā zĭ (Semen Benincasae), but these medicinals do not have the functions of moving blood and protecting the liver organ.

STUDY QUESTIONS 1. How does one differentiate the depressed steaming damp-heat pattern and the cold-damp obstruction pattern based upon the clinical features? 2. What is neonatal physiological jaundice?

Answers 1. The depressed steaming damp-heat and cold-dampness obstruction patterns can be differentiated based on the color of jaundice and cold or heat. The main feature of the depressed steaming damp-heat pattern is damp-heat congestion. Signs and symptoms include yellow body and eyes as light as the color of an orange, loud crying, diminished desire for milk, thirst, dry lips, possible fever, bound stools, dark yellow urine and a red tongue with a slimy yellow coating. The cold-dampness obstruction pattern has obvious features of deficiency cold with signs and symptoms including a yellow and lackluster face, eyes and skin, sustained jaundice, listlessness of spirit, lack of warmth in the limbs, torpid intake, grayish-white sloppy stools, scanty urination and a pale tongue with a slimy white coating. 2. Physiological jaundice appears within the first 2 to 3 days after birth, is most serious on days 4 to 5, and fades 10 to 14 days after the birth of a term infant. Fading may be delayed to the third or the fourth week after birth of a premature infant. During this period, the newborn is in a good

condition without other clinical symptoms. The serum total bilirubin is less than 85 μmol/L (5mg/dl).

CASE SCENARIOS The following cases present variations of this condition. After familiarizing yourself with the possible common pattern presentations and appropriate formulas for treatment, use the following exercises to test your overall understanding of the condition. 1. A child patient, 26 days after birth, has jaundice that has not faded. The skin is yellow and lacks luster. Other signs and symptoms include listlessness of spirit, lack of warmth in the limbs, sloppy grayish-white stools, and a pale tongue with a white coating. What are the correct principles of treatment? A. Clear heat and drain dampness B. Warm the middle and remove dampness C. Dissolve stasis and disperse accumulation D. Calm the liver and extinguish wind E. Warm yang to rescue desertion 2. A child patient, 30 days after birth, has sustained jaundice. The face, eyes and skin are yellow and lack luster. Focal distention is present under the right ribs, and other signs and symptoms include abdominal distention, prominent veins, and dark red mouth and lips. The disease pattern is identified as qi stagnation and blood stasis. On what is this diagnosis based? A. Abdominal distention

B. Sustained jaundice C. The face and eyes and skin are yellow D. Jaundice is dull and lacks luster E. Focal distention under the right ribs 3. A child patient, 7 days after birth, has a lustrous yellow face, eyes and skin. The jaundice quickly becomes serious and is accompanied by somnolence, unconsciousness, convulsion, and a red tongue with yellow coating. What is the most appropriate treatment formula? A. Yīn Chén Hāo Tāng (Virgate Wormwood Decoction) B. Yīn Chén Lǐ Zhōng Tāng (Virgate Wormwood Center-Regulating Decoction) C. Yīn Chén Wŭ Líng Săn (Five Substances Powder with Poria plus Virgate Wormwood) D. Xuè Fŭ Zhú Yū Tāng (Blood Mansion Stasis-Expelling Decoction) E. Líng Jiăo Gōu Téng Tāng (Antelope Horn and Uncaria Decoction) 4. A child patient, 23 days after birth, has face, eyes and skin that are lustrous yellow along with irritable crying, thirst, dry lips, yellow or reddish urine, dry bound stools, and a red tongue with a slimy yellow coating. Which Chinese prepared medicine should be administered by intravenous injection? A. Huáng Qí Zhù Shè Yè (Astragalus Injection) B. Dān Shēn Zhù Shè Yè (Salvia Injection)

C. Shēng Mài Zhù Shè Yè (Pulse-Engendering Injection) D. Shēn Fù Zhù Shè Yè (Ginseng and Aconite Injection) E. Yīn Zhī Huáng Zhù Shè Yè (Virgate Wormwood, Gardenia and Rhubarb Injection) 5. A child patient, 32 days after birth, has yellow and lackluster face, eyes and skin. There is sustained jaundice with abdominal distention, prominent veins, stasis macules scattered over the body, thin stools, stasis speckles on the tongue, and a yellow tongue coating. All of the following medicinals can be used except which ones? A. Chái hú (Radix Bupleuri) and yù jīn (Radix Curcumae) B. Zhĭ qiào (Fructus Aurantii) and dān shēn (Radix et Rhizoma Salviae Miltiorrhizae) C. Táo rén (Semen Persicae) and dāng guī (Radix Angelicae Sinensis) D. Dà huáng (Radix et Rhizoma Rhei) and máng xiāo (Natrii Sulfas) E. Chì sháo (Radix Paeoniae Rubra) and dān pí (Cortex Moutan)

Answers 1. B 2. E 3. E 4. E 5. D

PART VII Other Diseases Chapter 41 Summer Fever Summer fever, or summerheat syndrome, commonly attacks infants and young children during the summer season. Clinical manifestations include fever, thirst with increased fluid intake, polyuria, and hypohidrosis or anhidrosis. This disease usually affects those with congenital weakness that leads to kidney qi deficiency with spleen-stomach weakness or deficiencies of qi and yin; improper care can also be a contributing factor. Intolerable summerheat produces internal heat that consumes lung and stomach fluids bringing about fever which leads to thirst with increased fluid intake. When summerheat impairs the lung-wei, the lung fails to control opening and closing of the pores, also with impaired lung dispersion. Deficiency of fluids and qi contribute to adiaphoresis or ischidrosis. Common patterns include summerheat damaging the lung and stomach, and upper excess with lower deficiency.

CLINICAL ESSENTIALS Differentiation of summer fever, is based on the child’s physical condition and the clinical manifestations resulting from summerheat-evil impairing qi and yin, lung and stomach fluids, and kidney yang. At the onset, those with a healthy constitution will have a normal appetite and complexion, but there will be fever, thirst with increased fluid intake, polyuria, a red tongue and a rapid pulse; all of these are manifestations of summerheat impairing the lung and stomach. Those with chronic disease or a weak constitution will have a pale face, cold lower limbs, and loose stools in addition to the typical manifestations of summer fever, usually associated with patterns of upper excess and lower deficiency. The main treatment principles are to clear summerheat and discharge heat, and boost qi and engender fluids. The key to resolving summerheat lies in the removal of internal heat from the lung and stomach. Pungent and cooling medicinals should be used, and care must be taken not to overuse bitter-cold medicinals as they may impair yin. When boosting qi and engendering fluids, it is advisable to nourish the lung and stomach and assist center qi with sweet and moist medicinals; to prevent stagnation, overuse of these medicinals should also be avoided. Overuse of medicinals that drastically replenish qi or yang can also result in the accumulation of heat internally. For those with patterns of upper excess with lower deficiency, both heart and kidney are involved. Insufficient kidney yang leads to deficiency of kidney yin, and summerheat evil causes flaring up of heart-fire. In these cases, kidney yang must be warmed, while exuberant heart yang and heat-

evils affecting the heart must be eliminated. Dietary therapy can be effective during recovery, and of course summerheat should be avoided. Dr. Su Zhuo-huo holds that this disease is associated with the relatively delicate internal organs and the undeveloped yin and yang in children. Although it is a febrile disease, it cannot be treated by sweating to release the exterior, because qi desertion would follow perspiration loss. Clearing damp-heat through the stools must also not be allowed, as this can weaken the upright qi. Avoid extremely pungent and hot, or bitter and cold medicinals. Try not to use medicinals that are attacking and toxic because the internal organs could be damaged. If necessary, stop medicinal treatments when the desired effect has been achieved. Take care to assure that supplementing medicinals used to treat deficiency are enriching but not cloying, that medicinals for removing food accumulation are not too strong, that heatclearing drugs are not too cooling, and that dampness-dispelling medicinals are used properly. Dr. Su favors Wáng Shì Qīng Shŭ Yì Qì Tāng (Wang’s SummerheatClearing Qi-Boosting Decoction), but he omits medicinals that are bitter in taste and cold in property, such as huáng lián (Rhizoma Coptidis) and zhī mŭ (Rhizoma Anemarrhenae). He also suggests the use of fresh hé yè (Folium Nelumbinis) and xī guā pí (Exocarpium Citrulli) for clearing summerheat and promoting fluid production. Other commonly used medicinals include xī yáng shēn (Radix Panacis Quinquefolii), mài dōng (Radix Ophiopogonis), shí hú (Caulis Dendrobii), fú líng (Poria), dàn zhú yè (Herba Lophatheri), shí gāo (Gypsum Fibrosum), shān yào (Rhizoma Dioscoreae), gān căo (Radix et Rhizoma Glycyrrhizae), fresh hé yè (Folium Nelumbinis), and fresh xī guā pí (Exocarpium Citrulli).

CASE STUDY I Male, age 4. Initial Visit: August 10th, 2008 Chief Complaint: Fever, desire for drinks, diuresis and adiaphoresis for 1 month. History: The boy began to have a fever a month ago with the temperature of 38℃ to 39℃. He was hospitalized for 5 days and diagnosed with an acute upper respiratory infection. He was treated with penicillin and ribavirin, and with hormone and fluid infusions, but with no significant effect. He had a slightly lower temperature in the morning, but his temperature would gradually increase by noon. Thirst, polydipsia, polyuria, adiaphoresis and gradual loss of appetite were the main manifestations. Signs and Symptoms: Fever with no aversion to cold, cough or diarrhea, but with thirst, polydipsia, polyuria, long voidings of clear urine, adiaphoresis, a red tongue, a thin white coating with scant liquid, and a thready rapid pulse. Past History: Born premature with a weak constitution and susceptibility to cold. Physical Examination: BT 39℃, vexation and restlessness, burning sensation on the skin, blushing, dry lips, coarse breath sounds in both lungs, wet and dry rales not heard, strong heart sounds, pathological murmurs not heard, and soft abdomen with no tenderness or rebound tenderness. Laboratory Examination: No abnormalities were found.

Pattern Differentiation Summerheat consumed lung and stomach fluids which caused fever and thirst. Loss of fluids with exuberant interior heat led to the persistent fever, thirst and polydipsia. Heat impairing the lung resulted in adiaphoresis with a burning sensation on the skin. In addition to causing thirst and polydipsia, summerheat also consumes qi, which becomes unable to transform and distribute fluids; thus water accumulates in the bladder to result in polyuria and long voidings of clear urine in this case. Qi and fluid deficiency was reflected by the red tongue with a thin white coating, and the thready rapid pulse. The disease in this case was located in the lung and stomach, and diagnosed as qi and fluid deficiency associated with summerheat impairing the lung and stomach. This was a case of root deficiency with excess branch manifestations. This case should be differentiated from upper excess and lower deficiency, which would show indications of kidney yang deficiency marked by cold lower limbs, long voidings of clear urine, frequent urination, and loose stools, etc.

Diagnosis Summer fever due to summerheat impairing the lung and stomach with qi and fluid deficiency

Clinical Treatment

Principles: Clear summerheat, nourish yin to engender fluids Formula: Modified Wáng Shì Qīng Shŭ Yì Qì Tāng (Wang’s Summerheat-Clearing Qi-Boosting Decoction) [王氏清暑益⽓汤加减]

7 doses [Formula Analysis] Xī guā pí, hé gĕng and xiāng rú clear internal heat and resolve summerheat. Bĕi shā shēn, shí hú and mài dōng boost qi and promote fluid production. Zhī mŭ, zhú yè and huáng lián clear heat and drain fire. Gān căo reinforces the lung and stomach. Prepared Medicines

● Shēng Mài Yĭn Kŏu Fú Yè (Pulse-Engendering Beverage Oral Liquid), 5 ml, 3 times daily.

● Jiàn Ér Qīng Jiĕ Yè (Clearing and Draining for Infants’ Health Preservation Liquid), 5-10 ml, twice daily. Tui Na Push sān guān (三关), push liù fǔ (六腑) downward, separate yin and yang (fēn yīn yáng, 分阴阳), push pí jīng (脾经), and clear tiān hé shuĭ (天 河⽔). Then knead ST 44 (nèi tíng), ST 41 (jiĕ xī), ST 36 (zú sān lĭ) and SP 9 (yīn líng quán), and rub RN 6 (qì hăi) and RN 4 (guān yuán). 7 daily treatments comprise one course of treatment. Acupuncture Points: ST 36 (zú sān lĭ), RN 12 (zhōng wăn), DU 14 (dà zhuī), GB 20 (fēng chí), LI 4 (hé gŭ) Manipulations: Retain all needles for 2-3 min. Daily treatment for 7 days is one course of treatment; 1 to 2 courses are generally required. Second Visit After 7 days of treatment, the boy’s temperature returned to normal, but he still complained of thirst, a dry throat, poor appetite and fatigue. His urine and stools were normal. Signs and symptoms included a red tongue with little coating, and a thready rapid pulse. The summerheat-evil had been cleared and signs of excess were alleviated, but the root deficiency pattern marked by deficiency of fluids and qi in the lung and stomach remained. Principles: Nourish yin and replenish qi, clear heat Formula: Modified Shā Shēn Mài Dōng Tāng (Adenophora and Ophiopogon Decoction)

[沙参麦冬汤加减]

5 doses [Formula Analysis] Shā shēn, mài dōng, yù zhú, tiān huā fĕn and shí hú nourish the lung and stomach. Sāng yè removes heat-evil. Biăn dòu and gān căo harmonize and supplement stomach qi.

Prevention and Nursing Summer fever patients should be fed light foods. In cases of high fever, measures to promote physical cooling should also be employed.

CASE STUDY II Male, age 3. Initial Visit: July 20th, 1965 Chief Complaint: Fever for 16 days that is worse at night with no sweating, recurrent epistaxis over the last 2 days. History: The patient had taken medicines but still had fever, poor appetite, thirst, desire for cold drinks, and frequent copious urination, so he was brought to the hospital. Signs and Symptoms: Fever, a slightly sore throat, a wet tongue with greasy yellowish coating, and a deep wiry and rapid pulse. Past History: No history of measles, no recent contact with those afflicted. Physical Examination: BT 38.5℃, poor spirits, sore throat, 7 or 8 particle-like red spots seen on the right side of the oral mucosa, no abnormities detected in cardiac and pulmonary auscultation, soft abdomen, liver, kidney and spleen not palpable.

Diagnosis Summerheat with middle jiao dampness

Clinical Treatment Principles: Clear summerheat and eliminate dampness

Formula

One dose was prescribed. Second Visit July 21st There was no fever and no red spots in the oral mucosa, but there was thirst, no stools, intermittent abdominal pain, and a greasy yellowish tongue coating. Formula

One dose was prescribed. Third Visit July 22nd

No fever, no thirst, and thin white tongue coating; the patient was in high spirits with a good appetite. Source: He Shi-ying’s Pediatric Medical Records

COMMENTARY AND DISCUSSION Summerheat syndrome is characterized by a longstanding fever, thirst, polydipsia, diuresis, hypohidrosis or anhidrosis. All symptoms are easy to understand, except for manifestations of hypohidrosis or anhidrosis because summerheat is supposed to cause profuse sweating. Here, the hypohidrosis or anhidrosis is most likely caused by fluid exhaustion due to the summerheat-evil, or as a result of an external contraction of yin-pathogens such as cold or dampness. In the first case, the 4 year-old had suffered from the described symptoms for over a month. The red tongue with rough dry coating, and the thready rapid pulse show summerheat invasion with fluid deficiency. Therefore, Qīng Shŭ Yì Qì Tāng was applied. There are two prescriptions for clearing summerheat and supplementing qi in the history of Chinese formulas, and both of them are still commonly applied today. One is recorded in the Treatise on the Spleen and Stomach (Pí Wèi Lùn) by Li Dong-yuan (1249), and the other is from Warp and Woof of Warm-Heat Diseases (Wēn Rè Jīng Wĕi) by Wang Meng-ying (1852). The first formula is often called Lĭ Shì Qīng Shŭ Yì Qì Tāng (Li’s Summerheat-Clearing Qi-Boosting Decoction), which acts to supplement qi and eliminate summerheat and dampness. For this purpose, cāng zhú (Rhizoma Atractylodis), bái zhú (Rhizoma Atractylodis Macrocephalae) and zé xiè (Rhizoma Alismatis) are used to transform and dry dampness or

drain dampness by promoting urination. This makes great sense as summerheat often attacks with accompanying dampness-evil. The second formula, Wáng Shì Qīng Shŭ Yì Qì Tāng (Wang’s Summerheat-Clearing Qi-Boosting Decoction) focuses on clearing summerheat, supplementing qi and nourishing yin and fluids. It is most often applied for summerheat with qi and fluid deficiency but with only little or no invasion of dampness. Xiāng rú was added in this case to release the exterior, expel summerheat and transform dampness. It is commonly applied for externally-contracted cold in the summertime, as summerheat syndrome often shows hypohidrosis or anhidrosis. For this purpose, biăn dòu (Semen Lablab Album) can also be added to increase the effects. Sometimes, xiāng rú is replaced with qīng hāo (Medicinala Artemisiae Annuae), as qīng hāo can clear summerheat and also reduce fever from deficiency. Because this case had a long-lasting fever, qīng hāo together with xiāng rú would also have been an appropriate formula modification. It seems that in this case there was severe fluid deficiency, and therefore xī yáng shēn was replaced by bĕi shā shēn (Radix Glehniae) as northern shā shēn has stronger effect on nourishing body fluids. The other medicinals for fluid insufficiency in the modified formula included mài dōng, shí hú and xī guā pí. The watermelon (xī guā) is referred to as a natural form of Bái Hŭ Tāng (White Tiger Decoction) due to its ability to clear heat and nourish fluids; watermelon is the best dietary therapy for summerheat patterns.

STUDY QUESTIONS 1. What is the prognosis of this disease? 2. What were the respective manifestations of qi and yin deficiency at the second visit? 3. To what nursing considerations must one pay attention for this disease?

Answers 1. This case shows that the patient’s congenital weakness and the intolerable summerheat were main contributors. His body temperature rose as the environmental temperature rose. He had no complications. When the weather is cool in autumn, most cases would alleviate naturally, so the prognosis is good. 2. On the return visit, the patient’s complaints included fatigue and poor appetite along with other signs of qi deficiency. The thirst, dry throat, red tongue with little coating, and the thready rapid pulse all indicated yin deficiency. 3. When an infant has a high fever, it is important to keep the room temperature cool to around 26-29℃. It is important to feed light, highly nutritious foods. The patient should drink less hot water and take watermelon or honeysuckle juice instead of tea. Cán jiăn (Cocoon), hóng zăo (Fructus Jujubae), and wū méi (Fructus Mume) can also be decocted together as a beverage. In case of high fever, it is important to provide

physical cooling measures; showering with warm water can also help reduce the body temperature through sweating. To prevent complications, maintain cleanliness.

CASE SCENARIOS The following cases present variations of this condition. After familiarizing yourself with the possible common pattern presentations and appropriate formulas for treatment, use the following exercises to test your overall understanding of the condition. 1. A 9 month-old female infant has summerheat signs and symptoms, including lassitude, pale face, cold lower limbs, poor appetite, clear and frequent urination, loose stools, a persistent fever worse in the morning but better in the evening, thirst, polydipsia, a white tongue with a yellowish coating, and a thready rapid and weak pulse. What is the preferred treatment formula? A. Bái Hŭ Jiā Rén Shēn Tāng (White Tiger Decoction Plus Ginseng) B. Shā Shēn Mài Dōng Tāng (Adenophora and Ophiopogon Decoction) C. Wáng Shì Qīng Shŭ Yì Qì Tāng (Wang’s Summerheat-Clearing QiBoosting Decoction) D. Wēn Xià Qīng Shàng Tāng (Lower-Body Warming and UpperBody Clearing Decoction) E. Jīn Guì Shèn Qì Wán (Golden Cabinet’s Kidney Qi Pill) 2. A 2 year-old infant with a weak constitution has had a persistent fever since summer set in. Other signs and symptoms include thirst, polydipsia, polyuria, adiaphoresis, poor appetite, some fatigue, weak limbs

and a red tongue. There was no abnormality in the physical check-up and lab examination. What is the correct diagnosis? A. Summerheat syndrome B. Diabetes (xiāo kĕ, wasting-thirst) C. Dampness-prevalent diseases D. Summer non-acclimatization E. Cold caused by summerheat-evil 3. A 10 month-old child started to have a fever during the heat of the summer which has remained persistent; the higher the temperature is outside, the higher the infant’s body heat becomes. Other signs and symptoms include heat sensations on the skin, olighidria or adiaphoresis, thirst, frequent urination, urination immediately after drinking, irritability, loss of appetite, a red tongue with thin yellowish coating, a thready rapid pulse, and purple finger venules. What is the presenting pattern? A. Summerheat, upper excess with lower deficiency B. Summerheat, deficiency of both qi and yin C. Summerheat, summerheat-evil impairing lung and stomach D. Summer febrile disease, evil at the qi- and ying-levels E. Cold due to summerheat-evil 4. A one year-old child has had a persistent fever ever since the summer began. His body temperature tends to increase in the afternoon, and has shown no sign of decreasing. The higher the temperature is outside, the

higher his body heat becomes. Other signs and symptoms include thirst, adiaphoresis, dry skin, a normal appetite, weak limbs, frequent urination, and a red tongue with a thin yellowish coating. What is the best treatment formula? A. Bái Hŭ Jiā Rén Shēn Tāng (White Tiger Decoction Plus Ginseng) B. Bái Hŭ Tāng (White Tiger Decoction) and Shēng Mài Săn (PulseEngendering Powder) C. Zhú Yè Shí Gāo Tāng (Lophatherum and Gypsum Decoction) D. Qīng Shŭ Yì Qì Tāng (Summerheat-Clearing Qi-Boosting Decoction) E. Yù Nǚ Jiān (Jade Lady Decoction) 5. An 18 month-old child has a persistent fever that is worse in the morning but better in the evening. Other signs and symptoms include a pale face, thirst, polydipsia, adiaphoresis, dysphoria, lassitude, drowsiness, cold lower limbs, loose stools, clear and frequent urination, and white finger venules. The diagnosis is summerheat syndrome. What is the pathogenesis? A. Spleen-stomach deficiency B. Spleen-kidney yang deficiency C. Hypoactivity of spleen yang D. Exuberant heat in the stomach E. Slightly weakened kidney yang 6. A 3 year-old child, has had fever for over 20 days, beginning in the summer. His temperature has been 38-39℃. Other signs and symptoms

include olighidria, cold limbs, thirst, thirst, poor appetite, vexation, dry stools, increased urine, a red tongue with a white greasy coating, and a thready rapid pulse. What are the correct principles of treatment? A. Warm and supplement kidney yang, clear heat-evil from the heart, preserve fluids B. Clear summerheat, eliminate dampness, clear heat, release exterior C. Clear summerheat-evil and boost qi, nourish yin to engender fluids D. Clear heat-evil, drain dampness, nourish qi to invigorate the spleen E. Clear heat, eliminate toxins, release the exterior with pungentcoldness

Answers 1. D 2. A 3. C 4. D 5. B 6. C

Chapter 42 Mucocutaneous Lymph Node Syndrome Mucocutaneous lymph node syndrome, also called Kawasaki disease, is an acute febrile eruptive disorder with systemic vasculitis as the main pathology. The clinical manifestations include fever of unknown origin, erythema multiforme, conjunctival hyperemia, cervical lymphadenopathy, “strawberry tongue”, and indurative edema of the hands and feet. It is very common among infants. This disease is caused by warm-heat toxins which enter the mouth and nose to affect the lung-wei and accumulate in the muscle layer, resulting in fever. Then, heat immediately invades interiorly to the lung and stomach, exuberant yang heat blazes in the qi-level, scorching and steaming yingblood and resulting in blood and fluid consumption. The major signs and symptoms include high fever, macules and papules on the skin, and congestion of the conjunctiva and oral mucosa. On the one hand, heattoxins and phlegm obstruct the channels and collaterals, causing swelling and pain of lymph nodes. On the other hand, exuberant heat damages body fluids causing dry mouth and a red “strawberry-like” tongue. Also, heat flames ying-blood and blocks blood movement, causing blood stasis. In the advanced stage the internal heat resolves, but because qi is deficient and body fluids are depleted, there is fatigue and peeling of the skin on the fingers and toes.

Common clinical patterns involve the wei-level and qi-level simultaneously, with heat flaming and scorching both the qi- and yinglevels, also damaging both qi and yin.

CLINICAL ESSENTIALS In treating this disease, four-level pattern differentiation is the first priority. The disease begins in the lung-wei following the pathogenic invasion, with signs and symptoms of fever, aversion to wind and a red pharynx. Then, pathogenic heat rapidly invades interiorly to the lung and stomach with exuberant heat in the qi-level; manifestations include a persistent fever, dry mouth with thirst, and spreading rashes. At the next stage, as heat enters ying-blood, there are reddish-purple macules and papules, a strawberry tongue, fidgeting, and somnolence. Finally, both qi and yin are damaged; signs and symptoms include fatigue, profuse sweating, and peeling of the skin on the ends of the fingers and toes. The disease is also inclined to cause blood stasis manifesting with purple macules and papules, swelling of palms and soles, a red tongue, and dark purple finger venules. When static blood obstructs the channels and collaterals, there may be palpitations and lumps in the right rib-side. Treatment primarily focuses on clearing heat, resolving toxins, invigorating blood and dissolving stasis.

● At the early stage, acrid-cool medicinals are used to scatter wind, clear heat, resolve toxins and vent pathogens.

● With exuberant heat, bitter-cold medicinals are used to clear heat from the qi-level, cool the ying-level, and resolve toxins.

● At the final stage, there is consumption of qi and damage to yin, so sweet-cold medicinals are used to boost qi and nourish yin.

● Because the internal heat often transforms to fire, which is likely to damage yin, it is also necessary to nourish stomach-fluids and protect heart yin, depending on the particular stage of disease.

● Furthermore, because blood stasis is also a common development, treatments that invigorate blood and dissolve stasis must be applied from the early to the final stages.

CASE STUDY I Male, age 3. Initial Visit: October 25th, 2008 Chief Complaint: Fever for 10 days, rashes and swelling of the hands and feet for 1 week. History: The child had a fever of unknown origin with no chills, convulsion, cough, vomiting or diarrhea. His temperature was brought down only temporarily with antipyretics. 7 days ago, red rashes appearing like macules and papules were especially striking on the back. The rashes faded when pressed, became more obvious when the body temperature rose, and declined when body temperature dropped. Meanwhile, his hands and feet were so swollen that he could not stand. The child was treated in a local hospital with cephalosporin-type antibiotics for 6 days, with no significant alleviation of the symptoms. Signs and Symptoms: A high fever with red eyes and face, swelling of the hands and feet, no cough, chills or convulsion, irregular perspiration, indistinct macules and papules, a red strawberry-like tongue, and a rapid slippery pulse. Physical Examination: BT 38.9℃, red macules and papules all over the body that fade when pressed, especially obvious on the back, chest and buttocks, swelling of cervical lymph nodes, conjunctival congestion in the eyes, red dry lips, pharyngeal hyperemia, enlargement of both tonsils to degree Ⅱ, strawberry-like tongue, heart and lungs normal, obvious swelling of both palms and soles with pain when pressed with no membranous peeling.

Laboratory Examination: WBC 19.7 × 109/L, N 76%, L 24%, ESR (erythrocyte sedimentation rate) 112 mm/h, CRP (+), ASO (Antistreptolysin O antibody) < 400 U.

Pattern Differentiation First, the externally-contracted warm-febrile and seasonal pathogen with a soaring high fever transformed into heat blazing in the qi and ying that burned and steamed ying-blood, thus causing the indistinct maculae and papules. Second, heat congealing and blood stasis blocked the smooth flow of qi and blood, resulting in the obvious swelling of both palms and soles with pains when pressed. Third, the warmth-pathogen moved upwards and brought about pharyngeal hyperemia. With a mixture of heat and toxins, phlegm hindered the channels and collaterals, giving rise to the swollen lymph nodes. As the warmth-heat pathogen transformed to fire, body fluids were depleted, manifesting with a red strawberry tongue and dry red lips. These two symptoms, together with the rapid slippery pulses, are also manifestations of heat blazing in both qi and ying. Further examinations would include ultrasonic cardiogram, blood platelet, electrocardiogram, and others.

Diagnosis

Mucocutaneous lymph node syndrome due to heat blazing in both qi and ying

Clinical Treatment This case, in the critical stage of the disease, shows the exuberance of heat and has symptoms typical of blazing of qi and ying. The treatment should focus on the branches by clearing heat from the qi level, cooling the ying-level, and resolving toxins with bitter-cold medicinals. Combination with some Western medicines like gamma globulins and aspirin can quickly improve symptoms and prevent coronary artery ectasia and coronary aneurysm. Principles: Clear qi-level heat, cool the ying-level, resolve toxins, dissolve stasis Formula: Modified Qīng Wēn Bài Dú Yĭn (Epidemic-Clearing ToxinResolving Beverage) [清瘟败毒饮加减]

4 doses [Formula Analysis] Shuĭ niú jiăo, mŭ dān pí and chì sháo clear ying-level toxins, cool blood and dissolve stasis. Mài dōng and zhú yè act to clear heat and nourish yin with sweetcoldness. Shēng shí gāo and zhī mŭ clear qi-level heat. Huáng qín and zhī zĭ drain fire. Xuán shēn and shēng dì clear heat and nourish yin. Gān căo clears heat, resolves toxins, and harmonizes all the formula medicinals. Prepared Medicines

● Rè Dú Níng Zhù Shè Yè (Heat-Toxin Clearing Injection), 10 ml mixed with 100 ml 5% dextrose solution, intravenous infusion once daily.

● Dān Shēn Dī Wán (Salvia Dripping Pills), 1-3 pills, 3 times daily. Second Visit

After 4 days of treatment, the fever was reduced, but he was still tired and lacked strength. Other signs and symptoms included profuse sweating, dry pharynx and mouth, peeling of the finger and toe ends, a poor appetite, a red tongue with a thin coating, and a weak wiry and irregular pulse. After the above treatment, the typical symptoms of exuberant heat blazing in both qi and ying had almost disappeared; however, signs of qi and yin damage had not been resolved. Principles: Nourish yin, boost qi, clear remaining pathogens Formula: Modified Shā Shēn Mài Dōng Tāng (Adenophora and Ophiopogon Decoction) [沙参麦冬汤加减]

5 doses [Formula Analysis] Shā shēn, mài dōng, yù zhú, tiān huā fĕn and shí hú enrich and nourish the lung, stomach and body fluids. Sāng yè clears pathogenic heat. Biăn dòu and gān căo nourish stomach qi.

CASE STUDY II Female, age 3. Initial Visit: April 21st, 1990 Chief Complaint: Fever for 6 days with red macules and papules all over the body, especially on the neck and torso. History: Along with the fever, macules and papules, presenting signs and symptoms included cough with no phlegm, nausea, vomiting of clear water, abdominal pain, watery diarrhea three times per day, pain of the knee joints, red dry lips, strong thirst, a strawberry tongue, and a surging pulse. Physical Examination: BT 40.5℃, RR 40 times/min, PR 180 beats/min, BP 90/75 mmHg, red macules and papules in various sizes and shapes that fade when pressed, a broadbean-sized lymph node on the right lower jaw that was hard and painful when touched, conjunctival congestions in the eyes, red pharynx, swelling of the tonsils to degree Ⅱ, dry lips and mouth, and rough breathing sounds. Chest X-ray showed bronchitis and interstitial pneumonia. Electrocardiogram normal. Laboratory Examination: Renal function normal, liver function GPT 50 U/L, ESR 110 mm/h, WBC 18.9 × 109 /L, N 75%, L 25%, PLT 606 × 109/L.

Diagnosis Macules and papules due to heat blazing in qi and ying

Clinical Treatment Formula: Modified Qīng Wēn Bài Dú Yĭn (Epidemic-Clearing ToxinResolving Beverage) [清瘟败毒饮加减]

3 doses Second Visit After 3 doses of the above formula, her temperature fell to 38.9℃ and the abdominal pain, diarrhea and vomiting disappeared. The red dry lips and thirst persisted, while there was also swelling and pain of the fingers, burning pains of the knees, and coughing with no phlegm. These signs and symptoms were caused by damp-heat obstructing the channels and collaterals. Principles: Clear heat, percolate dampness, diffuse bi, unblock the collaterals Formula: Zhōng Jiāo Xuān Bì Tāng (Dispersing Bi in the Middle Decoction)

[中焦宣痹汤]

Third Visit The body temperature was 37.8 ℃; her appetite improved, and there was no knee swelling or pain. There was coughing with phlegm, dry red lips, lip corner erosion, nighttime thirst, phalangeal joint swelling, and a red tongue with a thin dry yellow coating. These signs and symptoms were caused by pathogenic heat consuming and damaging body fluids. Principles: Enrich yin, nourish body fluids, moisten the lung, relieve cough Formula: Modified Rùn Fèi Yĭn (Lung-Moistening Beverage) [润肺饮加减]

Fourth Visit After the above treatments, the child became vigorous with no fever or rashes. The remaining signs and symptoms included occasional cough, peeling of the finger-ends and thirst, but there were no lymph nodes on the lower jaw or anywhere else on the body. PLT 606 × 109/L, and the electrocardiogram normal with RF (-). Formula: Modified Xuè Fŭ Zhú Yū Tāng (Blood Mansion StasisExpelling Decoction) [⾎府逐瘀汤加减]

The child recovered from all the symptoms after 20 days in the hospital, with PLT 595 × 109/L and ESR 51 mm/h. She was discharged from the hospital and was given the formula for another half-month, with yù jīn (Radix Curcumae), dān shēn (Radix et Rhizoma Salviae Miltiorrhizae) and sū mù (Lignum Sappan) added according to the presenting symptoms.

Follow-up PLT 199 × 109/L, ESR and electrocardiogram both normal.

Note The child presented with a qi-level pattern manifesting with a high fever, fidgeting, thirst, and a surging pulse. At the same time, she had yinglevel signs such as the dense covering of macules and papules, red lips and mouth, and impaired fluids. The macules and papules were caused by pestilential toxins, while the presenting pattern was heat blazing in both qi and ying. A modification of Qīng Wēn Bài Dú Yĭn (Epidemic-Clearing Toxin-Resolving Beverage) was used to clear heat, resolve toxins and cool the blood to the effect of clearing both qi- and ying-levels. For the vomiting and diarrhea, modifications of Sū Yè Huáng Lián Tāng (Perilla Leaf and Coptis Decoction) and Xiāng Lián Wán (Costusroot and Coptis Pill) were added. In the course of the disease, treatments aimed at the joint swelling and pain were also applied, with Zhōng Jiāo Xuān Bì Tāng given to treat the obstruction of collaterals and channels. Rùn Fèi Yĭn was given to relieve

cough, moisten the lung and dissolve phlegm, with which the rashes and other symptoms all disappeared. Pathological changes mainly include systemic vasculitis, which may cause focal inflammation and necrosis of the internal organs. Inflammation of the arteries, especially the coronary artery, can lead to thrombosis, stenosis and aneurysm; in fact, this disease is a key cause of coronary heart disease in children. Because of this, when all other symptoms had disappeared except for the elevated ESR and PLT levels, in order to avoid the above situations from occurring, a modification Xuè Fŭ Zhú Yū Tāng was given to invigorate blood and dissolve stasis. After constant observation of changes in the electrocardiogram and PLT, within 2 months, ESR and PLT levels became normal.

COMMENTARY AND DISCUSSION Mucocutaneous lymph node syndrome manifests as a systemic, necrotizing, medium-sized vessel vasculitis, and is seen largely in children less than 5 years of age. It may affect many areas of body including the blood vessels, skin, mucous membranes and lymph nodes. However, mucocutaneous lymph node syndrome is also a leading cause of acquired heart disease in children. About 1 in 5 children with the disease develop heart problems which may include inflammation of the heart muscle (myocarditis), heart valve problems (mitral regurgitation), aneurysms leading to heart attack, inflammation of blood vessels, usually the coronary arteries, and abnormal heart rhythm (dysrhythmia). In Chinese medicine, this disease falls into the category of warm diseases caused by toxic-warmth and heat-evils. Regarding the transmission of warm disease, Ye Tianshi (1667-1746) stated in Treatise on Warm-Heat Diseases (Wēn Rè Lùn), “when warmevil attacks the upper part of the body, it first invades the lung, and it may also attack the pericardium, exceptionally” (温邪上受,⾸先犯肺,逆传⼼包). When evils invade the lung, the wei- and qi-levels are mainly affected, whereas when the heart and pericardium are attacked, the ying- and bloodlevels are involved. The characteristics of the disease transmission explain why there are many potential heart and blood vessel disorders, even though in accordance with the four-level pattern identification, the symptoms in the ying and blood levels are more likely to result in mental disorders (which are also considered as a heart dysfunction).

In this case, the high fever with red eyes and face are symptoms of qilevel pathogens, while the red macules and papules indicate that the yinglevel is also affected. Thus, both qi and ying are involved, and Qīng Wēn Bài Dú Yĭn (Epidemic-Clearing Toxin-Resolving Beverage) was applied to good effect. This prescription is a modification of four formulas: Bái Hŭ Tāng (White Tiger Decoction), Liáng Gé Săn (Diaphragm-Cooling Powder), Huáng Lián Jiĕ Dú Tāng (Coptis Toxin-Resolving Decoction) and Xī Jiăo Dì Huáng Tāng (Rhinoceros Horn and Rehmannia Decoction). Because of this, the formula acts to clear heat and toxins from both exterior and interior, and from the qi- and ying-levels. In Achievements in the Treatment of Epidemic Rashes (Yì Zhĕn Yī Dé), Yu Shi-yu said that the formula “drains fire from all twelve channels and is an essential prescription for all types of heat and fire, internally and externally with symptoms of mania, restlessness, thirst, sore throat, high fever, vomiting, insomnia, delirium and soliloquy, epistaxis and haematemesis, macules and papules”(治⼀切⽕热,表⾥俱盛,狂躁烦⼼。⼜⼲咽痛,⼤热⼲呕, 错语不眠,吐⾎衄⾎,热盛发斑). Yu Shi-yu discussed fifty-two syndromes in his book, all treated with modifications of Qīng Wēn Bài Dú Yĭn. This shows that Qīng Wēn Bài Dú Yǐn can be extensively applied for red macules and papules, whether in the beginning or ending stages. Returning to the beginning of the comments, since there is potential cardiovascular damage, attention should be paid to promoting blood circulation and treating and preventing blood stasis caused by the blood coagulation due to toxic-warmth and heat-evils. In this case, dān shēn was given at the first visit. Dān shēn acts to invigorate blood and resolve blood stasis. Modern research shows that dān shēn has a marked vasodilatory effect on the coronary arteries of guinea pig and rabbit heart specimens.

Dān shēn is also commonly applied for the treatment of angina pectoris. In this case, dān shēn effectively protects the heart and blood vessels, particularly the coronary blood vessels, so no heart complications appeared. As stated previously “…because blood stasis is a common development, treatments that invigorate blood and dissolve stasis must be applied from the early to the final stages”.

STUDY QUESTIONS 1. What is the prognosis of the disease? 2. What were the main manifestations of qi deficiency and yin deficiency at the second visit? 3. To what must one pay attention in nursing this disease?

Answers 1. The prognosis for the majority of the patients is good, with a selflimiting course; patients can be helped in their gradual recovery with proper treatment. Coronary artery aneurysm may happen in 15-30 percent of patients. Death caused by coronary artery aneurysm, thromboangiitis obliterans or myocarditis may occur in 1%-2% of all patients, even during the recovery phase. There are a few examples of subsequent ischemic heart disease. The disease recurs in 2%, and the death rate has dropped to 0.5%-1.0% in recent years. 2. At the second visit, the fatigue and poor appetite showed qi deficiency, while the dry pharynx and mouth and peeling of the finger and toes showed yin deficiency. The red tongue with a thin coating and the thin and weak irregular pulses are both manifestations of qi and yin deficiency. 3. Foods are to be light and fresh, and patients must drink sufficient water. Keep the mouth clean, and rest adequately in bed. Closely observe the changing state of the disease, and try to recognize any complications early on. Follow-up visits should continue for half a year to one year. Long-

term follow-up visits are necessary for coronary artery ectasia, with at least one bi-annual ultrasonic electrogram examination until the symptoms have disappeared.

CASE SCENARIOS The following cases present variations of this condition. After familiarizing yourself with the possible common pattern presentations and appropriate formulas for treatment, use the following exercises to test your overall understanding of the condition. 1. A patient, age 18 months, has a fever with a temperature of 39.3℃ to 39.8℃. Other signs and symptoms include conjunctival hyperemia, hard edema of the hands and feet, cervical lymphadenopathy, and no effective treatment from many antibiotics. What is the diagnosis? A. Acute conjunctivitis B. Acute glomerulonephritis C. Juvenile rheumatoid disease D. Mucocutaneous lymph node syndrome E. Infectious mononucleosis 2. A patient, age 24 months, got an abrupt high fever. Other signs and symptoms include red eyes and pharynx, flushing of palms and soles, obvious rashes on the torso and a floating and rapid pulse. What is the presenting pattern? A. Externally contracted wind-cold B. Summerheat

C. Disease of both the wei and qi D. Blazing of both qi and ying E. Heat entering the ying-blood 3. A patient, age 20 months, has high fever that serious at night but lighter during the day. Other signs and symptoms include dry chapped lips, macules and papules on the skin, and a red strawberry tongue. What are the correct principles of treatment? A. Boost qi and nourish yin B. Clear the remaining heat C. Release the exterior with acrid-coolness D. Clear qi-level heat and cool ying E. Release summerheat 4. A patient, age 22 months, has had an abrupt fever for 10 days. Other signs and symptoms include minor aversion to wind, red eyes and pharynx, rashes on the torso, occasional coughing, and a floating rapid pulse. What is the best treatment formula for this case? A. Xiè Xīn Tāng (Heart-Draining Decoction) B. Shēng Mài Yĭn (Pulse-Engendering Beverage) C. Yín Qiào Săn (Lonicera and Forsythia Powder) D. Shā Shēn Mài Dōng Tāng (Adenophora and Ophiopogon Decoction) E. Má Xìng Shí Gān Tāng (Ephedra, Apricot Kernel, Gypsum and Licorice Decoction)

5. A patient, age 19 months, has a high fever. Other signs and symptoms include cervical lymphadenopathy, hard edema of the hands and feet, red eyes and pharynx, macules and papules on the skin, a red tongue, and a forceful rapid pulse. What is the best treatment formula for this case? A. Má Xìng Shí Gān Tāng (Ephedra, Apricot Kernel, Gypsum and Licorice Decoction) B. Qīng Wēn Bài Dú Yĭn (Epidemic-Clearing Toxin-Resolving Beverage) C. Dà Chéng Qì Tāng (Major Purgative Decoction) D. Sāng Jú Yĭn (Mulberry Leaf and Chrysanthemum Beverage) E. Yín Qiào Săn (Lonicera and Forsythia Powder) 6. A patient, age 10, has had persistent high fever for 2 weeks. Other signs and symptoms include flushing of the mouth and lips, strawberry tongue, rashes appearing on the 4th day of fever, growth of WBC and neutrophils in a peripheral hemogram, reduction of lymphocytes, increase of PLT, abnormality of T wave and ST-segment in an electrocardiogram, and no effect from treatment with various antibiotics. What is the diagnosis? A. Infectious mononucleosis B. Vital myocarditis C. Mucocutaneous lymph node syndrome D. Infection of the upper respiratory tract E. Scarlet fever

7. A patient, age 2, has had a high fever of 40℃ for 7 days, worse at night than during day. Other signs and symptoms include red chapped lips, fidgeting, and skin rashes. What is the disease mechanism? A. Pathogens lingering in the lung and stomach B. Disease of both wei and qi C. Pathogens lingering in shaoyin D. Heat blazing in both qi and ying E. Pathogens lingering in taiyang 8. A patient, age 3, has had high fever for 10 days. Other signs and symptoms include red chapped lips, rashes on the skin, cervical lymphadenopathy, constipation, a red tongue, and a forceful rapid pulse. In addition to clearing heat and resolving toxins, for what else could dà huáng be added? A. To dredge the bowels B. To cool the blood and dissolve stasis C. To preserve yin with emergency purgation D. To drain warm-heat through the stool E. To soften hardness and dissipate masses 9. A patient, age 2, has had a fever for over 20 days that is gradually fading at present. Other signs and symptoms include a dry pharynx and chapped lips, strong thirst for drinks, hard stools, red tongue with a thin tongue coating, and a thin pulse.

In this case, what medicinal can be added to promote defecation? A. Dà huáng (Radix et Rhizoma Rhei) B. Máng xiāo (Natrii Sulfas) C. Guā lóu rén (Semen Trichosanthis) D. Fān xiè yè (Folium Sennae) E. Gān căo (Radix et Rhizoma Glycyrrhizae)

Answers 1. D 2. C 3. D 4. C 5. B 6. C 7. D 8. C 9. C

Appendix I: Pinyin-English Formula Cross Reference

Appendix II: List of Cited Sources